You are on page 1of 362

Joint Admission Test for M.Sc.

From IITs

Physics

Solved Papers
2 0 2 1 - 2 0 0 5
3 Practice Sets

Atique Hassan

ARIHANT PRAKASHAN (SERIES), MEERUT


Joint Admission Test for M.Sc. From IITs

Physics

Arihant Prakashan (Series), Meerut


All Rights Reserved

© Publisher

Administrative & Production Offices


Regd. Office
‘Ramchhaya’ 4577/15, Agarwal Road, Darya Ganj, New Delhi -110002
Tele: 011- 47630600, 43518550

Head Office
Kalindi, TP Nagar, Meerut (UP) - 250002, Tel: 0121-7156203, 7156204

Sales & Support Offices


Agra, Ahmedabad, Bengaluru, Bareilly, Chennai, Delhi, Guwahati,
Hyderabad, Jaipur, Jhansi, Kolkata, Lucknow, Nagpur & Pune.

ISBN 978-93-25299-34-4

PO No : TXT-XX-XXXXXXX-X-XX
Published by Arihant Publications (India) Ltd.
For further information about the books published by Arihant, log on to
www.arihantbooks.com or e-mail at info@arihantbooks.com
Follow us on
Joint Admission Test for M.Sc. From IITs

Physics

PREFACE
Joint Admission Test for M.Sc. (JAM) is an all India admission test conducted
jointly by the Indian Institutes of Technology (IITs) and Indian Institute of Science
(IISc), Bangalore on behalf of the Ministry of Human Resources Development.
This test enables a candidate to get admission into M.Sc. programmes, joint
Ph.D. dual-degree, and other post B.Sc programmes at various IITs. The main
objective of JAM is to consolidate Science as a career option by motivating the
students about the research infrastructure and the vibrant academic
environment of IISc Bangalore and IITs for bright students across the country.
So it is very considerable to get admission into the IITs if we choose the science
and research as the career options. So by keeping this point of view and after
exhaustive research, I have come up with the book Solved Papers and Practice
Sets of IIT JAM Physics published by Arihant Prakashan for the aspirants who are
preparing for the IIT JAM. This book having Solved Papers (2021-2005) and
3 Practice Sets will work as an ultimate tool to encounter the IIT JAM exam.

Salient Features of the Book


— This book contains previous years' solved papers (2021-2005) of IIT JAM Physics with
their detailed and authentic solutions to put you into the real practice.
— 3 Simulated Practice Sets designed as per existing test pattern.
— All the questions in practice sets are framed in a manner to provide the real feel to
the aspirants in terms of latest pattern and difficulty level.
— Step by step solution of each question in solved papers and practice sets has been
provided to increase the edificial knowledge of the aspirant.

I am thankful to Arihant Prakashan for giving me this opportunity to make such


a book which will help you to get 100% success in IIT JAM.
Suggestions for the further improvements of the book are most welcome.

Author
Joint Admission Test for M.Sc. From IITs

Physics

CONTENTS
SOLVED PAPERS
Solved Paper 2021 3-27
Solved Paper 2020 1-24
Solved Paper 2019 1-18
Solved Paper 2018 1-18
Solved Paper 2017 1-27
Solved Paper 2016 1-24
Solved Paper 2015 3-25
Solved Paper 2014 26-42
Solved Paper 2013 43-55
Solved Paper 2012 56-69
Solved Paper 2011 70-81
Solved Paper 2010 82-94
Solved Paper 2009 95-106
Solved Paper 2008 107-120
Solved Paper 2007 121-135
Solved Paper 2006 136-148
Solved Paper 2005 149-160

PRACTICE SETS
Practice Set 1 163-179
Practice Set 2 180-196
Practice Set 3 197-213
Joint Admission Test for M.Sc. From IITs

Physics

THE JOINT ADMISSION TEST FOR M.Sc. (JAM) IS AN ADMISSION TEST TO MASTER OF SCIENCE (M.Sc.) AND OTHER
POST-GRADUATE SCIENCE PROGRAMMES AT THE INDIAN INSTITUTES OF TECHNOLOGY, INDIAN INSTITUTE OF
SCIENCE, AND OTHER INSTITUTES. IITs STARTED CONDUCTING THE JAM IN THE 2004-2005 ACADEMIC SESSION.
THE MAIN OBJECTIVE OF JAM IS TO PROVIDE ADMISSIONS TO VARIOUS M.Sc., M.Sc.-Ph.D. DUAL-DEGREE AND
OTHER POST-GRADUATE PROGRAMMES BASED ON THE PERFORMANCE IN A SINGLE TEST. IN DUE COURSE, JAM IS
ALSO EXPECTED TO BECOME A BENCHMARK FOR NORMALISING UNDERGRADUATE-LEVEL SCIENCE EDUCATION
IN THE COUNTRY.

PATTERN OF TEST PAPERS


The JAM examination for all the seven test papers is carried out as ONLINE Computer Based Test (CBT) where
the questions shown to the candidates in a random sequence on a computer screen. For all the seven test
papers, the duration of the examination is 3 hours. The medium for all the test papers is English only. There is
a total of 60 questions carrying 100 marks. The entire paper is divided into three sections, A, B and C. All the
sections are compulsory. Questions in each section are of different types as given below:

Section–A
It contains a total of 30 Multiple Choice Questions (MCQs) carrying one or two marks each. Each MCQ type
question has four choices out of which only one choice is the correct answer. Candidates can mark the answer
by clicking the choice.

Section–B
It contains a total of 10 Multiple Select Questions (MSQs) carrying two marks each. Each MSQ type question
is similar to MCQ but with a difference that there may be one or more than one choice(s) that are correct out
of the four given choices. The candidate gets full credit if he/she selects all the correct answers only and no
wrong answers. Candidates can mark the answer(s) by clicking the choice(s).

Section–C
It contains a total of 20 Numerical Answer Type Questions (NATQs) carrying one or two marks each. For
these NAT type questions, the answer is a signed real number which needs to be entered using the virtual
numerical keypad shown on the monitor. No choices will be shown for these type of questions, candidates
have to enter the answer by using a virtual numerical keypad only.
Joint Admission Test for M.Sc. From IITs

Physics

Marking Pattern of the Test Papers


In all sections, questions not attempted will result in zero mark. In Section – A (MCQs), wrong answer
will result in negative marks. For all 1 mark questions, 1/3 marks will be deducted for each wrong
answer. For all
2 marks questions, 2/3 marks will be deducted for each wrong answer. In Section – B (MSQs), there
are no negative and no partial marking provisions. There is no negative marking in Section – C
(NATQs) as well.

Things Which are Strictly Prohibited


— There is provision of using online virtual calculator and hence, the candidates should not bring
any calculator with them.
— Mobile phones or any other electronic devices are
strictly prohibited inside examination hall. Charts,
graph sheets, tables, are also NOT allowed inside the
examination hall.
A scribble pad will be provided for rough work and
this has to be returned back at the end of the
examination.
— The candidates are required to select the answer for
MCQs and MSQs type questions, and to enter the answer for NATQs, use a mouse on a virtual
numeric keypad (the keyboard of the computer will be disabled). At the end of the 3-hours,
the computer will automatically close the examination.

ELIGIBILITY REQUIREMENTS (ERs) FOR ADMISSION


The candidates who qualify in JAM shall have to fulfill the following Eligibility Requirement (ER) for
admissions in IISc and IITs.
— All candidates admitted through JAM should have a Bachelor's degree.
— At least 55% aggregate marks, without rounding off, (taking into account all subjects, including
Languages and Subsidiaries, all years combined) for General/OBC-NCL Category Candidates
— At least 50% aggregate marks, without rounding off, (taking into account all subjects, including
Languages and Subsidiaries, all years combined) for SC/ST and PwD Category Candidates in the
qualifying degree.
Joint Admission Test for M.Sc. From IITs

Physics

SYLLABUS
Mathematical Methods Calculus of single and multiple variables, Partial derivatives, Jacobian,
Imperfect and perfect differentials, Taylor expansion, Fourier series, Vector algebra, Vector
calculus, Multiple integrals, Divergence theorem, Green's theorem, Stokes' theorem. First order
equations and linear second order differential equations with constant coefficients. Matrices
and determinants, Algebra of complex numbers.

Mechanics and General Properties of Matter Newton's laws of motion and applications,
Velocity and acceleration in cartesian, Polar and cylindrical coordinate systems, Uniformly
rotating frame, Centrifugal and coriolis forces, Motion under a central force, Kepler's laws,
Gravitational law and field, Conservative and non-conservative forces, System of particles,
Center of mass, Equation of motion of the CM, Conservation of linear and angular momentum,
Conservation of energy, Variable mass systems, Elastic and inelastic collisions, Rigid body
motion, Fixed axis Rotations, Rotation and translation, Moments of inertia and products of
inertia, Parallel and perpendicular axes theorem, Principal moments and axes, Kinematics of
moving fluids, Equation of continuity, Euler's equation, Bernoulli's theorem.

Oscillations, Waves and Optics Differential equation for simple harmonic oscillator and its
general solution, Superposition of two or more simple harmonic oscillators, Lissajous figures,
Damped and forced oscillators, Resonance, Wave equation, Traveling and standing waves in
one-dimension, Energy density and energy transmission in waves, Group velocity and phase
velocity, Sound waves in media, Doppler effect, Fermat's principle, General theory of image
formation, Thick lens, thin lens and lens combinations, Interference of light, Optical path
retardation, Fraunhofer diffraction, Rayleigh criterion and resolving power, Diffraction gratings,
Polarization: linear, circular and elliptic polarization, Double refraction and
optical rotation.

Electricity and Magnetism Coulomb's law, Gauss's law, Electric field and potential, Electrostatic
boundary conditions, Solution of laplace's equation for simple cases, Conductors, Capacitors,
Dielectrics, Dielectric polarization, Volume and surface charges, Electrostatic energy, Biot-Savart
law, Ampere's law, Faraday's law of electromagnetic induction, Self and mutual inductance,
Alternating currents, Simple DC and AC circuits with R, L and C components, Displacement
current, Maxwell's equations and plane electromagnetic waves, Poynting's theorem, Reflection
and refraction at a dielectric interface, Transmission and reflection coefficients (normal
incidence only), Lorentz force and motion of charged particles in electric and
magnetic fields.
Joint Admission Test for M.Sc. From IITs

Physics

Kinetic theory, Thermodynamics Elements of kinetic theory of gases, Velocity


distribution and equipartition of energy, Specific heat of mono-, di- and tri-atomic
gases, Ideal gas, Van-der-Waals gas and equation of state, Mean free path, Laws of
thermodynamics, Zeroth law and concept of thermal equilibrium, First law and its
consequences, Isothermal and adiabatic processes, Reversible, Irreversible and quasi-
static processes, Second law and entropy, Carnot cycle, Maxwell's thermodynamic
relations and simple applications, Thermodynamic potentials and their applications,
Phase transitions and Clausius-Clapeyron equation, Ideas of ensembles, Maxwell-
Boltzmann, Fermi-Dirac and Bose-Einstein distributions.

Modern Physics Inertial frames and galilean invariance, Postulates of special relativity,
Lorentz transformations, Length contraction, Time dilation, Relativistic velocity
addition theorem, Mass energy equivalence, Blackbody radiation, Photoelectric effect,
Compton effect, Bohr's atomic model, X-rays, Wave-particle duality, Uncertainty
principle, The superposition principle, Calculation of expectation values, Schrödinger
equation and its solution for one, Two and three dimensional boxes, Solution of
Schrödinger equation for the one dimensional harmonic oscillator, Reflection and
transmission at a step potential, Pauli exclusion principle, Structure of atomic nucleus,
mass and binding energy, Radioactivity and its applications, Laws of radioactive decay.

Solid State Physics, Devices and Electronics Crystal structure, Bravais lattices and
basis, Miller indices, X-ray diffraction and Bragg's law; Intrinsic and extrinsic
semiconductors, Variation of resistivity with temperature, Fermi level, p-n junction
diode, I-V characteristics, Zener diode and its applications, BJT: characteristics in CB,
CE, CC modes, Single stage amplifier, Two stage R-C coupled amplifiers, Simple
Oscillators: Barkhausen condition, sinusoidal oscillators, OP-AMP and applications:
Inverting and non-inverting amplifier, Boolean algebra: Binary number systems;
conversion from one system to another system; binary addition and subtraction, Logic
Gates AND, OR, NOT, NAND, NOR exclusive OR; Truth tables; combination of gates; de
Morgan's theorem.
IIT JAM PHYSICS
SOLVED PAPER 2021
MM : 100 Time : 3 hrs

1. The examination is of 3 hours duration. There are a total of 60 questions carrying 100 marks. The entire paper is
divided into three sections, A, B and C. All sections are compulsory. Questions in each section are of different
types.
2. Section - A contains a total of 30 Multiple Choice Questions (MCQs). Each MCQ type question has four
choices out of which only one choice is the correct answer. Question Q. 1- Q . 30 belong to this section and
carry a total of 50 marks. Q. 1- Q. 10 carry 1 mark each and Questions Q. 11 - Q. 30 carry 2 marks each.
3. Section - B contains a total of 10 Multiple Select Questions (MSQs). Each MSQ type question is similar to
MCQ but with a difference that there may be one or more than one choice (s) that are correct out of the four
given choices. The candidate gets full credit if he / she selects all the correct answers only and no wrong
answers. Questions Q. 31-Q.40 belong to this section and carry 2 marks each with a total of 20 marks.
4. Section - C contains a total of 20 Numerical Answer Type Questions (NATQs). For these NAT type
questions, the answer is a real number which needs to be entered using the virtual keyboard on the monitor. No
choices will be shown for these type of questions. Questions Q.41-Q.60 belong to this section and carry a total
of 30 marks. Q. 41- Q. 50 carry 1 mark each and Questions Q.51 - Q. 60 carry 2 marks each.
5. In all sections, questions not attempted will result in zero mark. In Section - A (MCQ), wrong answer will. result
in NEGATIVE marks. For all 1 mark questions, 1/3 marks will be deducted for each wrong answer. For all
2 marks questions, 2/3 marks will be deducted for each wrong answer. In Section - B (MSQ), there is NO
NEGATIVE and NO PARTIAL marking provisions. There is NO NEGATIVE marking in section - C (NAT) as well
6. Only Virtual Scientific Calculator is allowed. Charts, graph sheets, tables, cellular phone or other electronic
gadgets are NOT allowed in the examination hall.
7. The Scribble Pad will be provided for rough work.

Section A Multiple Choice Questions (MCQs)


In these type of questions, each question has four choices (a), (b), (c) and (d) out of which only
one option is correct.

Q.1-10 carry one mark each

1. A planet is in a highly eccentric orbit about a star. The distance of its closest approach is
300 times smaller than its farthest distance from the star. If the corresponding speeds are v c
and v f , then v c / v f is
1 1
(a) (b) (c) 300 (d) 300
300 300

2. For the given circuit, VD is the threshold voltage of the diode. The graph that best depicts the
variation of V out with V in is
4 IIT JAM Physics Solved Papers & Practice Sets

1 kΩ

1 kΩ
Vin –
Vout
+

Vout Vout Vout Vout

(a) 0
Vin (b) Vin (c) Vin (d) Vin
VD 0 0 VD 0

3. The moment of inertia of a solid sphere (radius R and mass M) about the axis which is at a
distance of R/2 from the centre is
1 13 3 9
(a) MR 2 (b) MR 2 (c) MR 2 (d) MR 2
2 20 20 10

4. Metallic lithium has bcc crystal structure. Each unit cell is a cube of side a. The number of
atoms per unit volume is
2 4 2 1
(a) 3
(b) 3
(c) 3
(d)
2a a a a3

5. Let M be a 2 × 2 matrix. Its trace is 6 and its determinant has value 8. Its eigenvalues are
(a) 3 and 3 (b) − 2 and − 3
(c) 2 and 6 (d) 2 and 4

6. Arrange the following telescopes, where D is the telescope’s diameter and λ is the wavelength,
in order of decreasing resolving power
1. D = 100 m, λ = 21cm 2. D = 2 m, λ = 500 nm
3. D = 1m, λ = 100 nm 4. D = 2 m, λ = 10 mm
(a) 3, 2, 4, 1 (b) 4, 3, 2, 1 (c) 2, 3, 1, 4 (d) 3, 2, 1, 4

7. The function e cos x is Taylor expanded about x = 0. The co-efficient of x 2 is


(a) e/2 (b) − 1/2 (c) zero (d) − e/2

8. For a semiconductor material, the conventional flat band energy diagram is shown in the
figure.
Conduction band

Valence band
X

The variables Y ,X , respectively, are


(a) distance, energy (b) momentum, energy
(c) energy, momentum (d) energy, distance
Solved Paper 2021 ●
5

9. An experiment with a Michelson interferometer is performed in vacuum using a laser of


wavelength 610 nm. One of the beams of the interferometer passes through a small glass
cavity 1.3 cm long. After the cavity is completely filled with a medium of refractive index n, 472
dark fringes are counted to move past a reference line. Given that, the speed of light is
3 × 10 8 m/s, the value of 77 is
(a) 1.01 (b) 1.10 (c) 1.06 (d) 1.04

10. An object of density ρ is floating is a liquid with 75% of its volume submerged. The density of
the liquid is
4 8 3
(a) ρ (b) ρ (c) ρ (d) 2 ρ
3 5 2

Q. 11- 30 carry two marks each


dy
11. The solution y (x ) of the differential equation y + 3x = 0, y (1) = 0, is described by
dx
(a) a circle (b) a straight line (c) an ellipse (d) a parabola

12. A particle, initially at the origin in an inertial frame S, has a constant velocity v $i. Frame S′ is
rotating about the Z-axis with angular velocity ω (anti-clockwise). The coordinate axes of S′
coincide with those of S at t = 0. The velocity of the particle (v ′x ,v ′y ) in the S′ frame, at
π
t= is

 3 vπ   vπ   vπ 
(a)  , − v (b) ( − v , − v ) (c)  , − v  (d)  − , − v
 2  2   2 

13. A system undergoes a thermodynamic transformation from state S1 to state S 2 via two
different paths 1 and 2. The heat absorbed and work done along path 1 are 50 J and 30 J,
respectively. If the heat absorbed along path 2 is 30 J, the work done along path 2 is
(a) zero (b) 10 J (c) 20 J (d) 30 J

14. In the figure below, point A is the object and point B is the image formed by the lens. Let l 1, l 2
and l 3 , denote the optical path lengths of the three rays 1, 2 and 3, respectively. Identify the
correct statement.

1
2
A B

(a) l 1 = l 3 < l 2 (b) l 1 > l 2 < l 3 (c) l 1 = l 2 = l 3 (d) l 1 = l 3 > l 2

15. The condition for maxima in the interference of two waves


k  k 
i  0 ( 3 x + y ) − ωt  i  0 ( x + y ) − ωt 
 2   2 
Ae and Ae

is given in terms of the wavelength λ and m, an integer by


6 IIT JAM Physics Solved Papers & Practice Sets

(a) ( 3 + 2 ) x + (1 − 2 ) y = 2 mλ (b) ( 3 − 2 ) x − (1 − 2 ) y = mλ
(c) ( 3 − 2 ) x + (1 − 2 ) y = (2 m + 1)λ (d) ( 3 − 2 ) x + (1 − 2 ) y = 2 mλ

16. The radial component of acceleration in plane polar coordinates is given by


2 2
d 2r  dθ  d 2r d 2r  dθ  dr dθ d 2θ
(a) + r  (b) (c) − r  (d) 2 +r 2
dt 2  dt  dt 2 dt 2  dt  dt dt dt

17. For the given circuit, the output Y is


A

(a) 0 (b) A (c) A (d) 1

18. The electric field of an electromagnetic wave has the form E = E 0 cos(ωt − kz )$i. At t = 0, a test
. ck$ , where c is the speed of light. The
particle of charge q is at z = 0 and has velocity v = 05
total instantaneous force on the particle is
qE0 $ qE0 $ $ qE0 $ $
(a) i (b) zero (c) (i − k) (d) ( i + j)
2 2 2

19. A linearly polarised light falls on a quarter wave plate and the emerging light is found to be
elliptically polarised.
The angle between the fast axis of the quarter wave plate and the plane of polarisation of the
incident light, can be
(a) 180º (b) 90º (c) 45º (d) 30º

20. A thin circular disc lying in the xy -plane has a surface mass density σ, given by

  r2
 σ 0 1 − 2  ,if r ≤ R
σ(r ) =   R 
 0 , if r > R

where, r is the distance from its centre. Its moment of inertia about the Z-axis, passing through
its centre is
σ 0R 4 π σ 0R 4
(a) σ 0R 4 (b) 2 πσ 0R 4 (c) (d)
4 6

21. At t = 0, N 0 number of a radioactive nuclei A start decaying into B with a decay constant λ a .
The daughter nuclei B decay into nuclei C with a decay constant λ b . Then, the number of
nuclei B at small time t (to the leading order) is
(a) λ b N0 t (b) ( λ a − λ b )N0 t (c) λ a N0 t (d) ( λ a + λ b )N0 t

22. The total charge contained within the cube (see figure), in which the electric field is given by
E = k ( 4x 2 $i + 3 y $j ), where ε 0 is the permittivity of free space, is
Solved Paper 2021 ●
7

(0 1 0)

(0 0 0)
X
(1 0 0)

(0 0 1)

(a) Zero (b) 7 kε 0 (c) 5kε 0 (d) 3kε 0

23. Three events, E 1(ct = 0, x = 0), E 2 (ct = 0, x = L ) and E 3 (ct = 0, x = − L ) occur, as observed in
an initial frame S. Frame S′ is moving with a speed v along the positive x-direction with respect
to S. In S′, let t ′ 1 ,t ′ 2 , t ′ 3 be the respective times at which E 1, E 2 and E 3 occured. Then,
(a) t ′3 < t ′ 1 < t ′2 (b) t ′3 < t ′ 2 < t ′1 (c) t ′1 = t ′2 = t ′3 (d) t ′2 < t ′1 < t ′3

24. In the Fourier series expansion of two functions f1(t ) = 4t 2 + 3 and f 2 (t ) = 6 t 3 + 7t in the
interval − T / 2 to + T / 2 , the Fourier co-efficients a n and b n (a n and b n are coefficients to
cos(nωt ) and sin(nωt ), respectively) satisfy.
(a) an ≠ 0 and bn ≠ 0 for f1(t ); an = 0 and bn ≠ 0 for f2(t )
(b) an ≠ 0 and bn = 0 for f1(t ); an = 0 and bn ≠ 0 for f2(t )
(c) an = 0 and bn ≠ 0 for f1(t ); an ≠ 0 and bn ≠ 0 for f2(t )
(d) an = 0 and bn ≠ 0 for f1(t ); an ≠ 0 and bn = 0 for f2(t )

25. A semiconductor pn-junction at thermal equilibrium has the space charge density ρ(x ) profile as
shown in the figure.
ρ

x
0
–w w
2 2

p-type n-type

The figure that best depicts the variation of the electric field E with x is (W denotes the width of
the depletion layer)
E E E E

W
–W –W
0 0 2 2 0 2
(a) x (b) x (c) x (d) x
0 W
–W W –W W
2 2 2 2 2
8 IIT JAM Physics Solved Papers & Practice Sets

26. Let (x, y ) denote the coordinates in a rectangular Cartesian coordinate system C. Let (x ′ , y ′ )
denote the coordinates in another coordinate system C ′, defined by x ′ = 2x + 3 y ,
y ′ = − 3x + 4 y . The area element in C ′, is
1
(a) x ′ dx ′ dy ′ (b) 12dx ′ dy ′ (c) dx ′ dy ′ (d) dx ′ dy ′
17

27. The rms velocity of molecules of oxygen gas is given by v at some temperature T. The
molecules of another gas have the same rms velocity at temperature T/16. The second gas is
(a) neon (b) nitrogen (c) helium (d) hydrogen

28. A mass m is connected to a massless spring of spring constant k, which is fixed to a wall.
Another mass 2m, having kinetic energy E, collides collinearly with the mass m completely
inelastically (see figure).

m 2m

The entire set up is placed on a frictionless floor. The maximum compression of the spring is
E 4 E E E
(a) (b) (c) (d)
5k 3 k 3k 7k

29. Four charges are placed very close to each other, as shown. The separation between the two
charges on the Y-axis is a. The separation between the two charges on the X-axis is also a.
The leading order (non-vanishing) form of the electrostatic potential, at point P, at a distance r
from the origin (r > > a ), is
P

+q

a 60°
+q 0 –q

–q
a

1 qa 1 qa
(a) ( 3 − 1) (b) (1 − 3 )
4π ε 0 2 r 2 4π ε 0 r 2
1 2qa 1 qa
(c) (d) ( 5 − 1)
4π ε 0 r 2 4π ε 0 r 2

30. The expression for the magnetic field that induces the electric field
E = k( yz i$ + 3z$j + 4 yk$ ) cos(ωt ) is
k k $
(a) − ( $i + y$j + zk$ )sin(ωt ) (b) − ( − i − y$j + zk$ )sin(ωt )
ω ω
k k
(c) − ( $i − y$j + zk$ )sin(ωt ) (d) − ( $i + y$j − zk$ )sin(ωt )
ω ω
Solved Paper 2021 ●
9

Section B Multiple Select Questions (MSQs)


In these type of questions, each question has four choices (a), (b), (c) and (d) out of which only
one or more than one is/are correct options.

Q. 31-40 carry two marks each

31. A Carnot engine operates between two temperatures, TL = 100 K and TH = 150 K. Each cycle
of the engine lasts for 0.5 s during which the power delivered is 500 J/s. Let QH be the
corresponding heat absorbed by the engine and QL be the heat lost. Identify the correct
statement(s).
QH 2
(a) ≤
QL 3
(b) The change in entropy of the engine and the hot bath in a cycle is 5J/K.
(c) Q H = 750 J
(d) The change in entropy of the engine in 0.5 s is zero.

32. Consider the following differential equation that describes the oscillations of a physical system:
d 2y dy
α 2
+β + γy = 0
dt dt

If α and β are held fixed, and γ is increased, then


(a) the frequency of oscillations increases
(b) the frequency of oscillations decreases
(c) the oscillations decay slower
(d) the oscillations decay faster

33. An isolated ideal gas is kept at a pressure p 1 and volume V1. The gas undergoes free
expansion and attains a pressure p 2 and volume V 2 . Identify the correct statement(s).
 Cp 
γ = 
 CV 
(a) p1V1γ = p2V2γ (b) This is an adiabatic process
(c) p1V1 = p2V2 (d) This is an isobaric process

34. A time dependent conservative force F has the form, F = 3y i$ + f (x, y )$j. Its magnitude at
x = y = 0 is 8. The allowed form(s) of f (x, y ) is/are
2
(a) 3 x + 8 (b) 3 x + 8e − y (c) 2 x + 8 cos y (d) 2 x + 8( y − 1)2

35. A particle of mass m is in an infinite square well potential of length L. It is in a superposed


1 2
state of the first two energy eigenstates, as given by ψ(φ) = ψ n = 1(x ) + ψ n = 2 (x ).
3 3
Identify the correct statement(s). (h is Planck’s constant)
3h2 3h
(a) E = (b) ∆p = (c) ∆x = 0 (d) p = 0
8mL2 2L
10 IIT JAM Physics Solved Papers & Practice Sets

36. A gaseous system, enclosed in an adiabatic container, is in equilibrium at pressure p 1 and


volume V1. Work is done on the system in a quasi-static manner due to which the pressure
and volume change to p 2 and V 2 , respectively, in the final equilibrium state. At every instant,
Cp
the pressure and volume obey the condition pV γ = C, where γ = and C is a constant. If
CV
the work done is zero, then identify the correct statement(s).
(a) p2V2 = ( γ + 1) p1V1 (b) p2V2 = γ p1V1 (c) p2V2 = ( γ − 1) p1V1 (d) p2V2 = p1V1

37. For the given circuit, identify the correct statement(s).


1 kΩ

1 kΩ

Vout
+
1 kΩ
+1V
1 kΩ
I0 RL=1.5k Ω

(a) I0 = 1 mA (b) V0 = 3 V
(c) If RL is doubled, V0 will change to 6 V. (d) If RL is doubled, I0 will change to 0.5 mA.

38. The figure shows the cross-section of a hollow cylindrical tank, 2.2 m in diameter, which is half
filled with water (refractive index of 1.33). The space above the water is filled with a gas of
unknown refractive index. A small laser moves along the bottom surface and aims a light
beam towards the centre (see figure).

Gas

Water
surface

S Laser

When the laser moves a distance of S = 1.09 m or beyond from the lowest point in the water,
no light enters the gas. Identify the correct statement(s) (speed of light is 3 × 10 8 m/s).
(a) The refractive index of the gas is 1.05.
(b) The time taken for the light beam to travel from the laser to the rim of the tank when S < 109
. m
is 8.9 ns.
(c) The critical angle for the water-gas interface is 56.77º.
(d) The time taken for the light beam to travel from the laser to the rim of the tank when S > 109
. m
is 9.7 ns.

39. Identify the correct statement(s) regarding nuclei.


(a) The energy of γ rays due to de-excitation of a nucleus can be of the order of MeV.
(b) The uncertainty in the momentum of a proton in a nucleus is roughly 105 times the uncertainty in
the momentum of the electron in the ground state of hydrogen atom.
Solved Paper 2021 ●
11

(c) The volume of a nucleus grows linearly with the number of nucleons in it.
56
(d) Fe is the most stable nucleus.

40. A beam of light travelling horizontally consists of an unpolarised component with intensity I 0
and a polarised component with intensity I p . The plane of polarisation is oriented at an angle θ
with respect to the vertical. The figure shows the total intensity I total after the light passes
through a polariser as a function of the angle a, that the axis of the polariser makes with
respect to the vertical. Identify the correct statement(s).
Itotal (W/m2)

25

20

15

10

0 50 100 150 200


a(degree)

(a) I0 = 10 W/m 2; Ip = 20 W/m 2 (b) I0 = 17.5 W/m 2


(c) θ = 125° (d) Ip = 5 W/m 2

Section C Numerical Answer Type Questions (NATQs)


This section contains Numerical Answer Type Questions (NATQs) for these NAT questions, the
answer is a real number.

Q.41-50 carry one mark each

41. A particle with positive charge 10 −3 C and mass 0.2 kg is thrown upwards from the ground at
an angle 45º with the horizontal with a speed of 5 m/s. The projectile moves through a
horizontal electric field of 10 V/m, which is in the same direction as the horizontal component
of the initial velocity of the particle. The acceleration due to gravity is 10 m/s 2 . The range is
……… m. (Round off to three decimal places.)

42. A small conducting square loop of side l is placed inside a concentric large conducting square
nµ 0l 2
loop of side L(L >> l). The value of mutual inductance of the system is expressed as .
πL
The value of n is ……… . (Round off to two decimal places.)

43. A particle is moving with a velocity 0.8 c $j (c is the speed of light) in an inertial frame S1.
Frame S 2 is moving with a velocity 0.8 c $i with respect to S1. Let E 1 and E 2 be the respective
energies of the particle in the two frames. Then, E 2 / E 1 is ……… .(Round off to two decimal
places.)
12 IIT JAM Physics Solved Papers & Practice Sets

44. A crystal has monoclinic structure, with lattice parameters, a = 5.14 Å, b = 5.20 Å, c = 5.30 Å
and angle β = 99 °. It undergoes a phase transition to tetragonal structure with lattice
parameters, a = 5.09 Å and c = 5.27 Å. The fractional change in the volume | ∆V / V | of the
crystal due to this transition is ……… . (Round off to two decimal places.)

45. A laser beam shines along a block of transparent material of length 2.5 m. Part of the beam
goes to the detector D1, while the other part travels through the block and then hits the
detector D 2 . The time delay between the arrivals of the two light beams is inferred to be
6.25 ns. The speed of light c = 3 × 10 8 m/s. The refractive index of the block is ……… .
(Round off to two decimal places.)

D1
Laser

D2
46. One of the roots of the equation, z 6 − 3z 4 − 16 = 0 is given by z 1 = 2. The value of the
product of the other five roots is ……… .

47. The following Zener diode voltage regulator circuit is used to obtain 20 V regulated output at
load resistance RL from a 35 V DC power supply. Zener diodes are rated at 5 W and 10 V.
The value of the resistance R is ……… Ω.
R

+
35 V RL

48. Consider N1 number of ideal gas particles enclosed in a volume V1 . If the volume is changed
to V 2 and the number of particles is reduced by half, the mean free path becomes four times of
its initial value. The ratio V1 / V 2 is ……… .(Round off to one decimal place.)

49. At some temperature T, two metals A and B, have Fermi energies ε A and εB , respectively. The
free electron density of A is 64 times that of B. The ratio ε A / εB is ……… .

50. An ideal black-body at temperature T, emits radiation of energy density u. The corresponding
T u
value for a material at temperature is . Its emissivity is ……… . (Round off to three
2 256
decimal places.)

Q. 51 - 60 Carry two marks each.

51. A current I is uniformly distributed across a long straight non-magnetic wire (µ r = 1) or circular
cross-section with radius a. Two points P and Q are at distances a / 3 and 9a, respectively,
from the axis of the wire. The ratio of the magnetic fields at points P and Q is ……… .
Solved Paper 2021 ●
13

52. An RC circuit is connected to two DC power supplies, as shown in the figure.


20 V
– +

4 kΩ

10 µF

6 kΩ

+ –
10 V

With switch S open, the capacitor is fully charged, S is then closed at time t = 0. The voltage
across the capacitor at t = 2.4 ms is ……… V. (Round off to one decimal place).

53. Consider a hemispherical glass lens (refractive index is 1.5) having radius of curvature
R = 12 cm for the curved surface. An incoming ray, parallel to the optical axis, is incident on
the curved surface at a height h = 1cm above the optical axis, as shown in the figure.

d
R

The distance d (from the flat surface of the lens) at which the ray crosses the optical axis is
………… cm. (Round off to two decimal places.)

54. A parallel plate capacitor having plate area of 50 cm 2 and separation of 0.1 mm is completely
filled with a dielectric (dielectric constant K = 10). The capacitor is connected to a 10 kΩ
resistance and an alternating voltage V = 10 sin(100 πt ), as shown in the figure.

10 kΩ

The switch S is initially open and then closed at t = 0. The ratio of the displacement current in
2
the capacitor, to the current in the resistance, at time t = s is ……… . (Round off to three
π
decimal places.)
14 IIT JAM Physics Solved Papers & Practice Sets

55. Twenty non-interacting spin 1/2 particles are trapped in a three-dimensional simple harmonic
oscillator potential of frequency ω. The ground state energy of the system, in units of hω, is
……… .

56. In an X-ray diffraction experiment with Cu crystals having lattice parameter 3.61 Å, X-rays of
wavelength of 0.090 nm are incident on the family of planes {1 1 0}. The highest order
present in the diffraction pattern is …… .

57. A thin film of alcohol is spread over a surface. When light from a tunable source is incident
normally, the intensity of reflected light at the detector is maximum for λ = 640 nm and
minimum for λ = 512 nm. Taking the refractive index of alcohol to be 1.36 for both the given
wavelengths, the minimum thickness of the film would be ……… nm. (Round off to two
decimal places.)

58. A particle A of mass m is moving with a velocity v $i, and collides elastically with a particle B, of
mass 2m. B is initially at rest. After collision, A moves with a velocity v A $j. If v B is the final
speed of B, then v A2 = kv B2 . The value of k is ……… .

59. For the Boolean expression Y = ABC + ABC + ABC + ABC , the number of combinations for
which the output Y = 1is ……… .

60. The wavelength of characteristics K α X-ray photons from Mo (atomic number 42) is ……… Å.
(Speed of light is 3 × 10 8 m/s; Rydberg constant R = 109
. × 10 7 /m) (Round off to one decimal
place)

Answers
1. (c) 2. (d) 3. (b) 4. (c) 5. (d) 6. (d) 7. (d) 8. (c) 9. (a) 10. (a)
11. (c) 12. (d) 13. (b) 14. (c) 15. (d) 16. (c) 17. (c) 18. (a) 19. (d) 20. (d)
21. (c) 22. (b) 23. (d) 24. (b) 25. (b) 26. (c) 27. (d) 28. (b) 29. (a) 30. (d)
31. (c, d) 32. (a) 33. (a, b, c) 34. (a, b) 35. (a, b, d) 36. (d) 37. (a, b, c) 38. (b, c, d) 39. (a,b,c,d) 40. (a)
41. (2.511 to 2.513) 42. (2.81 to 2.83) 43. (1.65 to 1.68) 44. (0.02) 45. (1.74 to 1.76) 46. (–8)
47. (30) 48. (0.5) 49. (16) 50. (0.063) 51. (3) 52. (18.7 to 18.9)
53. (16) 54. (0.006 to 0.008) 55. (1.5) 56. (5) 57. (470.57 to 470.59)
58. (1) 59. (4) 60. (0.6 to 0.8)
Answers with Solutions
1. (c) According to Kepler’s second law of planetary motion, 4. (c) Let’s focus on a bcc structure shown below
orbital speed is inversely proportional to the distance to
the central body.
vc df
So, =
vf dc
df Number of atoms in a unit cell
Given, = 300
dc 1 
=  × 8 + 1 = 2
vc 8 
So, = 300
vf 2
Hence, number of atoms per unit volume will be =
Hence, option (c) is correct. a3
Hence, option (c) is correct.
2. (d) Given, circuit is
1kΩ 5. (d) Since, matrix is of 2 × 2 order, its eigen values will be
λ 1 and λ 2 .
Given, λ1 + λ 2 = 6 = Trace …(i)
1kΩ – λ 1 λ 2 = 8 = Determinant
Vin Vout
+ ⇒ λ1 − λ 2 = ( λ1 + λ 2 )2 − 4 λ1 λ 2
= 36 − 4 ( 8) = 2
Clearly, diodes in the circuit are shorted by 1 kΩ feedback
or λ1 − λ 2 = 2 …(ii)
resistor, hence it will act as simple inverting amplifier.
R Solving Eqs. (i) and (ii), we get
⇒ Vout = − f × Vin
Ri λ1 = 4 and λ 2 = 2
Hence, option (d) is correct.
or Vout = − Vin
Hence, graph (d) represents the correct variation. 6. (d) Resolving power of a telescope is given by
D
RP =
3. (b) Moment of inertia about the central axis, 1.22λ
D
I I' ⇒ RP ∝
λ
In given combinations,
D 100 m 104
  = = ≈ 5 × 102
O  λ  1 21 cm 21
R
2 D 2m 2 × 107
  = =
 λ  2 500 nm 5
D 1m
  = = 107
 λ  3 100 nm
2
I = MR 2 D 2m
5   = = 2 × 102
 λ  4 10 mm
R 
Moment of inertia about the axis at   distance can be
 2 Clearly, (RP) 3 > (RP) 2 > (RP)1 > (RP) 4
determined by parallel axis theorem. Hence, option (d) is correct.
I ′ = I + md 2
2
7. (d) Given, y = e cos x
2 R 
⇒ I ′= MR 2 + M   So,
dy
= ( − sin x ) e cos x
5  2 dx
 2 1 d 2y
=  +  MR 2 and = − cosx e cos x − (sin x ) e cos x ( − sin x )
5 4 dx 2
13 d 2y
= MR 2 = − cos x e cos x + sin2 x e cos x
20 dx 2
Hence, option (b) is correct. So, while expanding across x = 0, coefficient of x 2 can be
found from 3rd term of the Taylor series
16 IIT JAM Physics Solved Papers & Practice Sets

f ′ (a ) f ′′(a ) By integration,
f (a ) + 1
(x − a ) + (x − a ) 2
L L2
Here, a =0
∫ ydy = − ∫ 3xdx
f ′′( 0) 1 d 2y y 2 3x 2
and term of x 2 = of ⇒ = +C
L 2
2 dx 2 2 −2
x =0

1 Given, y(1) = 0
So, coefficient of x 2 = | − cos x e cos x + sin2 x e cos x |x = 0
2 ⇒ y = 0 at x = 1
1 3 3
= [ − cos 0 e cos 0 + sin2 0 e cos 0] ⇒ C − = 0 or C =
2 2 2
1 e
= [ − e + 0] = − or 3x 2 + y 2 = 3
2 2
Hence, option (d) is correct. x2 y2
or + =1
1 3
8. (c) Energy-band diagrams are plotted between energy and
Which of the equation of an ellipse.
wave number (k ).
Hence, option (c) is correct.
Wave number k is corresponding here to the momentum
( p = hk ). 12. (d) Given,
Y
Clearly, energy band diagrams are plotted between Y′
energy (at Y-axis) and momentum (at X-axis).
Hence, option (c) is correct.
9. (a) Given, λ = 610 nm = 610 × 10− 9 m θ X′

t = 1.3 cm = 1.3 × 10− 2 m θ X


µ =n
and N = 472
Path difference introduce by the presence of medium Z =Z ′
∆ = (µ − 1)t vx = v
This path difference caused shift of 472 fringes hence and vy = 0
(µ − 1)t = N λ also x = vt
⇒ (n − 1)t = N λ From conversion of axis
610 × 10− 9 × 472 x ′ = x cos ωt + y sin ωt ...(i)
or (n − 1) =
1.3 × 10− 2 and y ′ = y cos ωt − x sin ωt ...(ii)
⇒ n − 1 = 2.214 × 10− 2 Clearly,
or n = 1 + 2.214 × 10− 2 dx ′
v x′ = = (v x cos ωt − xω sinωt )
n = 1.02 dt
No option is correct in the given options. + (v y sin ωt + yω cos ωt )
π
Hence, option (a) is most close option. at t = (or cos ωt = 0 ; sin(ωt ) = 1)

10. (a) Weight of floating body = Buyont force of water We get v x′ = ( 0 − xω) + ( 0 + 0)
3 
mg =  V  ρwg or v x′ = − xω or − v t ω
4 
π
4 m Again, at t =
⇒ ρw = × 2ω
3 V π π
4 v x′ = − v × ω= −v ...(iii)
ρw = × ρ 2ω 2
3
From Eq. (ii)
Hence, option (a) is correct. dy ′
v y′ = = (v y cos ωt − yω sin ωt )
11. (c) Given, equation y dy + 3x = 0, y(1) = 0 dt
dx − (v x sin ωt + xω cos ωt )
dy 3x π
So, =− Again, at t = , we get
dx y 2ω
⇒ ydy = − 3x dx v y′ = ( 0 − yω) − (v x + 0)
Solved Paper 2021 ● 17

= − yω − v x 17. (c) Given, circuit is


= − vyt − vx A

or v = −v
y ...(iv)
Y
Hence, option (d) is correct. P1 P2
13. (b) Given, for path 1,
∆W = 30 J
dQ = 50 J Given gate is X-NOR gate. One input of first gate is
We know that, dU = dQ − ∆W = 50 − 30 grounded and another input is a common input for all
⇒ dU = 20 J three gates.
Internal energy dU is a state function which does not A B (GND) P1 P2 = P1 ⊕ A Y = P2 ⊕ A
depend upon the path.
1 0 0 0 0
Hence, for path 2,
0 0 1 0 1
dU = 20 J
dQ = 30 J Clearly, Y = A
So, ∆W = dQ − dU = 30 − 20 Hence, option (c) is correct.
⇒ ∆W = 10 J 18. (a) Given, electromagnetic field is
Hence, option (b) is correct.
E = E 0 cos(ωt − kz )$i
14. (c) Since, optical paths of all rays passing through same lens Corresponding magnetic field can be found as
are always same when these rays meet at same point.
Hence, it should be same for all three given rays. ∂B
∇× E = −
∂t
Clearly, l1 = l 2 = l 3
$i $j k$
Hence, option (c) is correct.
∂ ∂ ∂
15. (d) Given, waves are Here, ∇×E=
∂x ∂y ∂z
i  0 ( 3 x + y ) − ωt 
k
E 0 cos (ωt − kz ) 0 0
 2 
y1 = A e
= $i( 0) − $j( −KE 0 sin(ωt − kz )) + k$ ( 0)
i  ( x + y ) − ωt 
k0
 
and y2 = A e 2
⇒ ∇ × E = + KE 0 sin(ωt − kz )$j
Comparing both with standard wave equation,
∂B
y = Aei (k .r − ωt ) , we get Again = ∇ × E = − KE 0 sin(ωt − kz )$j
∂t
 3 1   3 − 2
∆ x =  −  x =   x or B = − KE 0 $j ∫ sin(ωt − kz )dt
 2 2  2 
KE 0
1 1  1− 2 B=+ cos(ωt − kz )$j
and ∆y =  −  y =   y ω
2 2  2 
Now, force on the test charge,
Clearly, total path difference,
F = q( v × B)
∆ = ∆ x + ∆ y = mλ (for maxima)
1 KE 0 
 3 − 2 1− 2 = q  ck$ × cos (ωt − kz )$j 
⇒   x +   y = mλ 2 ω 
 2   2 
qc KE 0
= cos(ωt − kz ) ( k$ × $j )
⇒ ( 3 − 2 )x + (1 − 2 )y = 2mλ 2ω
Hence, option (d) is correct. qE 0 ω
F= cos(ωt − kz )$i [Qc = ]
16. (c) Radial acceleration in plane polar coordinates is given by 2 K

a = r&& + r (θ)2 At t = 0 and z = 0
r
2 ⇒ cos(ωt − kz ) = cos 0 = 1
d 2r  dθ 
= − r  qE 0 $
dt 2  dt  Hence, F= i
2
Note It is a direct formula based question.
Hence, option (a) is correct.
Hence, option (c) is correct.
18 IIT JAM Physics Solved Papers & Practice Sets

− λ bt
19. (d) We know that, relations between input angle and outputs and Nc = NB , 0 − NB , 0 e ...(iv)
of any quarter wave plate are as follows Since, we have asked only for small time
−λ
Wave type/Input Angles Output hence e at ≈ 1 − λ at

Linearly polarised, 45° Right circular or NB = N 0 − N 0(1 − λ a t )


⇒ NB = N 0 λ a t
Linearly polarised, − 45° Left circular
Hence, option (c) is correct.
Right circular Linear, θ = 45 ° 22. (b) Given, E = k( 4x 2$i + 3 y$j)
Left circular Linear, θ = 45 ° and cube has unit dimensions in each direction.
We know that,
θ ≠ 45 ° Elliptical q
∫ E ⋅ d A = ε0
θ = 0°, 180° or 90° Unchanged

Clearly, from the circled part of table only option (d) will
Here, ∫ E ⋅ dA = E x ⋅ Ax + E y ⋅ Ay + E z ⋅ Az

give us elliptical polarized light as output. = k 4x 2 ⋅ 1 + k 3 y ⋅ 1 + 0


20. (d) Given, density function, ⇒ E ⋅ dA = 4kx 2 + 3ky
  r2 
σ 1 − 2  , if r ≤ R
⇒ ∫ E⋅ dA = 4k + 3k = 7k
σ(r ) =  0  R  q
 0 , if r > R Now, = 7k ⇒ q = 7kε0
 ε0
Hence, option (b) is correct.
23. (d) Since, E1 and E 3 are moving away from S, their effective
d3
length in S ′ will decrease and time dilation will increase.
Since, length of E 3 is move than E1 for moving frame,
hence
t1′ < t 3′ ...(i)
Moment of inertia of this disc about central axis is
R
E 2 is moving closed to S frame, hence Its length will
I = ∫ dm r 2 increase or time dilation will decrease.
0
Obviously (as lengths are same), its dilation will be
Here, dm = σ × ds = σ (r ) × 2 πr dr minimum.
R
 r2  Clearly t 2′ is minimum
So, I = ∫ σ 0  1 − 2  r 2 × 2 πr dr
0  R  So, t 2′ < t1′ < t 3′ ...(ii)
'
σ 0 × 2π 5 S S
R R
= σ 0 × 2 π ∫ r 3 dr − ∫ r dr
0
R2 0
E1 v EQ
(x=0) (x=1)
R  2 π R 
4 6
= σ 0 × 2π   − σ0 × R 4  6 
 4   
R 4 R 4 
= σ 0 × 2π −
 4 6  E1
σ × 2 πR 4 σ 0 πR 4
= 0 =
12 6
Hence, option (d) is correct.
Hence, option (d) is correct.
Note Since, we are given three points only, we have
21. (c) As nuclei A is decaying into B. considered their x-position as their length for easy,
Number of nuclei of A at any instant concept-based solution of the problem.
N A = N 0 e − λ at ...(i)
24. (b) We know that for even functions
Clearly number of nuclei of B will be a n ≠ 0 and bn = 0 ...(i)
NB = N 0 − N A = N 0 − N 0e − λ at ...(ii) and for odd functions
Again nuclei of B are decaying into C, hence a n = 0 and bn ≠ 0 ...(ii)
− λ bt
NB = NB , 0e ...(iii) Given functions,
f1(t ) = 4t 2 + 3
Solved Paper 2021 ● 19

is an even function, hence it will follow Eq. (i) also Since, (v rms )1 = (v rms )2
Also, f 2(t ) = 6t 3 + 7t T1 T2
Hence, =
is an odd function, hence it will follow Eq. (ii). M1 M2
Hence, option (b) is correct. T2
or M2 = × M1
25. (b) From the given graph between ρ and x. It is clear that T1

ρ ∝x T
Given, T1 =T andT2 =
16
or ρ = Kx (let)
M1 = 16 (for oxygen)
P
1
So, M2 = × 16 = 1
–w/2 16
O –w/2 Hence, it must be hydrogen.
p-type n-type 28. (b) Given, spring constant =k
ρ Collising type = inelastic
We know that, ∇⋅ E = (Gauss theorem)
ρ0 Since, 2m mass has initial energy E. Let its velocity before
collision is v1 and velocity of the system after collision in
1
∈0 ∫
So, E= ρdx v 2.
1
1 So, E = ( 2m ) v12
∈0 ∫
= Kx dx 2
E
⇒ v1 =
2
...(i)
K 2
E= x +c m
∈0 Now, using linear momentum conservation law,
Clearly, only option (b) can be correct option as all other 0 + m1v1 = (m1 + m2 ) v 2
options are linear relations between E and x. ⇒ 2mv1 = 3mv 2
26. (c) Given, that coordinates in a rectangular Cartesian 2
⇒ v 2 = v1
coordinate system C is (x, y ) and that for system C′ is 3
(x ′ , y ′ ) 2 E
= ...(ii)
Area element is related of 3 m
dA′ = J dxdy Now, since energy of the system should be conserved,
or dx ′dy ′ = J dxdy hence
∂x ′ ∂dy ′ 1 2 1
kx = (m1 + m2 ) v 22
Here, Jacobian matrix, J = ∂x ∂x 2 2
∂x ′ ∂y ′
1 2 1 4 E 
∂y ∂dy ⇒ kx = × 3m ×  
2 2 9 m
Given that, x ′ = 2x + 3 y and y ′ = − 3x + 4 y 4 E
∂x ′ ∂y ′ ⇒ x2 = ×
= 2, =−3 3 k
∂x ∂x
4E
∂x ′ ∂y ′ or x=
= 3, =4 3 k
∂y ∂y
Hence, option (b) is correct.
2 −3
∴ J= 29. (a) Given, diagram is
3 4
= 2 × 4 − ( − 3 )( 3 ) +q
P
= 8 + 9 = 17
∴ ∂x ′ ∂y ′ = 17 ∂x∂y a
60º
1 +q O –q
or ∂ x∂ y = ∂x ′ ∂y ′
17
–q
27. (d) We know that, r.m.s velocity of gases
3RT a
v rms =
M
20 IIT JAM Physics Solved Papers & Practice Sets

Consider it as two mutually perpendicular dipoles as 31. (c, d) Given, TL = 100 K


shown below
+q
TH = 150 K
p Since it is Carnot cycle, entropy of the system must be

θ
º=
p zero.

30
p Hence, option (d) is correct and (b) is incorrect.
θ
60º Q T
Now, L = L (Heat lost will be in the form of power
+q p –q QH TH
a –q delivered)
(i) (ii) 500 100
⇒ =
QH 150
Electric potential due to electric dipole at distance
r (r >> a ) is given by ⇒ QH = 500 × 1.5 = 750 J
1 p ⋅ $r 1 p cos θ Hence, option (c) is correct.
V = =
4 πε0 r 2 4 πε0 r2 QL 100 2
Again = =
In Fig (i), θ = 180° − 60° = 120° QH 150 3

Hence, Hence, option (a) is incorrect.


1 p 1 qa So, option (c) and (d) are correct.
V1 = cos 120° = −
4 πε0 r 2 4 πε0 2r 2
32. (a) Given, equation
In Fig (ii), θ = 30°, hence d 2y dy
α +β + γy = 0
1 p dt 2
V2 = cos 30° dt
4 πε0 r 2 β γ
or y ′′ + y ′ + y = 0
1 qa 3 α α
= ×
4 πε0 r 2 2 R 1
Compare it with Q ′′ + Q ′ + Q=0
L LC
Clearly, total potential,
β γ
1 qa 3 We get, is damping factor, while is frequency
V =V1 + V2 = × α α
4 πε0 r 2 2
component.
1 qa 1
− × γ
4 πε0 r 2 2 So, ω=
α
1 qa
= ( 3 − 1) Since, only γ is increased.
4 πε0 2r 2
So, frequency will increase.
Hence, option (a) is correct.
Hence, option (a) is correct and (b) is incorrect.
30. (d) Given E = k( yz$i + 3 z$j + 4 yk$ ) cos(ωt ) Since, decay factor does not depend upon γ. It will remain
We know that (from Maxwell’s equation) unaffected.
∂B Clearly, (d) and (c) are incorrect.
∇×E=− ...(i)
∂t Hence, option (a) is correct.
$i $j k$ 33. (a, b, c) Since it is free expansion. It has to be adiabatic.
∂ ∂ ∂
Here, ∇× E=K cos ωt So, option (b) is correct.
∂x ∂y ∂z
yz 3 z 4 y Now, for adiabatic process,
pV γ = constant
= [[ 4 − 3] $i − [ 0 − y] $j + [ 0 − z] k$ ] k cos ωt γ γ
⇒ 1 1 = p2V2
pV
⇒ ∇ × E = ( $i + y$j − zk$ ) k cos ωt ...(ii)
So, option (a) is also correct.
So, B = − ∫ ( ∇ × E )∂t As per question, It is isolated ideal gas, so
= − ∫ ( $i + y$j − zk$ )k cos ωt dt 1 1 = p2V2 (Isolated = constant temperature)
pV
So, option (c) is also correct.
sin ωt
= − k( $i + y$j − zk$ ) Option (d) is clearly incorrect.
ω
k $ Hence, (a), (b) and (c) are correct.
⇒ $
B = − ( i + y j − zk ) sin ωt
$
ω 34. (a, b) Given function, F = 3y$i + f (x, y )$j
Hence, option (d) is correct. Also given, its magnitude is 8 at x = 0, y = 0.
Solved Paper 2021 ● 21

All four options satisfy the given condition. 36. (d) Given process is adiabatic.
Additionally, F is conservative, Hence, ∇ × F = 0. Work done in adiabatic process is
Let’s check ∇ × F for all four possible options. pV − p2V2
W = 1 1
Option I Option II 1− γ
$i $j k$ $i $j k$ Cp
Here, γ=
∂ ∂ ∂ ∂ ∂ ∂ CV
∇ × F=
∂x ∂y ∂z ∂x ∂y ∂z Clearly, W = 0 (given)
3 y 3x + 8 0 3y 3x + 8e − y
2
0 pV − p V
Hence, 1 1 2 2
=0
= $i | 0 | − $j | 0 | + k$ | 3 − 3 | = $i | 0 | − $j | 0 | + k$ | 3 − 3 | 1− γ
=0 =0 ⇒ 1 1 = p2V2
pV
Option III Option IV Clearly, except (d) all other options are wrong as all terms
$i $j $i $j are linear multiple of γ of above equation.
k$ k$
∂ ∂ ∂ ∂ ∂ ∂ Hence, only option (d) is correct.
∂x ∂y ∂z ∂x ∂y ∂z 37. (a, b, c) Given circuit has both inverting and non-inverting
3y 2x + 8 cos y 0 3y 2x + 8( y − 1)2 0 feedbacks
= $i | 0 | − $j | 0 | + k$ | 2 − 3 | = $i | 0 | + $j | 0 | + k$ | 2 − 3 | Hence, final output
= −1 = −1  R   R 
Vout =  1 + f  Vin −  − f  Vin
Hence, options (a) and (b) are correct.  Ri  NI  Ri  I
35. (a, b, d) Expectation value of energy,  1  1
=  1 +  1 +   1= 3 V
< E > = ( Coefficient of φn = 1(x ))2 E1 +  1  1

( Coefficient of φn = 2(x ))2 E 2 Hence, option (b) is correct.


2 Now, in non-inverting circuit using voltage dividing rule
 1   2 E 2
=  E1 +   E2 = 1 + E2
 (RF )Vout 1
 3  3 3 3 VL = = × 3 = 1.5 V
(Rf + Ri ) 2
E1 2
= + ( 4E1 ) (QE n = n 2E1 ) So, current through load resistor,
3 3
= 3E1 VL 1.5
IL = = = 1 mA
RL 1.5 × 103
 π 2h2   π 2h2 
= 3  QE1 = 
2
 2mL   2mL2  Hence, option (a) is also correct.
We know that, doubling RL will double Vout .
3h 2  h 
or <E> = Q h =  So, option (c) is correct and (d) is incorrect.
8mL2  2π 
Hence, (a), (b) and (c) are correct.
< p2 >
As we know, < E > =
2m 38. (b, c, d) When the laser moves a distance at s = 1.09 m or
or < p > = 2m < E >
2 beyond from the lowest point in the water, no light enters
 3h 2  3h 2 the gas, i.e. at this internal reflection will be happening.
= 2m  2
 = 2 i.e. (θ ≥ θc )
 8mL  4L

Uncertainty in momentum, ∆p = < p 2 > − < p > 2


Water
3h 2 surface
= −0
4L2
1.33 θc
3h
∆p =
2L
h Laser
From ∆x ⋅ ∆p ≥ S
2
So, ∆x ≠ 0 When the angle of incidence is equal to the critical angle it
means no light is travelling through other medium.
Hence, (a), (b), (d) are the correct options.
22 IIT JAM Physics Solved Papers & Practice Sets

If s<1.09 m, i.e. light will travelled through other medium as 40. (a) (i) We know that, when an unpolarised light passes
TIR will not place through a polariser at any angle, it gets polarised and
t = t medium 1 + t medium 2 its intensity remains half.
d d R R R (ii) We also know that, when a polarised light passes
= + = + = (µ 1 + µ 2 ) through a polariser, its intensity gets changed
V1 V2 C C C
µ1 µ2 depending on angle between polarised light and
polariser.
Arc S 1.09
θC = = = rad Considering both facts,
Radius R 1.1
I
1.09 180° Intensity contribution by unpolarised light = 0
or θC = × = 56.77° 2
1.1 3.14
Intensity contribution by polarised light = − I p to + I p
Using Snell’s law, µ 1 sin i = µ 2 sin r
(sinusoidal curve)
1.33 sin 56.77° = µ 2 × sin 90°
So, taking any peak of the curve
⇒ µ 2 = 1.11
I0
1.1 + I p = 25
∴ t = [1.33 + 1.11] 2
3 × 108
Clearly, only option (a) matches with the required
t = 8.9 ns condition
If s > 1.09 m i.e. TIR will be happening I 0 = 10 W/m 2
2R
∴Time taken, t = × µ1 ⇒
I0
= 5 W/m 2
C 2
and I p = 20 W/m 2
I 
⇒  0 + Ip  = 5 + 20 = 25 W/m 2 which matches with
2  max
R R the peak in the curve.
41. (2.511 to 2.513) According to given question,
q = 10− 3 C, v = 5 m/s
2 × 1.11 × 1.33 m = 0.2 kg, Angle of projection, θ = 45 °
=
3 × 108
E = 10 V/m (horizontal)
t = 9.8 ns ≈ 9.7 ns
and g = 10 m/s 2
So, options (b, c, d) are correct.
Y E=10 V/m
39. (a, b, c, d) Since energy of γ-radiations in nuclear reactions is
found in the range of MeV, so option (a) is correct. 5 m/s
Actul path in presence
( ∆p )proton = (m∆V )proton of electric filed
5 sin 45°
( ∆p )electron = (m∆V )electron
45º
X
( ∆p )Proton m ( ∆V )proton 5 cos 45°
⇒ = proton ×
( ∆p )electron melectron ( ∆V )electron 2uy
Time taken by charge particle to reach ground,T =
− 27 g
mproton 1.67 × 10
Here, = ≈ 105 2 × 5 sin 45 ° 1
melectron 9.1 × 10− 31 = = s
10 2
Hence, option (b) is also correct.
1 2
Volume of nucleus ∴Range, R = ux t + a xt
2
4
V = πR 3
3 1  qE  2  F qE 
or R = uxT +  T Qa x = = 
1 2m  m m
4
= πR 02 A (QR = R 0 A )
3
2
3 1 1  10− 3 × 10   1 
= 5 cos 45 ° × + ×  × 
⇒ V ∝A 2 2  0.2   2 
∴Option (c) is also correct. = 2.5 + 0.0125
Fe is most stable nucleus, so option (d) is also correct.
R = 2.5125 m
Hence, all options are correct.
Solved Paper 2021 ● 23

42. (2.81 to 2.83) v ′ = v ′2x + v ′2y + v ′2z


Let i current be flowing through outsider square loop as = ( − 0.8c )2 + (0.48c )2 + ( 0)2
shown
v ′ = 0.9329 c
v2
1−
E 2 γ 2m0c 2 γ 2 c2
i L = = =
E1 γ 1m0c 2 γ1 v ′2
1− 2
c
µ 0i 1 − (0.8)2 0.6
As, Bwire = (sinα + sinβ ) = =
4 πd 1 − (0.9329)2 0.36
Magnetic field due to loop (1), E2
= 1.666
µ 0i E1
B1 = 4 × [sin 45 ° + sin 45 °]
L
4 π  44. (0.02) For monoclinic structure, a = 5.14 Å, b = 5.20 Å,
 2
c = 5.30 Å
2 2 µ 0i
B1 = α = 90°, β = 99°, γ = 90°
πL
Since, given that L > l therefore the magnetic field of loop ∴Volume,V = abc sin β
(1) at the loop (2) can be assumed almost same. = (5.14 Å) × (5.20 Å) × (5.30Å) sin 99°
2 2 µ 0i = 139.94 × 10− 30 m 3
Flux through loop (2), φ2 = B ⋅ A = × l2
πL For tetragonal structure, a = 5.09 Å, c = 5.27 Å
φ ∴Volume,V ′ = a 2 c
As we know that, mutual inductance, M = 2
i = (5.09 Å)2 × (5.27 Å)
2 2 µ 0i = 136.535 × 10− 30 m 3
× l2
π L 2 2 µ0 2 ∆V V ′ −V
∴ M = = i Fractional change in the volume, =
i π L V V
On comparing with given expression, we get
136.535 × 10− 30 − 139.914 × 10− 30
n = 2 2~− 2.82 =
139.914 × 10− 30
43. (1.65 to 1.68) = 0.02415
In frame S1 , = 0.02 (on rounding off )

E2 =γ1m0 c2 45. (1.74 to 1.76)


x 2.5
Velocity in air c = ⇒t = s
t 3 × 108
vx =0 x
vy =0.8 c Velocity in block v =
vz =0 t′

In frame S 2 , and t ′ − t = 6.25 ns


2.5
So, t ′ = t + 6.25 ns = + 6.25 × 10− 9
3 × 108
E2 =γ2m0 c2
4375
So, t′ = ns
3
vx,' vy,' vz,' 2.5 × 3
Now, v= m/s
v −v 0 − 0.8c − 0.8c 43.75 × 10− 9
v x′ = x = = = − 0.8c Hence, refractive index of block,
vxv 0 ×0.8c 1− 0
1− 2 1−
c c 2 c 3 × 108 × 43.75 × 10− 9
n= =
v 2.5 × 3
v2 ( 0.8c )2
vy 1 −0.8c 1 − 43.75
v y′ =
2
c = c2 = = 1.75
vxv 0 − 0.8c 25
1− 2 1−
v c2 46. (–8) Given equation is
= 0.8 1 − ( 0.8) 2 = 0.48c z 6 − 3 z 4 − 16 = 0
v z′ = 0 On root is given by z1 = 2
24 IIT JAM Physics Solved Papers & Practice Sets

We know that, product of roots should be λ1 = λ


− 16 λ 2 = 4λ
z1 z 2 z 3 z 4 z 5 z 6 = = − 16
1 1 V
We know that, mean free path, λ =
− 16 16 2 πd N
2
So, z 2z 3 z 4 z 5z 6 = =− =−8
z1 2 λ1 V1 N 2
⇒ = ×
Note For an equation λ 2 V2 N1
ax 6 + bx 5 + cx 4 + dx 3 + ex 2 + fx + g = 0 V1 λ N λ N
⇒ = 1 × 1 = ×
b V2 λ 2 N 2 4 λ N / 2
Sum of roots, Σxi = x1 + x 2 + x 3 + x 4 + x 5 + x 6 = −
a V1 2
⇒ = = 0.5
Also, Σ xi x j =
c V2 4
i≠j a
49. (16) Free electron density,
d
Σ xi x j xk = − 3
i ≠ j ≠k a N 8 π  2mEF  2
=  
e V 3  h2 
Σ xi x j xk xl =
i ≠ j ≠k ≠l a N 3/2
or ∝ EF
f V
Σ xi x j xk xl x m = −
i ≠ j ≠k ≠l ≠ m a N 
  3

and product of roots xi x j xk xl x m =


g  V  A  E A  2 64
or =  = (given)
a N   EB  1
 
47. (30) Given circuit is  V B
R 2
2
E A  64  3  4  16
So, =  =  =
EB  1   1 1
+
35 V RL ∴ E A : EB = 16 : 1or 16

50. (0.063) As we know that, energy density, u = eσT 4
For perfectly black body e = 1
Rating of diodes 5W and 10V, ∴ u = σT 4 ...(i)
4
So, current and resistance of diodes u T 
In case (ii), = eσ  
P 5 256 2 
I = = = 0.5 A
V 10 u
or = eσT 4 ...(ii)
V 10 16
and R = = = 20Ω
I 0.5 From Eqs. (i) and (ii), we get
So, in conducting case, we can have equivalent circuit 1
σ= = 0.0625
Now, Vout = 20 V 16
R On rounding off, we get σ = 0.063

20 Ω 51. (3)
current
35 V Vout As we know, current density, J =
cross - section area
20 Ω I
=
πa 2
a
So, VR = 35 − 20 = 15 V Magnetic field at distance from centre,
3
and current through R
IR = 0.5Ω
V 15
So, R = = = 30 Ω
I 0.5 P
a/3
48. (0.5)
Given, N1 = N
N
N2 =
2
Solved Paper 2021 ● 25

From Ampere’s circuital law 10 + 20


i= = 3 mA
( 4 + 6) × 103
∫B⋅dl = µ I 0 enclosed

VC ( ∞ ) = 20 − ( 3 × 10−3 ) (4 × 103 )
∫ B dl cos θ = µ 0 J (area) (θ = 0°)
VC ( ∞ ) = 8 V
2
I a  Time constant replace battery by its internal resistance
B∫ dl = µ 0 ⋅ ⋅ π 
πa 2  3 (i.e R = 0)
a µ 0I µI 10 µF
B × 2π = or BP = 0
3 9 6 πa 4 kΩ
Magnetic field at distance 9a from centre
– +
10 µF
6 kΩ
4 k×6 k=2.4 kΩ
Q 4 k×6 k
9a

∴Time constant τ = R eq C = ( 2.4 k) (10.4) = 24 ms


Voltage across capacitor at any time t is given by
t

VC (t ) = VC ( ∞ ) + [VC ( 0− 1 ) − VC ( ∞ )]e τ
Again from Ampere’s circuital law,
t

∫ B dl cos θ = µ I 0 enclosed VC (t ) = 8 + [ 20 − 8] e τ

B × 2 π( 9a ) = µ 0I At t = 2.4 ms
2.4 m
µ 0I −
or BQ = VC (t ) = 8 + 12e 24 m
18 πa
µ 0I = 8 + 12e − 0.1
12
= πa = 3
BP 6 = 8 + 0.1
Ratio,
BQ µ 0I e
18 πa VC (t ) = 18.85 V
52. (18.7 to 18.9) 53. (16)
When switch is open (i.e. t < 0)
20 V
– + n
i=0
d
4 kΩ R

– +
10 µF
For first surface, µ 1 = 1, µ 2 = 1.5, u = − ∞ and R = + 12
At steady state i = 0 cm
∴ VC ( 0) = 20 V µ 2 µ1 µ 2 − µ1
Using, − =
− +
As we know VC ( 0 ) = VC ( 0 ) = 20 V V u R
When switch is closed and t = ∞, capacitor will behave as 1.5 1 1.5 − 1
− =
open circuit. V −∞ 12
20 V ⇒ V = + 36 cm
– +
This image will behave as the object for plane surface

i 4 kΩ µ 1 = 1.5, µ 2 = 1, u′ = 36 − 12 = 24 cm

– + i and R =∞
µ1 µ 2
10 µF Using, =
v ′ u′
6 kΩ
1 1.5
=
v ′ 24
i
10 V ⇒ v ′ = 16 cm
26 IIT JAM Physics Solved Papers & Practice Sets

54. (0.006 to 0.008) So, not more than two particles can be filled in one state
εA for energy in any state
Capacitance of parallel plate capacitor, C = 0 xK
d  3
E =  nx + ny + nz +  hω
Given that, A = 50 cm = 50 × 10
2 −4
m 2  2
d = 0.1 mm = 10− 4 m For ground state (0, 0, 0)
K = 10 3
E 0 = hω = 1.5 hω
8.85 × 10− 12 × 50 × 10− 4 2
C= × 10
10− 4 56. (5) Given, a = 3.61Å
C = 44.25 × 10− 10 F λ = 0.090 nm = 9 × 10− 11 m
After closing the switch, Family of planes {h k l } = {11 0}
C
Using Brag’s law,
nλ = 2d sin θ
10 kΩ For nmax → sin θ = 1
⇒ nmax λ = 2d
2d
or nmax =
λ
10 sin (100 πt) a 3.62
Here, d = = Å
h2 + k 2 + l 2 2
Displacement current,
3.61 × 1.41 × 10− 10
dQ d So, nmax =
id = = (CV ) 9 × 10− 11
dt dt
CdV Cd ⇒ nmax = 5.65
= = (10 sin 100 πt )
dt dt Clearly n = 5 is the heighest order of diffraction pattern.
= 44.25 × 10− 10 × 10 × 100 π × cos 100 πt 57. (470.57 to 470.59)
id = 44.25 × π × 10− 7 cos (100 πt ) With respect to the incident wave, the wave that reflects
2  2 from the top surface of the alcohol has a phase change of
At t = s, id = 44.25 × π × 10− 7 cos  100 π × 
π  π φ1 = π
id = 44.25 × π × 10 −7
cos ( 200) With respect to the incident wave, the wave that reflects
Current through capacitor, from the glass at the bottom surface of the alcohol has a
V 10 sin(100 πt ) phase change due to the additional path length and a
iR = = = 10− 3 sin (100πt) phase change of π on reflection.
R 10 × 103
 2t 
2  2 φ2 =   2π + π
At t = s, iR = 10− 3 sin  100 π ×   λ film 
π  π
iR = 10− 3 sin( 200) For constructive interference, the net phase change is
− 7  2t 
id 44.25 × π × 10 cos(200) φ =  2 π + π − π = m( π)
Ratio, =
iR 10− 3 sin(200)  λ film 

cos 200 where, m = 1, 2, 3, ...


= 0.0138 ×
sin 200 λ 1 film 1  λ1 
or t = (m ) =  m
id 2 2  µ film 
= 0.00759
iR For destructive interference, the net phase change is
55. (1.5) Number of particles = 20  2t 
φ =  2 π + π − π = ( 2n + 1) π,
In Case of − 3D oscillator,  λ 2 film 
1 where n = 0, 1, 2, ...
S=
2 1  λ2 
t =   ( 2n + 1)
Hence, degeneracy is 4  µ film 
g = 2s + 1 On combining two equations, we get
1 1 λ1 1 λ2
= 2× + 1 ×m= 2n + 1
2 2 µ film 4 µ film
⇒ g = 1+ 1= 2
Solved Paper 2021 ● 27

( 2n + 1) λ1 640 nm 5 or v A2 = vB2
or = = =
2m λ 2 540 nm 4 Given, equation v A2 = kvB2
It is clear that m = n = 2 Clearly k=1
The thickness of the film is 59. (4) Given boolean expression is
1 1  640 nm Y = ABC + A B C + A B C + A B C
t = λ1 film × m =  ×2
2 2  1.36 
Since, if the OR operation of four different expression
t = 470.58 nm sets, each expression will become ‘1’ in only one particle
58. (1) Given situation is combination of inputs.
ABC→ 1 1 1
m
vA A B C→ 0 0 1
A BC →010
m
v AB C →100
2m θ
vB 2m 60. (0.6 to 0.8) For K transition, ∆E = hν = Rhc( z − b )2  12 − 12 
1 2 
hc  1
= Rhc( z − b )2  1 − 
λ  4
Using momentum conservation in x and y-directions.
mv = 2m vB cos θ 1 3
or = R ( z − b )2 ×
λ 4
⇒ v = 2vB cos θ ...(i)
Given, R = 109
. × 107 /m, z = 42
mv A = 2mvB sin θ
b =1(for K-series)
⇒ v A = 2vB sin θ ...(ii)
4
By adding Eqs. (i) 2 and (ii) 2 , we get or λ=
3 × R × ( z − b )2
v + v = 4v
2 2
A
2
B ...(iii)
4
Since it is elastic collision, so using energy conservation λ=
3 × 1.09 × 107 × (42 − 1)2
1 1 1
mv 2 + 0 = mv A2 + ( 2m ) vB2 4
2 2 2 λ=
3 × 1.09 × 107 × 41 × 41
⇒ v 2 = v A2 + 2vB2 ...(iv)
= 0.727 × 10− 10 m
From Eqs. (iii) and (iv)
or λ = 0.727 Å
v A2 + 2vB2 + v A2 = 4vB2
⇒ 2v = 2v
2
A
2
B
IIT JAM PHYSICS
SOLVED PAPER 2020
MM : 100 Time : 3 hrs

1. The examination is of 3 hours duration. There are a total of 60 questions carrying 100 marks. The entire paper is
divided into three sections, A, B and C. All sections are compulsory. Questions in each section are of different
types.
2. Section - A contains a total of 30 Multiple Choice questions (MCQ). Each MCQ type question has four
choices out of which only one choice is the correct answer. Question Q. 1- Q . 30 belong to this section and
carry a total of 50 marks. Q. 1- Q. 10 carry 1 mark each and Questions Q. 11 - Q. 30 carry 2 marks each.
3. Section - B contains a total of 10 Multiple Select Questions (MSQ). Each MSQ type question is similar to
MCQ but with a difference that there may be one or more than one choice (s) that are correct out of the four
given choices. The candidate gets full credit if he / she selects all the correct answers only and no wrong
answers. Questions Q. 31-Q.40 belong to this section and carry 2 marks each with a total of 20 marks.
4. Section - C contains a total of 20 Numerical Answer Type (NAT) questions. For these NAT type questions,
the answer is a real number which needs to be entered using the virtual keyboard on the monitor. No choices
will be shown for these type of questions. Questions Q.41-Q.60 belong to this section and carry a total of 30
marks. Q. 41- Q. 50 carry 1 mark each and Questions Q.51 - Q. 60 carry 2 marks each.
5. In all sections, questions not attempted will result in zero mark. In Section - A (MCQ), wrong answer will. result
in NEGATIVE marks. For all 1 mark questions, 1/3 marks will be deducted for each wrong answer. For all
2 marks questions, 2/3 marks will be deducted for each wrong answer. In Section - B (MSQ), there is NO
NEGATIVE and NO PARTIAL marking provisions. There is NO NEGATIVE marking in section - C (NAT) as well
6. Only Virtual Scientific Calculator is allowed. Charts, graph sheets, tables, cellular phone or other electronic
gadgets are NOT allowed in the examination hall.
7. The Scribble Pad will be provided for rough work.

Section A Multiple Choice Questions (MCQs)


In these type of questions, each question has four choices (a), (b), (c) and (d) out of which only
one option is correct.

Q.1-10 carry one mark each.

1. Which one of the following functions has a discontinuity in the second derivative at x = 0,
where x is a real variable?
(a) f ( x ) = | x|3 (b) f ( x ) = x| x| (c) f ( x ) = cos(| x|) (d) f ( x ) = | x|2

2. A collimated beam of laser light of wavelength 514 nm is normally incident on a smooth glass
slab placed in air. Given, the refractive indices of glass and air are 1.47 and 1.0 respectively,
the percentage of light intensity reflected back is
(a) 0 (b) 4.0 (c) 3.6 (d) 4.2

3. Two stationary point particles with equal and opposite charges are at some fixed distance from
each other. The points having zero electric potential lie on
(a) a sphere (b) a plane (c) a cylinder (d) two parallel planes
2 IIT JAM Physics Solved Papers & Practice Sets

4. For a system undergoing a first order phase transition at a temperature Tc , which one of the
following graphs best describes the variation of entropy (S) as a function of temperature (T)?
S S S S

(a) (b) (c) (d)

Tc T Tc T Tc T Tc T

5. In a photoelectric effect experiment, a monochromatic light source emitting photons with


energy greater than the work-function of the metal under test is used. If the power of the light
source is doubled, which one of the following statements is correct?
(a) The number of emitted photoelectrons remains the same.
(b) The stopping potential remains the same.
(c) The number of emitted photoelectrons decreases.
(d) The stopping potential doubles.

6. The figure below shows a cubic unit cell with lattice constant a. The shaded crystallographic
plane intersects the X-axis at 0.5 a. The Miller indices of the shaded plane are
Z

(a) (210) (b) (210) (c) (110) (d) (102)

7. For a particle moving in a central potential, which one of the following statements is correct?
(a) The motion is restricted to a plane due to the conservation of angular momentum.
(b) The motion is restricted to a plane due to the conservation of energy only.
(c) The motion is restricted to a plane due to the conservation of linear momentum.
(d) The motion is not restricted to a plane.

8. Consider the motion of a quantum particle of mass m and energy E under the influence of a
step potential of height V 0 . If R denotes the reflection coefficient, which one of the following
statements is true?
V0

4 4 1 1
(a) If E = V0, R = 1 (b) If E = V0, R = 0 (c) If E = V0, R = 1 (d) If E = V0, R = 0.5
3 3 2 2
Solved Paper 2020 ●
3

9. The Boolean function PQ(P + Q )(Q + Q ) is equivalent to


(a) P (b) P (c) P Q (d) PQ

10. Three point charges each carrying a charge q are placed on the vertices of an equilateral
triangle of side L. The electrostatic potential energy of the configuration is
1 q2 2 q2 3 q2 1 q2
(a) (b) (c) (d)
4πε 0 L 4πε 0 L 4πε 0 L πε 0 L

Q. 11-30 carry two marks each.

11. Which one of the following statements is correct?


n!
Given,   =
n
is the binomial coefficient.
m m !(n − m )!

(a) cos nθ = cos n θ −   cos n − 2 θ sin2 θ +   cos n −`4 θ sin4 θ − ....... .


n n
2   4

(b) sin nθ =   cos n −`1 θ sin θ +   cos n − 3 θ sin3 θ + ...... .


n n
 1  3

(c) cos nθ = cos n θ +   cos n − 2 θ sin2 θ +   cos n − 4 θ sin4 θ + ..... .


n n
2   4

(d) sin nθ = cos n θ −   cos n − 2 θ sin2 θ +   cos n − 4 θ sin4 θ − ..... .


n n
2   4
2
−  
r
R   r$ 
12. The volume integral ∫ e ∇ ⋅  2  d 3 r , where V is the volume of a sphere of radius R
V r 
centred at the origin, is equal to
4
(a) 4π (b) 0 (c) πR 3 (d) 1
3

13. lim x x is equal to


x→ 0+
(a) 0 (b) ∞ (c) e (d) 1

14. A wheel is rotating at a frequency f 0 Hz about a fixed vertical axis. The wheel stops in t 0
seconds, with constant angular deceleration. The number of turns covered by the wheel before
it comes to rest is given by
f0 t 0 f0 t 0
(a) f0 t 0 (b) 2 f0 t 0 (c) (d)
2 2
v
15. Two objects of masses m and 2m are moving at speeds of v and , respectively. After
2
v
undergoing a completely inelastic collision, they move together with a speed of . The angle
3
between the initial velocity vectors of the two objects is
(a) 60° (b) 120° (c) 45° (d) 90°

16. Two planets P1 and P 2 having masses M 1 and M 2 revolve around the sun in elliptical orbits,
with time periods T1 and T2 , respectively. The minimum and maximum distances of planet P1
from the sun are R and 3R respectively, whereas for planet P 2 , these are 2R and 4R
respectively, where R is a constant. Assuming M 1 and M 2 are much smaller than the mass of
T
the sun, the magnitude of 2 is
T1
4 IIT JAM Physics Solved Papers & Practice Sets

2 2 M1 3 3M2 3 3 2 2
(a) (b) (c) (d)
3 3M2 2 2 M1 2 2 3 3

17. The intensity of the primary maximum in a two-slit interference pattern is given by I 2 and the
intensity of the primary maximum in a three-slit interference pattern is given by I 3 . Assuming
the far-field approximation, same slit parameters and intensity of the incident light in both the
cases, I 2 and I 3 are related as
3 9 2 4
(a) I2 = I3 (b) I2 = I3 (c) I2 = I3 (d) I2 = I3
2 4 3 9

18. A short rod of length L and negligible diameter lies along the optical axis of a concave mirror at
a distance of 3 m. The focal length of the mirror is 1 m and L << 1 m. If L′ is the length of
image of the object in the mirror, then
L′ L′ L′ 1 L′ 1
(a) =4 (b) =2 (c) = (d) =
L L L 16 L 4

19. A beam of unpolarised light of intensity I 0 falls on a system of four identical linear polarisers
placed in a line as shown in the figure. The transmission axes of any two successive
polarisers make an angle of 30° with each other. If the transmitted light has intensity I, the ratio
I
is
I0

81 9 27 27
(a) (b) (c) (d)
256 16 64 128

20. Consider an annular region in free space containing a uniform magnetic field in the z-direction,
schematically represented by the shaded region in the figure. A particle having charge Q and
mass M starts off from point P (a ,0, 0) in the +x-direction with constant speed v . If the radii of
inner and outer circles are a and b respectively, the minimum magnetic field required, so that
the particle returns to the inner circle is
Y

X
P

−1 −1 −1 −1
Mv  b 2 − a 2  Mv  b 2 − a 2  Mv  b 2 − a 2  Mv  b 2 − a 2 
(a)   (b)   (c)   (d)  
Q  b  Q  2b  Q  3b  Q  4b 
Solved Paper 2020 ●
5

21. A thin conducting square loop of side L is placed in the first quadrant of the xy -plane with one of
the vertices at the origin. If a changing magnetic field B(t ) = β 0 (5zyt x$ + zxt y$ + 3 y 2t z$ ) is
applied, where β 0 is a constant, then the magnitude of the induced electromotive force in the
loop is
(a) 4β 0L4 (b) 3β 0L4 (c) 2β 0L4 (d) β 0L4
−1
 ε−µ 
22. In which one of the following limits, the Fermi-Dirac distribution nF ( ε, T ) = e kBT + 1 and
 
 
−1
 ε−µ 
Bose-Einstein distribution nB ( ε, T ) = e kBT − 1 reduce to Maxwell-Boltzmann distribution?
 
 
(Here, ε is the energy of the state, µ is the chemical potential, kB is the Boltzmann constant
and T is the temperature.)
(a) µ = 0 (b) ( ε − µ ) << kBT (c) ( ε − µ ) >> kBT (d) µ >> kBT

23. Consider N classical particles at temperature T, each of which can have two possible energies
0 and ε. The number of particles in the lower energy level (N 0 ) and higher energy level (N ε )
levels are related by (kB is the Boltzmann constant)
ε ε t ε
− −
N0 N0 N0 N0
(a) =e k BT
(b) =e k BT
(c) = 1 + e k BT (d) = 1 − e k BT
Nε Nε Nε Nε

24. The root mean square (rms) speeds of H- atoms at 500 K, V H and He-atoms at 2000 K, V He
are related as
(a) VH > VHe (b) VH < VHe (c) VH = VHe (d) VH >> VHe

25. The normalised ground-state wave function of a one-dimensional quantum harmonic oscillator
1/ 4
α
with force constant K and mass m is ψ 0 (x ) =  
2 K
e − αx /2
, where α = mω 0 / h and ω 20 = .
 π m
Which one of the following is the probability of finding the particle outside the classically
allowed region?
(The classically allowed region is where the total energy is greater than the potential energy)
2 ∞ 2 2 ∞ −y2
(a) ∫
π 1
y 2e − y dy (b) ∫ e
π 1
dy (c) 0.5 (d) 0

$ acts on two orthonormal states of a system ψ and ψ as per following


26. A linear operator O 1 2

$ψ = ψ ,O
O $ ψ = 1 ( ψ + ψ ). The system is in a superposed state defined by
1 2 1 1 2
2
1 i
ψ= ψ1 + ψ 2 . The expectation value of O
$ in the state ψ is
2 2
1 1 1 1
(a) [1 + i ( 2 + 1)] (b) [1 − i ( 2 + 1)] (c) [1 + i ( 2 − 1)] (d) [1 − i ( 2 − 1)]
2 2 2 2 2 2 2 2

27. Consider a one-dimensional infinite potential well of width a. This system contains five
non-interacting electrons, each of mass m, at temperature T = 0 K. The energy of the highest
occupied state is
6 IIT JAM Physics Solved Papers & Practice Sets

25π 2h 2 10π 2h 2 5π 2h 2 9π 2h 2
(a) (b) (c) (d)
2 ma 2 2 ma 2 2 ma 2 2 ma 2

28. Consider the crystal structure shown in the figure, where black and grey spheres represent
atoms of two different elements and a denotes the lattice constant. The Bravais lattice for this
structure is
a

a
(a) simple cubic (b) face-centred cubic
(c) body-centred cubic (d) Triclinic

29. For an unbiased silicon n - p - n transistor in thermal equilibrium, which one of the following
electronic energy band diagrams is correct? (E c = conduction band minimum, E v = valence
band maximum and E F = Fermi level)
Ec
Ec
EF
(a) (b)
EF
Ev
Ev

Ec
EF Ec
(c) EF (d)
EF
Ev EF
Ev

30. In the circuit shown in the figure, both Op-amp are ideal. The output for the circuit V out is
10R 10R

R
– 5R

V1 + Vout
R +

V2

(a) 20 V1 + 10 V2 (b) − 20 V1 + 10 V2 (c) 10 V1 − 20 V2 (d) 20 V1 − 10 V2


Solved Paper 2020 ●
7

Section B Multiple Select Questions (MSQs)


In these type of questions, each question has four choices (a), (b), (c) and (d) out of which only
one or more than one is/are correct options.

Q. 31-40 carry two marks each.

31. If P and Q are Hermitian matrices, which of the following is/are true?
(A matrix P is Hermitian if P = P † , where the elements Pij† + P ji* )
(a) PQ + QP is always Hermitian (b) i ( PQ − QP ) is always Hermitian
(c) PQ is always Hermitian (d) PQ − QP is always Hermitian

32. Consider a vector function u ⋅ ( r ) and two scalar functions ψ( r ) and φ(r). The unit vector n$ is
normal to the elementary surface dS, dV is an infinitesimal volume, dl is an infinitesimal line
element, and ∂ / ∂n denotes the partial derivative along n.
$ Which of the following identities
is/are correct?
(a) ∫ ∇ ⋅ udV = ∫ S u ⋅ n$ dS , where surface S bounds the volume V .
V

 ∂φ ∂ψ 
(b) ∫ [ψ∇ 2φ − φ∇ 2ψ]dV = ∫ S ψ ∂n − φ ∂n  dS, where surface S bounds the volume V .
V

 ∂φ ∂ψ 
(c) ∫ [ψ∇ 2φ − φ∇ 2ψ] dV = ∫S ψ +φ dS , where surface S bounds the volume V .
V  ∂n ∂n 
(d) ∫ u ⋅ dl = ∫∫ S (∇ × u ) ⋅ n$ dS, where C is the boundary of surface S.
C

33. A thin rod of uniform density and length 2 3 m is undergoing small oscillations about a pivot
point. The time period of oscillation (Tm ) is minimum when the distance of the pivot point from
the centre of mass of the rod is x m . Which of the following is/are correct?
(Assume acceleration due to gravity, g = 10 m/s 2 )
3 2π 2π
(a) xm = 1m (b) xm = m (c) Tm = s (d) Tm = s
2 3 5

34. Three sinusoidal waves of the same frequency travel with the same speed along the positive
a a
x - direction. The amplitudes of the waves are a, and , and the phase constants of the
2 3
π 3π
waves are , π and , respectively. If Am and φ m are the amplitude and phase constant of
2 2
the wave resulting from the superposition of the three waves, which of the following is/are
correct?
5 π  3 7 2
(a) Am = a (b) φ m = + tan−1  (c) Am = a (d) φ m = tan−1 
6 2  4 6  3

35. An object executes simple harmonic motion along the x-direction with angular frequency ω and
amplitude a. The speed of the object is 4 cm/s and 2 cm/s, when it is at distances 2 cm and
6 cm respectively, from the equilibrium position. Which of the following is/are correct?
3 5 140 175
(a) ω = rad/s (b) ω = rad/s (c) a = cm (d) a = cm
8 6 3 6
8 IIT JAM Physics Solved Papers & Practice Sets

36. For electric and magnetic fields, E and B due to charge density ρ( r, t ) and a current density
J( r, t ), which of the following relations is/are always correct?
∂ρ
(a) ∇ × E = 0 (b) ∇ ⋅ B = 0 (c) ∇ ⋅ J − =0
∂t
(d) F = q( E + v × B ) , where F is the force on a particle with charge q moving with velocity v
k $
37. A spherical dielectric shell with inner radius a and outer radius b, has polarisation p = r,
r2
where k is a constant and r$ is the unit vector along the radial direction. Which of the following
statements is/are correct?
(a) The surface density of bound charges on the inner and outer surfaces are − k and + k,
respectively. The volume density of bound charges inside the dielectric is zero.
(b) The surface density of bound charges is zero on both the inner and outer surfaces. The volume
density of bound charges inside the dielectric is + k.
−k k
(c)The surface density of bound charges on the inner and outer surfaces are 2 and 2 ,
a b
respectively. The volume density of bound charges inside the dielectric is zero.
(d) The surface density of bound charges is zero on both the inner and outer surfaces. The volume
3k
density of bound charges inside the dielectric is .
4π( b 3 − a 3)

38. One mole of an ideal gas having specific heat ratio ( γ ) of 1.6 is mixed with one mole of another
ideal gas having specific heat ratio of 1.4. If CV and C p are the molar specific heat capacities
of the gas mixture at constant volume and pressure respectively, which of the following is/are
correct?
(R denotes the universal gas constant)
(a) CV = 2.08R (b) Cp = 2.9R (c) Cp = 148
. CV (d) Cp = 152
. CV

39. Two relativistic particles with opposite velocities collide head-on and come to rest by sticking
with each other. Which of the following quantities is/are conserved in the collision?
(a) Total momentum (b) Total energy (c) Total kinetic energy (d) Total rest mass

40. Figure shows a circuit diagram comprising Boolean logic gates and the corresponding timing
diagrams show the digital signals at various points in the circuit. Which of the following is/are
true?
1

2
1 2 3 5 6 4
7
4
5

(a) Points 3 and 7 are shorted.


(b) The NOT gate on the right is faulty.
(c) The AND gate is faulty and acts like a NOR gate.
(d) The AND gate is faulty and acts like an OR gate.
Solved Paper 2020 ●
9

Section C Numerical Answer Type Questions (NATQs)


This section contains Numerical Answer Type (NAT) questions, for these NAT questions, the
answer is a real number.

Q.41-50 carry one mark each.


41. The line integral of the vector function u (x, y ) = 2yi$ + x$j along the straight line from (0, 0) to
(2, 4) is ....... .

42. Consider a thin bi-convex lens of relative refractive index n = 1.5. The radius of curvature of
one surface of the lens is twice that of the other. The magnitude of larger radius of curvature in
units of the focal length of the lens is ………. (Round off to 1 decimal place)

43. Water flows in a horizontal pipe in a streamlined manner at an absolute pressure of


4 × 105 Pa and speed of 6 m/s. If it exits the pipe at a pressure of 105 Pa, the speed of water
at the exit point is ……… m/s. (Round off to 1 decimal place)
(The density of water is 1000 kg/m 3 )

44. Consider a retarder with refractive indices ne = 1.551 and no = 1.542 along the extraordinary
and ordinary axes, respectively. The thickness of this retarder for which a left circularly
polarised light of wavelength 600 nm will be converted into a right circularly polarised light is
……… µm. (Round off to 2 decimal places)

45. Using a battery, a 10 pF capacitor is charged to 50 V and then the battery is removed. After
that, a second uncharged capacitor is connected to the first capacitor in parallel. If the final
voltage across the second capacitor is 20 V, its capacitance is ……… pF.

46. Consider two spherical perfect blackbodies with radii R1 and R 2 at temperatures T1 = 1000 K
and T2 = 2000 K, respectively. They both emit radiation of power 1 kW. The ratio of their radii,
R1
is given by ……… .
R2

47. In a Compton scattering experiment, the wavelength of incident X-rays is 0.500 Å. If the
Compton wavelength λ C is 0.024 Å, the value of the longest wavelength possible for the
scattered X-ray is …… Å (Specify up to 3 decimal places)

48. A solid with FCC crystal structure is probed using X-rays of wavelength 0.2 nm. For the
crystallographic plane given by (2, 0, 0), a first order diffraction peak is observed for a Bragg
angle of 21°. The unit cell size is ………… nm. (Round off to 2 decimal places)

49. The figure shows a circuit containing two diodes D1 and D 2 with threshold voltages V TH of
0.7 V and 0.3 V, respectively. Considering the simplified diode model, which assumes diode
I - V characteristic as shown in the plot on the right, the current through the resistor R is ………
µA.
I

D1
10 V R=100 kΩ

V
VTH
D2
10 IIT JAM Physics Solved Papers & Practice Sets

50. An ideal gas undergoes and isothermal expansion along a path AB, adiabatic expansion along
BC, isobaric compression along CD, isothermal compression along DE and adiabatic
compression along EA, as shown in the figure. The work done by the gas along the process
BC is 10 J. The change in the internal energy along process EA is 16 J. The absolute value of
the change in the internal energy along the process CD is ……… J.
p
A

D C

Q. 51-60 carry two marks each.

d 2y dy
51. If a function y (x ) is described by the initial-value problem, 2
+5 + 6 y = 0, with initial
dx dx
 
dy
conditions y ( 0) = 2, and   = 0, then the value of y at x = 1is ……… .
 dx  x =0
(Round off to 2 decimal places)

52. A vehicle of mass 600 kg with an engine operating at constant power P accelerates from rest
on a straight horizontal road. The vehicle covers a distance of 600 m in 1 min. Neglecting all
losses, the magnitude of P is ........ kW. (Round off to 2 decimal places)

53. The angular momentum of a particle relative to origin varies with time (t ) as L = ( 4 x$ + αt 2 y$ )
kg-m 2 /s, where α = 1kg-m 2 /s 3 . The angle between L and the torque acting on the particle
becomes 45° after a time of ………… s.

54. Two transverse waves y 1 = 5 cos(kx − ωt ) cm and y 2 = 5 cos(kx + ωt ) cm, travel on a string
along X-axis. If the speed of a point at x = 0 is zero at t = 0 s, 0.25 s and 0.5 s, then the
minimum frequency of the waves is ………… Hz.

55. For the AC circuit shown in the figure, R = 100 kΩ and C = 10 pF, phase difference between
V in and V out is 90° at the input signal frequency of ………… kHz.
(Round off to 2 decimal places)

R C

Vout=VB – VA
Vin
A B

C R
Solved Paper 2020 ●
11

56. The magnetic fields in tesla in the two regions separated by the z = 0 plane are given by
B1 = 3 x$ + 5 z$ and Bz = x$ + 3 y$ + 5 z$ . The magnitude of the surface current density at the
interface between the two regions is α × 10 6 A/m. Given the permeability of the free space
µ 0 = 4 π × 10 −7 N/A 2 , the value of α is ……… . (Round off to 2 decimal places)

57. A body at a temperature T is brought into contact with a reservoir at temperature 2T. Thermal
equilibrium is established at constant pressure. The heat capacity of the body at constant
pressure is C p . The total change in entropy of the body and the reservoir in unit of C p is ....... .
(Round off to 2 decimal places)

58. One mole of an ideal monoatomic gas at pressure p, volume V and temperature T is
expanded isothermally to volume 4V. Thereafter, the gas is heated isochorically (at constant
volume) till its pressure becomes p. If R is the universal gas constant, the total heat transfer in
the process, in units or RT is ……… . (Round off to 2 decimal places)

59. In the transistor circuit given in the figure, the emitter-base junction has a voltage drop of
0.7 V.A collector-emitter voltage of 14 V reverse biases the collector. Assuming the collector
current to be the same as the emitter current, the value of RB is ……… kΩ.
20 V

RB IB 2 kΩ IC

β=100

1 kΩ IE

40 40
60. The radioactive nuclei K decay to . × 10 9 yr. The
Ar with a half-life of 125 40
K / 40 Ar
isotopic ratio for a particular rock is found to be 50. The age of the rock is m × 10 7 yr. The
value of m is ………… . (Round off to 2 decimal places)
Answers with Solutions
− x 2 , x < 0 3. (b) We are given with a dipole as shown in the figure.
1. (b) (ii) When f (x ) = x x=  2
 x ,x ≥ 0
P
− 2x , x < 0 − 2, x < 0
We have, f ′ (x ) =  and f ′ ′ (x ) =  r r
 2x , x ≥ 0  2, x ≥ 0
As, lim f ′ ′ (x ) = − 2 and lim f ′ ′ (x ) = 2 q+ –
x → 0− x→ 0 +
l l q
i.e. LHL (x = 0) ≠ RHL (x = 0)
Equatorial plane
Hence, the function [f (x )] has a discontinuity in second
derivative [f ′ ′ (x )] at x = 0. At any point on equatorial plane, potential due to
 − x 3, x < 0 system of charges,
(i) When f (x ) = | x |3 =  3
x , x ≥ 0 Vnet = V( + ) + V( − ) =
kq kq
− =0
 − 3x 2, x < 0 r r
We have, f ′ (x ) =  2
 3x , x ≥ 0 So, equatorial plane is an equipotential surface
with V = 0.
− 6x , x < 0
and f ′ ′ (x ) = 
 6x , x ≥ 0 4. (d) During first order phase transition, heat is absorbed at
a constant temperature.
Clearly, LHL f (x ) = lim
x→ 0 −
− 6x = 0.
x → 0− ∆Q
As, entropy change, ∆S =
RHL f (x ) = lim 6x = 0 T
x→0 + x→0 +
where, T = phase change temperature = constant.
Also, f ′ ′ ( 0) = 0
So, ∆S increases (as ∆Q is positive as heat is absorbed)
∴ f ′ ′ (x ) is continuous at x = 0.
at constant temperature. This is shown in following graph
− cos x , x < 0
(iii) When f (x ) = cosx; f ′ ′ (x ) = 
 − cos x , x ≥ 0
S

Clearly, f ′ ′ (x ) is continuous at x = 0.
(iv) When f (x ) =x2
x 2 , x < 0
=  2
x , x ≥ 0
Clearly f ′ ′ (x ) = 2∀ x ∈ R T
TC
∴ f ′ ′ (x ) is continuous at x = 0.
2. (c) In case of normal incidence θi = θr = 0°; and no 5. (b) When source power is doubled, intensity of light falling
distinction between S and P polarisation, fraction of over photosensitive surface is doubled, but frequency
incident power, i.e. reflected from the interface is given by; of source remains same.
Hence, number of photons reaching the surface
R = Intensity reflected ÷ Intensity incident
increases but energy of photon remains constant.
2
 n − n2  So, when source power is increased, stopping potential
= 1 
 n1 + n 2  does not changes as kinetic energy of photons remains
constant.
6. (a) Miller indices gives orientation of an atomic plane.
n1=1.0 θi=θr=0º Z
n2=1.47
a
5a
In given case with values given, we have 0.
2
 10 . 
. − 147 2
R=  = ( 0.19 ) = 0.036
 10 . 
. + 147
Hence, percentage of light reflected back Y
= 0.036 × 100 = 3.6 %
X
Solved Paper 2020 ●
13

We calculated Miller indices as follows: 9. (b) We have, following Boolean rules,


Axis X Y Z A + A =1
Intercept length a a ∞ 1⋅ A = A
2 A ⋅B = A + B
Reciprocal of intercepts 2 1 0 A+A=A
a a a A ⋅A = 0
Cleared fractions 2 1 0 We use above axioms and identities to solve given
Boolean function,
Miller indices = ( 210)
PQ (P + Q )(Q + Q )
7. (a) In classical potential theory, motion of a particle in a
= PQ + (P + Q )(1) = PQ (P + Q )
central potential field occurs due to a central force
(most probably negative). = (P + Q )(P + Q ) = P (P + Q ) + Q (P + Q )
A central force is always directed towards (or away) = P + P ⋅ Q + Q ⋅ P + Q ⋅ Q = P + P (Q + Q ) + 0
towards a fixed point.
= P + P (1) = P + P = P
The motion of a particle under influence of a central
force is always a planar motion. This is shown 10. (c) Potential energy of given system of charges,
mathematically here ; kQ1Q 2 kQ1Q 3 kQ 2Q 3
U = + +
Angular momentum of particle, r12 r13 r23
L = r × p = r × mv Q1 = q
+
So, time rate of change of angular momentum,
d d
L = m ( r × v)
dt dt L L
 dr dv
= m  × v + r ×  = m( v × v + r × a )
 dt dt 
+ +
dv L
But v × v = 0 and = acceleration = a Q3=q Q 2 =q
dt
dL Here, Q1 = Q 2 = Q 3 = q and r12 = r23 = r13 = L
⇒ = r × ma = r × F 1
dt Also, k =
4πε 0
As F is a central force F and r vectors are along same
line and so 3 q2
So, U =
r×F=0 4πε 0 L
dL
⇒ = 0 ⇒ L = constant. 11. (a) By binomial expansion, we have
dt
(cos θ + i sin θ )n = cosn θ + n cosn − 1 θ ⋅ i sin θ
Now, r ⋅ L = r ⋅ ( r × p) = p⋅ ( r × r ) = 0
n(n − 1)
(Note a ⋅ ( b × c) = 0 when a, b and c are coplanar) + cosn − 2 θ. i 2 sin2 θ + ........ .
1⋅ 2
Hence, particles position r and velocity v must be
coplanar vectors. As i 2 = − 1, i 3 = − i, i 4 = 1, i5 = i .……, we have
Hence, a particle moving in a central potential field is n(n − 1)
(cos θ + i sin θ )n = cosn θ − cosn − 2 θ ⋅ sin2 θ
restricted to a plane due to the conservation of angular 1. 2
momentum. n(n − 1)(n − 2)
+ cosn − 4 θ ⋅ sin4 θ + .....
8. (c) Reflection coefficient is defined as 1⋅ 2 ⋅ 3
 flux of reflected particles n(n − 1)(n − 2)
R= …… + i{n cosn − 1 θ ⋅ sin θ − cosn − 3 θ
 1⋅ 2 ⋅ 3
 flux of incident particles 
⋅ sin3 θ + ...... } …(i)
When E < V , we have
Now, by de-Moivre’s theorem, we have
| B |2 = | A |2
(cos nθ + i sin nθ ) = (cos θ + i sin θ )n .…(ii)
So, the reflected flux = incident flux
From Eqs. (i) and (ii) equating real parts, we have
and therefore R = 1 n(n − 1)
Hence, option (c) is correct. cos(nθ ) = cosn θ − cosn − 2 θ ⋅ sin2 θ
1⋅ 2
Also, when E > V , R ≠ 1 n(n − 1)(n − 2)(n − 3)
+ cosn − 4 θ ⋅ sin4 θ +…… .
and when E > V , R ≠ 0 1⋅ 2 ⋅ 3 ⋅ 4
14 IIT JAM Physics Solved Papers & Practice Sets

n n θ
= cosn θ −   cosn − 2 θ sin2 θ +   cosn − 4 θ ⋅ sin4 θ Now, as ω 0 = 2πf0 and number of rounds, n = .
 2  4 2π
− …… θ ( 2πf0 )t 0 f0t 0
So, n= = =
12. (a) We have, given volume integral, 2π 2 × 2π 2
r 2
−   15. (b) Initially (before collision),
R  r$ 
I =∫e ∇ ⋅  2  d 3r Y
V
r 
Integrating by parts, we have 2m
v/2 θ
r$ r$ X
I = − ∫ 2 ⋅ ∇(e −r )dτ + ∫ e −r 2 ⋅ da
V r S r
v
−r  ∂ −r  $ −r $
But ∇e =  e  r = e r
 ∂r  m
1 −r  r$  Finally (after collision),
So, I = ∫ e 4πr 2dr + ∫ e −r  2  (r 2 sin dθdφ ) r$
r 2
r  Y
R
−R v/3
= 4π ∫ e −r dr + e ∫ sin θdθdφ
0

= 4π [( − e −r )]R0 + 4πe −R α
Here, R → ∞ ⇒ e −R = 0 X
−∞ −0
∴ I = 4π( − e +e ) = 4π 3m

13. (d) Let y = x x Let angle between initial velocity vectors is θ and angle
of final velocity vector with X-axis is α.
Then, log y = x log x
As, momentum is conserved, i.e. ( p x )initial = ( p x )final
log x
or log y = and ( p y )initial = ( p y )final
 1
  We have,
x 
v  v 
Taking limit, we have 2m   + mv cosθ = 3m   cosα
 2  3
log x
⇒ lim log y = lim
x→0 +
x→0 +  1 v 
  and mv sin θ = 3m   sin α
x   3
Now, by L-Hospital’s rule, we have From above equations, we get
 1 1 + cos θ = cos α …(i)
 
x  and sin θ = sin α …(ii)
lim log y = lim
+ +  −1
x→0 x→0
 2 So, from Eq. (i), we have
x 
1 + cos θ = 1 − sin2 α
= lim ( − x ) = 0
x → 0+ and from Eq. (ii), we get
⇒ lim log y = 0 ⇒ lim e logy = e 0
+ +
x→0 x→0
1 + cosθ = 1 − sin 2θ
⇒ lim y = e 0 = 1
x → 0+ (1 + cos θ )2= cos2 θ
∴ lim x x = 1 1
x → 0+ ⇒ cosθ = − ⇒ θ = 120°
2
14. (c) Angular displacement of particle in time t 0, 16. (c) By Kepler’s law of periods,T 2 ∝ a 3
1 2
θ = ω 0t 0 + αt 0 where, T = time period of rotation and a = mean radius
2
of elliptical orbit.
0 − ω0 ω r +r
Here, α= =− 0 Also, a = 1 2
t0 t0 2
1  ω0  2 1 Here, r1 = radius at apogee (maximum distance)
So, θ = ω 0t 0 −   .t0 ⇒ θ= ω 0t 0
2  t0  2 and r2 = radius at perigee (minimum distance).
Solved Paper 2020 ●
15

So, ratio of time periods, Image ( A′ B′ ) of rod ( AB ) is formed as shown below.


2 3 3
 T2  a  (r1 + r2 )P32  2R + 4R 
  =  2 = =  L′
 T1   a1  (r1 + r2 )P31  R + 3R  B A A′ B′
3 3
T2  6  2  3 2 3 3 vB
⇒ =  =  =
T1  4  2 2 2 vA

17. (d) Let I 0 = intensity of incident light, then intensity patterns


Length of image, L′ = |v A − vB |
for two slits and three slits are as shown below. 3 3+L L
= − =
9 I0(three slits) 2 2 + L 2( 2 + L)
L
⇒ L′ =
4 I0(two slits) 4 + 2L
⇒ 4L′ + 2LL′ = L
L′
⇒ 4 + 2L′ = 1
λ L
λ ∆L=
∆L= 2 As L′ << 1, we are neglecting 2L′,
3
L′ L′ 1
Intensity maxima for N-slit interference is given by 4 ≈1 ⇒ ≈
2 L L 4
2
  1 Nδ
   N cos  19. (d) After first polaroid, intensity is reduced to half and then
sin N δ 
I max = I 0 lim  = I 0 lim  2 2  = I ⋅N 2
0 Malus law is applicable, i.e. I transmitted = I incident × cos2 θ
δ → 0 δ  δ→ 0
 1 δ 
 sin   cos
 2   2 2  Hence, intensities transmitted are as shown below.
2
∴ I max = N I 0
I0 I1 I2 I3 I4
For two slits, I max = I 2 = 4I 0
For three slits, I max = I 3 = 9I 0
I2 4 4I
So, = ⇒ I2 = 3
I3 9 9 I0
We have, I1 = ;
2
18. (d) Rod of length L is lying on optical axis as shown below. I 3
I 2 = 0 × cos2 30° = I 0
f=1m 2 8
3  9
B A I 3 =  I 0 × cos2 30° = I0
8  32
L 3m P
9  27
I4 =  I  × cos2 30° = I0
 32 0 128
I4 I 27
Now, for end A, ⇒ = =
I 0 I 0 128
u = − 3m , f = − 1m
1 1 1 20. (b) Since, it is given that the particle starts moving in
Using + = we have
v u f + x-direction in the presence of uniform magnetic field
1 1 1 which is in + z-direction. So, force on the particle is
= −
v ( −1) ( −3) given as
1 2 3 FB = Q ( v × B) = QvB sin θ
⇒ = − m ⇒ vA = − m = QvB (Qθ = 90°)
v 3 2
and for end B, Also, the trajectory followed by the particle will be
circular. So, we have
u = − ( 3 + L), f = − 1m, we have
1 1 1 1 1 Net centripetal force = Force on the particle due to B
= − = − ⇒ FC = FB
v f u ( − 1) − ( 3 + L)
1 1 − ( 2 + L) Mv 2
= − = ⇒ = QvB …(i)
3+L 1 3+L r
where, r is the radius of the circular path followed by
 3 + L
So, v3 = −   the particle.
 2 + L
16 IIT JAM Physics Solved Papers & Practice Sets

So, let us consider the trajectory of the particle in order Here, B = β 0(5zyt x$ + zxt y$ + 3y 2t z$ )
to returns to the inner circle, with O′ be the centre of
and dA = dxdy ⋅ z$
the circular path followed by it as shown below. L
So, Flux, φ = ∫ B ⋅ dA
0
L L

Path of
=∫ ∫ β0( 3y
2
tz$ ⋅ dxdy z$ )
r
particle 0 0
O′ L L
= β 0 3t ∫ y 2dy ∫ dx
+r 2

r
√a 2

0 0
O
L3 L
a P = β 0 3t ⋅ ⋅ = β 0L4t
b 3 1
So, magnitude of induced emf,
dφ d
ε= = β 0L4t = β 0L4
dt dt

22. (c) We can express Fermi-Dirac distribution given as


−1 −1
Note Diagram Z-axis is taken perpendicularly into the  ε −µ   ε− µ −1
 1 
plane of paper. nF = e kBT + 1 = e kBT  1 + ε − µ / kBT 
     e 
PO ′ = r , OP = a, ∠OPO ′ = 90° ⇒ OO ′ = a 2 + r 2    
From Eq. (i), we get Using binomial approximation, we have
 ε− µ   
−1
Mv 1
B= ⇒B ∝  1− 1 
nF = e
kB T

  ε−µ 
Qr r
 
So, in order to get minimum magnetic field, r should be    kBT 
e
maximum.
Mv Similarly for Bose-Einstein distribution, we have
⇒ Bmin = …(ii)
 ε − µ   
−1 −1
Qrmax  ε−µ 
   kBT  1 
nB = e kBT
−1 = e 1+ ε − µ 
From the figure shown above, we can write      
    
b = r 2 + a2 + r e kBT 
When both are same, we have
⇒ b 2 = r 2 + a 2 + r 2 + 2r r 2 + a 2 1 1
1− ε − µ = 1+ ε − µ
= 2r 2 + a 2 + 2r (b − r )
⇒ b = 2r 2 + a 2 + 2rb − 2r 2 ⇒ b 2 = a 2 + 2rb
2 e kBT e kBT
ε−µ
b2 − a2 1
or r (rmax ) = ⇒ ε−µ
≈0 ⇒ e kBT >> e 0
2b
e kBT
Substituting the value of rmax in Eq. (ii), we get
−1 ⇒ ε − µ >> kBT
Mv Mv b2 − a2
Bmin = =   23. (b) Using Ludwig Boltzmann’s equation, number of
b2 − a2 Q  2b 
Q  particles in higher state,
 2b  (E ε − E 0 )

Nε = N0e kBT
21. (d) Flux linked with coil at instant t.
φ = ∫ B ⋅dA Here, Eε = ε and E0 = 0
ε
Nε −
k T
Y So, =e B
N0
ε
N0 1 N0
⇒ = −ε
⇒ =e k BT
Nε Nε
e kBT
dy
k̂ 24. (c) Root mean square speed of a gas molecule,
dx 3kT 3RT
X v rms = =
m M
Z
Solved Paper 2020 ●
17

Here, m = mass of 1 molecule and M = molar mass. 1


= [1 − i ( 2 − 1)]
3R × 500 2 2
So, v rms of hydrogen at 500 K,v H =
1 × 103 27. (d) Energy of a particle confined to a box of width L,
and v rms of helium at 2000 K, n 2h 2
En =
3R × 2000 3R × 500 8mL2
v He = 3
=
4 × 10 1 × 103 Here, if we introduce
h
Hence, v H = v He h= or h = 2πh

25. (b) The turning point for oscillator occurs when kinetic
n 2 4 π 2 h2 n 2 π 2 h2
energy is zero, so that potential energy is equals to Then, En = 2
⇒ En =
8mL 2mL2
total energy.
If particle is an electron confined to a potential well of
For quantum number n, this is determined by width a, then each permissible level can accommodate
1 2  1 two electrons at most. This means fifth electron takes
Kx max = n +  hω
2  2 third permissible level (n = 3).
K 9 π 2 h2
Here, ω 0 = Hence, energy of fifth electron at 0K is; En = 3 =
m 2ma 2
mK 28. (b) Properties of FCC Bravais lattices are
and α= = mω 0 / h
h 4
Number of lattice points/atoms per unit cell =
2 h 2n + 1 4
∴ x max = ( 2n + 1) =
Km α  2
Nearest distance between lattice points = a  
So, P (x max ≤ x ≤ ∞ ) = P ( − ∞ ≤ x ≤ − x max )  2

=∫ | ψ (x )|2 dx Clearly, given crystal is a FCC crystal.
x max n
For n = 0, Poutside is; 29. (b) In an n - p - n transistor, Fermi energy level in emitter
1 (n-type) is near to conduction band and in base
∞ α 2 − αx 2 2 ∞ 2
Poutside = 2∫ 1   e dx = ∫1 e −y dy (p-type), it is near to valance band. As collector doping
 π π
α concentration is less than emitter, so Fermi energy
Above result shows there is around 16% chance that level is slightly distant from conduction band (than
oscillator will tunnel outside its classically allowed emitter). So, best suitable diagram is as shown below.
region.
Energy level of
$
26. (d) Given, Oψ =ψ , conduction
1 2 band (EC)
$ 1
Oψ 2 = ( ψ 1 + ψ 2 ),
2 Fermi energy
1 i level (EF)
ψ= ψ1 + ψ2
2 2 Energy level of
valance
$ = < ψO
$ψ > *
Expectation value of O band (EV)

1 * i * $ 1 i
=< ψ1 − ψ 2O ψ1 + ψ2 > 30. (d) The given circuit can be drawn as below.
2 2 2 2
10R 10R
 1 * i *   1 i  1 
= ψ − ψ 2  ψ + (ψ1 + ψ 2 ) 
 2 1 2   2 2 2  2 
R
– 5R
 1 * i  1 i i  –
= ψ − ψ 2*   ψ + ψ + ψ  + V0
 2 1 2   2 2 2 1 2 2 V1
R + Vout

1 i i
= ψ 1*ψ 2 + ψ 1*ψ 1 + ψ 1*ψ 2 V2
2 2 2 2 2
i * i2 i2
− ψ 2ψ 2 − ψ *2ψ 1 − ψ *2ψ 2
2 2 2 2 2 From the above circuit, we can write
2  10R 
=
i i
−− −
i
=
i i
− +
i V0 = −   V = − 10 V1
 R  1
2 2 2 2 2 2 2 2 2 2
18 IIT JAM Physics Solved Papers & Practice Sets

 10R   10R   For some angle θ, we have


So, Vout = −   ( − 10 V1) +  V
 5R   R  2  τ = I α = − mgx sin θ
= 20 V1 − 10 V2 d 2θ
⇒ I 2 = − mgx θ (for small θ, sin θ ≈ θ)
31. (a,b) As P and Q are Hermitian, we have dt
P = P † and Q = Q † d 2θ mgx
⇒ + =0
dt 2 I
Also i† = −i mgx
⇒ ω2 =
Now, for ‘PQ + QP ’, I
(PQ + QP )† = (PQ )† + (QP )† = Q †P † + P †Q † 2π I
So, time period of oscillation,T = = 2π
= QP + PQ = PQ + QP ω mgx
So, PQ + QP will be Hermitian. Also, for i(PQ − QP )† , where, I = I CM + mx 2
[i(PQ − QP )] † = i † (PQ − QP )† = − i{(PQ )† − (QP )† } By parallel axes theorem,
† † † †
= − i(Q P − P Q ) = − i(QP − PQ ) I CM + mx 2
hence T = 2π …(i)
= i(PQ − QP ) mgx
So, i(PQ − QP ) is a Hermition matrix. d 2
Now,T is minimum whenT 2 is minimum and T = 0.
32. (a,b,d) According to Gauss’s divergence theorem, integral dx
of a derivative (divergence) over a region (volume) is d  I CM + mx 2 
⇒   =0
equal to the value of the function at the boundary dx  mgx 
(surface that bounds the volume). ⇒ mgx ( 2mx ) − (I CM + mx 2 )mg = 0
So, identity given in option (a) is in accordance to this
ml 2
theorem, For a thin rod, I CM =
12

∀ ⋅ udV = u ⋅ ndS
$
∫ l2 l
V S So, 2x 2 − −x 2 = 0 ⇒ x =
12 12
According to Stokes’ theorem, integral of a derivative
(curl) over a region (patch of surface) is equal to the As, l =2 3 m
function at the boundary (perimeter of the patch). 2 3
⇒ xm = x when T is minimum = = 1m
So, identity given in option (d) is in accordance to this 12
theorem, Also, by substituting xm = 1m
∫ u ⋅ d l = ∫∫ ( v × u ) ⋅ n$ dS in Eq. (i), we get
C S
m( 2 3 )2
Also, according to Gauss’s identity are, if φ and ψ are + m ⋅ (1)2
12 2π
scalar functions sufficiently differentiables, then Tm = 2π = s
m × 10 × 1 5
∫ ( φ∇ ψ − ψ∇ 2φ )dV = ∫ ( φ∇ψ − ψ∇φ )dS
2

V S 34. (a, b) Wave phasors are as shown below.


 ∂ψ ∂φ  Y
=∫ φ −ψ  dS
 ∂n ∂n 
S
a
So, identity given in option (b) is correct, but in
option (c) is incorrect. π π/
2
33. (a, d) A thin rod pivoted at point P distant x from its centre a/2
X
of mass at O s shown below. O
3π a/3
P 2
x

θ
O To find resultant of there phasors, we resolve there
along X and Y-axis. Sum of components of phasors
along X-axis,
mg π a a 3π
Σa x = a cos + cos π + cos
2 2 3 2
Solved Paper 2020 ●
19


a
Σa x = − …(i) Hence, correct options are (b) and (d).
2 37. (c)
Sum of components alongY-axis,
π a a 3π n
Σa y = a sin + sin π + sin
2 2 3 2 n P
2a
⇒ Σa y = …(ii) P
3 a
So, resultant amplitude,
b
a 2 4a 2
a net = ( Σa x )2 + ( Σa y )2 = +
4 9
25a 2 5a
= =
36 6
Phase φ of resultant phasor is given by
Σa y 2a / 3 4 Bound surface charge density and polarisation are
tan φ = = =−
Σa x − a / 2 3 related as
π  3 σb = n$ out ⋅ P
⇒ φ= + tan−1 
2  4 n$ out = normal outward vector
k
35. (a,c ) For a particle oscillating along X-axis, and P = 2 ⋅ r$
r
X
O
• At r = a, P is antiparallel to n. $
a k $ $ −k
So, (σb )r = a = 2 ⋅| r || n| cos 180° = 2
Speed is given by v = ω a 2 − x 2 a a
At r = b, P is parallel to n. $
Here, at x = 2 cm,v = 4 cm/s and at x = 6 cm,v = 2 cm/s.
k $ $ k
So, ( σb )r = b = 2 ⋅| r|| n | cos 0° = 2
So, we have 4 = ω a 2 − 22 …(i) b b
and 2 = ω a 2 − 62 …(ii) Also, volume charge density γb = − ∇ ⋅ P = 0.

On dividing Eq. (i) by Eq. (ii), we get 38. (a, c) For a mixture of gases, ratio of specific heats,
2 2  n1γ1 n γ 
a −2 + 2 2
2=  γ − 1 γ − 1 
γ mix =  1 2
a 2 − 62 n n2 
 1 + 
⇒ 4(a 2 − 62 ) = a 2 − 22  γ1 − 1 γ 2 − 1 
⇒ 3a 2 = 4 × 62 − 22 = 32 Here, γ1 = 16
. , γ 2 = 14. , n1 = 1 and n 2 = 1
140 140  1× 16 . 1× 14.  16 . 14
.
⇒ a2 = ⇒a = cm + +
 16
. − 1 14 . − 1  0.6
3 3 So, γ mix =  = 0.4 = 148
.
1 1  1 1
Substituting for a in Eq. (i), we get  +  +
 16
. − 1 14 . − 1  0.6 0.4
140 3
4 =ω − 4 ⇒ω = rad s−1 Cp (mix)
3 8 So, γ mix = = 148
.
CV (mix)
36. (b, d) From Maxwell’s equations, we have
div E = ∇ ⋅ E = 0 or Cp (mix) = 148
. CV (mix)
div B = ∇ ⋅ B = 0 Also, Cp (mix) − CV (mix) = R
− 1 ∂B ⇒ . CV (mix) − CV (mix) = R
148
Curl E = ∇ × E = ⋅
c ∂t ⇒ 0.48CV (mix) = R
R
⇒ CV mix = = 2 .08R
1 ∂E 0.48
Curl B = ∇ × B = ⋅
c ∂t 39. (a, b) In case of a relativistic collision, both linear
Also, from equation of Lorentz force, momentum and total energy are conserved. Kinetic
F = q( E + v × B) energy is not conserved as collision is inelastic.
∂ρ Total rest mass is not conserved as the particles are
Also, ∇ ⋅ J − ≠0
∂t moving with relativistic speeds.
20 IIT JAM Physics Solved Papers & Practice Sets

40. (d) From input/output waveforms, we can state truth tables So, line integral,
1
of given gates,
1 2
∫c u ⋅ dr = ∫0 [ 2( 4t )$i + ( 2t )$j] ⋅ ( 2$i + 4$j) ⋅ dt
3
A 1
5 6 7 1  24t 2 
B C = ∫ (16t + 8t )dt =   = 12
4
0
 2 0
t0 t1 t2 t3 t4 t5 t6 t7 42. (1.5)
R1
R2
1
C2 P C1
2 and 3

4 By lens maker’s formula, focal length of lens is given


1  1 1
5 and 6 by = (n − 1) − 
f  R1 R2 
Here, n = 1. 5, R1 = R
and R2 = − 2R
7 1 1 1  1  3
So, = (15
. − 1) − =  
Truth tables obtained from observation of waveforms f  R ( −2R ) 2R  2
are 3
⇒ R= f
(i) For gate A, Y1 Y2 4
0 1 So, magnitude of larger radius of curvature in units of
1 0
focal length,
From waveforms 1 and 2, we observe that gate A is 3
| R2| = 2R = 2 × f = 15 .f
working correctly. 4
(ii) For gate B, Y2 Y4 Y5 = 1. 5 units of focal length
1 0 1
43. (25.21) For a horizontal tube by Bernoulli’s theorem, we
0 0 0
0 1 1 have
1 1 1
1
p1
But above is truth table of an OR gate.
So, gate B is faulty and it is acting like an OR gate. 2
p2
Also, from waveform, Y3 is not identical asY7, so points v1 v2
3 and 7 are not shorted.
NOT gate output Y6 is complementry of Y7, so NOT gate
on the right is working correctly.
41. (12) Vector parametric equation of a line that starts at r0 1 2 1
p1 + ρv1 = p 2 + ρv 22
and ends at r1, 2 2
r(t ) = (1 − t )r0 + t ( r1) 1
⇒ ρ(v12 − v 22 ) = p 2 − p1
where, 0 ≤t ≤1 2
Here, r = 0 and r = 2$i + 4$j 2( p1 − p 2 )
0 1 ⇒ v 22 − v12 =
ρ
So, r(t ) = (1 − t )0 + t ( 2i$ + 4$j) = 2t $i + 4t $j
Here, p1 = 4 × 105 Pa, p 2 = 1× 105 Pa,
So, x = 2t and y = 4t .
Now, line integral of vector function u along the line r(t ) v1 = 6ms−1, ρ = 1000 kg m −3
is given by; Substituting the values, we get
1
2 × ( 4 × 105 − 1× 105 )
∫c u ⋅ d r = ∫ 0u [ r(t )] ⋅ r′ (t ) ⋅ dt v 22 − 36 =
1000
1 d 
= ∫ ( 2y $i + x $j) ⋅  ( 2t $i + 4t $j) dt ⇒ v 2 = 636 = 25.21ms−1
0  dt 
Here, x = 2t and y = 4t
Solved Paper 2020 ●
21

44. (33.33) Thickness of a half-wave plate (retarder) to convert Here, we have two spheres of radii R1 and R2 at
a left circularly polarised light into a right circularly temperature,
polarised light is given by T1 = 1000 K and T2 = 2000 K.
2 2
1 λ  R1 T2   2000 R1
t=   So, =   =  ⇒ =4
2  ne − no  R2  T1   1000  R2
where, ne = refractive index for extraordinary ray = 1.551 47. (0.548) Compton scattering is as shown below.
no = refractive index for ordinary ray = 1.542

and λ = light’s wavelength = 600 × 10−9 m.
hf φ
Substituting the values, we have –
λi=0.5A
1  600 × 10−9  1  600 × 10−9  hf '
t=  =  
2  1.551 − 1.542 2  0.009  λf
1 Compton displacement or change in wavelength of
= × 66.66 × 10−6 m = 33.33 µm
2 incident photon ( ∆λ = λ f − λ i ) is given by
λ
45. (15) Initially capacitor C1 = 10 pF is charged to a voltage ∆λ = (1 − cos φ )
m0c
V1 = 50V
10pF This is maximum when φ = 180°.
+ – 2h
+ – ⇒ ∆λ max = = 2λ c
+ – m0c
+ –
λ
where, = λ c = Compton wavelength.
m0c
+ – Here, λ i = 0.500 Å
+ –
h
+
+

– λc = = 0.024Å
C m0c
When this charged capacitor is connected to another So, we have ∆λ max = λ f − λ i
uncharged capacitor of capacity C2, both capacitors ⇒ 2λ c = λ f − λ i
reaches at some common potentialV after transfer of ⇒ 2( 0.024) = λ f − 0.500
charge from charged to uncharged capacitor is complete. ⇒ λ f = 0. 500 + 0.048 ⇒ λ f = 0. 548 Å
10pF
+ – 48. (0.57) For a FCC crystal, for plane (2,0,0) by Bragg’s
+ –
+ – equation, we have
+ –
a
λ = 2d( 2, 0, 0) sin θ = 2 × × sin θ
2
λ
+ – So, unit cell size is, a =
+
+


sin θ
+ – Here, λ = 0.2 nm and θ = 21°
C
0.2nm 0.2nm
So, a = = = 0.5714 nm
sin 21° 0.35
Total charge C1V1
Then, V = = or a = 0.57 nm
Total capacity C1 + C2
49. (97) As current flows through least resistance path, so
Here, V = 20 V, C1 = 10 × 10−12F, V1 = 50 V
given circuit is reduced to
10 × 10−12 × 50
So, 20 =
10 × 10−12 + C2 – + R=100kΩ
−12 −12
⇒ 10 × 10 + C2 = 2 . 5 × 10 V=10V
−12
⇒ C2 = 15 × 10 F = 15 pF 0.3V

46. (4) Radiation emitted per second by a black body is given As potential drop across diode is 0.3V, so potential
by P = AσT 4 drop across resistance,
1 VR = 10 − 0.3 = 9.7 V
For same power, A ∝ ∴Current through resistor R,
T4
V 9.7
If radius of sphere is R, then A ∝ R 2 ∝
1 1
or R ∝ 2 I= R = = 97 × 10−6 A or I = 97 µA
T4 T R 100 × 1000
22 IIT JAM Physics Solved Papers & Practice Sets

50. (6) UA=UD+16 J Hence, substituting these values in Eq. (i), we have
A
y = 6e −2x − 4e −3x …(v)
∆U=0 So, when x = 1, from Eq. (v), we have
UB=UA=UD+16 J
B y (1) = 6e −2 − 4e −3 = 0.81 − 0.20 = 0.61

∆U=0 ∆W=–∆U 52. (1.12) Power, P = Fv


E
 dv  t v
∆W=16J ⇒ P = m  v ⇒ ∫ Pdt = m ∫ vdv
D C  dt  0 0
UC=UD+16 –10
UD =UD+6 J 1
mv 2  2Pt  2
⇒ P = or v =  
2t  m 
Let the value of internal energy at D is UD . Then, as
process DE is isothermal compression, 1 1
ds  2Pt  2 s t  2Pt  2
∆UDE = 0 ⇒ =
dt  m 
 ⇒ ∫ 0 ds = ∫ 0  m  dt
⇒ Internal energy at E, 1
3
UE = UD + ∆UDE = UD  2P  2 2 2 9ms 2
⇒ s =  ⋅ (t ) ⇒P =
Now, in process EA, ∆UEA = 16 J m 3 8t 3
⇒ Internal energy at A, Here, s = 600 m, t = 1min = 60 s and m = 600 kg.
U A = UE + ∆UEA = UD + 16J Substituting these values in expression of power, we get
Now, process AB is isothermal, hence ∆U AB = 0, so 9 × 600 × ( 600)2
P = = 1125 W or P = 1.125 kW
internal energy at B, 8 × ( 60)3
UB = U A = UD + 16J 53. (2) Angular momentum of particle at time t,
Now, process BC is adiabatic expansion ( ∆QBC = 0), so L = 4x$ + αt 2y$ ; α = 1kg m 2s−3 ⇒ L = 4x$ + t 2y$
∆UBC = − ∆WBC = − 10J Torque on the particle,
Hence, internal energy at C, dL d
τ= = ( 4x$ + t 2y$ ) ⇒ τ = 2ty$
UC = UB + ∆UBC = (UD + 16) − 10 = UD + 6J dt dt
So, change in internal energy in process CD, Let at some instant t angle of L and τ is 45°, then
∆UCD = UC − UD = UD + 6 − UD = 6J L ⋅ τ = | L|| τ| cos 45°
51. (0.61) Given differential equation, 1
⇒ ( 4x$ + t 2y$ ) ⋅ ( 2ty$ ) = ( 16 + t 4 )( 4t 2 )
d 2y dy 2
+5 + 6y = 0
dx 2
dx ⇒ 2 2t 3 = ( 16 + t 4 )2t
2
We have, (D + 5D + 6)y = 0 ⇒ 2t 4 = 16 + t 4 ⇒ t 4 = 16 or t = 2 s
dy
where, D= 54. (2) Net displacement of a point on string is due to
dx
superposition of both waves,
So, auxillary equation,
y1 = 5 cos(kx − ωt ) and y 2 = 5 cos(kx + ωt )
m 2 + 5m + 6 = 0
is given by
⇒ (m + 2)(m + 3) = 0
y = y1 + y 2 = 5 cos(kx − ωt ) + 5 cos(kx + ωt )
⇒ m = − 2 or m = − 3
Hence, solution of above equation, x + y  x −y 
Using cos x + cos y = 2 cos  ⋅ cos  , we get
 2   2 
y = C1e −2x + C2e −3x …(i)
−2 x −3 x y = 10 cos kx ⋅ cosωt
Now, y ′ = − 2C1e − 3C2e …(ii)
Velocity of particle,
It is given, y = 2 when x = 0, so Eq. (i) gives
∂y
2 = C1 + C2 …(iii) v = ` = − 10ω cos kx ⋅ sin ωt
∂t
dy
and it is also given that = 0 at x = 0. 1
dx Now, it is given velocity is zero at t = 0 s, t = s
4
So, from Eq. (ii), we have 1
0 = − 2C1 − 3C2 …(iv) and at t= s.
2
On solving Eqs. (iii) and (iv), we have As velocity is either maximum or minimum at mid time
C1 = 6 and C2 = − 4 instances.
Solved Paper 2020 ●
23

∴ Velocity maxima (or minima) occurs at Entropy change of body,


1 3 Tf dT T 
t = s or at t = s ∆S1 = Cp ∫ = Cp ln f 
8 8 Ti T  Ti 
t=1/8 t=3/8
Here, Tf = 2T and Ti = T
t=0 t=1/4 t=1/2
 2T 
dv 1 = Cp ln  = Cp ln 2 = 0.69 Cp
Hence, = 0 at x = 0 and at t = s T 
dt 8
Entropy change of reservoir,
dv 2
Now, = − 10ω cos ωt = 0 T − Ti 
dt x = 0 ∆S 2 = − C p  f 
 Tf 
π 2π 1 π 1
⇒ ωt = ⇒ × = or = f = 2 Hz
2 T 8 2 T (Negative sign shows loss of heat)
 2T − T 
55. (159.23) For the given R - C circuit, the required input = − Cp   = − 0. 5Cp
 2T 
1
frequency is given as f =
2πRC So, total entropy change,
Here, R = 100kΩ = 100 × 103 Ω, ( ∆S )universe = ∆S1 + ∆S 2 = 0.69Cp − 0.5Cp = 0.19Cp
C = 10 pF = 10 × 10−12 F 58. (5.88) Gas undergoes following processes
Substituting these values in the above equation, we get 1 mole
1 monoatonic Isothermal T,
f = = 159.23 Hz gas expansion 4V
2 × 3.14 × 100 × 103 × 10 × 10−12 p, V, T (1)
56. (2.87) X
B2
n
B1
(Region 1) Y (Region 2) Isochoric
p
heating
(2)
Z In process 1 (isothermal expansion), heat absorbed by
system,
V   4V 
Q1 = RT loge  f  = RT loge  
 Vi  V 
= 2RT loge 2 = 138
. RT …(i)
The dielectric interface is XOY-plane, Z = 0. In process 2 (isochoric heating), heat absorbed by
As a discontinuity in the tangential component of the system,
magnetic field intensity at the boundary must be  R 
supported by surface current flowing in a direction Q 2 = nCV ∆T = n   ∆T …(ii)
 γ − 1
perpendicular to this component of field, we have
B 5
B  As gas is ideal monoatomic, γ = .
n$ ×  1 − 2  = Js …(i) 3
µ0 µ0 
Now, we calculate ∆T for this process as follows
Here, n$ = unit normal vector from medium 1 to medium 2, Isothermal Isochoric
Js = surface current density, n=1 expansion T, p2, heating p3= p,
p, V, T 4V T3
B1 and B2 are magnetic fields in region 1 and 2. (1) (2)
Here, B = 3$i + 5k$ ,
1
For process 1,
B2 = $i + 3$j + 5k$ and n$ = k$ p
pV = p 2 ⋅ 4V ⇒ p2 =
So, from Eq. (i), we have 4
1 $ 1 $ For process 2,
( k × ( 2$i − 3$j)) = Js ⇒ Js = ( 2 j + 3i$ )
µ0 µ0 p
1  p2 p3 p
⇒ | Js | = ( 2$j + 3i$ )=
13
= 2 .87 × 106 Am−1 = ⇒ 4 = ⇒T3 = 4T
−7
µ 0  4π × 10 T T3 T T3
Hence, for isochoric process,
57. (0.19) Assuming body is small compared to reservoir, final
temperature of body is 2T at thermal equilibrium. ∆T = 4T − T = 3T
24 IIT JAM Physics Solved Papers & Practice Sets

Now, by Eq. (ii), heat added in process 2, By Kirchhoff’s voltage rule, we have
 R  R 9 VCC − iBRB − VBE − iE RE =`0
Q2 = n   ∆T = 1 × × 3T = RT = 4. 5RT
 γ − 1 5  2 20 − iBRB − 0.7 − 2 × 10−3 × 1000 = 0
 − 1
3 
⇒ iBRB = 20 − 0.7 − 2
So, total heat added iC
As iB − , we have
= Q1 + Q 2 = 1. 38RT + 4. 50RT = 5.88RT β
59. (865) Consider the path shown (collector to emitter) RB = 17.3 × 100 / 2 × 10−3
VCC 20V ⇒ RB = 8.65 × 105 Ω = 865 kΩ
40
60. (3.60) Let N0 potassium K atoms are present at t = 0.
ic
Then, number of potassium atom at the time of
RB RL 2kΩ analysis,
NK = N0e −λt …(i)
IB C
β=100 where, λ = disintegration constant and t = age of rock.
E So, number of argon ( 40 Ar) atoms present at the time
of analysis,
RE 1kΩ NAr = N0 − NK …(ii)
iE From Eqs. (i) and (ii), we have
NAr = NK (e λt − 1)

By Kirchhoff’s voltage rule, we have 1  N 


⇒ t= ⋅ ln1 + Ar 
λ  NK 
VCC − iCRL − VBE − iE RE =`0
⇒ 20 − iC × 2000 − 14 − iE × 1000 = 0  N 
1 + Ar 
But given iE ≈ iC , so 20 − 14 = 3000iC  NK 
⇒ t = T1/ 2 ln
⇒ iC = iE = 2 × 10−3 A ln 2
9 NAr 1
Now, for path shown (collector-base-emitter), Here T1/ 2 = 125
. × 10 yr; =
VCC NK 50
 1
. × 109 × ln 1 + 
125
iB iC  50
⇒ t=
ln2
RB RL=2kΩ 9
. × 10 × 0.02
125
= = 0.036 × 109 yr
C
0.69
= 3.6 × 107 yr
B E
So, m = 3.60
iE

RE=1kΩ
Solved Paper 2019 ●
1

IIT JAM PHYSICS


SOLVED PAPER 2019
MM : 100 Time : 3 hrs

1. The examination is of 3 hours duration. There are a total of 60 questions carrying 100 marks. The entire paper is
divided into three sections, A, B and C. All sections are compulsory. Questions in each section are of different
types.
2. Section - A contains a total of 30 Multiple Choice questions (MCQ). Each MCQ type question has four
choices out of which only one choice is the correct answer. Question Q. 1- Q . 30 belong to this section and
carry a total of 50 marks. Q. 1- Q. 10 carry 1 mark each and Questions Q. 11 - Q. 30 carry 2 marks each.
3. Section - B contains a total of 10 Multiple Select Questions (MSQ). Each MSQ type question is similar to
MCQ but with a difference that there may be one or more than one choice (s) that are correct out of the four
given choices. The candidate gets full credit if he / she selects all the correct answers only and no wrong
answers. Questions Q. 31-Q.40 belong to this section and carry 2 marks each with a total of 20 marks.
4. Section - C contains a total of 20 Numerical Answer Type (NAT) questions. For these NAT type questions,
the answer is a real number which needs to be entered using the virtual keyboard on the monitor. No choices
will be shown for these type of questions. Questions Q.41-Q.60 belong to this section and carry a total of 30
marks. Q. 41- Q. 50 carry 1 mark each and Questions Q.51 - Q. 60 carry 2 marks each.
5. In all sections, questions not attempted will result in zero mark. In Section - A (MCQ), wrong answer will. result
in NEGATIVE marks. For all 1 mark questions, 1/3 marks will be deducted for each wrong answer. For all
2 marks questions, 2/3 marks will be deducted for each wrong answer. In Section - B (MSQ), there is NO
NEGATIVE and NO PARTIAL marking provisions. There is NO NEGATIVE marking in section - C (NAT) as well
6. Only Virtual Scientific Calculator is allowed. Charts, graph sheets, tables, cellular phone or other electronic
gadgets are NOT allowed in the examination hall.
7. The Scribble Pad will be provided for rough work.

Section A Multiple Choice Questions (MCQs)


In these type of questions, each question has four choices (a), (b), (c) and (d) out of which only
one option is correct.
Q.1-10 carry one mark each.
8x
1. The function f (x ) = 2
is continuous everywhere except at
x +9
(a) x = 0 (b) x = ± 9 (c) x = ± 9i (d) x = ± 3i

2. A classical particle has total energy E. The plot of U


potential energy U as a function of distance r from the
centre of force located at r = 0 is shown in the figure. I III
E
Which of the regions are forbidden for the particle ? IV
II
(a) I and II (b) II and IV
(c) I and IV (d) I and III
0 r
2 IIT JAM Physics Solved Papers & Practice Sets

235
3. In the thermal neutron induced fission of U, the distribution of relative number of the
observed fission fragments (Yield) versus mass number A is given by

Yield

Yield
Yield

Yield
(a) (b) (c) (d)

80 120 160 80 120 160 80 120 160 80 120 160


A A A A

4. Which one of the following crystallographic planes represent (101) miller indices of a cubic unit
cell ?
Z Z Z Z

(a) (b) (c) (d)


Y Y Y Y

X X X X

5. The Fermi-Dirac distribution function [n( ε)] is


( K B is the Boltzmann constant, T is the temperature and εF is the Fermi energy)
1 1 1 1
(a) n( ε ) = ε − εF (b) n( ε ) = εF − ε (c) n( ε ) = ε − εF (d) n( ε ) = εF − ε
e K BT
−1 e K BT
−1 e K BT
+1 e K BT
+1

6. If φ(x, y ,z ) is a scalar function which satisfies the Laplace equation, then the gradient of φ is
(a) solenoidal and irrotational (b) solenoidal but not irrotational
(c) irrotational but not solenoidal (d) Neither solenoidal nor irrotational

7. In a heat engine based on the Carnot cycle, heat is added to the working substance at constant
(a) entropy (b) pressure (c) temperature (d) volume

8. Isothermal compressibility is given by


1  δV  1  δp  1  δV  1  δp 
(a)   (b)   (c) −   (d) −  
V  δp  T p  δV  T V  δp  T p  δV  T

9. For using a transistor as an amplifier, choose the correct option regarding the resistances of
base-emitter (R BE ) and base-collector (R BC ) junctions
(a) Both RBE and RBC are very low (b) Very low RBE and very high RBC
(c) Very high RBE and very low RBC (d) Both RBE and RBC are very high

10. A unit vector perpendicular to the plane containing A = i$ + $j − 2 k$ and B = 2$i − $j + k$ is


1 1 1 1
(a) ( − $i + 3$j − 4k$ ) (b) ( − $i + 3$j − 3k$ ) (c) ( − $i + 5$j − 3k$ ) (d) ( − $i − 5$j − 3k$ )
26 19 35 35

Q. 11- 30 Carry two marks each.


3 4
11. A thin lens of refractive index is kept inside a liquid of refractive index . If the focal length of
2 3
the lens in air is 10 cm, then its focal length inside the liquid is
(a) 10 cm (b) 30 cm (c) 40 cm (d) 50 cm
Solved Paper 2019 ●
3

3 i 0
 
12. The eigenvalues of  −i 3 0 are
0 0 6

(a) 2, 4 and 6 (b) 2i, 4i and 6 (c) 2i, 4 and 8 (d) 0, 4 and 8

13. For a quantum particle confined inside a cubic box of side L, the ground state energy is given
by E 0 . The energy of the first excited state is
(a) 2 E0 (b) 2 E0 (c) 3E0 (d) 6 E0

14. A small spherical ball having charge q and mass m, is tied to a thin
massless non-conducting string of length l. The other end of the string is
fixed to an infinitely extended thin
non-conducting sheet with uniform surface charge density σ. Under
equilibrium, the string makes an angle 45° with the sheet as shown in the 45° l
figure. Then σ is given by (g is the acceleration due to gravity and ε 0 is the
σ q
permittivity of free space)
mgε 0 mgε 0 m
(a) (b) 2
q q
mgε 0 mgε 0
(c) 2 (d)
q q 2

15. Consider the normal incidence of a plane v1


x
Medium 1 v2
electromagnetic wave with electric field given k1 k2
Medium 2
by E = E 0 exp [i (k1z − ωt )]x$ over an interface
at z = 0 separating two media (wave velocities
v 1 and v 2 (v 2 > v 1 ) and wave vectors k1 and k 2 , O z
respectively) as shown in figure. The magnetic
field vector of the reflected wave is (ω is the y
angular frequency)
E0 E0
(a) exp [ i ( k1z − ωt )]y$ (b) exp [ i ( − k1z − ωt )]y$
v1 v1
E0 −E0
(c) − exp [ i ( − k1z − ωt )]y$ (d) exp [ i ( k1z − ωt )]y$
v1 v1

16. The output of following logic circuit can be simplified to


X
Y

(a) X + YZ (b) Y + XZ (c) XYZ (d) X + Y + Z


4 IIT JAM Physics Solved Papers & Practice Sets

17. A red star having radius rR at a temperature TR and a white star having radius rW at a
temperature TW , radiate the same total power. If these stars radiate as perfect black bodies,
then
(a) rR > rW and TR > TW (b) rR < rW and TR > TW (c) rR > rW and TR < TW (d) rR < rW and TR < TW

18. The mass per unit length of a rod (length 2 m) varies as ρ = 3x kg/m. The moment of inertia
(in kg-m 2 ) of the rod about a perpendicular axis passing through the tip of the rod (at x = 0) is
(a) 10 (b) 12 (c) 14 (d) 16

19. For a forward biased p-n junction diode, which one of the following energy band diagrams is
correct? (εF is the Fermi energy)
Conduction Band Conduction Band
p-type n-type p-type n-type
Electron
Energy

εF(p)

Electron
εF(n) εF

Energy
(a) (b)
Valance Band
Valance Band

Conduction Band Conduction Band


p-type n-type p-type n-type
εF(p) εF(p)

Electron
Electron

Energy
Energy

εF(n)
(c) (d) εF(n)
Valance Band Valance Band

c 2c
20. The amount of work done to increase the speed of an electron from to is
3 3
(c = 3 × 10 8 m/s and rest mass of electron is 0.511 MeV)
(a) 56.50 keV (b) 143.58 keV
(c) 168.20 keV (d) 511.00 keV

21. The location of Cs + and Cl − ions inside the unit cell of z

CsCl crystal is shown in the figure.


The Bravais lattice of CsCl is Cl–
(a) simple cubic
(b) body centered orthorhombic a y Cs+

(c) face centered cubic a


a
(d) base centered orthorhombic x

22. A γ-ray photon emitted from a 137 Cs source collides


with an electron at rest. If the Compton shift of the photon is 3.25 × 10 −13 m, then the
scattering angle is closest to
(Planck’s constant h = 6.626 × 10 −34 J-s, electron mass me = 9.109 × 10 −31 kg and velocity of
light in free space c = 3 × 10 8 m/s)
(a) 45° (b) 60° (c) 30° (d) 90°

23. During free expansion of an ideal gas under adiabatic condition, the internal energy of the gas
(a) decreases (b) initially decreases and then increases
(c) increases (d) remains constant
Solved Paper 2019 ●
5

24. In the given phase diagram for a pure substance, regions I, II, III, IV, respectively represent

P
Critical
Point
III

IV
II I

T
(a) vapour, gas, solid, liquid (b) gas, vapour, liquid, solid
(c) gas, liquid, vapour, solid (d) vapour, gas, liquid, solid

25. Light of wavelength λ (in free space) propagates through a dispersive medium with refractive
index n( λ ) = 1.5 + 0 .6 λ . The group velocity of a wave travelling inside this medium in units of
10 8 m/s is
(a) 1.5 (b) 2.0 (c) 3.0 (d) 4.0

26. The maximum number of intensity minima that can be observed in the Fraunhofer diffraction
pattern of a single slit (width 10 µm) illuminated by a laser beam (wavelength 0.630 µm) will be
(a) 4 (b) 7 (c) 12 (d) 15

27. During the charging of a capacitor C in a series RC circuit, the typical variations in the
magnitude of the charge q (t ) deposited on one of the capacitor plates and the current i (t ) in the
circuit, respectively are best represented by

q i q i

0 t 0 t 0 t 0 t
I II III IV
(a) I and II (b) I and IV (c) III and II (d) III and IV

28. Which one of the following is an impossible magnetic field B ?


 z3 
(a) B = 3 x 2 z 2x$ − 2 xz 3z$ (b) B = − 2 xyx$ + yz 2y$ + 2 yz −  z$
 3
 z2 
(c) B = ( xz + 4 y )x$ − yx 3y$ +  x 3 z −  z$ (d) B = − 6 xzx$ + 3 yz 2y$
 2

29. If the motion of a particle is described by x = 5 cos 8 πt, y = 5 sin 8 πt and z = 5t , then the
trajectory of the particle is
(a) circular (b) elliptical (c) helical (d) spiral

30. A ball of mass m is falling freely under gravity through a viscous medium in which the drag force
is proportional to the instantaneous velocity v of the ball. Neglecting the buoyancy force of the
medium, which one of the following fig best describes the variation of v as a function of time t ?
v v v v
(a) (b)
(c) (d)

t t 0 t 0 t
0 0
6 IIT JAM Physics Solved Papers & Practice Sets

Section B Multiple Select Questions (MSQs)


In these type of questions, each question has four choices (a), (b), (c) and (d) out of which only
one or more than one is/are correct options.

Q. 31-40 carry two marks each.

. A1/ 3 fm,
31. The relation between the nuclear radius R and the mass number A, given by R = 12
implies that
(a) the central density of nuclei is independent of A
(b) the volume energy per nucleon is a constant
(c) the attractive part of the nuclear force has a long range
(d) the nuclear force is charge dependent

32. Consider an object moving with a velocity v in a frame which rotates with a constant angular
velocity ω. The Coriolis force experienced by the object is
(a) along ν (b) along ω
(c) perpendicular to both ν and ω (d) always directed towards the axis of rotation.

33. The gradient of a scalar field S(x, y ,z ) has the following characteristics.
(a) Line integral of a gradient is path-independent
(b) Closed line integral of a gradient is zero
(c) Gradient of S is a measure of the maximum rate of change in the field S
(d) Gradient of S is a scalar quantity

34. A thermodynamic system is described by the p ,V ,T coordinates. Choose the valid expression
(s) for the system,
 δp   δV   δp   δp   δV   δp 
(a)     = −   (b)     =  
 δV  T  δT  p  δT  V  δV  T  δT  p  δT  V

 δp   δT   δV   δV   δT   δV 
(c)     = −   (d)     =  
 δV  p  δp  V  δp  T  δT  p  δp  V  δp  T

35. Which of the following statement(s) is/are true?


(a) Newton’s laws of motion and Maxwell’s equations are both invariant under Lorentz
transformations.
(b) Newton’s laws of motion and Maxwell’s equations are both invariant under Galilean
transformations.
(c) Newton’s laws of motion are invariant under Galilean transformations and Maxwell’s equations
are invariant under Lorentz transformations.
(d) Newton’s laws of motion are invariant under Lorentz transformations and Maxwell’s equations
are invariant under Galilean transformations.

36. For an underdamped harmonic oscillator with velocity v (t ),


(a) rate of energy dissipation varies linearly with v (t )
(b) rate of energy dissipation varies as square of v (t )
(c) the reduction in the oscillator frequency, compared to the undamped case, is independent of
v (t )
(d) for weak damping, the amplitude decays exponentially to zero
Solved Paper 2019 ●
7

37. Out of the following statements, choose the correct option about a perfect conductor
(a) The conductor has an equipotential surface
(b) Net charge, if any resides only on the surface of conductor
(c) Electric field cannot exist inside the conductor
(d) Just outside the conductor, the electric field is always perpendicular to its surface

38. In the X-rays diffraction pattern recorded for a simple cubic solid (lattice parameter a = 1 Å)
using X-rays of wavelength 1 Å, the first order diffraction peak would appear for the
(a) (100) planes (b) (112) planes (c) (210) planes (d) (220) planes

39. Consider a classical particle subjected to an attractive inverse square force field. The total
energy of the particle is E and the eccentricity is ε. The particle will follow a parabolic orbit, if
(a) E > 0 and ε = 1 (b) E < 0 and ε < 1
(c) E = 0 and ε = 1 (d) E < 0 and ε = 1

40. An atomic nucleus X with half-life TX decays to a nucleus Y, which has half-life TY . The
condition(s) for secular equilibrium is/are
− TY
(a) TX ~ (b) TX < TY
(c) TX << TY (d) TX >> TY

Section C Numerical Answer Type Questions (NATQs)


This section contains Numerical Answer Type (NAT) questions, for these NAT questions, the
answer is a real number.

Q.41-50 carry one mark each.

41. In a typical human body, the amount of radioactive K is 3.24 × 10 −5 percent of its mass. The
40

activity due to 40 K in a human body of mass 70 kg is ……… kBq.


(Round off to 2 decimal places) (Half - life of 40 K= 3.942 × 1016 s, Avogadro’s number
N A = 6.022 × 10 23 mol −1)

42. Sodium (Na) exhibits body-centered cubic (BCC) crystal structure with atomic radius 0.186 nm.
The lattice parameter of Na unit cell is ……… nm. (Round off to 2 decimal places)

43. Light of wavelength 680 nm is incident normally on a diffraction grating having 4000 lines / cm.
The diffraction angle (in degrees) corresponding to the third order maximum is____.
(Round off to 2 decimal places)

44. Two gases having molecular diameters D1 and D 2 and mean free paths λ 1 and λ 2 ,
respectively are trapped separately in identical containers.
λ
If D 2 = 2D1, then 1 = _____.
λ2
(Assume there is not change in other thermodynamic parameters).

45. An object of 2 cm height is placed at a distance of 30 cm in front of a concave mirror with


radius of curvature 40 cm. The height of the image is …… cm.
8 IIT JAM Physics Solved Papers & Practice Sets

46. The flux of the function F = ( y 2 )x$ + ( 3xy − z 2 )y$ + ( 4 yz )z$ passing through the surface ABCD
along n$ is ____. (Round off to 2 decimal places)
z
1 C
D
n
(0,0,0) 1 y
B
1
A
x

1
47. The electrostatic energy (in units of J) of a uniformly charged spherical shell of total
4 πε 0
charge 5 C and radius 4 m is____. (Round off to 2 decimal places)

48. An infinitely long very thin straight wire carries uniform line charge density 8 π × 10 −2 C/m. The
magnitude of electric displacement vector at a point located 20 mm away from the axis of the
wire is ……… C/m 2 .

49. The 7th bright fringe in the Young’s double slit experiment using a light of wavelength 550 nm
shifts to the central maxima after covering the two slits with two sheets of different refractive
indices n1 and n 2 but having same thickness 6 µm. The value of | n1 − n 2 | is _____ .
(Round off to 2 decimal places)

50. For the input voltage Vi = (200mV) sin 400t, the amplitude of the output voltage (V 0 ) of the
given OPAMP circuit is ……… V. (Round off to 2 decimal places)
35kΩ 35kΩ
Rf Rf Rf
10kΩ 10kΩ
– – 10kΩ
R1 + + –
R2 R3 +
V0
Vi

Q. 51 - 60 Carry two marks each.

51. The value of emitter current in the given circuit is ……… µA.
(Round off to 1 decimal places)
+VCC=10V

2 MΩ 4 kΩ

C
B
β=100
E
0.3 V

2 kΩ +
–CE
Solved Paper 2019 ●
9

3 +i 2

52. The value of ∫ (z )dz , along the line 3y = x, where z = x + iy is_____.


0

(Round off to 1 decimal place)

53. If the wavelength of Kα 2 X- ray line of an element is 1.544Å, then the atomic number ( Z ) of
the element is_____ (Rydberg constant R = 1097
. × 10 7 m −1 and velocity of light c = 3 × 10 8
m/s)

54. A proton is confined within a nucleus of size 10 −13 cm. The uncertainty in its velocity is
___ ×10 8 m/s. (Round off to 2 decimal places)
(Planck’s constant h = 6.626 × 10 −34 J-s and proton of mass m p = 1672
. × 10 −27 kg)

2 π 
55. Given the wave function of a particle ψ(x ) = sin  x for 0 < x < L and 0 elsewhere, the
L L 
L
probability of finding the particle between x = 0 and x = is___. (Round off to 1 decimal place)
2

56. The Zener current I Z for the given circuit is ……… mA.
+ VR –
R=10kΩ IZ
RL=20kΩ
VZ=20V
Vin=40V

VL

57. If the diameter of the Earth is increased by 4% without changing the mass, then the length of
the day is ____ hours.
(Take the length of the day before the increment as 24 hours. Assume the earth to be a
sphere with uniform density).
(Round off to 2 decimal places)
58. A diatomic gas undergoes adiabatic expansion against the piston of a cylinder. As a result, the
temperature of the gas drops from 1150 K to 400 K. The number of moles of the gas required
to obtain 2300 J of the work from the expansion is ____.
(The gas constant R = 8.314 J mol −1 K −1).
(Round off to 2 decimal places)
59. The decimal equivalent of the binary number 110.101 is ____.
60. A surface current K = 100x$ A/m flows on the surface z = 0, which separates two media with
magnetic permeabilities µ 1 and µ 2 as shown in the figure. If the magnetic field in the region 1
is B1 = 4 x$ − 6 y$ + 2z$ mT, then the magnitude of the normal component of B2 will be_____mT.
z>0
µ1=5×10–6H/m
B1=4x–6y+2z mT
K=100x A/m
x
z=0

B2
n
µ2=10×10–6H/m
z<0
Answers with Solutions
1. (d) Given function, 6. (a) Function φ(x , y , z ) is satisfies the Laplace equation,
8x ∇ 2 φ(x , y , z ) = 0
f (x ) = …(i)
x2 + 9 For irrotation, ∇×p=0
1 Here, p = ∇φ
A function will not be continuous where its value is .
0 Hence, ∇ × ( ∇φ) = 0
On putting all values in Eq. (i), we get
For solenoidal, ∇⋅ p = 0
at x = 0, f (x ) = 0
Hence, ∇ ⋅ ( ∇φ) = ∇ 2 φ = 0
70i
at x = ± 9, f (x ) = Hence, function φ(x , y , z ) is a solenoidal and irrotational
90
function.
at x = ± 9i , f (x ) = − 1
24i 7. (c) A Carnot cycle involve two adiabatic and two isothermal
at x = ± 3i , f (x ) = =∞
0 processes.
So, given function is non-continuous at x = ± 3i . No heat transfers in adiabatic process, so heat is only added
to working substance in isothermal compression in which
2. (d) Regions I and III are forbidden as for these regions temperature remains constant.
PE(U ) > TE(E ) , which is not possible. T = constant
3. (a) % yield of Tc 99 is 6.13% and of Cs135 is 6.9%. 8. (c) Isothermal compressibility is a measure of relative volume
Tc( A = 99) Cs( A = 135 ) change of a fluid or solid as a response to a pressure
So, yield of Tc and Cs are almost equal and also maximum in change at constant temperature.
all of the fragments. So, correct depiction of yield versus It is given by
mass number curve. 1  ∂V 
K =−  
Yield V  ∂p  T
9. (b) Transistor works as an amplifier in active mode in which BE
junction is forward bias, (i.e. low R BE) and BC junction is
reverse bias, (i.e. high R BC).
10. (d) Given, A = $i + $j − 2k$ , B = 2$i − $j + k$
Unit vector perpendicular to both A and B is given by
99 A
80 TC 120 Cs 135 160 A ×B
n$ = …(i)
| A × B|
4. (b) Given, Miller indices = 101
$i $j k$ 
Let intercept length of plane on X , Y and Z -axis are a , b, c .
A × B = 1 1 −2
1 1 1 
 

Then, a = = 1, b = = ∞, c = = 1 2 −1 1
1 0 1  
Then corresponding plane would be = $i(1 − 2) − $j(1 + 4 ) + k$ ( −1 − 2)
Z A × B = − $i − 5 $j − 3 k$ …(ii)
L Magnitude of A × B,
c | A × B| = ( −1) 2 + ( −5 ) 2 + ( −3 ) 2 = 35 …(iii)
On putting values from (ii) and (iii) to Eq. (i), we get
− $i − 5 $j − 3 k$
n$ =
Y 35
a 1
3
11. (c) Refractive index of lens of glass, ng =
1 b 2
X
4
5. (c) Fermi-Dirac statistics describe a distribution of particle over Refractive index of liquid, n l =
3
energy state in systems consisting of many identical
Refractive index of air, na = 1
particles that obey the Pauli exclusion principle.
By lens Maker’s formula focal length in air
This function is given by
1 1  ng  1 1
n
n( ε) = i = = − 1  − 
gi εF f a  na   R1 R 2 
e αe K B T + 1 1 3/2  1 1
εF 1 = − 1  − 
Since, α =− ⇒ n( ε) = ε − ε fa  1   R1 R 2 
KBT F

e
K BT
+1 Given, f a = 10 cm
Here, εF = Fermi energy, KB = Boltzmann constant  1 1 1
So,  −  =
andT = temperature.  R1 R 2  5
Solved Paper 2019 ● 11

Now focal length in liquid, Electric field due to infinite non-conducting sheet,
1  ng  1 1  3/2   1 σ
= − 1  − = − 1   E =
fl  l
n  1
R R 2 4/3 5 2ε0
1 1 1 1 Here, σ = surface charge density.
= × = ⇒ f l = 40 cm So, electric force on point charge q is.
fl 8 5 40

 3 $i 0 Fe = qE = and Fg = mg
2ε0
12. (a) Given matrix =  − $i 3 0
As particle is in equilibrium, so from the figure
 0 0 6
T cos 45° = mg andT sin 45° = qE
For eigenvalues | A − λI | = 0  σ 
q 
Here, λ = eigenvalues, A = matrix (given) I = unit matrix T sin 45 ° qE  2ε0 
Now, = =
 3 $i 0  1 0 0 T cos 45 ° mg mg
So,  $i 3 0 − λ  0 1 0 = 0 qσ 2ε0mg
tan 45 ° = ⇒ σ =
 0 0 6
  
 2ε0mg
   0 0 1 q

3 − λ $i 0  15. (c) Given, electromagnetic wave with electric field,
⇒  − $i 3−λ 0 = 0 E = E 0 exp [i (k1 z − ωt )] x$

 
 Now electric field equation of reflected wave,
0 0 6− λ
 
Er = − E 0 exp[i (k1 ( − z ) − ωt )] x$
⇒ 3 − λ[( 3 − λ )( 6 − λ ) − 0] − $i[ − $i( 6 − λ) − 0] + 0 = 0
Er = − E 0 exp[i ( −k1 z − ωt )] x$
⇒ [( 6 − λ )( 3 − λ ) 2 − ( 6 − λ )] = 0 As we know, magnetic field,
⇒ ( 6 − λ )[( 3 − λ ) 2 − 1] = 0 E K ×E
Br = r and direction of B =
⇒ λ = 6 and ( 3 − λ ) 2 − 1 = 0 v ω
λ2 − 6λ + 8 = 0 ⇒ which is y-direction
⇒ ( λ − 4 )( λ − 2) = 0 E
So, B = − 0 exp[i ( −k1 z − ωt )] y$
⇒ λ = 4 and λ = 2 v1

So, λ = 2, 4, 6 16. (b) In the given logic circuit,


X XY
13. (a) Energy of quantum particle confined in a box of side L is
Y
given by
Y+Z Total
h2
E nx , n y , n z = (nx2 + ny2 + nz2 ) Z XY+(Y+Z)X+(Y+Z)Y
8mL2 (Y+Z)X
For ground state (nx , ny , n z ) = (1, 1, 1)
h2 3h 2
E0 = 2
(1 + 1 + 1) = …(i)
8mL 8mL2 (Y+Z)Y
For 1st excited state nx , ny , nz could be (2, 1, 1)
Output = XY + X (Y + Z ) + Y (Y + Z )
or (1, 2, 1) or (1, 1, 2). So, putting any value
= XY + XY + XZ + Y + YZ
h2
E1 = ( 4 + 1 + 1) = XY + XZ + Y QY + YZ = Y
8mL2
⇒ E1 = 6h / 8mL 2 2 = XZ + XY + Y QY + XY = Y
From Eq. (i), we get = XZ + Y
E1 = 2E 0 17. (c) Hot star emits white light and cooler star emits red light.
14. (c) Hence, Tw > TR
σ +
By Stefan’s law
+
+ Power radiated, P = A σT4
+ +
+ +
+ where, A = area, σ = Stefan’s constant
+ +
+
+ + T = temperature
+ +
+
45°+ T cos 45° As A ∝r2
+
+ + ++ l Where, r = radius
+
+ ++ + T Hence, P ∝ r 2T 4
+ ++
+ + + According to the question, total power of stars is same.
+ + q
+ ++ m PW = PR
Fe=qE
+ T sin 45°
+ or rW2TW4 = rR2TR4
Hence, rR > rW (QTW > TR )
Fg=Mg
From above result option (c) is correct.
12 IIT JAM Physics Solved Papers & Practice Sets

Put the value of v1 and v 2 , we get


18. (b)
l=2m  
 1 1 
= m 0c 2  − 
dx  1− 4 1−
1
 9 9 
x
 3 3 
Consider an element of length dx at a distance x from one = m 0c 2 −
 5 8 
end. 
Let its mass is dm. Given, m 0c 2 = 0511
. MeV
Then MI of element,  1 1 
= 3 × 0511
. MeV −
dI = dm ⋅ x 2  2.236 2.828 
kg = 01433
. MeV = 143.3 keV
As density (given), ρ = 3x
m W ~
− 143.58 keV
So, dm = ρ ⋅ dx = 3x ⋅ dx
21. (a) In the given cubic of CsCl x = y = z = a and α = β = γ = 90°
dI = ( 3x ⋅ dx ) ⋅ x 2 = 3x 3 ⋅ dx
The given cubic look like body centered but it is not.
Now, MI of entire rod,
l =2 Because for body centered atom in the center should be
I = ∫ 3x ⋅ dx
2
identical to the atoms at the corners, so it is just ‘‘simple
l =0 cubic’’.
2
x 4  22. (c) Compton shift (i.e. difference in wavelength of incident and
I = 3 
 4 0 outgoing photon) is given by
h
I = 12 kg-m 2 ∆λ = (1 − cos θ)
me c
p-type
n-type θ = scattering angle
19. (a) Conduction Band Planck constant,
Electron

εF(n)
Energy

εF(p) h = 6.626 × 10−34 J


me = 9109
. × 10−31 kg
Given, ∆λ = 3.25 × 10−13 m
Valance Band 6.626 × 10−34
3.25 × 10−13 = (1 − cos θ)
9109
. × 10−31 × 3 × 108
When a p-n junction is forward biased, the electrons in the
n-type, which have been elevated to the conduction band and 1 − cos θ = 01340
.
which have diffused themselves across the junction at a cos θ = 0.865
higher energy than holes in p-type material. These electron θ = cos −1 ( 0.865 )
readily combine with those holes, making possible a or θ = 30°
continuous forward current through junction. 23. (d) From Eqs.,
20. (b) Given rest mass of e − , m 0c 2 = 0511
. MeV ∆Q = ∆U + ∆W
and speed of light, c = 3 × 108 m/s In adiabatic process, ∆Q = 0
By work energy theorem, So, ∆U = − ∆W
Work done = Change in KE In free expansion, no work is done by the system.
W = K 2 − K1 …(i) So, ∆W = 0
c 2c and change in internal energy, ∆U = 0
Let moving masses of electron at speed v1 = and v 2 =
3 3 i.e. U = constant
are m1 and m 2 . Note Free expansion of gas occurs against the vacuum, so
Then, K1 = (m1 − m 0 )c 2 no force is exert by the gas in expansion and no work is done.
K 2 = (m 2 − m 0 )c 2 24. (b) p
So, K 2 − K1 = (m 2 − m1 )c 2 Critical
Liquid Point
Here moving mass,
me

III
ltin

m0 m0 on Gas
m1 = ⇒ m2 = Solid a ti
g

ris I
v12 v2 IV po
1− 2 1 − 22 Va
c c n
tio II
ma
Put these values in Eq. (i), we get bli Vapour
Su
  T
 1 1  Pure substance whose chemical composition does not
W = m 0c 2  − 
 change during phase change.
v12 v 
2

 1 − 1 − 22  In above diagram regions I, II, III and IV represent gas,


 c2 c 
vapour, liquid and solid respectively.
Solved Paper 2019 ● 13

25. (b) Given, 28. (d) According to Maxwell’s 2nd equation of electromagnetism
Refractive index of dispersive medium ∇ ⋅ B = 0 (for possible magnetic field)
n( λ ) = 15
. + 0.6λ …(i) Now, checking the options
Group velocity, vg = ...... × 108 m/s (a) B = 3x 2 z 2 x$ − 2xz 3 z$
Group velocity (vg ) and refractive index [n( λ )] is related as ∂ ∂ ∂
∇⋅B = ( 3x 2 z 2 ) + ( 0) + ( −2xz 3 )
c ∂x ∂y ∂z
vg =
n( λ ) − λ
dn ∇ ⋅ B = 6xz 2 − 6xz 2 = 0
dλ  z3  $
(b) B = − 2xyx$ + yz 2 y$ +  2yz − z
From Eq. (i), we get  3
dn
= 0 + 0.6 = 0.6 ∂ ∂ ∂  z3 
dλ ∇⋅B = ( −2xy ) + ( yz 2 ) +  2yz − 
∂x ∂y ∂z  3
c
vg =
. + 0.6λ − λ( 0.6)
15 ∇ ⋅ B = − 2y + z 2 + 2y − z 2 = 0
3 × 108  z2  $
= (c) B = (xz + 4 y )x$ − yx 3 y$ +  x 3 z − z
.
15  2
vg = 2 × 108 m/s ∂ ∂ ∂  3 z2 
∇⋅B = (xz + 4 y ) + ( − yx 3 ) + x z − 
26. (d) For diffraction, ∂x ∂y ∂z  2
a sinθ = nλ ∇⋅B = z − x3 − x3 − z = 0
Given, width of single slit, a = 10 µm (d) B = − 6xzx$ + 3 yz 2 y$
Wavelength of light, λ = 0.630 µm ∂ ∂
∇⋅B = ( −6xz ) + ( 3 yz 2 )
For maximum number of intensity minima, ∂x ∂y
θ = 90° {sin 90° = 1} ∇ ⋅ B = − 6z + 3 z 2 ≠ 0
So, a = nλ Hence, option (d) represents the impossible magnetic field.
a 10 × 10−6
n= = 29. (c) Given,
λ 0.630 × 10−6
x = 5 cos 8 πt …(i)
n = 15.87 y = 5 sin 8 πt …(ii)
n~− 15 z = 5t …(iii)
27. (a) During the charging of a capacitor in RC circuit equation of From Eqs. (i) and (ii), we get
charge and current is given by x 2 + y 2 = 5(cos 2 8 πt + sin2 8 πt )
(i) Charge at time t, x2 + y2 = 5 (Q sin2 θ + cos 2 θ = 1)
q (t ) = q 0 (1 − e − t /RC )
at t = 0, q = 0 (i) This equation represents a circular motion.
at t = ∞, q = q 0 (ii) And Z = 5t represents straight line motion with
So, correct q-t graph is constant velocity.
q Now, combination of circular and straight line motion is
‘‘helical’’ motion.
q0 30. (d) A ball of mass m falling freely under gravity in a viscous
medium, this resistive viscous force is directly proportional
to instantaneous velocity v of ball (Fv ∝ ν).
So, net force F = Fg − Fv

kv
t
(ii) Current in circuit, i = i 0e − t /RC
At t = 0, i = i 0 and at t = ∞, i = 0 m
i

i0
mg
⇒ F = mg − kv {k is constant}
dv
⇒ m⋅ = mg − kv
dt
dv
⇒ m + kv = mg
dt
t dv k
⇒ + v =g
dt m
14 IIT JAM Physics Solved Papers & Practice Sets

31. (a, b) Given, 34. (a, c) From the property of partial derivatives.
R = 12. A1 / 3 fm  δA   δB   δC 
⇒       = −1
Density of nucleus,  δB  C  δC  A  δA  B
Mass A amu  δA   δB  1  δA 
d = = ⇒
Volume 4     =− = − 
πR 3  δB  C  δC  A  δC   δC  B
3  
 δA  B
A amu
d = So, comparing with this property option (a) and (c) are
4
π(12
. A1 / 3 ) 3 correct.
3
 δp   δV   δp 
On solving, d = 2.7 × 1017 kg/m (constant so independent of A)     = −  option (a)
 δV  T  δT  p  δT  V
(option a)
Also, due to the property that a given nucleon influences only  δV   δT   δV 
    = −  option (c)
nucleons which are close to it (called property of saturation of  δT  p  δp  V  δp  T
nuclear force), volume energy per nucleon is a constant.
35. (c) As Maxwell’s equations are invarient in lorentz
(option b) transformation but not invarient in gallilian transformation
Note Nuclear force has short range and charge independent. thats why Newton’s laws of motion are invarient in gallilian
32. (c) Coriolis force It is a force acting on a rotating particle, but not in lorentz transformation.
perpendicular to both motion of the particle and axis of 36. (b, c, d) (a) is wrong.
rotation, i.e. coriolis force is perpendicular to both v and ω (b) is right (energy change’s rate in underdamped oscillation
It is given by dE
is given by = –bv 2 , Here, ‘b’ is the damping constant.
F = 2m(ω × v ) dt
Note object deflects to the right in northern hemisphere and (c) is right
to the left in southern hemisphere because of coriolis force In case of underdamped (simple) oscillations :
due to the rotation of earth. Decrease in frequency, ∆ω = 0
33. (a, b, c) Decrease in frequency, ∆ω = ω 20 − y 2
Given scalar field S (x , y , z ) = S b
(Here, y = = damping constant)
∇S = gradient S m
Option (a) So, decrease in frequency does not depend upon the
Line integral of gradient is speed v (t ).
→ →
(d) If right amplitude in weak damping is given by
dS = ∫ ∇S dl A(t ) = A 0e − αt (Here ‘α’ is a constant)
S+∆S and position of the oscillator is given by
x (t ) = A 0e − αt cos(ωt + φ)
37. (a, b, c, d)
S (a) Surface of a conductor is equipotential, because no work
is done in moving a charge on surface.
It only depends on initial and final position (correct).
(b) Charge reside on surface because conduction electron try
Option (b)
to get maximum separation between them due to
As it is path independent then close integral will be zero, i.e.
repulsion.
∫ ∇S ⋅ dl = 0 (correct) (c) Electric field inside conductor is zero because all extra
Option (c) charges reside on surface and charge inclosed by
→ → Σq
As, dS = ∇S .dl = ∇Sdl cosθ gaussian surface is zero. ∫ E ⋅ ds =
ε0
For cosθ = 1(maximum)
as Σq = 0, so E = 0
So, |dS |max = ∇Sdl
(d) As surface is equipotential also on surface charge is
dS static, so there is no tangential component of electric field
= ∇S
dl max on surface.
dS
Clearly, here  = ∇S (correct) 38. (a) Simple cubic solid is given for which lattice parameter,
 dl max a = 1Å
Option (d) Wavelength of X-ray,
Gradient S can never be scalar quantity because gradient or λ = 1Å
∂ $ ∂ $ Geometrical structure factor
∇= i+ j is vector and S is scalar.
∂x ∂y 2πi (u $ h + u $ k + w $ l )
s = Σe j j j
…(i)
So, ∇S must be vector (incorrect).
Solved Paper 2019 ● 15

Half-life of K 40 ,
T = 3.942 × 1016
0.693
R = × 34.15 × 1019
3.942 × 1016
R = 6.003 × 103 Bq = 6.003 kBq
42. (0.43) Given, Atomic radius of sodium, r = 0186
. nm
In BCC, relation between atomic radius (r ) and lattice
(0, 0, 0)
parameter (a ).
4r = 3a
4r 4 × 0186
.
a = = nm
3 1732
.
For origin (uj$ , v j$ , w j$ ) = ( 0, 0 , 0)
a = 0.429 nm
when we put these values in Eq. (i), we get
a ~
− 0.43 nm
s=1
F = f .s here f = atomic structure as s = 1 43. (54.68°) In a grating 4000 lines/cm
F =f width of 1 slit,
Intensity, I = |F |2 1 × 10−2 1
∆l = = × 10−5 m = 25
. × 10−6 m
Clearly intensity does not allow on h, k and so all planes are 4000 4
allow. Given, λ = 680 nm
First peak will be for the plane 100. From the formula,
∆l sinθ = 3 λ
39. (c) Eccentricity of particle,
3 λ 3 × 680 × 10−9
 2El 3 
1/ 2
sinθ = =
ε = 1 +  ∆l 25. × 10−6
 mk 2 
sinθ = 0.816
Here, E is energy, l is angular momentum, m is mass and k is θ = sin−1 ( 0186
. )
constant. θ = 54.68°
We know that, 1
For parabolic energy, 44. (4) As, λ ∝
D2
E =0
Here, λ = mean free path and D = diameter.
So, ε = (1 + 0)1 / 2 = 1
λ1 D22
⇒ Energy, E = 0 and ε = 1 So, =
λ 2 D12
40. (d) Secular equilibrium is a situation in which the quantity of D2
radioactive isotops remains constant because its production Given, =2
D1
rate is equal to its decay rate. 2
As X decay into Y λ1  2  4
=  =
For secular equilibrium, λ 2  1 1
When λX << λY , thenTX >> TY 45. (4)
 λ = decay constant  Given, Object distance, u = − 30 cm
 
 T = half - life  R = − 40 cm (concave mirror)
41. (6.003) Given, R
So, focal length, f = = − 20 cm
Mass of human body = 70 kg 2
% of K 40 in human body = 3.24 × 10−5 % of mass Activity due Object height, hO = 2 cm
to K 40 in human body = ? Using mirror formula,
Now, amount of K 40 in human body (in kg) 1 1 1
= +
3.24 × 10−5 f v u
= × 70 kg
100 1 1 1
= −
Q 40 g or 40 × 10−3 kg of K 40 contain atoms v f u
= 6.023 × 1023 1 1 1
= −
∴ Number of K element in human body,
40 v −20 −30
6.023 × 1023 3.24 × 10−3 1 1
N = × × 70 = −
40 × 10−3 100 30 20
v = − 60 cm
N = 34.15 × 1019
Linear magnification,
Now activity,
h v h −60
R = λN m= I =− ⇒ I =− , hI = − 4 cm
hO u 2 −30
0.693
R = N
T So, height of the image is 4 cm.
16 IIT JAM Physics Solved Papers & Practice Sets

46. (1.167) Electric displacement vector is given by D = ε0 εr E


z For vacuum, ε0 = 8.85 × 10−12 , εr = 1
E = electric field due to wire
C
λ $
E= r
D 2 πε0r
n=y λ $ λ
D = ε0 r = = r$
1 2 πε0r 2 πr
y
B 8 π × 10−2
D =
2 π × 2 × 10−2
A
x D = 2r$ = 2
Given function, 49. (0.64) Position of nth fringe in Young’s double slit experiment,
F = y 2 x$ + ( 3xy − z 2 )y$ + ( 4 yz )z$ nλ D
Xn =
Consider small elemental area ds on the given face ( ABCD ) d
ds = dxdzy$
7th
flux through the surface bright
φ = ∫ ∫ F ⋅ ds

φ= ∫ ∫ [y x$ + ( 3xy − z 2 )y$ + 4 yzz$ ] ⋅ ds


2
S1
y =1
d1
φ = ∫ ∫ ( 3xy − z 2 )dx ⋅ dz
(1, 1)
S2
 3x 2  
= ∫ y − z 2x dz 
 2   (0, 0) D
(1, 1)
 3x 2 z z 3x  3 1 9− 2 7 Wavelength, λ = 550 nm
= − = − = = = 1167
.
 2 3  (0, 0) 2 3 6 6 D = separation between slits and screen
For 7th bright fringe (n = 7)
47. (3.124) A charged spherical shell is can isolated capacitor and

energy stored will be x7 =
1 d
E = CV 2 Now, when we put glass slab of refraction index n1 and n 2 and
2
thickness t = 6 µm.
Here, C = capacitance of shell and r = radius
7th bright shifts to central maxima.
C = 4 πε0r ,
Position of central fringe,
1 q
Potential, V = −D
4 πε 0 r x0 = [(n2 − 1) t − (n1 − 1)t ]
2
d
1  1 q According to the question,
E = ( 4 πε0r ) 
2  4 πε 0 r  x7 = x0
Given q = 5C, r = 4m 7λD −D
= [(n2 − 1)t − (n1 − 1)t ]
q 2  1 d d
E =  J
 2r  4 πε0 7λ = − t (n2 − n1 )
7λ 7 × 550 × 10−9
(5 ) 2 1 |n1 − n2 | = = = 0.64
E = × J t 6 × 10−6
2 × 4 4 πε 0
1 50. (11.02)
E = 3.124 × J
4 πε 0 35kΩ 35kΩ 35kΩ
48. (2) Given, Rf Rf Rf
Linear charge density, λ = 8 π × 10−2 C/m 10kΩ 10kΩ
– 10kΩ
Position, r = 20 mm, = 2 × 10−2 m R1 + – –
V01 R2 + V01 R3 +
+ V0
λ
+ Vi

+
r P
+
Input voltage, (given) V i = ( 200mV ) sin 400t
+ Let intermediate outputs are V 01 andV 02 .
+
Solved Paper 2019 ● 17

V 01  R   35 × 103  On differential above equation, we get


Now, = 1 + f  = 1 + 
Vi  R1   10 × 103  dz = dx + idy
= 4.5 (Non-inverting amplifier) Also, 3 y = x ⇒ 3dy = dx
V 01 = 4.5 V i …(i) Hence, dz = 3dy + idy = ( 3 + i )dy
Now, for inverting amplifier, and z = 3 y − iy = ( 3 − i )y
V R 35 Now, put the values of dz and z in given integral.
AV = 02 = − f = −
V 01 R1 10 y =1
2
1
2

V 02 = −
35
× V 01 = − 3.5 × 4.5 × V i
⇒ ∫ ( 3 − i )y .( 3 + i )dy = ( 3 − i )( 3 + i )∫ y .dy
y =0 0
10 2
1 2
V
= 0 =− f
R
(inverting mode) y2  1
AV3 ⇒ 10  = 10 × = 25
V 01 R1  2 0 2
V0 35
=− Note In official answer key, the answer is 111.0
V 01 10
53. (29) Given, Wavelength of K α 2 X-ray line, λ = 1544
. Å
⇒ V 0 = − 3.5 × ( −3.5 × 4.5 × V i )
= 3.5 × 3.5 × 4.5 × 200 Atomic number of element, Z = ?
= 11025 mV = 1102 . V 2p l2
n=2
51. (445) +VCC=10V 2s l1

2 MΩ 4 kΩ

C
B 1s n=1
β=100
1 1 1
E = ( Z − b )2R  2 − 2 
0.3 V λ  n1 n2 
For K α series b = 1
2 kΩ +
–CE For K α transaction of e − is from n2 = 2 to n1 = 1
1  1 3 4
= ( Z − 1) 2 R  1 −  = ( Z − 1) 2 R ⇒ ( Z − 1) 2 =
λ  4 4 3 λR

Apply the Kirchhoff’s loop rule. Given R = 1.097 × 107 m−1


V cc = IBRB + V BE + IERE 4
( Z − 1) 2 =
From the above figure, 3 × 1544
. × 10−10 × 1097
. × 107
10 = IB × 2 × 106 + 0.3 + IE × 2 × 103 ( Z − 1) = 787
9.7 = 2IB × 106 + 2 × 103 IE …(i)
( Z − 1) = 28.01
As we know that,
Z = 29.01
I
β = C = 100 (given) So Z = 29
IB
IC = 100IB 54. (0.315) By Heisenberg uncertainty principle
h
IE = IC + IB ∆ x∆ p ≥
2
IE = 101 IB
where, ∆x = position and ∆p = momentum.
Put this value in Eq. (i), we get
h
9.7 = 2IB × 106 + 2 × 103 × 101IB ∆x ( m∆v ) ≥
2
9.7 = 2 × 103 [103 + 101]IB
9.7 h
IB = × 10−3 ∆ x∆ v ≥
1101 × 2 4 πm

IB = 4.40 µA Given, ∆x = 10−13 cm = 10−15 m


As, IE = 101IB h
∆v ≥
IE = 4.40 × 101 4 πm∆x
IE = 444.91 µA 6.626 × 10−34
∆v ≥
IE ≈ 445 µA 4 π × 1672
. × 10−27 × 10−15

52. (*) To get the value of ∆v ≥ 0.3153 × 108 m/s


2
3,1 55. (0.50) Given wave function of a particle,
∫ z ⋅ dz along the line 3y = x
2 π 
0, 0 ψ(x ) = sin x 
As z = x + iy …(i) L L 
18 IIT JAM Physics Solved Papers & Practice Sets

For particle in a box having length L probability of particle in Since I is momentum of inertia and ω is angular frequency.
L
range x = 0 to is. R12 R 22 R2
⇒ = or = constant
2 T1 T2 T
L L
x= x=
2 2 ∆R 4
or = 4% =
∫ P (x )dx = ∫ |ψ(x )|
2
dx R 100
x=0 x=0
∆T 2∆R 8
x=
L
2 x=
L ⇒ = = 2 × 4% = 8% =
 2  πx  
2
2 2  πx  T R 100
P = ∫  sin
L x ∫= 0  L 
⇒   dx = sin  dx 24 × 8 96
x = 0 L
 L  ⇒ ∆T = = = 192
. hours/day
100 50
x= 
L 2 πx 
1 − cos Hence, length of the day is, T + ∆T = 24 + 1.92
2 2 
= ∫
L x=0

2
L  dx

= 25.92 hours
  58. (0.15) Given,
x=
L For diatomic ideal gas
 2 πx  2
sin Work,W = 2300 J
1 L 
x− Ti = 1150 K
L π 
 2  Tf = 400 K
 L x = 0
We know that,
Applying limits
 2 Cp 
1 L  1 Work, W = µCV (Ti − Tf ) ∴ γ = 1 + = 
Probability, P = − 0 = = 05
.  f CV 
L  2  2
µR
56. (1) = (Ti − Tf )
γ −1
I + VR – Here, R = gas constant, µ = number of moles, Ti = initial
R=10kW IZ IL temperature andTf = final temperature.
µ × 8.4
2300 = (1150 − 400)
VZ=20 V

RL=20kΩ
Vin=40V

2/5
VL µ × 8.4
2300 = × 750
2/5
2300 × 2 230 230
⇒ µ = = = = 0146
.
8.4 × 5 × 750 21 × 75 1575
We have to calculate the Zener current I z . Number of moles of the gas, µ ~− 015
.
V − Vz 40V − 20V 59. (6.625) Given, binary = 110101
.
Current, I = in =
R 10 kΩ 110 = 1 × 22 + 1 × 21 + 0 × 2° = 4 + 2 + 0 = 6
I = 2 mA .101 = 1 × 2−1 + 0 × 2−2 + 1 × 2−3
As load and Zener diode is in parallel 1 1 5
= + 0 + = = 0.625
So, VL = V Z = 20 V 2 8 8
V 20 V . ) 2 = ( 6.625 )10
So, (110101
Now current, IL = L =
RL 20 kΩ
60. (2) Continuity condition for interface of different mediums
IL = 1mA B1, 1 = B1, 2
As, I = I z + IL Here, surface is in XY-plane, so z-axis is the perpendicular
Iz = I − IL direction to the surface.
Iz = 2 mA − 1 mA →
Here, B1 = 4x$ − 6y$ + 2z$ mT (Given)
Iz = 1mA

57. (25.92) Given diameter of earth increased by 4% then time So, B1, 1 = 2z$ mT or B1, 1 = mT
period of earth increases. Using continuity equation,
From law of conservation of angular momentum Normal component of B2 , B1, 2 = 2 mT
I1ω1 = I 2ω 2
IIT JAM PHYSICS
SOLVED PAPER 2018
MM : 100 Time : 3 hrs

n
This test paper has a total of 60 questions carrying 100 marks. The entire question paper is divided into
three Sections A, B and C. All sections are compulsory. Questions in each section are of different types.
n
Section A contains Multiple Choice Questions (MCQ). Each MCQ type question has four choices out of
which only one choice is the correct answer. This section has 30 Questions and carry a total of 50 marks.
(Q.1 – Q.10) carry 1 mark each and (Q.11 – Q.30) carry 2 marks each.
n
Section B contains Multiple Select Questions (MSQ). Each MSQ type question is similar to MCQ but
with a difference that there may be one or more than one choice(s) that are correct out of the four given
choices. The candidate gets full credit if he/she selects all the correct choices only and no wrong
choices. This section has 10 questions (Q. 31 – Q. 40) and carry 2 marks each with a total of 20 marks.
n
Section C contains Numerical Answer Type Questions (NATQ). For these NAT type questions, the answer
is a real number which needs to be entered using the virtual numerical keypad on the monitor. No
choices will be shown for these type of questions. This section has 20 questions and carry a total of
30 marks. (Q.41 – Q. 50) carry 1 mark each and (Q.51 – Q.60) carry 2 marks each.
n
Do not write more than one answer for the same question. In case you attempt a subjective question
more than once, please cancel the answer(s) you consider wrong. Otherwise, the answer appearing last
only will be evaluated.

Section A Multiple Choice Questions (MCQs)


In these type of questions, each question has four choices (a), (b), (c) and (d) out of which only
one option is correct.
Q.1-10 carry one mark each.

1. Let f (x , y ) = x 3 − 2y 3 . The curve along which ∇ 2f = 0 is


y
(a) x = 2 y (b) x = 2 y (c) x = 6 y (d) x = −
2

2. A curve is given by r ( t ) = t i$ + t 2 $j + t 3 k.
$ The unit vector of the tangent to the curve at t = 1is
$i + $j + k$ $i + $j + 2 k$
(a) (b)
3 6
$i + 2 $j + 2k$ $i + 2 $j + 3k$
(c) (d)
3 14

3. There are three planets in circular orbits around a star at distances a, 4a and 9a, respectively.
At time t = t 0 , the star and the three planets are in a straight line. The period of revolution of
the closest planet is T. How long after t 0 will they again be in the same straight line?
(a) 8T (b) 27T (c) 216T (d) 512T
2 IIT JAM Physics Solved Papers & Practice Sets

4. A current I is flowing through the sides of an equilateral triangle of side a. The magnitude of
the magnetic field at the centroid of the triangle is
9µ 0 I µ0 I 3µ 0 I 3µ 0 I
(a) (b) (c) (d)
2π a πa 2π a πa

5. Two vehicles A and B are approaching an observer O at rest with equal speed as shown in the
figure. Both vehicles have identical sirens blowing at a frequency fs . The observer hears these
sirens at frequency f A and fB , respectively from the two vehicles. Which one of the following is
correct?
A

θ
O
B

(a) fA = fB < fs (b) fA = fB > fs (c) fA > fB > fs (d) fA < fB < fs

6. Three infinite plane sheets carrying uniform charge densities − σ , 2σ , 3σ are placed parallel to
the x-z plane at y = a , 3a , 4a respectively. The electric field at the point (0, 2a , 0 ) is
4σ $ 3σ $
(a) j (b) − j
ε0 ε0
2σ $ σ $
(c) − j (d) j
ε0 ε0

7. Two boxes A and B contain an equal number of molecules of the same gas. If the volumes
are V A and VB , and λ A and λ B denote respective mean free paths, then
λ A λB
(a) λ A = λ B (b) =
VA VB
λA λB
(c) = (d) λ AV A = λ BVB
V A1/ 3 VB1/ 3

8. Let Tg and Te be the kinetic energies of the electron in the ground and the third excited states
Tg
of a hydrogen atom, respectively. According to the Bohr model, the ratio is
Te
(a) 3 (b) 4
(c) 9 (d) 16

9. Which one of the following arrangements of optical components can be used to distinguish
between an unpolarised light and a circularly polarised light?
λ/2 λ/2
plate analyser polariser plate analyser
light light

(a) (b)

λ/4 λ/4
plate analyser polariser plate analyser
light light

(c) (d)
Solved Paper 2018 ●
3

10. Which one of the following graphs shows the correct variation of V 0 with Vi ? Here, Vd is the
voltage drop across the diode and the Op-Amp is assumed to be ideal.
Vi Vd
+
Vo

RL

Vo Vo

(a) Vi (b) Vi
0 0 Vd

Vo Vo
(c) (d)

Vi Vi
0 Vd 0

Q. 11-30 carry two marks each.

11. Which one of the figures correctly represents the T-S diagram of a Carnot engine?
T T T T

(a) (b) (c) (d)

S S S S

12. The plane of polarisation of a plane polarized light rotates by 60° after passing through a wave
plate. The pass-axis of the wave plate is at an angle α with respect to the plane of polarisation
of the incident light. The wave plate and α are
(a) λ / 4, 60° (b) λ /2, 30°
(c) λ / 2, 120º (d) λ / 4, 30º

13. A rectangular loop of dimensions l and w moves with a constant speed of v through a region
containing a uniform magnetic field B directed into the paper and extending a distance of 4w.
Which of the following figures correctly represents the variation of emf (ε) with the position (x)
of the front end of the loop?
v
B
l
w

0 x 4w
4 IIT JAM Physics Solved Papers & Practice Sets

ε ε

+Bwv +Bwv
0 w 4w
(a) 0 x (b) 0
w
x
4w 0
–Bwv –Bwv

ε ε
+Blv +Blv

0 w 4w
(c) 0 x (d) 0 x
4w 0 w

–Blv –Blv

14. The equation of state for one mole of a non-ideal gas is given by pV = A 1 +
B
 , where the
 V
coefficients A and B are temperature dependent. If the volume changes from V1to V 2 in an
isothermal process, the work done by the gas is
1 1 V 
(a) AB  −  (b) AB In  2 
 V1 V2   V1 
V  1 1  V − V1
(c) A In  2  + AB  −  (d) A In  2  +B
 V1   V1 V2   V1 

15. An ideal gas consists of three dimensional polyatomic molecules. The temperature is such that
only one vibrational mode is excited. If R denotes the gas constant, then the specific heat at
constant volume of one mole of the gas at this temperature is
7 9
(a) 3R (b) R (c) 4R (d) R
2 2
^
16. A long solenoid is carrying a time dependent current such that the magnetic k
field inside has the form B (t ) = B 0t 2k$ , where k$ is along the axis of the
solenoid. The displacement current at the point P on a circle of radius r in a
plane perpendicular to the axis r P
(a) is inversely proportional to r and radially outward.
(b) is inversely proportional to r and tangential.
(c) increases linearly with time and is tangential.
(d) is inversely proportional to r 2 and tangential.

17. Consider an ensemble of thermodynamic systems, each of which is characterized by the same
number of particles, pressure and temperature. The thermodynamic function describing the
ensemble is
(a) Enthalpy (b) Helmholtz free energy (c) Gibbs free energy (d) Entropy

kr 2, r < R
18. Given a spherically symmetric charge density ρ(r ) =  . (k being a constant), the
0 , r > R
electric field for r < R is (take the total charge as Q)
Qr 3 3Qr 2 5Qr 3 Q
(a) r$ (b) r$ (c) r$ (d) r$
4πε 0R 5
4πε 0R 4
8πε 0R 5
4πε 0r 2
Solved Paper 2018 ●
5

19. An infinitely long solenoid, with its axis along k$ carries a current l. In addition there is a uniform
^ $
line charge density λ along the axis. If S is the energy flux in cylindrical coordinates (ρ, φ, k),
then
(a) S is along ρ (b) S is along k$
(c) S has non zero components along ρ and k$ (d) S is along ρ × k$

20. Consider two waves y 1 = a cos (ωt − kz ) and y 2 = a cos [(ω + ∆ω ) t − (k + ∆k )z]. The group
velocity of the superposed wave will be (∆ω << ω and ∆k << k)
(ω − ∆ω ) (2ω + ∆ω ) ∆ω (ω + ∆ω )
(a) (b) (c) (d)
( k − ∆k ) (2 k + ∆k ) ∆k ( k + ∆k )

21. Consider a convex lens of focal length f. A point object moves towards the lens along its axis
between 2f and f. If the speed of the object is v o , then its image would move with speed v I
which of the following is correct?
(a) v I = vo ; the image moves away from the lens.
(b) v I = − vo ; the image moves towards the lens.
(c) v I > vo ; the image moves away from the lens.
(d) v I < vo ; the image moves away from the lens.

22. A disc of radius R1 having uniform surface density has a concentric hole of radius R 2 < R1. If
its mass is M, the principal moments of inertia are
M( R12 − R22 ) M( R12 − R22 ) M( R12 − R22 ) M( R12 + R22 ) M( R12 + R22 ) M( R12 + R22 )
(a) , , (b) , ,
2 4 4 2 4 4
M( R12 + R22 ) M( R12 + R22 ) M( R12 + R22 ) M( R12 − R22 ) M( R12 − R22 ) M( R12 − R22 )
(c) , , (d) , ,
2 4 8 2 4 8

 x, − π < x < 0
23. The function f (x ) =  is expanded as a Fourier series of the form
 −x, 0 < x < π

a n cos (nx ) + ∑ ∞ b n sin (nx ). Which of the following is true?



a0 + ∑
n=1 n=1
(a) a0 ≠ 0, bn = 0 (b) a0 ≠ 0, bn ≠ 0
(c) a0 = 0, bn = 0 (d) a0 = 0, bn ≠ 0

24. Which one of the following curves correctly represents (schematically) the solution for the
df
equation + 2f = 3;f ( 0) = 0 ?
dx

f(x) f(x) f(x) f(x)


3
1 3 1 —
(a) —
2 (b) —
2 (c) —
2 (d) 2

0 x 0 x 0 x 0 x

25. The mean momentum p of a nucleon in a nucleus of mass number A and atomic number Z
depends on A, Z as
1 1 1 2
− −
(a) p ∝ A 3 (b) p ∝ Z 3 (c) p ∝ A 3 (d) p ∝ ( AZ ) 3
6 IIT JAM Physics Solved Papers & Practice Sets

26. The Boolean expression ( AB ) ( A + B ) ( A + B ) can be simplified to


(a) A + B (b) AB (c) A + B (d) AB

27. Consider the transformation to a new set of coordinates (ξ, η ) from rectangular Cartesian
coordinates (x,y), where ξ = 2x + 3 y and η = 3x − 2y In the ( ξ, η) coordinate system, the area
element dxdy is
1 2 3
(a) dξ dη (b) dξ dη (c) 5dξ dη (d) dξ dη
13 13 5

 0, 0 < x < L
28. A particle of mass m is in a one dimensional potential V (x ) = 
 ∞, otherwise
1 2
At some instant its wave function is given by ψ(x ) = ψ 1(x ) + i ψ 2 (x ), where ψ 1(x ) and
3 3
ψ 2 (x ) are the ground and the first excited states, respectively. Identify the correct statement.
L h 2 3π 2 2L h2 π 2
(a) x = ; E = (b) x = ; E =
2 2 m L2 3 2 m L2
L h 2 8π 2 2L h2 π 2
(c) x = ; E = (d) x = ; E =
2 2 m L2 3 2 m 3 L2

29. A raindrop falls under gravity and captures water molecules from atmosphere. Its mass
changes at the rate λm(t), where λ is a positive constant and m(t ) is the instantaneous mass.
Assume that acceleration due to gravity is constant and water molecules are at rest with
respect to earth before capture. Which of the following statements is correct?
(a) The speed of the raindrop increases linearly with time.
(b) The speed of the raindrop increases exponentially with time.
(c) The speed of the raindrop approaches a constant value when λt >> 1.
(d) The speed of the raindrop approaches a constant value when λt << 1.
z
30. A particle P of mass m is constrained to move on the surface of a cylinder
under a force −k r as shown in figure (k is the positive constant). Which of the
P
following statements is correct? (Neglect friction.)
(a) Total energy of the particle is not conserved. r
y
O
(b) The motion along z-direction is simple harmonic.
x
(c) Angular momentum of the particle about O increases with time.
(d) Linear momentum of the particle is conserved.

Section B Multiple Select Questions (MSQs)


In these type of questions, each question has four choices (a), (b), (c) and (d) out of which only
one or more than one is/are correct options.

Q. 31-40 carry two marks each.

 4 x
31. Let matrix M =   If det (M ) = 0, then
 6 9
(a) M is symmetric (b) M is invertible
(c) One eigenvalue is 13 (d) Its eigenvectors are orthogonal
Solved Paper 2018 ●
7

32. Let f(x ) = 3x 6 − 2x 2 − 8. Which of the following statements is (are) true?


(a) The sum of all its roots is zero (b) The product of its roots is −8 / 3
(c) The sum of all its roots is 2/3. (d) Complex roots are conjugates of each other

33. Two projectiles of identical mass are projected from the ground with same u u
initial angle (α) with respect to earth surface and same initial velocity (u) in
the same plane. They collide at the highest point of their trajectories and α α
stick to each other. Which of the following statements is (are) correct?
(a) The momentum of the combined object immediately after the collision is zero.
(b) Kinetic energy is conserved in the collision.
(c) The combined object moves vertically downward.
(d) The combined object moves in a parabolic path.

34. Two beams of light in the visible range (400 nm-700 nm) interfere with each other at a point.
The optical path difference between them is 5000 nm. Which of the following wavelengths will
interfere constructively at the given point?
(a) 416.67 nm (b) 555.55 nm (c) 625 nm (d) 666.66 nm

35. Which of the following relations is (are) true for thermodynamic variables?
 ∂P   ∂V 
(a) TdS = CVdT + T   dV (b) TdS = CPdT − T   dP
 ∂T  V  ∂T  P
(c) dF = − SdT + PdV (d) dG = − SdT + VdP

36. Consider a convex lens of focal length f. The lens is cut along a diameter in two parts. The two
lens parts and an object are kept as shown in the figure. The images are formed at following
distances from the object

2f
x
0 f

(a) 2f (b) 3f (c) 4f (d) ∞


→ 2 2
37. Let the electric field in some region R be given by E = e − y $i + e − x $j. From this we may
conclude that
(a) R has a non-uniform charge distribution. (b) R has no charge distribution.
(c) R has a time dependent magnetic field. (d) The energy flux in R is zero everywhere.

38. In presence of a magnetic field B $j and an electric field ( − E ) k,


$ a particle moves undeflected.
Which of the following statements is (are) correct?
E$
(a) The particle has positive charge, velocity = − i
B
E$
(b) The particle has positive charge, velocity = i
B
E
(c) The particle has negative charge, velocity = − $i
B
E$
(d) The particle has negative charge, velocity = i
B
8 IIT JAM Physics Solved Papers & Practice Sets

39. In a p-n junction, dopant concentration on the p-side is higher than that on the n-side. Which
of the following statements is (are) correct, when the junction is unbiased?
(a) The width of the depletion layer is larger on the n-side.
(b) At thermal equilibrium the Fermi energy is higher on the p–side.
(c) In the depletion region, number of negative charges per unit area on the p-side is equal to
number of positive charges per unit area on the n–side.
(d) The value of the built-in potential barrier depends on the dopant concentration.

40. Which of the combinations of crystal structure and their coordination number is (are) correct?
(a) body centered cubic – 8 (b) face centered cubic – 6
(c) diamond – 4 (d) hexagonal closed packed – 12

Section C Numerical Answer Type Questions (NATQs)


This section contains Numerical Answer Type (NAT) questions, for these NAT questions, the
answer is a real number.

Q.41-50 carry one mark each.

41. The coefficient of x 3 in the Taylor expansion of sin (sin x) around x = 0 is ______.
(Specify your answer upto two digits after the decimal point.)

42. A particle of mass m is moving along the positive x direction under a potential
1 2 λ
V (x ) = kx + 2 (k and λ are positive constants). If the particle is slightly displaced from its
2 2x
equilibrium position, it oscillates with an angular frequency (ω) __________.
k
(Specify your answer in units of as an integer.)
m

43. A planet has average density same as that of the earth but it has only 1/8. of the mass of the
earth. If the acceleration due to gravity at the surface is g p and g e for the planet and earth,
gp
respectively, then = ____________.
ge
(Specify your answer upto one digit after the decimal point.)

44. In a grating with grating constant d = a + b , where a is the slit width and b is the separation
b
between the slits, the diffraction pattern has the fourth order missing. The value of
a
is ________. (Specify your answer as an integer.)

45. Consider an electromagnetic plane wave E = E 0 ( i$ + b$j ) cos  {ct - (x − 3 y ) } ,where λ is
λ 
the wavelength, c is the speed of light and b is a constant. The value of b is ________.
(Specify your answer upto two digits after the decimal point.) P
P2
46. Consider a monoatomic ideal gas operating in a closed cycle as shown
P1
in the P-V diagram given below. The ratio is _____. (Specify your adiabatic
P2
answer upto two digits after the decimal point.) P1
V
V1 3V1
Solved Paper 2018 ●
9

47. Consider the first order phase transition of the sublimation of zinc. Assume the vapor to be an
ideal gas and the molar volume of solid to be negligible. Experimentally, it is found that
C
log10 (P ) = − 1 + C 2 where P is the vapor pressure in Pascal, T is in K, C1 = 6790 K and
T
C 2 = 9. The latent heat of sublimation of zinc from the Clausius - Clapeyron equation is ____
kJ/mole. (R = 8.314 J/mole. K) (Specify your answer in kJ/mole upto one digit after the decimal
point.)

48. A system of 8 non-interacting electrons is confined by a three dimensional potential


1
V (r ) = mω 2r 2 . The ground state energy of the system in units of hω is __________ .
2
(Specify your answer as an integer.)

49. For the given circuit, value of the base current (Ib ) of the n-p-n transistor will be _____ mA. (β is
the current gain and assume Op-Amp as ideal.) (Specify your answer in mA upto two digits
after the decimal point.)
+10V
1kΩ
+5V + Ib

β=50

1kΩ

50. The lattice constant of unit cell of NaCl crystal is 0.563 nm. X-rays of wavelength 0.141 nm are
diffracted by this crystal. The angle at which the first order maximum occurs is
________degrees. (Specify your answer in degrees upto two digits after the decimal point.)

Q. 51-60 carry two marks each.

51. For the following circuit, the collector voltage with respect to ground will be ________V.
(Emitter diode voltage is 0.7 V and β DC of the transistor is large.) (Specify your answer in volts
upto one digit after the decimal point.)
+10V

3kΩ

3kΩ 1kΩ

–3V

52. A body of mass 1 kg is moving under a central force in an elliptic orbit with semi major axis
1000 m and semi minor axis 100 m. The orbital angular momentum of the body is
100 kg m 2 s −1. The time period of motion of the body is __________ hours. (Specify your
answer in hours upto two digits after the decimal point.)
10 IIT JAM Physics Solved Papers & Practice Sets

53. The moon moves around the earth in a circular orbit with a period of 27 days. The radius of
the earth (R) is 6.4 × 10 6 m and the acceleration due to gravity on the earth surface is
9.8 ms −2 . If D is the distance of the moon from the center of the earth, the value of D/R will be
_____. (Specify your answer upto one digit after the decimal point.)

54. A syringe is used to exert 1.5 atmospheric pressure to release water horizontally. The speed
of water immediately after ejection is ________. (take 1 atmospheric pressure = 105 Pascal,
density of water = 10 3 kg m −3 ) (Specify your answer in ms −1 as an integer.)

55. Consider a slit of width 18 µm which is being illuminated simultaneously with light of orange
color (wavelength 600 nm) and of blue color (wavelength 450 nm). The diffraction pattern is
observed on a screen kept at a distance in front of the slit. The smallest angle at which only
the orange color is observed is θ1, and the smallest angle at which only the blue color is
observed is θ 2 . The angular difference θ 2 − θ1 (in degrees) is _____. (Specify your answers
upto two digits after the decimal point.)

56. A particle of mass m is moving in a circular orbit given by x = R cos (ωt ); y = R sin (ωt ), as
observed in an inertial frame S1. Another inertial frame S 2 moves with uniform velocity
v = ωR i$. with respect to S1 . S1 and S 2 are related by Galilean transformation, such that the

origins coincide at t = 0. The magnitude of the angular momentum of the particle at t = , as
ω
observed in S 2 about its origin, is expressed as (mR 2ω) x. Then x is _______. (Specify your
answer upto two digits after the decimal point.)

57. Rod R1 has a rest length 1m and rod R 2 has a rest length of 2m. R1 and R 2 are moving with
respect to the laboratory frame with velocities +v $i and −v $i, respectively. If R 2 has a length of
v
1m in the rest frame of R1, is given by ________. (Specify your answer upto two digits after
c
the decimal point.)

58. Two events E 1 and E 2 take place in an inertial frame S with respective time-space coordinates
(in SI units) : E 1(t1 = 0, r1 = 0) and E 2 (t 2 = 0,x 2 = 0,x 2 = 10 8 , y 2 = 0,z 2 = 0). Another inertial
r
frame S ′ is moving with respect to S with a velocity v = 0.8c i$. The time difference (t' 2 − t' 1)as
observed in S′ is ____ s. (c = 3 × 10 8 ms −1) (Specify your answer in seconds upto two digits
after the decimal point.)

59. In the following circuit, the time constant RC is much greater than the period of the input
signal. Assume diode as ideal and resistance R to be large. The dc output voltage across
resistance R will be ___V. (Specify your answer in volts upto one digit after the decimal point.)
C

24 Vrms
~ C R

60. For a metal, the electron density is 6.4 × 10 28 m −3 . The Fermi energy is ........ eV.
(h = 6.626 × 10 −34 J-s, me = 9.11 × 10 −31 kg, 1 eV = 1.6 × 10 −19 J)
(Specify your answer in electron volts (eV) upto one digit after the decimal point.)
Answers with Solutions
Section A Multiple Choice Questions  3  3 µ 0I
2 3 µ 0I
= ⋅   =
1. (b) The Laplacian operator 2 πa 2 πa
→ → ∂2 ∂2 ∂2 3µ0 I
∇ 2 = ∇⋅ ∇ = + + ∴ Magnetic field due to all 3-sides = 3 ×
∂x 2
∂y 2
∂z 2 πa
 ∂ 2
∂ 2
∂  3
2 = 9 µ 0 I / 2 πa
So, ∇ 2f =  2 + +  (x − 2y 3 )
 ∂ x ∂ y 2
∂ z 2 5. (b) Angle between directions of approach of vechicles
= 6x − 12y does not affect the change in frequency observed due
∴ ∇ 2f = 0 when 6x − 12y = 0 to doppler’s effect. ∴ fA = fB
i.e. x = 2y As speeds of both vehicles is same.
Also, fA = fB > fs . As the vechicles are approaching the
2. (d) Given, r(t ) = t $i + t 2$j + t 3k$
observer, observed frequency is higher than true
dr $
∴ = i + 2t $j + 3t 2k$ frequency.
dt
dr 6. (b) Given situation is
= $i + 2$j + 3k$ z
dt t = 1
–σ
∴ A unit vector tangent to the curve r(t )|t = 1 is given as,
dr 2σ
$ $ $ $ $ $ 3σ
$n = dt = i + 2 j + 3k = i + 2 j + 3k –σ a
dr 12 + 22 + 32 14
dt a
3. (c) At t = 0, given situation is shown as y
x

z
–σ 2σ 3σ
E1
Star E3
a E2
y
a 2a 3a 4a
9a 4a x

1 2 3
∴ Time period of rotation of a planet is proportional to
In region (0, 2a, 0) all three electric fields are in same
its orbital radius is, T ∝ a 3 / 2; where a = radius of orbit
direction.
∴ In given condition, we have σ 2σ 3σ
T1 : T2 : T3 : : a 3 / 2 : ( 4a )3 / 2 : (9a )3 / 2 ∴ Enet = ( − $j) + ( − $j) + ( − $j)
2ε 0 2ε 0 2ε 0
or T1 : T2 : T3 : : T : 43 / 2 T : 9 3 / 2 T
6σ 3σ $
or T1 : T2 : T3 : : T : 8T : 27T = ( − $j) = ( − j)
2ε 0 ε0
LCM of 1, 8 and 27 is 216. Hence, planets again align
after time 216 T. 7. (b) Mean free path λ is given by,
4. (a) Magnetic field at centroid of the triangle due to one of 1
λ=
the conductor (one of side) is given by N
2 π d2⋅
µI V
B = 0 (sin α 2 + sin α 1 ) A
4 πr where, d = diameter of a gas molecule, N is total
µ 0I number of molecules present in volume V.
= (sin 60° + sin 60° )
 a  λ 1
4π   ∴ = = a constant in given case.
 2 3 α2
°

2 πd 2N
1 20

V
O
 a  α1 λ A λB
Q r =  ∴ =
 2 3 VA VB
C B
12 IIT JAM Physics Solved Papers & Practice Sets

8. (d) In Bohr’s model energy of nth shell level is, dφ


13. (c) Here induced emf in wire loop, ε = −
13.6 dt
En = − (for a hydrogen electron).
n2 d  dw 
=− Blv = − Bl   = − Blv
− 13.6 dt  dt 
Tg 12 Also, induced emf lasts only till the flux is changing.
So, = = 16 So, graph between ε v/s x is
Te − 13 .6
( 4 )2
Blv
9. (c) In given arrangement,
λ/4 0 w
plate Analyser x
4w 5w
Light
–Blv

14. (c) As coefficients A and B are temperature dependent


and they remains constant in given is isothermal
On rotating analyser at θ = 90°, a complete darkness process.
appears which confirms that incoming light is circularly V2
∴ Work done is given by W = ∫ pdV
polarised. V1

10. (a) For an ideal Op–Amp; correct graph of Vo and Vi is  A AB 


V2
=∫
 + 2  dV
Vo
V V 
V1

 B
As, pV = A 1+ 
 V
V
 1 2
∴ W = A [lnV ]VV21 + AB − 
Vi  V V1
V  1 1
Vo appears as soon asVi is positive (just greater than = A ln  2  + AB  − 
zero.)  V1  V1 V2 
Also Vo increases linearly withVi ; (Vi > 0). 15. (c) A polyatomic 3-dimensional gas molecule has 6
11. (b) For a carnot-cycle degrees of freedom at ordinary temperatures.
Now, if one vibration mode is active at given
p T temperature, so total number of degrees of freedom
A A Isothermal B are
Isothermal B T1
f =6+ 2=8
Adiabatic

Adiabatic

Adiabatic
Adiabatic

T1 f
∴ CV = ⋅ R ⇒ CV = 4R
2
D T2 T2 16. (b) From the Maxwell’s equations,
C D Isothermal C
Isothermal d
V S
we have, ∫ E ⋅ d s = − dt ∫ B ⋅ d s
d
As a reversible adiabatic process is known as or E ∫ ds = − (B0t 2 ) ∫ ds ′
dt
isentropic process, so entropy of system remains
constant during BC and DA processes. ^
k
12. (b) When a plane polarized light is incident upon a half
λ
wave plate (i.e. ) which makes an angle α with the
2 ID
direction of the optic axis, then the emergent light
is also plane polarized with vibrations inclined at r P
angle 2α.
In given case,
2α = 60° or α = 30°
Solved Paper 2018 ● 13

where, ∫ dS = surface area of cylinder 20. (c) Group velocity of resultant superimposed wave is
ω1 − ω 2
and, ∫ dS′ = surface of cross-section, i.e. (surface area of vg =
k1 − k 2
circle) ∆ω
d i.e. =
∴ E ⋅ 2 πr = − (B0t 2 ) π a 2 ∆k
dt
1 21. (c)
⇒ We get, E ∝ , which also a time dependent vo
r
quantity.
∂E 2f f
Q Displacement current, ID = ε 0
∂t
1
∴ ID ∝ As object is moving from 2f towards f ; its image is
r
moving away from 2f to ∞.
which is tangential to the surface.
1 1 1
17. (c) We have, Gibbs function, G = H − TS From lens formula, − =
v x f
For a small reversible process Differentiating w.r.t. time, we get
∆G = ∆H − T∆S − S∆T −1 dv 1 du v v
⋅ + 2 = 0 or i2 = 02
But ∆H = T∆S + V∆P 2
v dt u dt v u
∴ ∆G = V∆p − S∆T As v >u
At constant pressure and temperature, ∴ vi > v 0
∆p = ∆T = 0 Hence, the image moves away from the lens.
∴ ∆G = 0 or G = a constant. 22. (b) As, moment of inertia I = ∫ R 2dm
18. (a) For a spherical shell of thickness dr and radius r,
volume is 4πr 2dr.
∴ Total charge in given volume is R2
R R
Q = ∫ ρdV = ∫ kr 2 ⋅ 4πr 2dr
0 0 dR R1
4 πk 5 R
⇒ Q=
5
r
0
[ ]
 4 πk  5
⇒ Q = R
 5 
5Q
⇒ k = ∴ I =∫
R2
2πρhR 3dR
4 πR 5 R1
Now, at a distance r (r < R), using Gauss’s theorem  R 4 − R14 
Q enclosed = 2πρh  2 
∫ E ⋅ dA = ε 0  4 
πρh 2
Q r5 = (R2 − R12 ) (R22 + R12 )
⇒ E ( 4 πr 2 ) = 2
ε 0R 5 M M
As, ρ= =
Qr 3 Qr 3 $ V πh(R22 − R12 )
⇒ E= 5
or E = r
4πε 0R 4πε 0R 5 M
∴ I= (R12 + R22 )
1 2
19. (d) The energy flux is given as, S = ( E × B)
Hence, principal moments of inertia using
µ0
perpendicular axes theorem are
Here, E is along ρ$
M 2 M M
B is along k$ (R1 + R22 ); (R12 + R22 ); (R12 + R22 )
2 4 4
∴ E × B is along ρ$ × k$
23. (a) For Fourier expansion,
As magnetic field of solenoids is along k$ and electric ∞ ∞

field of charge density λ is along ρ;


$ then energy flux S f (x ) = a 0 + ∑
n =1
a n cos nx + ∑
n =1
bn sin nx
exists in direction of E × B.
1 π
$
Hence, S is along ρ$ × k. we have, a0 =
π ∫ − π
f (x ) ⋅ dx
14 IIT JAM Physics Solved Papers & Practice Sets

1 π 1 2
π ∫− π
a0 = f (x ) cos nx ⋅ dx Given, ψ (x ) = ψ 1(x ) + i ψ 2(x )
3 3
1 π
and bn = ∫ f (x ) sin nx ⋅ dx 1 2
π −π ∴ ψ *(x ) = ψ 1(x ) − i ψ 2(x )
3 3
∴ Given function is even function { as,f (x ) = − | x |} L

bn = 0 and a 0 ≠ 0. Further, ∫ ψ *(x ) x ψ (x )dx


0

dy L  1 2 
24. (b) As from the given equation, + 2y = 3 = ∫  ψ 1(x ) + i ψ 2(x )
dx 0
 3 3 
∴ Integrating factor (IF) = e ∫ = e 2x
2dx
 1 2 
x  ψ 1(x ) − i ψ 2(x ) dx
Further solution of given equation is  3 3 
y (IF) = ∫ 3 (IF) ⋅ dx L  1 2  L x 2 L L2
= ∫  x + x  dx = ∫ x dx =
2 ∫0
=
0 3 3  0 2
⇒ ye 2x = 3 ∫ e 2x dx
L L  1 2 
3 2x Also, ∫ ψ *(x ) ψ (x ) dx = ∫  ψ 1(x ) + i ψ 2(x )
⇒ ye 2x =
e +C 0 0
 3 3 
2
3  1 2 
As at x = 0, y = 0. So, C = − x  ψ 1(x ) − i ψ 2(x ) ⋅ dx
2  3 3 
3  e 2x − 1 L  1 2
Hence, y =   =∫  +  dx
2  e 2x  0  3 3
L
So, most suitable graph is as in option (b). = ∫ dx = L
0
L
25. (c) As the mean momentum of a nucleon in a nucleus is
inversely proportional to the radius of nucleus. ∴ Expectation value, <x > =
∫ 0
ψ *(x ) ψ (x ) dx
L

∴ p∝
1 ∫ 0
ψ *ψ dx
R L2
1 = 2 =
L
or p∝ (Q R = R0A1/ 3 )
R0A1/ 3 L 2
Now, expectation value of energy is given as,
or p ∝ A − 1/ 3
P E + P2E2
<E > = 1 1
26. (c) We have; P1 + P2
( A + B ) ( A + B ) = AA + AB + BA + BB 1 π 2 h2 2  4 π 2 h2 
= AB + AB (∴ AA = BB = 0) × +  
3 2mL 2
3  2mL2   n 2π 2h2 
= Q En = 
So, ( AB ) ( AB + AB ) = ( AB ) ( AB ) + ( AB ) ( AB ) 1 2  2mL2 
+
= ( AB ) ( AB ) 3 3
= (A + B ) (A + B) = (A + B) 1 π 2 h2
= (1 + 8)
3 2mL2
27. (a) The Jacobian of transformation is, 3π 2h2 h2  3π 2 
∂ξ ∂ξ = 2
=  
2mL 2m  L2 
∂ ( ξ, η) ∂
= x ∂y = 2 3 = −4 − 9 = −13
∂η ∂η 3 −2 dm
∂ (x , y ) 29. (c) We have, = λm , i.e. m′ = λm
∂x ∂y dt
Further rate of change of momentum is,
∴Area of element dx ⋅ dy in ( ξ, η) coordinate system
‘dξ ⋅ dη’ are related as (mv )′ = gm
⇒ mv ′ + vm′ = gm
∂ ( ξ, η)
A≈ dx ⋅ dy ⇒ mv ′ + vλm = gm
∂ (x , y )
⇒ v ′ + vλ = g
or dξ ⋅ dη = | − 13 | (dx ⋅ dy ) dv
⇒ = g − λv
1 dt
⇒ dxdy = dξdη
13 Solution of above equation is
28. (a) We know that, expectation value is given as, log | g − λv | = − λt − λc
⇒ g − kv = Ae − λt ; v( 0) = 0
<x > =
∫ ψ *(x ) x ψ (x ) dx
As, v( 0) = 0
∫ ψ *(x ) ψ (x ) dx ⇒ A =g
Solved Paper 2018 ● 15

So, kv = g − ge − λt Now, as per given option,



g
v = (1− e − λt ) when λ = 416.67 nm), n ≈ 12
K when λ = 555.55 nm, n ≈ 9
As t → ∞,e − λt → 0 when λ = 625 nm, n = 8
g Hence, these wavelengths, can produce a maxima, i.e.
∴ lim v =
t → ∞ k gives constructive interference
30. (b) Hamilton’s equations for the particle are 35. (a,b, d) We have Helmholtz function,
∂H p z F = U − TS
z& = =
∂p x m ⇒ dF = dU − TdS − SdT
∂ H p = (TdS − PdV ) − TdS − SdT
and θ& = = θ
∂p θ mR 2 = −PdV − SdT
∴ p& z = − kz ⇒ m&z& = − kz Also dG = −SdT + VdP
∴ The angular momentum is conserved while particle Here (a) and (b) options represents the energy
executes harmonic motion along z axis. equations.
∴ Options (a) (b) and (d) are correct.
Section B Multiple Select Questions 36. (c,d) For first lens, object is at focus, so its image is
formed at infinity.
 4 x
31. (a,c,d) M =  ⇒ |M | = 0
 6 9
⇒ 36 − 6x = 0 or x = 6
 4 6 f Image
∴ M =  at ∞
 6 9
 4 6 For second lens object is at distance 2f . So, its image
And, M T =  ∴ M is symmetric. is also at 2f .
 6 9
4f
Also, eigen value of the matrix | M − λI ) = 0 2f
⇒ λ ( λ − 13) = 0 O
⇒ λ = 0 or λ = 13 Image
Also, eigenvectors are orthogonal.
32. (a,b,d) Sum of roots of a polynomial equation.
37. (b,c) We have, E = e − y $i + e − x $j
2 2

 coefficient of second highest power of x 


=−  $i $j $
 Coefficient of highest power of x  k
∂ ∂ ∂
= 0 (in this case as coefficient of x 5 = 0) Further, ∇ × E =
And, product of roots ∂x ∂y ∂z
2 2

constant 8 e−y e−x 0


= =− $ −x 2 2
coefficient of highest power of x 3 = k ( −2xe − 2y e − y ) ≠ 0
Also, complex roots always occurs in pairs, so options ∂B
but, ∇ ×E= − ≠0
(a,b,d) are correct ∂t
33. (a,c) At highest point sum of initial momentum in ∴ R has a time dependent magnetic field, i.e. option (c)
Y–direction Σpyi = 0 ∂ 2 ∂ 2
Now, ∇⋅ E = (e − y ) + (e − x )
∂x ∂y
Also sum of initial momentums in x-direction
= 0+ 0= 0
= mu cos α + ( −mu cos α ) = 0
q
Hence, momentum of combined object after collision is But, ∇⋅ E = =0
ε0
zero and it falls down vertically under action of gravity.
K.E. is not conserved as collision is completely ∴R no charge distribution, i.e. option (b).
inelastic. Also, flux energy is calculated as
1
34. (a, b, c) For constructive interference, path difference S = ( E × B)
µ0
∆L = nλ Q ∆L = 5000 nm
∆L From above eq, we get
∴ λ= 2 2 ∂B
n ∇ × E = − k$ 2(xe − x + ye − y ) = −
5000 nm 5000 nm ∂t
⇒ λ= or n = 2 2

n λ ∴ B = 2k$ (xe − x + ye − y ) t
16 IIT JAM Physics Solved Papers & Practice Sets

Q E×B ≠ 0 1 λ  4k
= k + 4  =
∴ S ≠0 m x0  m
Hence, only options (b) and (c) are correct. ∴ It oscillate with angular frequency (ω )
38. (b,d) Here, magnetic force on a charge particle is given k
or ω0 = 2 =2
as m
F = q ( E + v × B) = 0 1
(∴ Particle is undeffected) 43. (0.5) Given, Mass of planet = [Mass of earth]
8
∴ E=−v×B 4 1 4 R
∴ πRp3ρ = × πRe3ρ ⇒ RP = e
Or − E k$ = − v × {B $j} 3 8 3 2
E$ 1
⇒ k = ( v × $j ) G ⋅ Me
B G ⋅ MP 8
Now, gP = =
E E Rp2 Re2
∴ |v|= or c = $j
B B 4
∴The particle either positive charge or has negative 1  GMe  1
E =  2  = ge
charge, when velocity v = $i. 2  Re  2
B gp 1
Hence options (b) and (d) are correct. ∴ = = 0.5
ge 2
39. (a,c,d) In an unbiased p -n junction,
44. (3) In diffraction grafting a missing order is realized at
(i) Thickness of depletion layer is less, if concentration
angle θ for which these two must be simultaneously
of impurities is higher.
satisfied.
(ii) Depletion layer does not contains any excess free
charge carrier. i.e, d sin θ = mλ , m = 1, 2, 3, K
(iii) Barrier potential depends on junction field and and, a sin θ = nλ , n = 1, 2, 3, K
dopant. d m
⇒ =
a n
40. (a,c,d) Face centred cubic (fcc) has a coordination
number 12. ∴ All other options are correct. Also, d =a + b
a +b m
So, = = 4;
a n
Section C Numerical Answer Type Questions ⇒
b
1+ = 4
41. (– 0.33) Taylor’s expansion of sin (sin x ) about x = 0 is a
b
x 3 x 5 8x 7 ⇒ =3
sin (sin x ) = x − + − +K a
3 10 315
−1 45. (0.57) For plane electromagnetic wave,
∴ Coefficient of x 3 = = −0.33
3 ∂E ∂E
= …(i)
42. (2) As potential function, ∂x ∂y
1 2 λ According to question,
V (x ) = kx +
2 2x 2 ∂E  2π  2π
= E0 sin {ct − (x − 3y )}  ×
Total potential energy of the particle is ∂x λ  λ
1 λm
U (x ) = mkx 2 + ∂E  2π  2π
2 2x 2 and = E0 b sin  {ct − (x − 3y ) × ⋅ 3
dU 1 λm −2 ∂y λ  λ
∴ = mk ( 2x ) + ⋅
dx 2 2 x3 ∂E ∂E
Using the value of and in Eq. (i), we get
dU ∂x ∂y
P.E. is minimum, when =0
dx  2π 2π 
mλ E0b sin  {ct − (x − 3y )] × ⋅ 3
⇒ mkx = 3 λ λ 
x
λ  2π  2π
⇒ x =
4 = E0 sin  {ct − (x − 3y ) ×
k  λ  λ
1  d 2
U  ⇒ 3b =1
Now, ω 20 = 2  2 
m  dx  x = x 1
0 ⇒ b= = 0.5780
3
1  3mλ 
= mk + 4  ∴ Correct answer upto two decimal point is b = 0.57.
m2  x0 
Solved Paper 2018 ● 17

46. (0.17) For adiabatic process. PV γ = Constant 50. (14.50°) In this problem, if first order minimum occurs at
γ
∴ P2V1 = P1( 3V1 )γ angle θ.
P2 λ
⇒ = 3γ Then, using bragy’s equation d sin θ = λ or sin θ =
P1 d
0.141nm
P1 1 1 ⇒ sin θ =
⇒ = = = 0.17 0.563 nm
P2 3γ 31. 6
⇒ sin θ = 0.250
47. (131) In this problem , if we take integration of ⇒ θ = sin−1 (0.250)
Clausius-Clapeyron equation in between limits T1 and T2 = 14.50° degrees
then,
51. (3.1) Given, VBE = 0.7 V;
 −L   1 1
lnP =  e   −  …(i) By KVL to emitter side loop we have,
 R  T Tb 
VEE = IERE + VBE
Converting ln P to base 10, we get V −V
log10 P ⇒ IE = EE BE
ln P = RE
log10 e
3 − 0.7
Now substituting given values in (i) we get ⇒ IE = = 2.3 mA
1× 103
kJ
Latent heat of time = 131 . As β is large, IE ≈ IC
mole
48. (18) Given, three dimensional potential – –
10 V= Vcc
1
V (r ) = mω 2r 2
2 3kΩ = RC

5
(1,0,0) (0,1,0) (0,0,1) — hω
2 1kΩ = RE
1 kΩ= RB
3
(0,0,0) — hω – 3V = VEE
2 +

Also, energy is 3-D potential, potential well is given as


3 By KVL on collector loop side, we have
E = (n x + n y + n z + ) hω VCB = VCC − IC RC
2
3 5 = 10 − 2.3 × 3
= 2 × hω + 6 × hω = 3hω + 15hω = 18 hω = 10 − 6.9 = 3.1 V
2 2
∴ E = 18hω 2m π ab
52. (1.74) Time period in an elliptical orbit is; T = ,
L
49. (0.09) From the given fig. it is clear that,
where, L = angular momentum
5 = Ie (1× 103 ) (QR = 1k Ω)
2 × 1 × π × 1000 × 100
∴ Ie = 5 mA Using given values,T =
100
+10V
= 2000 π sec = 174
. hours.
1 kΩ
+5V + Ib 53. (60) Orbital radius of moon is around

D = 3.84 × 108 m (Calculated from given data)
Ie
5V Also, R = 6.4 × 106m
β=50 D 3.84 × 108
1 kΩ so, ratio, = = 60
R 6.4 × 106
54. (10) We use Bernoulli’s Eq. with assumption that velocity
Also, we have,
of fluid in syringe is zero,
Ib + Ic = 5 mA
 P − P2 
or Ib (1+ β ) = 5 (Q Ic = βIb ) ∴ Voutflorw = 2  1 
 ρ 
or Ib (1+ 50) = 5
5 1
Ib = 2× × 105
51 = 2 = 10 m/s
∴ Ib = 0.09 mA 103
18 IIT JAM Physics Solved Papers & Practice Sets

55. (0.47) Only orange color is observed at angular position but, according to questions
of first order minima of blue color and vise-versa. L = (mR 2ω ) x …(iv)
 λ − λ 1  180 From Eq. (iii) and (iv), we get
∴ θ 2 − θ1 =  2 × degrees
 d  π x = − 5.28
( 600 − 450) × 10−9 × 180 57. (0.6) Using equation for contraction of length
=
18 × 10−6 × π (Lorentz-Fitzgerald contraction) we have,
150 × 10−3 × 180  v
1/ 2
= = 0.47 degrees  1− 
18 × π L′
= c
56. (– 5.28) A Galilean transformation is used to transform L0  1+ v 
 
 c
between the co-ordinates of two reference frames which 1/ 2
difference only by the constant relative another. These  v
1−
transformation together with spatial rotatory and 1  c
⇒ = 
translations is the space and time form the in 2  1+ v 
 
homogeneous Galilean group while without the  c
translation in space and time the group is the v
1−
homogeneous Galilean group. ⇒ c = 1 ⇒ 4 − 4v = 1+ v
For inertial frame s 1 1+
v 4 c c
x = R cos(ωt ) c
3 v v
y = R sin(ωt ) ⇒ = or = 0.6
5 c c
For s 2
v = cos R$i with respect to s 58. (– 0.44) Using Lorentz equations, we have time difference
1
w.r.t. frame S′ is
According to Galilean transformation .
[(β (x 2 − x1 )]
x ′ = x = vt t 2′ − t1′ = r (t 2 − t1 ) = r
c
= R cos(ωt ) + ωR ⋅ t 1 (x 2 − x1 )
y′ = y = ⋅β
(1 − β 2 )1/ 2 c
z′ = z
v
and t′ = t Here, β = = 0.8
dx ′ (t ) c
∴ v′ = = Rx − sin ωt × ω + ω ⋅ R × 1 1 108
dt ∴ t 2′ − t1′ = ⋅ ( 0.8 ) ⋅
= − ωR sin ωt + ωR (1 − (0.8)2 )1/ 2 3 × 108
⇒ v ′ = ω R (1 − sin ωt ) 0.8
= = 0.44
= ωR (1 − ω t ) …(i) 0.6 × 3
{for small angle sin θ ≈ θ} Also the time is delayed in S′.
As Angular momentum. ∴ t 2′ − t1′ = – 0.44
L = m v′ R …(ii) 59. (67.68) For a given circuit, VDC = 2 × Vmax
[Using value ofv′ in Eq. (ii)] = 2 × Vrms × 2
or L = m ω R (1 − ωt ) ⋅ R
= 2 × 2 × 24 ≈ 67.68 V
 2π   2π 
= m R 2ω 1 − ω ×  Qt=  h 2c 2   3 
2/ 3
 ω   ω  60. (5.8) Fermi energy, EF =   ×  × n 2/ 3
= mR ω (1 − 2π )
2  8mc 2   π 
= m R 2ω (1 − 6.28) Using the given values, we get
or L = m R 2ω ( − 5.28) …(iii) EF = 5.8 eV
Solved Paper
2017
IITJAM Physics
MM : 100 Time : 3 hrs

n
This test paper has a total of 60 questions carrying 100 marks. The entire question paper is divided into
three Sections A, B and C. All sections are compulsory. Questions in each section are of different types.
n
Section A contains Multiple Choice Questions (MCQ). Each MCQ type question has four choices out of
which only one choice is the correct answer. This section has 30 Questions and carry a total of 50 marks.
(Q.1 – Q.10) carry 1 mark each and (Q.11 – Q.30) carry 2 marks each.
n
Section B contains Multiple Select Questions (MSQ). Each MSQ type question is similar to MCQ but
with a difference that there may be one or more than one choice(s) that are correct out of the four given
choices. The candidate gets full credit if he/she selects all the correct choices only and no wrong
choices. This section has 10 questions (Q. 31 – Q. 40) and carry 2 marks each with a total of 20 marks.
n
Section C contains Numerical Answer Type Questions (NATQ). For these NAT type questions, the answer
is a real number which needs to be entered using the virtual numerical keypad on the monitor. No
choices will be shown for these type of questions. This section has 20 questions and carry a total of
30 marks. (Q.41 – Q. 50) carry 1 mark each and (Q.51 – Q.60) carry 2 marks each.
n
Do not write more than one answer for the same question. In case you attempt a subjective question
more than once, please cancel the answer(s) you consider wrong. Otherwise, the answer appearing last
only will be evaluated.
n
Clip board, log tables, slide rule, calculator, cellular phone and electronic gadgets in any form are not
allowed.

Section A Multiple Choice Questions (MCQs)


In these type of questions, each question has four choices (a), (b), (c) and (d) out of which only
one option is correct.
Q.1-Q.10 carry one mark each.
1. The dispersion relation for electromagnetic waves travelling in a plasma is given as
ω 2 = c 2k 2 + ω 2p , where c and ω p are constants. In this plasma, the group velocity is
(a) proportional to but not equal to the phase velocity
(b) inversely proportional to the phase velocity
(c) equal to the phase velocity
(d) a constant
2 IIT JAM Physics Solved Papers & Practice Sets

2. Consider the following circuit with two identical Si diodes. The input AC voltage waveform has
the peak voltage, VP = 2 V as shown in the figure.
R
P
Vin

2V ∼

t
R

The voltage waveform across PQ will be represented by


VPQ VPQ

(a) (b)
t t

VPQ VPQ

(c) (d)
t t

3. A current I = 10 A flows in an infinitely long wire along the axis of a hemisphere (see figure).
The value of ∫ (∇ × B) ⋅ d S over the hemispherical surface as shown in the figure is

I=10A

(a) 10 µ 0 (b) 5 µ 0 (c) 0 (d) 7.5 µ 0

4. Consider two, single turn, co-planar, concentric coils of radii R1 and R 2 with R1 >> R 2 . The
mutual inductance between the two coils is proportional to

R2

R1

(a) R1 / R2 (b) R2 / R1 (c) R22 / R1 (d) R12 / R2

5. If the Boolean function Z = PQ + PQR + PQRS + PQRST + PQRSTU, then Z is


(a) P Q + R (S + T + U ) (b) P Q
(c) P + Q (d) P + Q + R + S + T + U
Solved Paper 2017 ●
3

6. Shown in the figure is a combination of logic gates. The output values at P and Q are correctly
represented by which of the following?
1
P

Q
0

(a) 0 0 (b) 1 1 (c) 0 1 (d) 1 0

7. Which of the following is due to inhomogeneous refractive index of earth’s atmosphere?


(a) Red colour of the evening sun (b) Blue colour of the sky
(c) Oval shape of the evening sun (d) Large apparent size of the evening sun

8. For the three matrices given below, which one of the choices is correct?
 0 1  0 i 1 0
σ1 =  , σ 2 =   and σ 3 =  
 1 0  − i 0  0 −1
(a) σ 1σ 2 = − iσ 3 (b) σ 1σ 2 = iσ 3 (c) σ 1σ 2 + σ 2σ 1 = 1 (d) σ 3σ 2 = − iσ 1

9. A plane in a cubic lattice makes intercepts of a , a / 2 and 2a / 3 with the three crystallographic
axes, respectively. The Miller indices for this plane are
(a) (2 4 3) (b) ( 3 4 2 ) (c) (6 3 4) (d) (12 3)

10. Which one of the following schematic curves best represents the variation of conductivity σ of
a metal with temperature T?

σ σ
(a) (b)

0 T 0 T

σ σ
(c) (d)

0 T 0 T

Q. 11-Q. 30 carry two marks each.

11. Consider an inertial frame S′ moving at speed c / 2 away from another inertial frame S along
the common XX ′-axis, where c is the speed of light. As observed from S′, a particle is moving
with speed c / 2 in the y ′-direction, as shown in the figure. The speed of the particle as seen
from S is
c/2

y y′
c/2
S x S′ x′

(a) c / 2 (b) c /2 (c) 7 c / 4 (d) 3 c / 5


4 IIT JAM Physics Solved Papers & Practice Sets

12. Consider a uniform thin circular disc of radius R and mass M. A concentric square of side R / 2
is cut out from the disc (see figure). What is the moment of inertia of the resultant disc about
an axis passing through the centre of the disc and perpendicular to it?

R/2

MR 2  1  MR 2  1  MR 2  1  MR 2  1 
(a) I = 1 − 48 π  (b) I = 1 − 48 π  (c) I = 1 − 24 π  (d) I = 1 − 24 π 
4   2   4   2  

13. Consider a system of N particles obeying classical statistics, each of which can have an
energy 0 or E. The system is in thermal contact with a reservoir maintained at a temperature
T. Let k denote the Boltzmann’s constant. Which one of the following statements regarding the
total energy U and the heat capacity C of the system is correct?
NE NE e E / k T NE E NE eE/ k T
(a) U = and C = k (b) U = and C = k
1 + e E / kT k T (1 + e E / kT )2 k T 1 + e E / kT k T (1 + e − E / kT )2
NE NE 2 eE/ k T NE NE 2 eE/ k T
(c) U = and C = k (d) U = and C = k
1+ e E / kT
( k T ) (1 + e
2 E / kT 2
) 1+ e E / kT
( k T ) (1 + e E / kT )2
2

40
14. The integral of the vector A (ρ, φ,z ) = cos φ ρ$ (standard notation for cylindrical coordinates is
ρ
used) over the volume of a cylinder of height L and radius R 0 is
(a) 20πR0( $i + $j ) (b) 0 (c) 40πR0L $j (d) 40πR0L $i

15. Consider Rydberg (hydrogen-like) atoms in a highly excited state with n around 300. The
wavelength of radiation coming out of these atoms for transitions to the adjacent states lies in
the range
(a) gamma rays ( λ ~ pm ) (b) UV ( λ ~ nm )
(c) infrared ( λ ~ µm ) (d) RF ( λ ~ m )

16. A uniform rigid meter scale is held horizontally with one of its end at the edge of a table and
the other supported by hand. Some coins of negligible mass are kept on the meter scale as
shown in the figure.

As the hand supporting the scale is removed, the scale starts rotating about its edge on the
table and the coins start moving. If a photograph of the rotating scale is taken soon after, it will
look closest to

(a) (b) (c) (d)


Solved Paper 2017 ●
5

17. Consider two identical, finite, isolated systems of constant heat capacity C at temperatures T1
and T2 (T1 > T2 ). An engine works between them until their temperatures become equal. Taking
into account that the work performed by the engine will be maximum ( = W max ), if the process
is reversible (equivalently, the entropy change of the entire system is zero), then the value of
W max is
(a) C(T1 − T2 ) (b) C(T1 − T2 ) / 2
(c) C(T1 + T2 − T1T2 ) (d) C( T1 − T2 )2

18. A white dwarf star has volume V and contains N electrons, so that the density of electrons is
N
n= . Taking the temperature of the star to be 0 K, the average energy per electron in the
V
3h 2
star is ε 0 = ( 3 π 2n ) 2/ 3 , where m is the mass of the electron. The electronic pressure in the
10m
star is
1 2
(a) nε 0 (b) 2 nε 0 (c) nε 0 (d) nε 0
3 3

19. A pendulum is made of a massless string of length L and a small bob of negligible size and
mass m. It is released making an angle θ 0 (<<1 rad) from the vertical. When passing through
the vertical, then the string slips a bit from the pivot, so that its length increases by a small
amount δ(δ << L ) in negligible time. If it swings upto angle θ1 on the other side before starting
to swing back, then to a good approximation, which of the following expressions is correct?

θ0 θ1

L L+δ

m m

 δ  δ  3δ 
(a) θ 1 = θ 0 (b) θ 1 = θ 0 1 −  (c) θ 1 = θ 0 1 −  (d) θ 1 = θ 0 1 − 
 2 L  L  2 L

20. To demonstrate Bernoulli’s principle, an instructor arranges two circular horizontal plates of
radii b each with distance d (d << b ) between them (see figure). The upper plate has a hole of
radius a in the middle. On blowing air at a speed v 0 through the hole, so that the flow rate of
air is πa 2v 0 , it is seen that the lower plate does not fall. If the density of air is ρ, then the
upward force on the lower plate is well approximated by the formula (assume that the region
with r < a does not contribute to the upward force and the speed of air at the edges is
negligible)
a

b
πρv 02a 4  b πρv 02a 2b 2  b
(a) ln   (b) ln  
4d 2  a 4d 2  a
πρv 02d 4 b 2 πρv 02a 4  b
(c) ln   (d) ln  
2 ab  a d 2  a
6 IIT JAM Physics Solved Papers & Practice Sets

21. KCl has the NaCl type structure which is fcc with two atom basis, one at (0,0,0) and the other
 1 1 1
at  , ,  . Assume that the atomic form factors of K + and Cl − are identical. In an X-ray
 2 2 2
diffraction experiment of KCl, which of the following (h k l) peaks will be observed?
(a) (1 0 0) (b) (1 1 0) (c) (1 1 1) (d) (2 0 0)
df
22. Which one of the following graphs represents the derivative f ′ (x ) = of the function
dx
1
f (x ) = most closely (graphs are schematic and not drawn to scale)?
1+ x 2
f ′(x) f ′(x)

(a) (b)
x x

f ′(x) f ′(x)

(c) (d)
x x

23. In the radiation emitted by a black body, the ratio of the spectral densities at frequencies 2ν
and ν will vary with ν as
(a) [e hν / kBT − 1] −1 (b) [e hν / kBT + 1] −1 (c) [e hν / kBT − 1] (d) [e hν / kBT + 1]

24. Consider a thin long insulator coated conducting wire carrying current I. It is now wound once
around an insulating thin disc of radius R to bring the wire back on the same side, as shown in
the figure. The magnetic field at the centre of the disc is equal to
I

R
I

µ 0I µ 0I  2 µ 0I  2 µ 0I  1
(a)
2R
(b)
4R 3 + π  (c)
4R 1 + π  (d)
2R 1 + π 

25. Consider two coherent point sources (S1 and S 2 ) separated by a small distance along a
vertical line and two screens P1 and P 2 placed as shown in figure. Which one of the choices
represents the shapes of the interference fringes at the central regions on the screens?
P2

S1
S2

P1

(a) Circular on P1 and straight lines on P2 (b) Circular on P1 and circular on P2


(c) Straight lines on P1 and straight lines on P2 (d) Straight lines on P1 and circular on P2
Solved Paper 2017 ●
7

26. The electric field of an electromagnetic wave is given by


E = ( 2 k$ − 3 $j ) × 10 −3 sin[10 7 (x + 2y+3z − βt )]
The value of β is (c is the speed of light)
(a) 14 c (b) 12 c
(c) 10 c (d) 7 c

27. Unpolarised light is incident on a combination of a polariser, a λ / 2 plate and a λ / 4 plate kept
one after the other. What will be the output polarisation for the following configurations?
Configuration 1 : Axes of the polariser, the λ / 2 plate and the λ / 4 plate are all parallel to each
other.
Configuration 2 : The λ / 2 plate is rotated by 45° with respect to configuration 1.
Configuration 3 : The λ / 4 plate is rotated by 45° with respect to configuration 1.
(a) Linear for configuration 1, linear for configuration 2, circular for configuration 3
(b) Linear for configuration 1, circular for configuration 2, circular for configuration 3
(c) Circular for configuration 1, circular for configuration 2, circular for configuration 3
(d) Circular for configuration 1, linear for configuration 2, circular for configuration 3

28. For the Fourier series of the following function of period 2 π


0, − π < x < 0
f (x ) = 
1, 0< x < π

The ratio (to the nearest integer) of the Fourier coefficients of the first and the third
harmonic is
(a) 1 (b) 2
(c) 3 (d) 6

29. An n-p-n transistor is connected in a circuit as shown in the figure.


If IC = 1 mA, β = 50, VBE = 0.7 V and the current through R 2 is 10 IB , where IB is the base
current. Then, the ratio R1 / R 2 is
6V

R1 2.5 K

VC
VCE=3 V
VE=0.5 V

R2 0.5 K

(a) 0.375 (b) 0.25


(c) 0.5 (d) 0.275
8 IIT JAM Physics Solved Papers & Practice Sets

30. A rectangular loop of dimension L and width w moves with a constant velocity v away from an
infinitely long straight wire carrying a current I in the plane of the loop as shown in the figure
below. Let R be the resistance of the loop. What is the current in the loop at the instant, the
near side is at a distance r from the wire?
v

R w

L
r
I

µ 0IL wv µ 0IL wv µ 0IL wv µ 0IL wv


(a) (b) (c) (d)
2 πR r [ r + 2 w ] 2 πR r [2 r + w ] 2 πR r [ r + w ] 2 πR 2 r [ r + w ]

Section B Multiple Select Questions (MSQs)


In these type of questions, each question has four choices (a), (b), (c) and (d) out of which only
one or more than one is/are correct options.

Q. 31-Q. 40 carry two marks each.

31. A photon of frequency ν strikes an electron of mass m initially at rest. After scattering at an
angle φ, the photon loses half of its energy. If the electron recoils at an angle θ, then which of
the following is (are) true?
 mc 2 
(a) cos φ = 1 − 
 hν 
 mc 2 
(b) sinθ = 1 − 
 hν 
sin φ
(c) The ratio of the magnitudes of momenta of the recoiled electron and scattered photon is
sin θ
h
(d) Change in photon wavelength is (1 − 2 cos φ )
mc

32. For an atomic nucleus with atomic number Z and mass number A, which of the following is
(are) correct?
(a) Nuclear matter and nuclear charge are distributed identically in the nuclear volume
(b) Nuclei with Z > 83 and A > 209 emit α-radiation
(c) The surface contribution to the binding energy is proportional to A 2/ 3
(d) β-decay occurs when the proton to neutron ratio is large, but not when it is small

33. Consider a one-dimensional harmonic oscillator of angular frequency ω. If 5 identical particles


occupy the energy levels of this oscillator at zero temperature, which of the following statement
(s) about their ground state energy E 0 is (are) correct?
13 25
(a) If the particles are electrons, E0 = hω (b) If the particles are protons, E0 =

2 2
25 5
(c) If the particles are spin less fermions, E0 = hω (d) If the particles are bosons, E0 = hω
2 2
Solved Paper 2017 ●
9

34. For a point dipole of dipole moment p = p z$ located at the origin, which of the following is (are)
correct?
(a) The electric field at ( 0, b, 0) is zero
qp
(b) The work done in moving a charge q from ( 0, b, 0) to ( 0, 0, b ) is
4πε 0b 2
(c) The electrostatic potential at ( b, 0, 0) is zero
qp
(d) If a charge q is kept at ( 0, 0, b ), then it will exert a force of magnitude on the dipole
4πε 0b 3

35. A dielectric sphere of radius R has constant polarisation P = P 0 z$ , so that the field inside the
P0
sphere is E in = − z$ . Then, which of the following is (are) correct?
3ε 0
(a) The bound surface charge density is P0 cos θ
1
(b) The electric field at a distance r on the Z-axis varies as for r >> R
r2
PR
(c) The electric potential at a distance 2R on the Z-axis is 0
12ε 0
(d) The electric field outside is equivalent to that of a dipole at the origin

36. Consider a circular parallel plate capacitor of radius R with separation d between the plates
(d << R ). The plates are placed symmetrically about the origin. If a sinusoidal voltage
V = V 0 sinωt is applied between the plates, which of the following statement(s) is (are) true?
V02εω R
(a) The maximum value of the Poynting vector at r = R is
4d 2
(b) The average energy per cycle flowing out of the capacitor is zero
(c) The magnetic field inside the capacitor is constant
(d) The magnetic field lines inside the capacitor are circular with the curl being independent of r

 x2
37. The linear mass density of a rod of length L varies from one end to the other as λ 0 1+ ,
 L2 
where x is the distance from one end with tensions T1 and T2 in them (see figure), and λ 0 is a
constant. The rod is suspended from a ceiling by two massless strings. Then, which of the
following statement(s) is (are) correct?

T1 T2

0 L x

2 λ 0L
(a) The mass of the rod is
3
9L
(b) The centre of gravity of the rod is located at x =
16
7 λ 0Lg
(c) The tension T1 in the left string is
12
3λ 0Lg
(d) The tension T2 in the right string is
2
10 IIT JAM Physics Solved Papers & Practice Sets

38. An object of mass m with non-zero angular momentum L is moving under the influence of
gravitational force of a much larger mass (ignore drag). Which of the following statement(s) is
(are) correct?
(a) If the total energy of the system is negative, then the orbit is always circular
(b) The motion of m always occurs in a two-dimensional plane
(c) If the total energy of the system is zero, then the orbit is a parabola
(d) If the area of the particle’s bound orbit is S, then its time period is 2mS / L

39. A particle of mass m fixed in space is observed from a frame rotating about its Z-axis with
angular speed ω. The particle is in the frame’s xy -plane at a distance R from its origin. If the
Coriolis and centrifugal forces on the particle are FCOR and FCFG , respectively, then (all the
symbols have their standard meaning and refer to the rotating frame),
(a) FCOR + FCFG = 0 (b) FCOR + FCFG = −mω 2R r$
(c) FCOR = −2 mω 2R r$ (d) FCFG = −mω 2R r$

40. An isolated box is divided into two equal compartments by a partition (see figure). One
compartment contains a van der Waals’ gas while the other compartment is empty. The
partition between the two compartments is now removed. After the gas has filled the entire box
and equilibrium has been achieved, which of the following statement(s) is (are) correct?

(a) Internal energy of the gas has not changed


(b) Internal energy of the gas has decreased
(c) Temperature of the gas has increased
(d) Temperature of the gas has decreased

Section C Numerical Answer Type Questions (NATQs)


This section contains Numerical Answer Type (NAT) questions, for these NAT questions, the
answer is a real number.

Q.41-Q.50 carry one mark each.

41. An intrinsic semiconductor of band gap 1.25 eV has an electron concentration 1010 cm −3 at
300 K. Assume that its band gap is independent of temperature and that the electron
concentration depends only exponentially on the temperature. If the electron concentration at
200 K is Y × 10N cm −3 (1 < Y < 10; N = integer), then the value of N is ________ .

42. A particle of unit mass is moving in a one-dimensional potential V (x ) = x 2 − x 4 . The minimum


mechanical energy (in the same units as V (x ) above which the motion of the particle cannot be
bounded for any given initial condition is ________ .
(Specify your answer to two digits after the decimal point.)
Solved Paper 2017 ●
11

43. In a coaxial cable, the radius of the inner conductor is 2 mm and that of the outer one is 5 mm.
The inner conductor is at a potential of 10 V, while the outer conductor is grounded. The value
of the potential at a distance of 3.5 mm from the axis is ________ .
(Specify your answer in volts to two digits after the decimal point.)

10V

44. Sand falls on a conveyor belt at the rate of 1.5 kg/s. If the belt is moving with a constant speed
of 7 m/s, then the power needed to keep the conveyer belt running is ________ .
(Specify your answer in watts to two digits after the decimal point.)

45. A particle of mass m is placed in a three-dimensional cubic box of side a. What is the
 h 2π 2 
degeneracy of its energy level with energy 14  ?
 2ma 2 

(Express your answer as an integer.)

46. The wave number of an electromagnetic wave incident on a metal surface is ( 20π + 750 i ) m −1
inside the metal, where i = −1. The skin depth of the wave in the metal is ________ .
(Specify your answer in mm to two digits after the decimal point.)

47. Consider a Carnot engine operating between temperatures of 600 K and 400 K. The engine
performs 1000 J of work per cycle. The heat (in joules) extracted per cycle from the high
temperature reservoir is ________ .
(Specify your answer to two digits after the decimal point.)

48. Unpolarised light of intensity I 0 passes through a polariser P1. The light coming out of
the polariser falls on a quarter wave plate with its optical axis at 45° with respect to
the polarisation axis of P1 and then passes through another polariser P 2 with its polarisation
axis perpendicular to that of P1. The intensity of the light coming out of P 2 is I. The ratio I 0 / l
is ________ .
(Specify your answer to two digits after the decimal point.)

49. An anti-reflection film coating of thickness 0.1 µm is to be deposited on a glass plate for normal
incidence of light of wavelength 0.5 µm. What should be the refractive index of the film?
(Specify your answer to two digits after the decimal point.)
12 IIT JAM Physics Solved Papers & Practice Sets

50. Intensity versus distance curve for a double slit diffraction experiment is shown in the figure
below. If the width of each of the slits is 0.7 µm, then what is the separation between the two
slits in micrometers?
(Specify your answer to two digits after the decimal point.)

Intensity
Distance

Q. 51-Q.60 carry two marks each.

51. The volume integral of the function f (r , θ, φ) = r 2 cos θ over the region ( 0 ≤ r ≤ 2, 0 ≤ θ ≤ π / 3
and 0 ≤ φ ≤ 2π) is ________ .
(Specify your answer to two digits after the decimal point.)
R
52. A sphere of radius R has a uniform charge density ρ. A sphere of smaller radius is cut out
2
from the original sphere, as shown in the figure below. The centre of the cut out sphere lies at
R R
z = . After the smaller sphere has been cut out, the magnitude of the electric field at z = −
2 2
ρR
is . The value of the integer n is ________ .
nε 0
R
2

53. In planar polar coordinates, an object’s position at time t is given as (r , θ) = (e t m, 8t rad). The
magnitude of its acceleration in m/ s 2 at t = 0 (to the nearest integer) is ________ .

54. Consider two particles moving along the X-axis. In terms of their coordinates x 1 and x 2 , their
dx 1 dx
velocities are given as = x 2 − x 1 and 2 = x 1 − x 2 , then respectively. When they start
dt dt
moving from their initial locations of x 1( 0) = 1 and x 2 ( 0) = − 1, then the time dependence of
both x 1 and x 2 contains a term of the form e at , where a is a constant. The value of a (an
integer) is ________ .
Solved Paper 2017 ●
13

55. For a proton to capture an electron to form a neutron and a neutrino (assumed massless), the
electron must have some minimum energy. For such an electron, the de-Broglie wavelength in
picometers is ________ .
(Specify your answer to two digits after the decimal point.)

56. Starting with the equation TdS = dU + pdV and using the appropriate Maxwell’s relation along
∂ p
with the expression for heat capacity C p (see useful information), the derivative   for a
 ∂TS
substance can be expressed in terms of its specific heat c p , density ρ, coefficient of volume
expansion β and temperature T. For ice, c p = 2010 J/kg- K, ρ = 10 3 kg/m 3 and
 ∂p 
β = 1.6 × 10 −4 / ° K. If the value of   at 270 K is N × 10 7 Pa/K, then the value of N is
 ∂T  S
________ .

(Specify your answer to two digits after the decimal point.)

57. In an electron microscope, electrons are accelerated through a potential difference of 200 kV.
What is the best possible resolution of the microscope?
(Specify your answer in picometers to two digits after the decimal point.)

58. Consider the differential equation y ′′ + 2y ′ + y = 0. If y ( 0) = 0 and y ′ ( 0) = 1, then the value of


y ( 2) is ________ .
(Specify your answer to two digits after the decimal point.)

59. An op-amp is connected in a circuit with a Zener diode as shown in the figure. The value of
resistance R in k Ω for obtaining a regulated output V 0 = 9 V is ________ .
(Specify your answer to two digits after the decimal point.)
1kΩ

R
V0
1kΩ
Vin=12V Vz=4.7 V

60. At t = 0, a particle of mass m having velocity v 0 starts moving through a liquid kept in a
 dx 
horizontal tube and experiences a drag force Fd = − k  . It covers a distance L before
 dt 
coming to rest. If the times taken to cover the distances L/ 2 and L/ 4 are t 2 and t 4 respectively,
then the ratio t 2 / t 4 (ignoring gravity) is ________ .
(Specify your answer to two digits after the decimal point.)
Answers with Solutions
Section A Multiple Choice Questions
1. (b) From the definition of group velocity, Case II For reversed biased,
dω R
vg = …(i) P
dk
Given that,
ω 2 = c 2k 2 + ω p2
1
VP
⇒ ω = (c 2k 2 + ω p2 ) 2 …(ii)
On differentiating Eq. (ii) w.r.t. k, we get R
dω 1 1
= × 2c 2k 2 …(iii) Q
dk 2 (c 2k 2 + ω p2 )1/ 2
By using formula, Vout = VP − V0
Putting the value from Eq. (iii) into Eq. (i), we get Vout = 2 − 0.7 = 1.3 V …(ii)
1 1 From Eqs. (i) and (ii), It is clear that, the voltage
vg = × 2c 2k 2
2 (c 2k 2 + ω 2p )1/ 2 waveform across PQ will be represented by
1
= × 2c 2k [from Eq. (ii)]
2ω VPQ
c 2k
=
ω t
c2
= 3. (a) From Ampere’s circuital law in integral form,
ω
k ∫ B ⋅ d l = ∫ ( ∇ × B ) ⋅ dS = µ I
0

c2  ω S
vg = Qv =
vp  P k  ∴ ∫ ( ∇ × B) ⋅ ds = 10µ 0

where, v p = phase velocity 10A


1
∴ vg ∝
vp Amperion loop
Hence, group velocity (vg ) is inversely proportional to (base of hemisphere)
the phase velocity (v p ).
2. (c) We know that the voltage drop across each Si diode is
V0 = 0.7 V 4. (c) Suppose the current flow in outer ring is I1.
Case I For forward biased,
R1
R
P O
R2

VP
Then, we know that the magnetic field at the centre of
R the ring due to I1 is
µI  µ I
Q B1 = 0 1 QB = 0 …(i)
2R1 
 2R 
By using formula, Vout = VP − 2V0 Since R1 >> R2, we approximate the magnetic field
Vout = 2 − 2 × 0.7 [given Vp = 2 V ] through the entire inner coil be B1.
Vout = 2 − 1.4 Hence, the flux through the inner coil is
Vout = 0.6 V …(i) φ 21 = B1A2 …(ii)
Solved Paper 2017 ●
15

Here, A2 = area of inner ring 0 1


8. (a) Given, σ1 =  
∴ A2 = πR22 …(iii)  1 0
On putting the values from Eqs. (i) and (iii) into Eq. (ii), 0 i 1 0 
we get σ2 =  and σ 3 = 
− i 0 
0 − 1
µI
φ 21 = 0 1 π R22 …(iv) 0 1  0 i  − i 0
2R1 Now, σ1 ⋅ σ 2 =   = 
 1 0 − i 0  0 i 
Thus, the mutual inductance is given by
1 0 
φ = −i  
M = 21 0 − 1
I1
σ1σ 2 = − i σ 3 , property of Pauli spin matrix.
µ 0 π R12
⇒ M = 9. (a) Ratio of intercepts,
2R1
a 2a
R2 ( pa : qb : rc ) = a : :
⇒ M ∝ 2 2 3
R1 Ratio of coefficients for intercepts,
1 2
5. (c) For Boolean algebra, 1 + A = 1 ( p : q : r ) = 1: :
2 3
∴ 1 + A + B + C + D + .... = 1 Their reciprocals,
A + B = A ⋅B  1 1 1 3
 : :  = 1: 2 :
p q r 2
A ⋅B = A + B
Multiply from LCM for denominator,
So, according to question,
1 1 1  3
Z = PQ + PQR + PQRS + PQRST + PQRSTU  × LCM : × LCM : × LCM = 1× 2 : 2 × 2 : × 2
p q r  2
= PQ (1 + R + RS + RST + RSTU ) Hence, the Miller indices of the plane = ( 2, 4, 3).
Z = PQ [Q1 + R + RS + RST + RSTU = 1]
10. (a) We know that, conductivity of metal (conductor),
∴ Z = PQ = P + Q ne 2τ
σ=
6. (c) m
1 1· x =1 = 0
P Here, τ = relaxation time
x
n = number of electrons
e = charge of electron
0
m = mass of electron
x Q We also know that relaxation time decrease with
0
increase in temperature, hence conductivity of metals
Let output at Q is x,  1
decrease rapidly  2  with temperature, but in
r 
i.e. x =Q …(i)
relaxation time conductivity is same.
Now, from first AND gate operation,
For τ
for Boolean algebra, (in relaxation time)
 1⋅ x = 1 
(1⋅ x ) = P  1 = 0
 T>τ
 1
  
1 =P   r2
0 =1
 
  c
11. (c) In frame S, ux = 0 , uy =
0 =P …(ii) 2
From second NAND gate operation, In frame S′ , u x′ = ? and u y′ = ?
( 0 ⋅ 0) = Q By the Lorentz transformation of velocity,
u −v
0 =Q u x′ = x
uv
1= Q …(iii) 1 − x2
c
Hence, P = 0, Q = 1  c
0 − − 
7. (c) The rays of light from upper and lower edges of sun  2
u x′ =
bend unequally due to inhomogeneous refractive index  −c 
0 
of earth’s atmosphere. Due to unequal bending of light,  2
1−
the image of sun appears oval and larger in size. c2
16 IIT JAM Physics Solved Papers & Practice Sets

c ∴ I = I1 − I 2
c MR 2
u x′ = 2 = 1
I = MR 2 −
1− 0 2 2 96π
v2 1 2  1 
uy 1− I = MR 1 −
c2 2  48π 
u y′ =
uv
1 − x2 13. (d) We know that from Boltzmann’s distribution,
c −E

 −c 
2 Ni = N0e kT …(i)
 
 2 But N0 + Ni = N
c /2 1 − E
u y′ = c2 ∴ N0 + N0e

kT
=N
 c 
0 −  −
E
 2 N0 (1 + e kT
)=N
1−
c2 N
⇒ N0 = …(ii)
c 1  E

1− −
1 + e kT 
′ 2 4 3
uy = = c  
1− 0 4
From Eqs. (i) and (ii), we get
So, resultant velocity is E
N −
v = (u x′ )2 + (u y′ )2 Ni = E
⋅ e kT …(iii)

2 2 1 + e kT
c   3 
=   + c ∴ U = N0 × 0 + Ni × E
 2  4  U = Ni × E
E
c2 3 2 1 3 N −
= + c =c + U = E
⋅ e kT ⋅ E [from eq. (iii)]
4 16 4 16 −
1 + e kT
7
v =c NE NE
4 = =
1 e −E /kT e +E /kT + 1
+ −E /kT
12. (b) We know that, moment of inertia of disc, e −E /kT e
1 NE
I1 = MR 2 …(i) U =
2 E

1 + e kT
40
14. (d) Given, A (ρ, φ, z ) =cos φρ$ …(i)
ρ
R We know that in cylindrical coordinates system,
O
ρ$ = cos φ $i + sin φ $j …(ii)
R/2 $ $
Here, i and j are unit vectors in Cartesian coordinates
We also know that, moment of inertia of square, system.
1 From Eqs. (i) and (ii), we get
I 2 = M ′ L2 …(ii) 40
6 A (ρ, φ, z ) = cos φ (cos φ$i + sin φ $j )
ρ
Here, M′ = mass of square 40 40
M′ = area of square × σ A (ρ, φ, z ) = cos2 φ $i + cos φ sin φ $j …(iii)
ρ ρ
R2 M  M  On taking volume integral of Eq. (iii),
M′ = × Qσ =
4 πR 2 
 πR 2  L 2π R 0
40
∫ ∫0 ∫0 ∫0 ρ cos φρ dρ dφ dz i
A (ρ, φ, z ) d τ = 2 $
M
M′ = …(iii)
4π L 2π R 0
40
∴ L=
R
…(iv)
+ ∫∫∫ ρ
cos φ sin φρ dρ dφ dz $j
2 0 0 0

On putting the values from Eqs. (iii) and (iv) into Eq. (ii), [Q cos φ sin φρ dρ dφ dz $j = 0]
L 2π R 0
we get 40
1 M R2 ∫ A (ρ, φ, z ) dτ = ∫ ∫∫
ρ
cos2φ ρ dρ dφ dz $i …(iv)
I2 = × 0 0 0
6 4π 4
∫ A(r , φ, z ) dτ = 40πR0L i
$
MR 2
I2 = …(v) So, volume of cylinder is 40πR L $i.
96π 0
Solved Paper 2017 ●
17

15. (d) We know that for hydrogen like atom, 2L


Till r = , velocity of point on rod is less than velocity
hc 1 1  3
= 13.6  2 − [formula of transition] of coin.
λ  n (n + 1)2 
So, coin be in contact with rod there after coin will get
  detached from rod.
hc 13.6  1 
= 2 1 − 2
 17. (d) Consider unit mass substance (m = 1) .
λ n   1 
 1 +   Temperature becomes equal when θ1 = θ 2
  n 
C(T1 − Tf ) = C(Tf − T2 )
hc 13.6   1 
−2
T + T2
= 2 1 − 1 +   ⇒ Tf = 1
λ n   n  2
hc 13.6   T 
2  Entropy, S1 = C ln  f 
= 2 1 − 1 − + ……  T1 
λ n   n 
hc 13.6  2  T 
= 2 S 2 = C ln  2 
λ n n   Tf 
hc 27.2 For reversible process, ∆S = S1 − S 2 = 0
= 3
λ n T  T 
C ln  f  − C ln  f  = 0
(hc )n 3 T1  Tf 
⇒ λ= …(i)
27.2 Tf 2
Q hc = 1240 eV - nm …(ii) C ln =0
TT
1 2
Given, n = 300
Tf 2
From Eq. (i), we get ⇒ ln =0
1240 × ( 300)2 1240 × 9 × 104 TT
1 2
λ= nm = nm Tf 2
27.2 27.2 = e0
= 1.2 × 109 nm TT
1 2

λ = 1.2 m Tf 2
⇒ =1
λ~m TT
1 2

16. (b) Angular acceleration given by Tf = TT


1 2

τ = Iα ∴Work done, W = Q1 − Q 2
τ = C (T1 − Tf ) − C (Tf − T2 )
⇒ α= …(i)
I = C {T1 − 2Tf + T2]
∴Moment of inertia of the rod, = C (T1 − 2 TT 1 2 + T2 )
1
I = ML2 …(ii) W = C ( T1 − T2 )2
3
∴ τ = F ⋅r N
18. (d) Given that, n =
L V
τ = Mg ⋅ …(iii)
2 E
Average energy, E0 =
Putting these values from Eqs. (ii) and (iii) into Eq. (i), N
we get ⇒ E = NE0
L Work done, dE = − pdV
Mg ⋅
α= 2  ∂E 
1 2 We know that, pressure, p = −  
ML  ∂V 
3 2

3g 3 h2  2 N  3
α= E =N⋅  3π 
2L 10 m  V
We know that, angular velocity, 3 h2
E =N ( 3π 2N )2/ 3 V −2/ 3 …(i)
ω = αt 10m
3g  ∂E 
ω= t p = −  …(ii)
2L  ∂V 
Relation between angular and linear velocity is On differential Eq. (i) w.r.t. V and put in Eq. (ii), we get
v = ωr 3 h2  2 − 5
3g p = −N ( 3π 2N )2/ 3  − V 3 
v = tr 10m  3 
2L
18 IIT JAM Physics Solved Papers & Practice Sets

3 h2  2  20. (a) Let v 0 and v1 are velocity at r = a and r = r


p = −N ×   ( 3π 2N )2/ 3 V −2/ 3 V −1
10m  3 Apply continuity equation,
2
2 3h2  2 N  3 −1 ( 2πrd ) v1 = πa 2v 0
= N  3π  V a 2v 0
3 10m  V v1 = …(i)
2 N 3h2  N  2rd
= ( 3π 2n )2/ 3 Q =n Pressure difference,
3 V 10m  V 
1 2
2 ∆p = p1 − p 0 = ρv 1
p = nE0 2
3 2
3 h2 1  a 2v 0 
where, E0 = ( 3π 2n )2/ 3 ∆p = ρ  [from Eq. (i)]
10m 2  2rd 
ρa 4v 02
19. (b) ∆p = …(ii)
8 r 2d 2
L θ0 θ1
∴ Force, dF = ∆p × Area
m
L+δ dF = ∆p × 2πr dr
ρ a 4v 2
dF = 2 20 ⋅ 2πr dr [from Eq. (ii)]
m 8r d
ρπ a 4v 02
dF = dr …(iii)
4d 2r
b

Net force, F = ∫ dF
1
Kinetic energy at mean position is mgL (1 − cos θ 0 ) a
2
ρπa 4v 02  dr 
b

Potential energy at mean position is F =∫  


4d 2  r 
1 a
mg(L + δ )(1− cos θ1 ) ρπ a 4v 02
2 = (ln r )ba
By the law of conservation of energy, 4d 2
ρπ a 4v 02
1 1 = (ln b − ln a )
mgL(1 − cos θ 0 ) = mg(L + δ ) (1 − cos θ1 ) 4d 2
2 2
ρ πa 4v 02  b 
⇒ L(1 − cos θ 0 ) = (L + δ ) (1 − cos θ1 ) F = ln 
4d 2  a 
θ θ
⇒ L sin2 0 = (L + δ ) sin2 1
2 2 21. (d) From Bragg’s diffraction condition,
 θ 2 d h k l sin θ = nλ
Q 1 − cos θ = sin2
 2  1
2⋅ sin θ = nλ
 θ 
2
 θ
2
h2 + k 2 + l2
⇒ L  sin 0  = (L + δ )  sin 1 
 2  2 ⇒ sin2 θ ∝ (h 2 + k 2 + l 2 )
2 2
θ  θ  if θ = 0, then For fcc lattice,
⇒ L 0  = (L + δ )  1   sin θ = θ 
 2  2   peaks observed, if
θ 20 θ2 h 2 + k 2 + l 2 = 3, 4, 8, 11, 12, 14
⇒ L = (L + δ ) 1
4 4 ⇒ (hkl ) − plane = (111) ( 200) ( 220)…
∴ θ1 =
2 L
θ 20 ⇒ (hkl ) = ( 200)
(L + δ ) 1
1
22. (a) Given, f (x ) = ⇒ fmax at x = 0
⇒ θ12 = θ2 1+ x 2
 δ 0
1 +  f ′(x)
 L
1

 δ 2
⇒ θ1 = 1 +  θ 0
 L x
 δ −2x
⇒ θ1 = 1 −  θ 0 f ′ (x ) =
 2L (1 + x 2 )2
Solved Paper 2017 ●
19

f ′ at x = − 1 26. (a)Given that, the electric field of an electromagnetic wave


⇒ f ′ (x ) =  max
 f ′ max at x = 1 is
f ′(x) E = ( 2k$ − 3$j ) × 10−3 sin [107(x + 2y + 3z − βt )] …(i)
We know that, general equation of electromagnetic
x=1 wave is
x = –1 x $ = E sin(k$ ⋅ r$ − ωt )
E …(ii)
0

On comparing Eqs. (i) and (ii), we get


E = ( 2 k$ − 3$j ) × 10−3
0

23. (b) We know that, spectral density at frequency, (k$ ⋅ r$ − ωt ) = 107(x + 2y + 3z − βt )


8 π hν 3 1 where, k$ ⋅ r$ − ωt is phase
u( ν ) = …(i)
c 3 (e hν /k BT − 1) and k = k x2 + k y2 + k z2 = (1)2 + ( 2)2 + ( 3)2
8 πh ( 2 ν ) 3
1
u( 2ν ) = …(ii) k = 1 + 4 + 9 = 14
c3 (e h 2ν /k BT − 1)
We also know that,
On dividing Eq. (ii) by Eq. (i), we get
ω
8 π h ( 2 ν )3 1 k =
h 2ν /k BT c
u( 2ν ) c 3
(e − 1)
= ⇒ ω = k .c
u( ν ) 8 π h ν3 1
c3 (e hν /k BT − 1) ω = 14 ⋅ c
hν /k BT
u( 2ν ) e −1 From Eqs. (i) and (ii), we get
⇒ ∝ hν /k BT ∝ (e hν /k BT + 1)−1
u( ν ) (e − 1) ωt = β t
u( 2ν ) 1 ω =β
= = [e hν /k BT + 1] −1
u( ν ) (e hν /k BT + 1) So, β = 14 c
24. (b) λ
Semi-circular loop
Long wire 27. (a) In plate,
I
2
λ
= + Path difference =
I 2
Circular loop 2π λ
Long wire Phase difference = ⋅ =π
λ 2
Total magnetic field,
λ
B = Bcircular loop + Bsemi-circular loop + 2 Blong wire In plate,
4
µI 1 µ 0I µI λ
B= 0 + +2 0 Path difference =
2R 2 2R 4 πR 4
µ 0I  2 2π λ π
B= 2 + 1+  Phase difference = ⋅ =
4R  π λ 4 2
µ 0I  2 Configuration 1
B= 3 + 
4R  π Linearly
polarised
25. (a) From Young double slit experiment,
Unpolarised Linearly Linearly
light polarised polarised
Screen P1
Polariser λ/2 plate
λ/4 plate
S1 Configuration 2
S1 S2

S2

Circular frings 45°


Linearly Linearly
polarised polarised
Screen P2
Straight line frings λ/2 plate
20 IIT JAM Physics Solved Papers & Practice Sets

Configuration 3 30. (c)


R
w

L w+r
45° x r
I
Linearly Circularly
polarised polarised µ 0I
Magnetic flux, dφ = BL dx = L dx
2 πx
λ/4 plate r +w µ I
φ = ∫ dφ = ∫ 0
L dx
r 2 πx
0 − π < x < 0 µ I L r +w µ IL
28. (c) Given, f (x ) =  = 0 ∫ dx /x = 0 [log x ]rr +w
1 0 < x < π 2π r 2π
π µIL
1 φ = 0 [log(r + w ) − log r ]
π −∫π
Coefficient, a n = f (x ) cos nπ dx 2π
dφ dφ dr dφ  dr 
π Q Induced emf, ε = = ⋅ =v Q =v
1  dt 
π ∫0
= 1 × cos nπ dx dt dr dt dt
µ ILv  1 1
1 1 ε= 0  − 
= [sin nπ ] π0 = [sin nπ − sin 0° ] 2π r + w r 
nπ nπ |ε|
1 Induced current, i =
= [ 0 − 0] = 0 R

π µ 0I L  vw 
1 i=  
Coefficient bn = ∫ f (x ) sin nx dx 2πR  r (v + w )
π −π
0 π
1 1
π −∫π
bn = f (x ) sin nx dx + ∫ f (x ) sin nx dx
π 0
1 1 π 0
Section B Multiple Select Questions
= ( 0) + ∫ 1⋅ sin nx dx [Q ∫ f (x ) sin nx dx = 0]
π π − π − π 31. (a,c)
1
= [ − cos nπ ] π0
nπ hν′ Photon y
1  0 for even number
bn = [1 − cos nπ ] =  2 φ
nπ for odd number


θ x
2 2 b
∴ b1 = , b3 = ⇒ 1 =3
π 3π b3
VE 0.5
29. (d)∴ IE = = = 1mA
RE 0.5 K
e– Electron
I 1
IB = C = mA Photon loss of half energy,
β 50
hc 1 hc
On applying KVL in the given circuit =
λ′ 2 λ
6 − R1 × 11IB − R2 × 10IB = 0 1 1
11R1 + 10R2 = 300 … (i) ∴ =
λ + λ 0(1 − cos φ ) 2λ
and 6 − 11IB × R1 − 0.7 − 0.5 = 0 … (ii)
h
On solving above equations ⇒ 2λ = λ + (1 − cos φ )
m0c
4.8 × 50
R2 = = 218
. kΩ h
11 λ= (1 − cos φ)
and R1 = 6.02 m0c
R2 218 . c h  c
Now, = = (1 − cos φ ) Qλ =
R1 6.02 ν m0c  ν 
R2 m0c 2
= 3.62 (1 − cos φ ) =
R1 hν
R1 6.02 m c2
and = = 0.275 cos φ = 1 − 0
R2 218 . hν
Solved Paper 2017 ●
21

Conserving linear momentum in y-direction, (d)


h β–1 decay
0= sin φ − p sin θ
λ′ N
=1
p sin φ p
⇒ = Neutron (N)
(h / λ ′ ) sin θ
Conserving linear momentum in x-direction,
h h β+ decay
= cos φ + p cos θ
λ λ′
2h h  λ′  Proton (p)
⇒ = cos φ + p sin θ Qλ =
λ′ λ′  2  N
sin φ β = decay occurs when >1
⇒ 2 = cos φ + cos θ ⇒ θ≠φ p
sin θ
N>p
32. (b,c) Nuclear matter and nuclear charge are distributed This is incorrect statement.
identically. 33. (a,c,d) Energy of one-dimensional harmonic oscillator is
209
(b) Nuclei Z > 83 < A > 209, i.e. Bi, higher nucleus emits  1
En = n +  hω
53  2
α-particles to gain stability as shown by binding energy 1
curve. Fermions → electrons and proton (spin )
2
(c) Surface energy, ES = aSA 2/ 3 Spin less fermions → Classical partical
• Fermions Follow Pauli exclusive principle (i.e. no
two particles in same state)
• Classical particles (Maxwell-Boltzmaan particles)
→ Atleast one particle in a state

• Bosons All particles can be same (lowest) state.

n=4 n=4 n=4


n=3 n=3 n=3
n=2 n=2 n=2
n=1 n=1 n=1
n=0 n=0 n=0

Fermions Classical Boson

 1  1  1  1
E n = 2  0 +  hω E n =  0 +  hω + 1 +  hω E n = 5  0 +  hω
 2  2  2  2
 1  1  1
+ 2 1 +  hω + 1 2 +  hω +  2 +  hω
 2  2  2
 1  1
+  3 +  hω +  4 +  hω
 2  2

13 25 5
En = hω En = hω En = hω
2 2 2
22 IIT JAM Physics Solved Papers & Practice Sets

34. (b,c) We know that, electric field at any point due to dipole, 36. (a,c,d) (a) Displacement current density,
p ε 0 ∂E ε 0 ∂V  V 
E= [ 2 cos θ r$ + sin θ θ$ ] Jd = = QE = 0
4πε 0r 3 ∂t d ∂t  d 
and electric potential, ε0 d
= (V0 sin ωt ) [QV = V0 sin ωt ]
p cosθ d dt
V =
4πε 0r 2 ε V ω cos ωt
Jd = 0 0
(a) At point ( 0, b, 0), θ = 90° d
p ∴Magnetic field intensity,
E= [ 0 + θ$ ] I J ⋅ π R 2 Jd R
4πε 0r 3 H= = d =
p 2πR 2 πR 2
= θ$ ≠ 0 ε 0V0ωR cos ωt
4πε 0r 3 =
2d
So, option (a) is wrong.
Poynting vector,
(b) At point ( 0, b, 0), θ = 90° and at point ( 0, 0, b ); θ = 0° S =E ×H
Vinitial = 0 , V ε V ωR cos ωt  V
S = × 0 0 QE =
Vfinal =
p d 2d  d 
4πε 0b 2 V0 sin ωt ε 0V0ωR cos ωt
∴ S = × [QV = V0 sin ωt ]
Work done in moving charge q from ( 0, b, 0) to ( 0, 0, b ), d 2d
 p  ε V ωR 2

W = q(V final − Vinitial ) = q  − 0 S = 0 0 2 sin ωt cos ωt


 4πε 0b 2  2d
ε 0 V02 ωR
=
qp S = sin( 2 ωt ) [Q 2 sin θ cos θ = sin 2θ]
4πε 0b 2 4d 2
ε V 2 ωR
So, option (b) is correct. S max = 0 0 2 × 1 [Q sin( 2ωt )max = 1]
4d
(c) At point (b, 0, 0), θ = 90° ε V 2ωR
p cos90° S max = 0 0 2
V = =0 4d
4πε 0b 2
Hence, option (a) is correct.
So, option (c) is correct.
(b) Average energy flowing out of capacitor is zero,
(d) There is no force of dipole due to charge, so option (d) because capacitor stores energy but it is not dissipated.
is wrong. Hence, option (b) is correct.
35. (a,c,d) (c) Magnetic field inside capacitor varies with distance r,
(a) Bound surface charge density, i.e. B = B(r ) ≈ B0r
Q P$ = P 2$  Hence, option (c) is wrong.
$ ⋅ n$
σb = P 0
$ $  (d) Magnetic field lines are circular.
n = 2 cos θ 
Hence, option (d) is correct.
∴ σb = P0 2$ ⋅ 2$ cos θ
σb = P0 cos θ( 2$ ⋅ 2$ ) = P0 cos θ
Hence, option (a) is correct.
P
(b) E r >>R = Ein + 0
2ε 0
−P
Q Ein = 0
3ε 0  x 2
37. (b,c) Given, λ = λ 0 1 + 
−P P  L2 
So, E r >>R = 0 + 0 = 0
3ε 0 3ε 0 Centre of
Option (b) is wrong. T1 gravity T2
(c) Option (c) is correct.
xCM xCM
(d) Outside, total charge, q = 0
∴Electric field outside, E = 0 0 L x
(L – xCM)
Dipole eorigin = 0
–q (0,0) –q L L
 x 2
Mass, M = ∫ λ dx = ∫ λ 0 1 + 2  dx
Hence, option (d) is correct. 0 0
 L 
Solved Paper 2017 ●
23

L
 x3  4 39. (b,c) ω
M = λ 0 x + 2  = λ 0L …(i) ω
3 FCFG
 L 0 3 FC
Hence, option (a) is wrong.
Centre of gravity (mass x CM ) R
θ
∫ x dm = 1 x dm O
x CM = ∫
∫ dm M
L
 x 2
∫ x λ 0 

1 +  dx
L2  9L
x CM = 0 = …(ii)
4 16
λ0 L Total force on an object on earth surface due to
3 rotation,
Hence, option (b) is correct. Fi = Fr + FCOR + FCFG
For equilibrium,
We know that,
4
T1 + T2 = Mg = λ 0Lg …(iii) F COR = − 2m (ω × v ) = − 2mω 2R r$
3
and FCFG = m(ω × ω × R ) = mω 2R r$
and T1x CM = T2 (L − x CM )
∴ FCOR + FCFG = − 2 mω 2Rr$ + mω 2Rr$
From Eqs. (i) and (ii), we get
= − mω 2Rr$
 9L  9L  7L
T1   = T2 L −  = T2   So, option (b, c) are correct.
 16   16   16 
7 40. (a,d) By the energy relation,
⇒ T1 = T2 …(iv)
9  ∂U   ∂p 
  =T   −p …(i)
On solving Eqs. (iii) and (iv), we get  ∂V  T  ∂T  V
7 By van der Walls’ gas equation,
T1 = λ 0Lg
12  a
3λ 0Lg  p + 2  (V − b ) = RT
and T2 =  V 
4 RT a
Hence, option (c) is correct and option (d) is wrong. p= −
V −b V2
38. (b,c,d) Trajectory of a satellite based on total energy E.  ∂p  R
⇒   = ...(ii)
 ∂T  V V − b
Energy E Orbit
From Eqs. (i) and (ii), we get
E>0 Hyperbola  ∂U   R 
  =T   −p
 ∂V  T V − b 
E =0 Parabola  ∂U   a a
∴   = p + 2 − p = 2
 ∂V  T  V  V
E<0 Ellipse
 ∂U  a
⇒   =
 ∂V  T V 2
Option (a) is wrong and option (c) is correct.
We know that the motion is under central force is U decrease with volume.
always in a plane. We know that temperature decrease with volume.
Hence, option (b) is correct. So, option (a,d) are correct.
dS L
Areal velocity = =
dt 2m
L Section C Numerical Answer Type Question
dS = dt −E g /kT
m 41. (4)∴ n = n 0e …(i)
L T Ist condition,
⇒ ∫ dS = m ∫0dt −
1.25 eV

LT 2mS 1010 = n 0 e k ⋅ 300


…(ii)
S = = IInd condition,
2m L
1.25 eV

Hence, option (d) is correct. Y × 10N = n 0e k ⋅ 200
…(iii)
24 IIT JAM Physics Solved Papers & Practice Sets

On dividing Eq. (iii) by Eq. (ii), we get 20


1.25 eV
C2 = − …(ii)
− 3
Y × 10N n 0 e k ⋅ 300
= 1.25 eV
C1 = 5 C2
1010 −
 20
n 0 e k ⋅200 C1 = 5 ×  − 
1. 25 eV  1 1   3
Y × 10N −  − 
= e k  200 300  …(iv) 100
10 10
C1 = − …(iii)
3
Q Boltz mann constant k = 1.38 × 10−23
Putting these values from Eqs. (ii) and (iii) into Eq. (i),
1 eV = 1.6 × 10−19 J
we get
Put the above values in Eq. (iv), we get 100 20
1. 25 × 1. 6 × 10−19  1  V (r ) = −
Y × 10N −
1. 38 × 10−23
 
 6 3r 3
=e
1010 100
V( 3.5) = − 20

1. 25 × 1. 6
× 104 −
2
× 104 7
=e 1. 38 × 6
=e 1. 38 × 6 3×
2
Y × 10N 200 20
V( 3.5) = −
4
= e − 0.2415 × 10
1010
21 3
By the solution above equation, we get 20
V ( 3.5) =
N=4 7
V( 3.5) = 2.85
42. (0.25) Given, V (x ) = x 2 − x 4 = x 2 (1 − x 2 )
44. (73.5) By the definition of force,
dp
F =
dt
d (mv )
F = [Q p = mv ]
dt
dv dm
F =m +v
dt dt
dv
Qv = constant, =0
dt
dm
∴ F =v
dt
dV (x )
= 2x − 4x 3 = 2x (1 − 2x 2 ) By the definition of power,
dx
P = F ⋅v
1
x = 0, ± dm dm
2 P =v v =v 2
dt dt
 1 1 1 1 Given that, v = 7 m/s
Vmax = V  ± = − =
 2 2 4 4 ∴ Power, P = ( 7)2 × 1.5
Vmax = 0.25 = 49 × .15 = 73.5 W
43. (2.85) ∇ V = 0
2
45. Energy eigenvalues in three-dimensional cubic box,
1 d  2 dV  π 2 h2
r  =0 E(n x , n y , n z ) = (n x2 + n y2 + n z2 ) …(i)
r 2 dr  dr  2ma 2
dV C1  π 2h2 
= 2 Given, E=  14 …(ii)
dr r  2ma 2 
− C1
V = + C2 …(i) On comparing Eqs. (i) and (ii), we get
r n x2 + n y2 + n z2 = 14
At r = 2 mm, V = 10 V ⇒ (n x , n y , n z ) = (1, 2, 3)
C
⇒ − 1 + C2 = 10 Degeneracy : (n x , n y , n z ) = (123) (132) (213) (231)
2
(312) (321)
At r = 5 mm, V = 0
There is six different energy states, so degeneracy
C
⇒ − 1 + C2 = 0 (6 to 6).
5
C1 = 5 C2 46. (4.6) We know that wave vector in conducting medium is
5 C2 k =α + iβ …(i)
⇒ − + C2 = 10
2 Given, k = 20π + 750 i …(ii)
Solved Paper 2017 ●
25

On comparing Eqs. (i) and (ii), we get 50. (3.50) Slits in micrometers
α = 120π 
…(iii)
β = 750 

Intensity
a
(a+b) Central maxima
but 2 αβ = µσω b Ist order maxima
2 1
⇒ = …(iv) IInd order maxima
µσω αβ
Skin depth is defined as
IIIrd order maxima
2
δ=
µ σω IVth order maxima
1 Vth order maxima
δ= [from Eq. (iv)] (missing)
αβ
Now put the values of α and β in above equation. Distance
1 Grating spectra conditions,
δ=
20 π × 750 a sin θ = nλ (diffraction maxima) …(i)
1 (a + b ) sin θ = mλ (interference minima) …(ii)
δ=
15000 π Here, a = slit width
δ = 4.6 mm b = length of open portion
47. (3000) Efficiency of Carnot engine is defined as (a + b ) = grating element or slit separation
Work done T From curve n = 1, m = 5
η= = 1− 2
Heat taken T1 On dividing Eq (ii) by Eq (i), we get
W T2 (a + b ) sin θ mλ
= 1− =
Q1 T1 a sin θ nλ
1000 400 a+b m
⇒ = 1− ⇒ =
Q1 600 a n
1000 200 1 a+b 5
= = =
Q1 600 3 a 1
Q1 = 3000 J (a + b ) = 5a [Qa = 0.7 µm]
⇒ Slit separation = 5 × 0.7 µm = 3.50 µm
48. (4)
51. (15.1) Volume integral,
45° V =∫ ∫ ∫ f (r , θ, φ )r
2
sin θ dr dθ dφ
I0
π
Unpolarised I0 I0 I0 2π 3 2
cos2 45°
V = ∫ ∫ ∫r cos θr 2 sin θ dr dθ dφ
2
2 2 2
φ = 0 θ = 0r = 0

From Malus’s law, Solution of this integration,


2
I 32 1  3 
I = 0 cos2 θ V = 2π × × ×
2 
2
5 2  2
I0 I  1
I = (cos 45° )2 = 0   16 3 24π
2 2  2 = 2π × × =
5 4 5
I0 1 I0 I0 24
I= × = ⇒ =4 = × 3.14 = 4.8 × 3.14 = 15.072
2 2 4 I 5
49. (1.25) For anti-reflection coating, path difference is given by V = 15.1
λ
∆ = 2 µt = 52.
2
R 1
λ
t= 2
4µ ρ 2
∴Refractive index, –ρ
= +
λ 0.5 µm 5 O R
µ= = = E1
4 t 4 × 0.1µm 4 E2
µ = 1.25
26 IIT JAM Physics Solved Papers & Practice Sets

R Eigenvalues of matrix,


ρ 
 2  −1 1 
E1 = in downward direction
3 ε0  1 − 1 ⇒ λ = 0, − 2
 
3
R λ 1 = 0, λ 2 = − 2
ρ 
 2 ∴
dx1
= λ 1x1
E2 = in upward direction
3 ε 0R 2 dt
dx1
So, net electric field, = λ 1dt
x1
E = E1 − E2
ρR ρR ρR On taking integration both sides, we get
E= − = dx1
6ε 0 24ε 0 8ε 0 ∫ x1 = ∫ λ1dt
ρR
E= …(i) ln x1 = λ 1t + C
8 ε0
⇒ x1 = e λ 1t + C 1 …(iii)
But given that, dx 2 dx
ρR = λ 2x 2 ⇒ 2 = λ 2 dt
E= …(ii) dt x2
n ε0
On taking integration both sides, we get
On comparing Eqs. (i) and (ii), we get dx 2
n=8 ∫ x 2 = ∫ λ 2 dt
53. (9) In polar coordinate (r , θ ) velocity is given by ln x 2 = λ 2t + C2
• •
v = r e$r + r θ e$ θ ⇒ x 2 = e λ 2t + C2
…(iv)
At x1( 0) = 1 ⇒ C1 = 1
dv
Acceleration, a= At x 2( 0) = − 1 ⇒ C2 = − 1
dt
+1
•• • •• • • At x1(t ) = e 0t [from Eq. (iii)]
a = ( r − r θ 2 ) e$r + (r θ + 2 r θ ) e$ θ …(i)
x1(t ) = e = e1

Given,
x 2(t ) = e −2t −1
[from Eq. (iv)] …(v)
r = et , θ = 8t
at − 1
• •• x 2(t ) = e …(vi)
⇒ r = et , r = et
• ••
On comparing Eqs. (v) and (vi), we get
θ = 8, θ =0 a=−2
Now, put the above values in Eq. (i), we get 55. (1.08) Reaction, p + e → n + ν
a = (et − et 8) e$r + (et × 0 + 2et 8 ) e$ θ
Rest mass energy = 939.12 − 938.57
a = − 7 et e$r + 2 8 et e$ θ = 129
. MeV
At t = 0, We know that, de-Broglie wavelength,
at = − 7e 0e$r + 2 8 e 0e$ θ h h
= 0 λ= =
p 2 mE
a = − 7e$r + 2 8 e$ θ
6.62 × 10−34
Magnitude of a, λ=
2 × 9.1 × 10−31 × 129 . × 10−19
. × 106 × 16
a = ( − 7 )2 + ( 2 8 )2
. × 10−12 m
λ = 108
= 49 + 4 × 8
λ = 108
. pm
= 49 + 32 = 81 = 9
56. (4.56) We know that from Maxwell’s equation,
a =9  ∂p   ∂S   ∂S ∂T   ∂S   ∂T 
  =  = ⋅  =  ⋅ 
dx  ∂T  S  ∂V  p  ∂T ∂V  p  ∂T  p  ∂V  p
54. (-2) Given, 1 = x 2 − x1 …(i)
dt  ∂S  1  ∂T   ∂Q  1  ∂T 
dx 2 =T   ⋅   =  ⋅  
= x1 − x 2 …(ii)  ∂T  p T  ∂V  p  ∂T  p T  ∂V  p
dt
1  ∂T  1  ∂T 
Now, write Eqs. (i) and (ii) in matrix form, = mCp ⋅   = ρC p V  
T  ∂V  p T  ∂V  p
 dx1 
 dt  −1 1  x1  ρC p 1
 = =
  T 1  ∂V 
dx 2   1 − 1 x 2   
V  ∂T  p
 dt 
Solved Paper 2017 ●
27

 ∂p  ρC p 1kΩ
  = …(i)
 ∂T  S T β
R
Given, Cp = 2010 J/kg-K
B V0 = 9V
ρ = 103 kg/m 3 A
β = 1.6 × 10−4 1°/K 1kΩ
Vin=12V Vz=4.7 V
 ∂p 
 = N × 10 Pa/K
7

 ∂T  S V=0
T = 270 K VB − 0 4.7
Now, put above values in Eq. (i), we get Resistance, R= =
I 4.3 mA
103 × 2010
N × 107 = R = 1.09 kΩ
. × 10−4
270 × 16
N × 107 = 4.65 × 107 60. (2.41) Equation of motion,
dv
N = 4.65 m = − kv
dt
57. (2.74) From de-Broglie equation, dv dx dv
h h ⇒ m ⋅ = − kv ⇒ mv = − kv
λ= = [Q p = 2mE ] dx dt dx
p 2mE dv k
⇒ m = − k ⇒ dv = − dx
6.62 × 10−34 dx m
λ=
2 × 9.1 × 10 −31
. × 10−19
× 200 × 103 × 16 On taking integration both sides, we get
k
=
6.62 × 10−12 ∫ dv = − m ∫ dx
2 × 9.1 × 0.2 × 16
. k
v =− x +C
6.62 × 10−12 m
= = 2.74 × 10−12 m
2.413 At x = 0, v = v 0 ⇒ C = v 0
λ = 2.74 pm k
v =v0 − x
m
58. (0.271) (D 2 + 2D + 1)y = 0 k v v
At x = L, v = 0 ⇒ = 0 ⇒v = v 0 − 0 x
AE m L L
(m 2 + 2m + 1) = 0 dx v0 v 0(L − x ) dx v
⇒ =v0 − x = ⇒ = 0 dt
⇒ (m + 1)2 = 0 dt L L (L − x ) L
⇒ m = − 1, − 1 On taking integration both sides, we get
CF
L − x  v0
x t
dx v0
y (x ) = (C1 + C2x )e − x ∫0 L − x = L ∫0 dt ⇒ − log L  = L t
Given conditions,
y( 0) = 0 L L − x 
t =− log 
⇒ (C1 + C2 ⋅ 0)e 0 = 0 v0  L 
⇒ C1 = 0 L  L 
t= log  
∴ y (x ) = C2xe − x v0 L −x 
dy
y ′ (x ) = = C2(1 − x )e − x L
dx At x = , t = t 2 (given)
2
⇒ y′ ( 0) = 1 ⇒ C2 = 1 L
∴ y (x ) = xe − x At x = , t = t 4
4
2
y ( 2) = 2e −2 = 2 ∴
L
t 2 = log 2
e v0
2
= = 0.271 L  4
( 2.731)2 t 4 = log 
v0  3
y( 2) = 0.271 [Qe = 2.731]
t2 log 2
59. (1.09) VB = VA = 4.7 V ∴ =
t4  4
log  
V0 − VB  3
Current, I=
R t 2 0.3010
9 − 4.7 = = 2.409
I= t 4 0.1249
1kΩ
t2
I = 4.3 mA = 2.41
t4
Solved Paper
2016
IITJAM Physics
MM : 100 Time : 3 hrs

n
This test paper has a total of 60 questions carrying 100 marks. The entire question paper is divided into
three Sections A, B and C. All sections are compulsory. Questions in each section are of different types.
n
Section A contains Multiple Choice Questions (MCQ). Each MCQ type question has four choices out of
which only one choice is the correct answer. This section has 30 Questions and carry a total of 50 marks.
(Q.1 – Q.10) carry 1 mark each and questions (Q.11 – Q.30) carry 2 marks each.
n Section B contains Multiple Select Questions (MSQ). Each MSQ type question is similar to MCQ but
with a difference that there may be one or more than one choice(s) that are correct out of the four given
choices. The candidate gets full credit if he/she selects all the correct choices only and no wrong
choices. This section has 10 questions (Q. 31 – Q. 40) and carry 2 marks each with a total of 20 marks.
n
Section C contains Numerical Answer Type Questions (NATQ). For these NAT type questions, the answer
is a real number which needs to be entered using the virtual numerical keypad on the monitor. No
choices will be shown for these type of questions. This section has 20 questions and carry a total of 30
marks. (Q.41 – Q. 50) carry 1 mark each and Questions (Q.51 – Q.60) carry 2 marks each.
n
Do not write more than one answer for the same question. In case you attempt a subjective question
more than once, please cancel the answer(s) you consider wrong. Otherwise, the answer appearing last
only will be evaluated.
n Clip board, log tables, slide rule, calculator, cellular phone and electronic gadgets in any form are not
allowed.

Section A Multiple Choice Questions (MCQs)


In these type of questions, each question has four choices (a), (b), (c) and (d) out of which only
one option is correct.

1. For an infinitely long wire with uniform line charge density λ , along the Z-axis, the electric field
at a point (a , b , 0) away from the origin is ______ ( e$ x , e$ y and e$ z are unit vectors in Cartesian
coordinate system).
λ λ
(a) (e$ x + e$ y ) (b) ( ae$ x + be$ y )
2
2 πε 0 a + b 2 2 πε 0 ( a2 + b2 )
λ λ
(c) e$ x (d) e$ z
2 πε 0 a2 + b2 2 πε 0 a2 + b2
2 IIT JAM Physics Solved Papers & Practice Sets

2. A 1 W point source at origin emits light uniformly in all the directions. If the units for both the
axes are measured in centimetre, then the Poynting vector at the point (1, 1, 0) in W/cm 2 is
(e$ x , e$ y and e$ z are unit vectors in Cartesian coordinate system)
1 1
(a) (e$ x + e$ y ) (b) (e$ x + e$ y )
8π 2 16π
1 1
(c) (e$ x + e$ y ) (d) (e$ x + e$ y )
16π 2 4π 2

3. A charged particle in a uniform magnetic field B = B 0e$ z starts moving from the origin with
velocity v = ( 3e$ x + 2 e$ y ) m/s. The trajectory of the particle and the time t at which it reaches
2 m above the XY-plane are (e$ x , e$ y and e$ z are unit vectors in Cartesian coordinate system.)
(a) Helical path, t = 1s (b) Helical path, t = 2/3 s
(c) Circular path, t = 1s (d) Circular path, t = 2/3 s

4. Consider a particle of mass m following a trajectory given by x = x 0 cosω 1t and


y = y 0 sinω 2t, where x 0 , y 0 , ω 1 and ω 2 are constants of appropriate dimensions. The force
on the particle is
(a) central only if ω 1 = ω 2 (b) central only if x0 = y0 and ω 1 = ω 2
(c) always central (d) central only if x0 = y0 and ω 1 ≠ ω 2
1
5. Which one of the following points represent the complex number = ?
1− i
Y Y

1 1
0.5 0.5
(a) X (b) X
–1 – 0.5 0.5 1 –1 – 0.5 0.5 1
– 0.5 – 0.5
–1 –1

Y Y

1 1
0.5 0.5

(c) X (d) X
– 1 – 0.5 0.5 1 –1 – 0.5 0.5 1
– 0.5 – 0.5
–1 –1

6. The eigenvalues of the matrix representing the following pair of linear equations x + iy = 0 and
ix + y = 0 are
(a) 1 + i , 1 + i (b) 1 − i , 1 − i (c) 1, i (d) 1 + i , 1 − i

7. The solution of the Boolean equation, Y = A + B + AB is


(a) 1 (b) AB (c) A B (d) A + B
Solved Paper 2016 ●
3

8. A spherical closed container with smooth inner wall contains a monoatomic ideal gas. If the
collisions between the wall and the atoms are elastic, then the Maxwell speed-distribution
 dn 
function  v  for the atoms is best represented by
 dV 

dnv dnv
(a) (b)
dV dV

O v O v

dnv dnv
(c) (d) dV
dV

O v O v

9. Two sinusoidal signals of frequencies ω x and ω y having same amplitude are applied to x and
y channels of a Cathode Ray Oscilloscope (CRO), respectively. The following stationary figure
will be observed when
y
1

x
–1 1

–1

(a) ω y = ω x (b) phase difference is 0


(c) ω y = 2ω x (d) phase difference is π / 2

10. The phase difference (δ ) between input and output voltage for the following circuits (i) and (ii)
C R

Vi ~ C Vo Vi ~ C Vo

(i) (ii)

will be
(a) 0 and 0
(b) π / 2 and 0 < δ ≤ π / 2 respectively
(c) π / 2 and π / 2
(d) 0 and 0 < δ ≤ π / 2 respectively
4 IIT JAM Physics Solved Papers & Practice Sets

11. For the given set of equations


x+y =1
y +z = 1
x +z = 1
which one of the following statements is correct?
(a) Equations are inconsistent
(b) Equations are consistent and a single non-trivial solution exists
(c) Equations are consistent and many solutions exist
(d) Equations are consistent and only a trivial solution exists

12. The tangent line to the curve x 2 + xy + 5 = 0 at (1, 1) is represented by


(a) y = 3 x − 2
(b) y = −3 x + 4
(c) x = 3 y − 2
(d) x = −3 y + 4

13. In the following R-C circuit, the capacitor was charged in two different ways.
(i) The capacitor was first charged to 5V by moving the toggle switch to position P and
then it was charged to 10V by moving the toggle switch to position Q.
(ii) The capacitor was directly charged to 10V, by keeping the toggle switch at position Q.
Assuming the capacitor to be ideal, which one of the following statements is correct?
R C

P 5V

Q 10V

(a) The energy dissipation in cases (i) and (ii) will be equal and non-zero
(b) The energy dissipation for case (i) will be more than that for case (ii)
(c) The energy dissipation for case (i) will be less than that for case (ii)
(d) The energy will not be dissipated in either case

14. If a constant voltage +V is applied to the input of the following op-amp circuit for a time t, then
the output voltage Vo will approach
C

R
+V
Vo

(a) + V exponentially (b) − V exponentially


(c) + V linearly (d) − V linearly
Solved Paper 2016 ●
5

1 V
15. In the following R-C network, for an input signal frequency, f = , the voltage gain o
2πRC Vi
and the phase angle φ between Vo and Vi , respectively are
R C

Vo

R Vo

1 1 1 π 1 π
(a) and 0 (b) and 0 (c) and (d) and
2 3 2 2 3 2

16. Light travelling between two points takes a path for which
(a) time of flight is always minimum (b) distance is always minimum
(c) time of flight is extremum (d) distance is extremum

17. Consider a free electron (e) and a photon (pH) both having 10 eV of energy. If λ and p
represent wavelength and momentum respectively, then
(mass of electron = 9.1 × 10 −31 kg, speed of light = 3 × 10 8 m/s)
(a) λ e = λ pH and pe = ppH (b) λ e < λ pH and pe > ppH
(c) λ e > λ pH and pe < ppH (d) λ e < λ pH and pe < p pH

18. For an ideal gas, which one of the following T - S diagrams is valid?
or

(a) T (b) T
ch

ar

b
Is o
Iso

r
ba

ho

Iso
oc

Is

S S
r

(c) T (d) T
ho
oc

Is

S S

19. If U, F, H and G represent internal energy, Helmholtz free energy, enthalpy and Gibbs
free energy respectively, then which one of the following is a correct thermodynamic
relation?
(a) dU = pdV − TdS (b) dH = Vdp + TdS
(c) dF = − pdV + SdT (d) dG = Vdp + SdT
6 IIT JAM Physics Solved Papers & Practice Sets

20. A train passes through a station with a constant speed. A stationary observer at the station
platform measures the tone of the train whistle as 484 Hz when it approaches the station and
442 Hz when it leaves the station. If the sound velocity in air is 330 m/s, then the tone of the
whistle and the speed of the train are
(a) 462 Hz, 54 km/h (b) 463 Hz, 52 km/h (c) 463 Hz, 56 km/h (d) 464 Hz, 52 km/h

21. The minimum length of a plane mirror to see the entire full length image of an object is half of
the object’s height. Suppose δ is the distance between eye and top of the head of person of
height h. The person will be able to see his entire full length image with a mirror of height h / 2
fixed on the wall
(a) when the bottom edge of mirror is kept h / 2 above the floor
(b) when the bottom edge of mirror is kept ( h + δ ) / 2 above the floor
(c) when the bottom edge of mirror is kept ( h − δ )/2 above the floor
(d) when the centre of the mirror is at the same height as centre of the person

22. A particle is moving in a plane with a constant radial velocity of 12 m/s and constant angular
velocity of 2 rad/s. When the particle is at a distance r = 8 m from the origin, the magnitude of
the instantaneous velocity of the particle in m/s is
(a) 8 15 (b) 20 (c) 2 37 (d) 10

23. A cylindrical rod of length L has a mass density distribution given by ρ(x ) = ρ 0 1 +  , where x
x
 L
is measured from one end of the rod and ρ 0 is a constant of appropriate dimensions. The
centre of mass of the rod is
5 4 1 1
(a) L (b) L (c) L (d) L
9 9 9 2

24. A particle travels in a medium along a horizontal linear path. The initial velocity of the particle
is v 0 and the viscous force acting on it is proportional to its instantaneous velocity. In the
absence of any other forces, which one of the following figures correctly represents the velocity
of the particle as a function of time?

(a) v(t) (b) v(t)

t
t

(c) v(t) (d) v(t)

t t
Solved Paper 2016 ●
7

25. A lightly damped harmonic oscillator with natural frequency ω 0 is driven by a periodic force of
frequency ω . The amplitude of oscillation is maximum when
(a) ω is slightly lower than ω 0 (b) ω = ω 0
(c) ω is slightly higher than ω 0 (d) the force is in phase with the displacement

26. A block of mass 0.38 kg is kept at rest on a frictionless surface and attached to a wall with a
spring of negligible mass. A bullet weighing 0.02 kg moving with a speed of 200 m/s hits the
block at time t = 0 and gets stuck to it. The displacement of the block (in metre) with respect to
the equilibrium position is given by

(Spring constant = 640 N/m)


(a) 2 sin5 t (b) cos 10 t (c) 0.4 cos25 t (d) 0.25 sin40 t

27. One mole of an ideal gas with average molecular speed v o is kept in a container of fixed
volume. If the temperature of the gas is increased such that the average speed gets doubled,
then
(a) the mean free path of the gas molecule will increase
(b) the mean free path of the gas molecule will not change
(c) the mean free path of the gas molecule will decrease
(d) the collision frequency of the gas molecule with wall of the container remains unchanged

28. An arbitrarily shaped conductor encloses a charge q and is surrounded by a conducting hollow
sphere as shown in the figure. Four different regions of space, 1, 2, 3, and 4 are indicated in
the figure. Which one of the following statements is correct?

q
3 4
1
2

(a) The electric field lines in region 2 are not affected by the position of the charge q
(b) The surface charge density of the inner wall of the hollow sphere is uniform
(c) The surface charge density on the outer surface of the sphere is always uniform irrespective of
the position of charge q in region 1
(d) The electric field in region 2 has a radial symmetry

29. Consider a small bar magnet undergoing Simple Harmonic Motion (SHM) along the X-axis. A
coil whose plane is perpendicular to the X-axis is placed such that the magnet passes in and
out of it during its motion. Which one of the following statements is correct? Neglect damping
effects.
(a) Induced emf is minimum when the center of the bar magnet crosses the coil
(b) The frequency of the induced current in the coil is half of the frequency of the SHM
(c) Induced emf in the coil will not change with the velocity of the magnet
(d) The sign of the emf depends on the pole (N or S) face of the magnet which enters into the coil
8 IIT JAM Physics Solved Papers & Practice Sets

30. An incompressible, non-viscous fluid is injected into a conical pipe at its orifice as
schematically shown in the figure. The pressure at the orifice of area A0 is p 0 . Neglecting the
effect of gravity and assuming streamline flow, which one of the following plots correctly
predicts the pressure along axis of the cone?

π/4

X0(p0 ,v0)

p p

(a) p0 (b) p0

X0 X X0 X

p p

(c) p0 (d) p0

X0 X X0 X

Section B Multiple Select Questions (MSQs)


In these type of questions, each question has four choices (a), (b), (c) and (d) out of which only
one or more than one is/are correct options.

31. Consider a spherical dielectric material of radius a centered at origin. If the polarisation vector
p = p 0 e$ x , where p 0 is a constant of appropriate dimensions, then
(e$ x , e$ y and e$ z are unit vectors in Cartesian coordinate system)
(a) the bound volume charge density is zero
(b) the bound surface charge density is zero at (0, 0, a)
(c) the electric field is zero inside the dielectric
(d) the sign of the surface charge density changes over the surface

32. For an electric dipole with moment p = p 0 e$ z placed at the origin, (p 0 is a constant of
appropriate dimensions and e$ x , e$ y and e$ z are unit vectors in Cartesian coordinate system)
1
(a) potential falls as , where r is the distance from origin
r2
(b) a spherical surface centered at origin is an equipotential surface
(c) electric flux through a spherical surface enclosing the origin is zero
(d) radial component of E is zero on the XY-plane
Solved Paper 2016 ●
9

33. Three infinitely long conductors carrying currents I1, I 2 and I 3 lie perpendicular to the plane of
the paper as shown below.
C3
C2 I2
I3

C1
I1

If the value of the integral ∫CB ⋅ dl for the loops C1, C 2 and C 3 are 2µ 0 , 4µ 0 and µ 0 in the
units of N/A respectively, then
(a) I1 = 3A into the paper
(b) I2 = 5A out of the paper
(c) I3 = 0
(d) I3 = 1A out of the paper

34. In the optical arrangement as shown below, the axes of two polarising sheets P and Q are
oriented such that no light is detected. Now, when a third polarising sheet (R) is placed in
between P and Q, then light is detected. Which of the following statement (s) is/are true?

Detector
Unpolarised
light P Q

(a) Polarisation axes of P and Q are perpendicular to each other


(b) Polarisation axis of R is not parallel to P
(c) Polarisation axis of R is not parallel to Q
(d) Polarisation axes of P and Q are parallel to each other

35. The p-V diagram below shows three possible paths for an ideal gas to reach the final state f
from an initial state i. Which among the following statement(s) is/are correct?

p i 3

2
f
1
V

(a) The work done by the gas is maximum along path-3


(b) Minimum change in the internal energy occurs along path-2
(c) Maximum heat transfer is for path-1
(d) Heat transfer is path independent
10 IIT JAM Physics Solved Papers & Practice Sets

36. Potential energy U as a function of r is shown below. If a particle of mass m1 and energy E 1,
starting from r >> a , moves towards the origin, then

U(r) m1

E1

r
0 b a
Umin
c

(a) there is only one turning point for the particle


(b) velocity of the particle starts to increase at r = a and reaches its maximum at r = c
(c) velocities of the particle at r = a and r = b are equal
(d) the particle gets bounded if it elastically collides with a stationary particle of mass m2 at r = c,
imparting a momentum greater than 2 m2 E1

37. A particle moves in a circular path in the XY-plane centered at the origin. If the speed of the
particle is constant, then its angular momentum
(a) about the origin is constant both in magnitude and direction
(b) about (0, 0, 1) is constant in magnitude but not in direction
(c) about (0, 0, 1) varies both in magnitude and direction
(d) about (0, 0, 1) is constant in direction but not in magnitude

38. A p-n junction was formed with a heavily doped (1018 cm −3 ) p-region and lightly doped
(1014 cm −3 ) n -region. Which of the following statement(s) is/are correct?
(a) The width of the depletion layer will be more in the n-side of the junction
(b) The width of the depletion layer will be more in the p -side of the junction
(c) The width of the depletion layer will be same on both sides of the junction
(d) If the p-n junction is reverse biased, then the width of the depletion region increases

39. A slit has width d along the x-direction. If a beam of electrons, accelerated in y -direction to a
particular velocity by applying a potential difference of 100 ± 0.1 kV passes through the slit,
then which of the following statement(s) is/are correct?
(a) The uncertainty in the position of electrons in x-direction before passing the slit is zero
(b) The momentum of electrons in x-direction ~ h / d immediately after passing the slit
(c) The uncertainty in the position of electrons in y-direction before passing the slit is zero
(d) The presence of the slit does not affect the uncertainty in momentum of electrons in y-direction

40. A free particle of energy E collides with a one-dimensional square potential barrier of height V
and width W . Which one of the following statement(s) is/are correct?
(a) For E > V , the transmission coefficient for the particle across the barrier will always be unity
(b) For E < V , the transmission coefficient changes more rapidly with W than with V
(c) For E < V , if V is doubled, the transmission coefficient will also be doubled
(d) Sum of the reflection and the transmission coefficient is always one
Solved Paper 2016 ●
11

Section C Numerical Answer Type Questions (NATQs)


This section contains Numerical Answer Type (NAT) questions. For these NAT type questions, the
answer is a real number.

41. Fourier series of a given function f (x ) in the interval 0 to L is


∞ ∞
a0  2πnx   2πnx 
f (x ) = + ∑ a n cos   + ∑ b n sin  .
2 n =1  L  n =1
 L 

If f (x ) = x in the region ( 0, π ), b 2 = __________.

42. Consider a function f (x, y ) = x 3 + y 3 , where y represents a parabolic curve x 2 + 1 . The total
derivative of f with respect to x, at x = 1is _________.

43. A rectangular area ( A1 ) is formed by two vectors x and y as shown in Fig. (i). A new set of
vectors, representing the area ( A2 ) as shown in Fig. (ii), are given as u1 = x , u 2 = k x + y,
where k is a dimensionless constant.

u2
Y

X u1
(i) (ii)

The Jacobian of the frame (u 1,u 2 ) with respect to ( x, y ) is _________.

44. A particular radioisotope has a half-life of 5 days. In 15 days, the probability of decay in
percentage will be______ .

45. In a photoelectric experiment both sodium (work function = 2 .3 eV) and tungsten (work
function= 4.5 eV) metals were illuminated by an ultraviolet light of same wavelength. If the
stopping potential for tungsten is measured to be 1.8 V, then the value of the stopping
potential for sodium will be _______ V.

46. The addition of two binary numbers 1000.01 and 0001.11 in binary representation is ______.
47. The number of second-nearest neighbour ions to a Na + ion in NaCl crystal is ________ .
48. The output voltage Vo of the op-amp circuit given below is ......... V.
2R

Vo
R
1V
R
2V
R
3V
12 IIT JAM Physics Solved Papers & Practice Sets

49. A cylinder contains 16 g of O 2 . The work done when the gas is compressed to 75% of the
original volume at constant temperature of 27° C is _______ J.
[Universal gas constant R = 8.31 J/(mole K)]

50. When sunlight is focused on a paper using a bi-convex lens, it starts to burn in the shortest
time, if the lens is kept 0.5 m above it. If the radius of curvature of the lens is 0.75 m, then the
refractive index of the material is _______

51. Consider a closed triangular contour traversed in counter-clockwise direction as shown in the
figure below.
Y

π π
4 4
O P(2,0) X

The value of the integral ∫ F ⋅ dl evaluated along this contour, for a vector field, F = y e$ x − x e$ y ,
is _______.
(e$ x , e$ y and e$ z are unit vectors in Cartesian coordinates system)

52. A hemispherical shell is placed on the XY-plane centred at the origin. For a vector field
E = ( − y e$ + x e$ y ) / (x 2 + y 2 ), the value of the integral ∫ (∇ × E) ⋅ d a over the hemispherical
s
surface is _______ π.
(d a is the elemental surface are e$ x , e$ y and e$ z are unit vectors in Cartesian coordinates
system.)

53. The shape of a dielectric lamina is defined by the two curves y = 0 and y = 1 − x 2 . If the
charge density of the lamina σ = 15y C/m 2 , then the total charge on the lamina is ______ C.

54. In the circuit given below, the collector to emitter voltage VCE is ______ V.
(Neglect VBE , take β = 100 )
VCC=10 V

5 kW
5 kW
VCE

5 kW

10 kW

55. The de-Broglie wavelength of a relativistic electron having 1 MeV of is ... ×10 −12 m. (Take the
rest mass energy of the electron to be 0.5 MeV. Planck’s constant = 6.63 × 10 −34 J-s, speed of
light = 3 × 10 8 m/s and electronic charge = 1.6 × 10 −19 C).
Solved Paper 2016 ●
13

56. X-ray diffraction of a cubic crystal gives an intensity maximum for Bragg angle of 20°
corresponding to the (110) plane. The lattice parameter of the crystal is ______ nm.

57. X-rays of 20 keV energy is scattered inelastically from a carbon target. The kinetic energy
transferred to the recoiling electron by photons scattered at 90° with respect to the incident
beam is ______ keV.
(Planck’s constant = 6.6 × 10 −34 J-s, speed of light = 3 × 10 8 m/s, electron mass = 9.1 × 10 −31 kg
and electronic charge = 1.6 × 10 −19 C)

58. An aluminum plate of mass 0.1 kg at 95° C is immersed in 0.5 L of water at 20° C kept inside
an insulating container and is then removed. If the temperature of the water is found to be
23°C, then the temperature of the aluminum plate is ______ °C.
(The specific heat of water and aluminum are 4200 J/kg-K respectively, the density of water is
1000 kg/m 3 )

59. The maximum and minimum speeds of a comet that orbits the Sun are 80 and 10 km/s,
respectively. The ratio of the aphelion distance of the comet to the radius of the earth’s orbit is
_______.
(Assume that, the earth moves in a circular orbit of radius 1.5 × 10 8 km with a speed of
30 km/s)

60. If there is a 10% decrease in the atmospheric pressure at a hill compared to the pressure at
sea level, then the change in the boiling point of water ______ is °C.
(Take latent heat of vaporisation of water ad 2270 kJ/kg and the change in the specific volume
at the boiling point to be 1.2 m 3 /kg)

Answers
1. (b) 2. (a) 3. (a) 4. (a) 5. (a) 6. (d) 7. (d) 8. (c) 9. (b) 10. (d)
11. (b) 12. (b) 13. (c) 14. (d) 15. (b) 16. (a) 17. (b) 18. (a) 19. (b) 20. (a)
21. (c) 22. (b) 23. (a) 24. (d) 25. (a) 26. (d) 27. (b) 28. (c) 29. (a) 30. (a)
Answers with Explanations
Section A Multiple Choice Questions
1. (b) Consider an infinitely long charged wire of charge Force along x and z direction is zero, therefore
density λ as shown in figure. From the symmetry of the trajectory of the particle is a helix with axis along
figure it is clear that electric field will be along OP. Y-axis. Velocity along X and Z axis will remain
According to Gauss’ law, electric field at point P is same. Therefore, distance above the XY-plane at any
Z + time t is
+ z = (v z ) × t
Y
+ ⇒ 2 = ( 2) × t ⇒ t = 1s
+
+ P(a,b,0)
4. (a) Trajectory of the particle is given by
X
x = x 0 cos ω1t
O +
and y = y 0 sin ω 2 t
+
If the force is central, then trajectory of the particle will
+
be circular or elliptical.
+
+ Now we have,
x 2 + y 2 = x 02 cos2 ω1t + y 02 sin2 ω 2t
1 λ
EP = OP If ω1 = ω 2 = ω
2π ∈0 OP
⇒ x 2 + y 2 = x 02 cos2 ωt + y 02 sin2 ωt
1 λ  a e$ x + b e$ y 
= ⋅   Let φ = ωt = constant at any given instant
2π ∈0 a 2 + b 2  a 2 + b 2  ∴ x 2 + y 2 = x 02 cos2 φ + y 02 sin2 φ
1 λ = constant
EP = ⋅ 2 (a e$ x + b e$ y )
2π ∈0 (a + b 2 ) This represents a closed path obtained by the particle
due to the central force.
2. (a) Power of the source, P = 1 W
5. (a) Given complex number is
Distance of the point (1, 1, 0) from the origin, where 1
source can be assumed to be placed is z =
1− i
r = 12 + 12 + 02 = 2 1 1+ i 1+ i
= × =
Intensity of the source at this point is 1 − i 1 + i (1)2 − (i )2
P 1 1 1+ i
I= = = =
4πr 2 4π( 2 )2 8π 1 − ( − 1)
We know that magnitude of Poynting vector is same as 1+ i
= = 0.5 + ( 0.5) i
intensity of the P source. 2
1 ⇒ x + iy = ( 0.5) + ( 0.5) (i )
Let Poynting vector P = ( e$ x + e$ y )
8π 2 x = 0.5, y = 0.5
1 1
⇒ P = 1+ 1 = =I These coordinates correspond to a point in the first
8π 2 8π quadrant.
Therefore, this is the required Poynting vector.
6. (d) Given matrix is
3. (a) Given magnetic field 1 i 
B = B0e$ z A= 
i 1
and velocity v = ( 3e$ x + 2e$ z ) m/s
Force on the particle due to magnetic field is Let eigenvalues are represented by λ.
F = q( v × B) ∴ A − λI = 0
= q( 3e$ x + 2e$ z ) × (B0e$ z ) 1 i   1 0
⇒ i 1 − λ 0 1 = 0
= − q( 3B0e$ y + 0)    
= − 3qB0e$ y ⇒ 1 − (i )2 − λ × 1 = 0
where, q is charge on the particle. ⇒ λ = 1 − (i )2
This force is along negativeY-axis. = (1 + i ), (1 − i )
Solved Paper 2016 ● 15

7. (d) Given Boolean equation is 13. (c) When a capacitor is charged, amount of heat
Y = A + B + AB dissipated in the wire is same as energy stored in the
=AB + A +B capacitor.
Therefore, energy dissipated in first case is
= A (B + 1) + B
1 1
=A +B [Q B + 1 = 1] U1 = (C ) (5)2 + (C ) (10 − 5)2
2 2
1 50 C
8. (c) The Maxwell’s speed distribution function 
dnv 
 for the = C ( 25 + 25) =
 dv  2 2
atoms will be [c].
Energy dissipated in second case
In Maxwell’s theory, the atoms acquire maximum 1 100 C
velocity and impart to other atoms by electric collisions U 2 = C (10)2 =
2 2
and again their velocity decreases.
∴ U 2 >U1
9. (b) When the phase difference between the sinusoidal
signals is zero, the superposition of signals will produce 14. (d) Given, circuit is an integrator
1
RC ∫ in
figure given in the question. ∴Output voltage, Vo = − (V )dt
10. (d) In circuit (i), the phase difference produce by 1
⇒ Vo = − (V ) ∫ dt
condenser C, will be reversed by another condenser C, RC
so Vo and Vi will be in the same phase. In circuit (ii) V
=− t +K
when AC passes through resistor, there will be no RC
phase difference. As soon as it passes through where, K is a constant.
π
condenser C, an output voltage phase difference of Therefore, Vo varies linearly as given by above
2
is produced. expression.
π
∴0 and 0 < δ ≤ 1
2 15. (b) Given that, f =
2πRC
11. (b) Given set of equation are 1
⇒ ω = 2 πf =
x + y = 1, y + z = 1, x + z = 1 RC
⇒ x + y + 0( z ) = 1 ...(i) The circuit consists of series and parallel RC networks,
⇒ 0(x ) + y + z = 1 ...(ii) which are in opposite phases. Therefore, net phase
⇒ x + 0(y ) + z = 1 ...(iii) angle between output and input voltage is zero.
1 1 0 Now, we have
Now we have, ∆ = A = 0 1 1 1

1 0 1 Output impedance, Z o = ω C
2
= 1(1 − 0) + 0 + 1(1 − 0)  1 
R2 +  
⇒ ∆=2≠0  ωC 
Therefore, given equations are consistent and a single R R
= =
non-trivial solution exists. Rω C +1
2 2 2
2
12. (b) Given equation of the curve is  1 
2
R
x 2 + xy + 5 = 0 Input impedance, Z i = R 2 +   +
 ωC  2
Now, differentiating w.r.t. x, we have
R R
dy = R2 + R2 + = 2R +
2x + x +y =0 2 2
dx
dy Vo Zo R/ 2 R 1
⇒ x = − y − 2x ∴ Voltage gain, = = = =
dx Vi Zi 2R +
R 3R 3
dy − y − 2 x 2
⇒ =
dx x
− 1− 2
16. (a) Fermet’s principle, path travelled by light is one for
 dy 
⇒   = =−3 which time taken is least.
 dx  (1, 1) 1
Now, required equation of tangent is
17. (b) Given energy of electron and photon,
 dy  y −1 E = 10 eV
  = For electron de-Broglie wavelength is given by
 dx  (1, 1) x − 1
y −1 h
λe = =
h
⇒ −3=
x −1 p 2meE
⇒ y − 1 = − 3x + 3 ⇒ y = − 3x + 4 where, me = mass of electron, h = planck’s constant.
16 IIT JAM Physics Solved Papers & Practice Sets

For a photon, energy is given by ( 330 + vs )


1.09 =
hc ( 330 − vs )
E=
λ ph 330 + vs = 1.09 (330 − vs )
hc 31.35
⇒ λ ph = vs = = 15 m/s
E 2.09
λe h E E 1 = 54 km/h ...(iii)
∴ = × = ×
λ ph 2meE hc 2me C Substitution of vs in Eq. (i) gives
− 19  330 
10 × 1.6 × 10 1 484 = n  
= ×  330 − 15
2 × 9.1 × 10− 31 3 × 108
4 1  330
= 5 × × 1012 × =n  
 315
3 3 × 108
484 × 21
4 1 n= = 462 Hz
= × 10− 2 × × 2.236 < 1 22
3 3
⇒ λ ph > λ e 21. (c) Top edge of mirror must lie at above eye level of
Momentum of electron, person.
H F
pe = 2meE δ
M1
Momentum of photon is given by E
p ph = E /c h/2
pe 2meE 2mec 2E
∴ = =
p ph (E / c ) E2 h
M2
2mec 2
=
E
p ph E 1
⇒ = × < 1 [From previous part] F
pe 2me c
∴ p ph < pe h δ
∴ Distance from bottom = h −  + 
 2 2
18. (a) The graph as shown in option (a) is more steep in
comparison to other three. So, it will be the valid graph h δ  h − δ
= − = 
for T - S diagram. 2 2  2 
19. (b) E = V ⋅ dp (workdone) 22. (b)
and H = E + T .S
/s

(Enthalpy)
2m

∴ dH = Vdp + T ⋅ dS
r 1
v=

20. (a) When train approaches towards station, the frequency v 2 = rω


T
listen by the observer =16 m/s
 v   330 
8m

n1 = n   =n  
v − vs   330 − vs 
Here, v = 330 m/s
n = actual tone of whistle
 330 
∴ 484 = n   ...(i) Y′
 330 − vs 
When train leaves the station, the apparent frequency | v | = vr2 + vT2
 v   330  = (12)2 + ( 8 × 2)2
n2 = n   =n  
v + vs   330 + vs 
= 42 ( 32 + 42 )
 330 
442 = n   ...(ii) = 42 (9 + 16)
 330 + vs 
Divide Eqs. (i) by (ii), we get = 42 × 52
484 330 + vs = 4 ×5
=
442 330 − vs = 20 m/s
Solved Paper 2016 ● 17

 x Now, applying conservation of linear momentum just


23. (a) f (x ) = f0 1 + 
 L before and after the bullet gets stuck into the block.
L = length mv = (m + M ) V
L  x where, V = common speed of block and bullet
∫ x . f (x ) dx = ∫  L 
L
f x 1+ 0 . dx mv
x CM = 0
0
∴ V =
m+M
∫ f (x ) dx f ∫ 1 + L  . dx
L
x L
0 0
0 Now, applying conservation of mechanical energy, we
L get
x 2
x  3
L2 L2 1 1 1
 2 + 3L  + (m + M )V 2 = (m + M ) ω 2A 2 = kA 2
 0 2 3 2 2 2
= =
 x2
L
L2 where, A = amplitude of oscillation of spring + bullet
L+
x + 2L  2L + block system
 0
 5  5 ω = angular speed
L2   L2   1 m2v 2 1
 6  6 5 ∴ (m + M ) = kA 2
= = = L 2 (m + M )2 2
L  3 9
L+ L 
2  2 m 2v 2
⇒ kA 2 =
m+M
24. (d) Let the instantaneous velocity bev. 1
F ∝ −v  m 2v 2  1  2
dv ⇒ A =   
=m = − kv  m + M  k 
dt
mv 0.02 × 200
dv − k = =
= = . dt …(i) (m + M )k 0.40 × 640
v m
On integrating Eq. (i), we get 2×2
=
 dv  − kt 4 × 64
log   = +C
v  m 2×2 2 1
= = = = 0.25 m
C = constant of integration 2×8 8 4
⇒ v = e −kt / m + C …(ii) Initially spring is in natural length, therefore
∴ According to Eq. (ii), the velocity decreases displacement of the system must be represented by
exponentially with time. sine function.
∴ x = A sin ω t
25. (a) Frequency of the damped oscillator is given by
where, ω is angular frequency of oscillation.
k b2
ω= − k 640
m 4m 2 ⇒ =
where, symbols have their standard meaning. m+M 0.4
b2 = 16 × 100 = 40 rad/s
⇒ ω = ω 20 −
4m 2 ∴Displacement of the block with respect to equilibrium
For lightly damped oscillator, position.
b 2 < 4 mk x = A sin ω t = 0.25 sin 40 t
b2 k 27. (b) The mean free path or average distance between
⇒ 2
< = ω 20
4m m collisions for a gas molecule is given by
b2 1
⇒ ω 20 > λ=
4m 2 2 π d 2n
b2
⇒ ω 2 = ω 20 − < ω 20 where, d = diameter of molecules
4m 2
n = molecules per unit volume
⇒ ω < ω0
λ is independent of temperature, therefore with
26. (d) Mass of the bullet, m = 0.02 kg increase in temperature, it will not change.
Mass of the block, M = 0.38 kg 28. (c) Charge q is not placed at the centre, therefore it is
Speed of the bullet,v = 200 m/s located at symmetric position within the conductor. Due
Spring constant, k = 640 N/m to this asymmetric location of the charge, induced
When bullet stucks into the block, let amplitude of charge on inner surface of the conductor will be
oscillation of the spring + block + bullet system be A . non-uniform.
18 IIT JAM Physics Solved Papers & Practice Sets

Electric field lines in region 2 are effected by location of π


At the point (0, 0, a), angle between p and n$ is .
the charge q. It is radial and symmetric if charge q is 2
placed at the centre of spherical conductor and it is π
∴ σb = p . n$ = | p || n$ | cos = 0
asymmetric, if charge q is placed off the centre. 2
Due to mutual repulsion, charges on outer surface of We can see that surface charge density depends upon
the conductor is always uniform irrespective of location angle between p and n. $ Therefore, sign of surface
of the charge q as shown in figure. charge density ( σb ) changes according to cosine of
+ angle between p and n$ over surface of the sphere.
+ +
+ + 32. (a, c, d)
+ –q + (i) Consider an electric dipole placed along Z -axis as
+q shown is figure.
+
q
+ Dipole moment of the dipole is p = p 0 e$ z
Z Q(x,y,z)
+ +
Y
+ +
+ p

29. (a) Induced emf in the coil depends upon rate of change of O
X
velocity of the magnet. It is given that, the magnet is
executing SHM along the axis of the coil. Therefore, its
Electric potential at any point Q (x , y , z ) is given by
mean position will be located at the centre of the coil.
1 p . r$ 1 p . r$
Hence, velocity of the magnet will be maximum at the V = =
4 π ∈0 | r | 2
4π ∈0 r 2
centre and rate of change of velocity will be zero.Thus,
induced emf will be minimum at the centre. where, r$ = x e$ x + y e$ y + z e$ z
(ii) A spherical surface centred at origin cannot be an
30. (a) As an incompressible, non-viscous fluid passes into a
equipotential surface because charges of the dipole
conical pipe, the velocity of the fluid will increase, while are located at asymmetric positions with respect to
pressure will decrease. centre of the spherical surface.
(iii) Net charge enclosed by a spherical surface enclosing
π/4 the origin is zero. Therefore, according to Gauss’ law,
electric flux associated with this surface is zero.
(iv) Any point on the the XY-plane is located at
equatorial position with respect to the dipole.
Electric field at this point will be along Z -axis, i. e.,
radial component is zero.
X0(p0, V0)
33. (a, b) According to Ampere law
When the fluid advances along the axis of the cone, the
velocity decreases and pressure slightly increases.
∫c1
B . dl = µ 0 (I1 + I 2 ) = 2 µ 0
⇒ I1 + I 2 = 2 … (i)
Soon the fluid attain the stream line flow, so the velocity
and pressure both almost remain unchanged. Here, we have assumed that all the currents are
∴Graph (a) will be correct for P and X along X-axis. directed out of the paper and positive.
Similarly, ∫ B ⋅ dl = µ 0 (I 2 + I 3 ) = 4 µ 0
c2

Section B Multiple Select Questions ⇒ I2 + I3 = 4 … (ii)

31. (a, b, d) Given, Polarisation vector p = p 0e$ x


and ∫ c3
B . dl = µ 0 (I1 + I 2 + I 3 ) = µ 0

The spherical dielectric is placed in electric field. ⇒ I1 + I 2 + I 3 = 1 … (iii)


Bound volume charge density of the sphere is given by From Eqs. (i) and (iii), we get
ρb = − ∇ . p I 3 = 1 − (I1 + I 2 ) = 1 − ( 2) = − 1
 ∂ $ ∂ ∂ $  ∴ I 3 = 1 A into the paper.
=− e + e$ y + ez  ( p 0e$ x ) From Eqs. (ii) and (iii), we get
 ∂x x ∂y ∂z 
I1 = 1 − (I 2 + I 3 ) = 1 − 4 = − 3
 ∂ 
=− p + 0+ 0 = 0 [Q po = constant] ∴ I1 = 3A into the paper.
 ∂x 0 
Now from Eq. (i)
Bound surface charge density of the sphere is given by
I 2 = 2 − I1 = 2 − ( − 3) = 5
σb = p . n$
where, n$ is a normal vector at a given point. ∴ I 2 = 5A out of the paper.
Solved Paper 2016 ● 19

34. (a, b, c) When no light is observed, the polarisation axes of P 37. (a, b)
Y
and Q are perpendicular to each other, this cross-position.
When a third polarising sheet
v

Detector (0,0,0) X
O

P R Q v
(R) is placed in between P and Q, then some light detected.
When the particle is moving in a circular path in
Therefore, the axis of P and Q should be exactly parallel to XY-plane, centre at O, the magnitude and direction
P and Q. of angular momentum of the particle will constant
35. (a) If we observe the p − V diagram of ideal gas, the workdone about the origin [ A].
is given by the area obtained on the graph between p and V About the point (0, 0, 1), the magnitude of angular
for that gas. From graph, it is clear that the area followed by momentum will be constant but direction will
the gas through path 3, from initial state to final state is change, because the particle is moving in circular
maximum. path in XY- plane.
p
38. (a, d) p n
i
3
2 The charge density in p-side is 1018 cm− 3 . The
1 f charge density in n-side is 1014 cm− 3 . From charge
densities, it is clear that width of depletion layer will
be more in n-side of junction.
a b V
When the p - n junction is reverse biased, the width
∴The workdone by the gas is maximum along path − 3. of the depletion region increases because charge
Area = i f b a i carriers holes and electrons drift away from the
depletion layer.
36. (a, b, c) Given graph shows potential energy of a particle as a
function of distance r from the origin. During movement of 39. (b, d)
y
the particle from ∞ to the point
r = a, U (r ) = 0 d
S x
Force on the particle during movement ∞ → a is
dU (r ) e
F =− =0
dr
So, the particle moves with constant velocity. Consider a beam of electrons accelerated in
Now, consider movement of the particle from y-direction, passes through a slit S having width S
r = a to r = c . along x-direction.
dU (r ) Given, potential difference,V = (100 ± 0.1) kV
During this region, = positive According to Heisenberg uncertainty principle, we
dr
dU (r ) can write
So, attractive force on the particle is, F = h
dr ∆x ⋅ ∆p x ≥
2
So, velocity of the particle increases in the region r = a to
where, ∆x = uncertainty in measurement of position.
r = c.
and ∆p x = uncertainty in the measurement of
As the particle has accelerated motion between a and c, its
x-component of momentum.
velocity increases from r = a and reaches to maximum at
r = c. For the x-direction, we can write
d
Again consider movement of the particle from ∆x ≈
2
r = c to r = b.
d h h
dU (r ) . ∆p x ~ ⇒ ∆p x ~
During this region, = negative 2 2 d
dr
Slit is along the x-direction. Therefore, it will not
So, motion of the particle is retarded in the same fashion.
affect the uncertainty in momentum of electron in
Thus, the particle at r = b will be same as that of r = a.
y-direction.
20 IIT JAM Physics Solved Papers & Practice Sets

40. (b, d) It is given that, 43. (1) Given vectors are u1 = x, u2 = kx + y


Energy of the particle = E Jacobian matrix of the frame (u1, u2 ) with respect to
Potential barrier = V ( x, y ) is
Width = W  ∂u1 ∂u1 
 ∂x ∂y 
When a stream of particles collides with a J = 
one-dimensional square potential barrier with E < V . A  ∂u2 ∂u2 
portion of the particles is reflected back and the rest is  ∂x ∂y 
transmitted such that
 1 0
T + R =1 J = 
where, T = transmission coefficient K 1
R = reflection coefficient. ∴Required Jacobian = | J |
Transmission coefficient is given by  1 0
=  = 1− 0 = 1
16E(V − E )  −2 W  K 1
T = exp 2 m(V − E ) … (i)
V2  h 
44. (87.5) T1/ 2 = 5 days
where, m = mass of the particle
15 days
When E < V ⇒V − E > 0, it is
clear from Eq. (i) that T changes linearly withV and N N N
5 days , 10 days − , 15 −
exponentially with widthW , which is more rapid variation 2 4 8
than linear variation. N
Decayed percentage = × 100
8
= N × 12.5
Section C Numerical Answer Type Questions Probability of decay in percentage
41. ( −0.5) Given, = 100 − 12.5
∞ ∞
a  2πnx   2πnx  = 87.5
f (x ) = 0 +
2
∑ a n cos
 L 
+ ∑ bn sin
 L 

n =1 n =1
45. (4) WNa = 2.3 eV
f (x ) = x ; 0 < x < L WTn = 4.5 eV
Now, we have
hν = eV + W0
2 L 2πnx
bn = ∫ f (x ) sin dx eV = hν − W0
L 0 L
2 π 2π × 2 2 π eVNa = hν − 2.3 eV
b2 = ∫ x sin xdx = ∫ x sin 4xdx e1.8 = hν − 4.5 eV
π 0 π π 0
π e(VNa − 1.8) = 2.2 eV
Let, I = ∫ x sin 4xdx
0 VNa = 2.2 + 1.8 V
= (x ∫ sin 4xdx ) − ∫ (1 × ∫ sin 4xdx )∫ dx 
π π
=4V
 0 0 
46. (1010.00) From addition of two binary numbers
 −x × cos 4x  π π cos 4x 
⇒ I =   +∫ dx  1000.01
 4 0 0 4
  0001.11
 1  sin 4x  
π
1010.00
= − ( π cos 4π − 0) +  
 16  0 
 4  47. (12) NaCl has fcc type of crystal lattice. Here, chloride
 1  π anions ( Cl− ) occupy all the octahedral site and sodium
= − ( π × 1 − 0) + 0 = −
 4  4 cations ( Na + ) occupy all the octahedral holes in the fcc
2 2  −π  lattice. Each ion is octahedrally coordinated by six
∴ b2 = (I ) =  
π π 4 counter ions, and so this structure has (6, 6)
−1 coordination.
⇒ = − 0.5
2 To find out the nearest neighbour of NaCl . Consider
42. (27) Given function is f (x1y ) = x + y 3 3 the coordination of the sodium ion at the centre of the
dy unit cell. It has six nearest neighbours which are Cl− .
f ′ (x , y ) = 3x 2 + 3y 2 × The next nearest neighbours are 12 sodium cations
dx
= 3x 2 + 3(x 2 + 1)2 × ( 2x ) = 3x 2 + 6x (x 2 + 1)2 (Na + ) located on the middle of each of the edges of
the cube. Beyond that there are 8 Cl− ions, at corners
∴ [f ′ (x , y )] x = 1 = 3 × (1)2 + 6 × 1(1 + 1)2
of the cube.
= 3 + 6 × 4 = 3 + 24 = 27
Solved Paper 2016 ● 21

48. (6) Consider the op-amp circuit as given below. 50. (1.75) Consider the ray diagram as shown. To burn the
2R paper in the shortest time, height of the lens above the
paper must be equal to focal length of the lens.

R
V Lens
I R Vo
V
1V h
I′
R Paper
2V
R i.e. h = f = 0.5 m
3V
Applying Lens maker’s formula, we have
1 1 1  2
= (µ − 1)  −  = (µ − 1)  
f  R −R  R
Let |V− | = |V + | = V
where, f = focal length of the lens
Current through feedback network is
µ = refractive index of the material of the lens.
V − 0 Vo − V R = radius of curvature of the lens.
= =I
R 2R = 0.75 m.
⇒ 2V = Vo − V 1  2 
∴ = (µ − 1)  
⇒ Vo = output voltage = 3 V 0.5  0.75
0.75
Again for the lower network ⇒ = 2 (µ − 1)
0.5
1−V 2 −V 3 −V 0.75
I′ = + + =0 ⇒ µ − 1= = 0.75
R R R 0.5 × 2
⇒ 6 − 3V = 0 ⇒ µ = 1.75
⇒ V =2V
51. ( −2) Given vector field F = y ex − xey .
⇒ Vo = 3 ( V )
Consider the closed triangular contour as shown in
=3×2
figure.
=6V y

49. (358.56) Given mass of O2, m = 16 g


Q
m
Number of moles of O2, n =
M π
π/4 4
where M = molecular weight of O2 = 32 g O
x
P (2,0)
16 1
⇒ n= =
32 2 Let, dl = dx ex + dy ey
Temperature T = 27° C = 300 K
For OP, dy = 0
 75  3
V2 = V1   = V ⇒ dl = dx ex
 100 4 1
where V1 = original volume Also, y = 0 ⇒ F = −x ey

V2 = final volume ∴ ∫ F ⋅ dl = ∫ ( −x e
OP
y ) ⋅ dl
∴ Work done by the gas in the isothermal process.
= − ∫ x ey ⋅ ( ex dx )
V 
W0 = nRT loge  2  =0 [Q ey ⊥ ex ]
 V1 
For the line PQ,
3  Q ≡ (1, 1), dl = dx ex + dy ey
1  V1 
= × 300 × 8.31 × loge  4  Equation of the line is
2  V1  y − 0 = ( −1)(x − 2)
 
⇒ y = −x + 2 …(i)
 3 ⇒ dy = − dx …(ii)
= 150 × 8.31 × loge  
 4 y = 1
x =1
= − 358.56 J ⇒ ∫ F ⋅ dl = ∫ (ye x − x ey ) ⋅ (dx ex + dy ey )
∴ Work done on the gas,W = − W0 = 358.56 J PQ x=2
y=0
22 IIT JAM Physics Solved Papers & Practice Sets

y = 1
x =1 Area of this strip is dA = ydx = (1 − x 2 ) dx
∫ F ⋅ dl = ∫ (y dx − x dy ) Charge as closed by this area
PQ x=2
y=0
dQ = σdA = (15y ) (1 − x 2 ) dx
x = 1 Therefore, total charge on the lamina enclosed
= ∫ [( − x + 2 ) dx − x ( −dx )] between the two curves is
x=2
Q = ∫ σdA = ∫ (15y ) (1 − x 2 ) dx
[From Eqs. (i) and (ii)]
x =1 Given y = 1 − x 2 ⇒ dy = − 2xdx
= ∫ [ −xdx + 2dx + xdx ] ∴
 dy 
Q = ∫ (15y )(y )  − 
x = 2  2x 
x =1
− 15 y 2dy −15 y 2dy
= ∫ 2 dx = 2 (x ) = 2 (1 − 2)
2 ∫ x 2 ∫ 1− y
1
2 = =
x = 2

= −2 Let z = 1 − y ⇒ dz = − dy
Similarly for the line QO, and y = 1 − z ⇒ y 2 = (1 − z )2
Q ≡ (1, 1), O ≡ ( 0, 0), dl = dx e$ x + dy e$ y at y = 0⇒z =1
Equation of the line is and y = 1⇒ z = 0
y − 1 = 1(x − 1) 15 (1 + z 2 − 2 z )
2 ∫
y = x ⇒ dy = dx ⇒ Q =− dz
y = 0
z
x = 0 1 3 1
15 −

2 ∫
=− ( + − 2 ) dz
∫ ∫ y e$
2 2 2
∴ F ⋅ dl = − x e$ y ) ⋅ (dx e$ x + dy e$ y ) z z z
x
QO x =1 0
y = 1  − 1 +1 1
+1

3/2 +1
y = 0 15  z 2
Z 2z 2

x = 0 y = 0 =− + −
2 − 1 + 1 3 1 
= ∫ (y dx − x dy ) =
x =1 y =1
∫ (y dy − y dy ) [Q x = y ] 
 2 2
+1
2
+1
1
y =1 0
 1 5 3 
=0 15  z 2 z 2 2 z 2 
=− + −
∴ ∫ F ⋅ dl = ∫ F ⋅ dl = ∫ F ⋅ dl = ∫ F ⋅ dl 2  1 5 3 
 
OPQO OP PQ QO
 2 2 2 1
= 0 + ( −2 ) + ( 0 ) = − 2 15   2 4 
=− 0 − 2 + −  
−y e$ x + x e$ y 2   5 3 
52. (2 or − 2) Given, E =
x2 + y2 15  30 + 6 − 20 
=+
2  15 
e$ x e$ y e$ z 1
= [16] = 8 C
δ δ δ 2
Now, we have ∇ × E =
δx δy δz 54. (2.5) Consider the given circuit as shown in the figure.
y y
− 2 0 Let collector current be IC and current in the back side
x + y2 x2 + y2
resistors be I. We will neglect the base current as it is
Now, we can calculate very small compared to I and IC . Also, as given in
( e$ x + e$ y + e$ z )
∫ ( ∇ × E) ⋅ da =∫ (∆ × E) ⋅
s
x2 + y2 + z2
the question we are neglecting base to emitter
voltage VBE .
= − 2π or + 2π VCC=10 V
For upper or lower hemisphere, respectively. I IC
53. (8)Given, two curves are y = 0 and y = 1 − x 2 5 kΩ 5 kΩ
Also charge density, σ = 15y c/m . 2

Consider the curves as shown in the figure.


Assume a small strip of width dx at x. B VCE
y
5 kΩ
y=1–x2
y=1–x2 10 kΩ
dx
–1 y=0 O y=0 x
Solved Paper 2016 ● 23

For the front side of the transistor, we have 57. (0.7) Change in wavelength of scattered photon
VCC = IC × (5 kΩ ) + VCE + IE × (10 kΩ ) h
λ′ − λ = (1 − cos θ )
Taking IC ≈ IE , we get mec
10 = IC (15 kΩ ) + VCE
where, me = mass of electron = 9.1 × 10− 31 kg
⇒ VCE = 10 − IC × (15 kΩ ) …(i)
h = 6.6 × 10− 34 Js,
Again for back side of the transistor we can write as
c = 3 × 108 ms −1, θ = 90°
VCC = I × 5 kΩ + I × 5 kΩ
6.6 × 10− 34
⇒ 10 = (10 kΩ ) I ∴ ∆λ = (1 − cos 90° )
⇒ I = 1mA 9.1 × 10− 31 × 3 × 108
Now, from the circuit, we have 0.725 × 10− 3
=
VB = V10 kΩ 3 × 108
⇒ I × 5 kΩ = IC × 10 kΩ 0.725 × 10− 3
⇒ λ′ − λ =
I 3 × 108
⇒ IC = = 0 . 5 mA
2 hc hc 0.725 × 10− 11
⇒ − =
Therefore, from Eq. (i), we get E′ E 3
VCE = 10 − IC × (15 kΩ ) 1 1 0.725 × 10−11
⇒ − =
= 10 − ( 0.5 mA) (15 kΩ ) E′ 20 × 103 × 1.6 × 10− 19 3 × hc
= 10 − 7.5 = 2.5 V 1 1 0.725 × 10− 11
⇒ = − 16
+
55. (1.414) Energy of a relativistic electron is given by E ′ 20 × 1.6 × 10 3 × hc
 h2  1016 0.725 × 10− 11
E 2 = m 2c 4  2 2 2 + 1 = +
m c λ  32 3 × 6.62 × 10− 34 × 3 × 108
where, E = energy = 1MeV 1016 0.725 × 10− 11
= +
mc 2 = rest mass energy = 0.5 MeV 32 9 × 6.62 × 10− 26
λ = wavelength
1 1016 0.725 × 1015
 h 2
 ⇒ = +
∴ (1 MeV )2 = (mc 2 )2  + 1 E′ 32 9 × 6.62
2 2
mc mλ   10 0.725  15
 h2  = +  10
= ( 0.5 MeV)2  + 1  32 9 × 6.62
2
 ( 0.5 MeV) mλ  = 0.324 × 1015 J
 − 34 2
( 6.63 × 10 )  ⇒ E′ = 3.08 × 10− 15 J
( 2 )2 =  − 19
+ 1
 0.5 × 10 6
× 1.6 × 10 × 9.1 = 1.93 × 104 eV

 × 10− 31 × λ2  = 19.3 × 103 eV
43.95 × 10− 68 = 19.3 keV
⇒ 4= +1
7.28 × 10− 44 × λ2 ∴ The kinetic energy transferred to the recoiling
10− 68 + 44 electron
⇒ λ2 = 43.95 × = 2.012 × 10− 24 ∆K = E − E ′
7.28 × 3
= 20 keV − 19.3 keV
⇒ λ = 2 × 10− 12 m = 1.414 × 10− 12 m
= 0.7 keV
56. (0.31) According to Braag’s law for X-ray diffraction, 58. (25) Mass of the aluminium plate
2d sin θ = nλ ma = 0.1 kg
where, θ = Bragg angle = 20° Initial temperature of the aluminium plate, Qia = 95° C
λ = wavelength of X-ray = 0.15 nm, n = 1, 2 , 3 , ..... Final temperature of the aluminium plate
d = interplane separation Qfa = Q (say)
Taking n = 1for maximum intensity of maxima, we have Initial temperature of water
2d sin 20° = λ = 0.15 nm Qiw = 20° C
 0.15  Final temperature of water
⇒ d =  nm = 0.219 nm
 2 sin 20° Qfw = 23° C
Heat lost by aluminium plate
∴ Lattice parameter corresponding to 110 plane is,
∆Ha = (ma ) ( ∆Q ) Ca
a =d 12 + 12 + 02 = ( 0.1) (Qia − Q ) (900)
= ( 0.219 ) ( 2 ) nm = 0.31nm = 90 (95 − Q )
24 IIT JAM Physics Solved Papers & Practice Sets

Heat gained by water 60. (2) If latent heat of vaporisation and vapour pressure of
∆Hw = (mw ) ( ∆Q ) CW water at certain temperature is known, the boiling
= (0.5)(Qfw − Qiw )( 4200) point can be calculated as
−1
= (0.5)( 23 − 20)( 4200)  p  
 R ln  0  
= ( 2100)( 3) 1  p0 
As water is kept inside an insulating container QB =  − 
Q 0 2 
∴ ∆Ha = ∆Hw  
⇒ 90 (95 − Q ) = 2100 ( 3)  
2100 210 where,
⇒ 95 − Q = ×3= = 70
90 3 QB = the boiling point at the required pressure (in K)
Q = 95 − 70 = 25° C p = vapour pressure of the liquid at given pressure
59. (1.9) Let v1 and v 2 be the speeds of the planet at perihelion and  10 
= p 0 1 − 
 100
aphelion positions.
r1 = a(1 − e ) and r2 = a (1 + e ) 90
= p0 ×
100
GM  1 + e 
⇒ v max = v1 =   = 80 km/s p 0 = pressure corresponding to temperature
a  1− e 
Q 0` = atmospheric pressure
where, a = semi major axis, e = eccentricity = 105 Pa
and M = mass of the sun L = heat of vapourisation
GM  1 − e  = 2270 × 103 J/kg
Similarly, v min = v 2 =   = 10 km/s
a 1+ e Q 0 = the boiling temperature
v1 1+ e 1+ e 80 = 373.15 K
∴ = × = =8 −1
v2 1 − e (1 − e ) 10   90  
  p0 × 
1+ e 8.314 ln  100 
⇒ =8 
1− e   p0 
⇒ 1+ e = 8 − 8e  1  
∴ QB = −
⇒ 9e =7  373.15 2270 × 103 
7  
⇒ e=  
9  
So, ratio of the aphelion distance of the correct to the radius  
−1
of the fourth’s orbit is   90  
r2 a (1 + e )  1 8.314 ln  
= =  100 
1.5 × 108 1.5 × 108 = _ 
 373.15 2270 × 10 3

 7   
1+
 9  16
=a  = ×a = 375.15 K
1.5 × 108  9 × 1.5 × 108
 
  ∴ Rise in boiling point of water
Taking standard value of ‘a’, we will get the ratio as 1.9. = ∆QB ≈ 2° C
Solved Paper
2015
IITJAM Physics
MM : 100 Time : 3 hrs

n
This test paper has a total of 60 questions carrying 100 marks. The entire question paper is divided into
three Sections A, B and C. All sections are compulsory. Questions in each section are of different types.
n
Section A contains Multiple Choice Questions (MCQ). Each MCQ type question has four choices out of
which only one choice is the correct answer. This section has 30 Questions and carry a total of 50 marks.
Q.1 – Q.10 carry 1 mark each and Questions Q.11 – Q.30 carry 2 marks each.
n
Section B contains Multiple Select Questions (MSQ). Each MSQ type question is similar to MCQ but
with a difference that there may be one or more than one choice(s) that are correct out of the four given
choices. The candidate gets full credit if he/she selects all the correct choices only and no wrong
choices. This section has 10 Questions (Q. 31 – Q. 40) and carry 2 marks each with a total of 20 marks.
n
Section C contains Numerical Answer Type (NAT) questions. For these NAT type questions, the answer is
a real number which needs to be entered using the virtual numerical keypad on the monitor. No choices
will be shown for these type of questions. This section has 20 Questions and carry a total of 30 marks.
Q.41 – Q. 50 carry 1 mark each and Questions Q.51 – Q.60 carry 2 marks each.
n
Do not write more than one answer for the same question. In case you attempt a subjective question
more than once, please cancel the answer(s) you consider wrong. Otherwise, the answer appearing last
only will be evaluated.
n
Clip board, log tables, slide rule, calculator, cellular phone and electronic gadgets in any form are not
allowed.

Section A Multiple Choice Questions (MCQs)


In these type of questions, each question has four choices (a), (b), (c) and (d) out of which only
one option is correct.

1. A system consists of N number of particles, N >> 1. Each particle can have only one of the two
energies E 1 or E 1 + ε ( ε > 0). If the system is in equilibrium at a temperature T, the average
number of particles with energy E 1 is
N N N
(a) (b) (c) (d) N e −ε/ kT
2 e ε/ kT + 1 e − ε/ kT + 1
4 IIT JAM Physics Solved Papers & Practice Sets

2. A mass m, lying on a horizontal, frictionless surface is connected to


one end of a spring. The other end of the spring is connected to a
wall, as shown in the figure. At t = 0, the mass is given an impulse.
The time dependence of the displacement and the velocity of the m
Impulse

mass (in terms of non-zero constants A and B ) are given by


(a) x(t ) = A sin ωt , v (t ) = B cos ωt (b) x(t ) = A sin ωt , v (t ) = B sin ωt
(c) x(t ) = A cos ωt , v (t ) = B sin ωt (d) x(t ) = A cos ωt , v (t ) = B cos ωt

3. A particle with energy E is incident on a potential given by


 0, x < 0
V (x ) = 
V 0 , x ≥ 0
The wave function of the particle for E < V 0 , in the region x > 0 (in terms of positive constants
A, B and k ) is
(a) Ae kx + Be − kx (b) Ae − kx (c) Ae ikx + Be − ikx (d) zero

4. The electric field of a light wave is given by


  π
E = E 0 $i sin (ωt − kz ) + $j sin ωt − kz −   .
  4 
The polarisation state of the wave is
(a) left handed circular (b) right handed circular
(c) left handed elliptical (d) right handed elliptical
x +y x −y
5. Consider the coordinate transformation x ′ = , y′ =. The relation between the area
2 2
elements dx ′ dy ′ and dx dy is given by dx ′ dy ′ = J dxdy . The value of J is
(a) 2 (b) 1 (c) −1 (d) −2

6. The trace of a 2 × 2 matrix is 4 and its determinant is 8. If one of the eigen values is 2 (1 + i ),
the other eigen value is
(a) 2(1 − i ) (b) 2(1 + i ) (c) (1 + 2 i ) (d) (1 − 2 i )

7. Temperature dependence of resistivity of a metal can be described by


R R R R

(a) (b) (c) (d)

T T T T

8. A proton from outer space is moving towards earth with velocity 0.99c as measured in earth’s
frame. A spaceship, travelling parallel to the proton, measures proton’s velocity to be 0.97c.
The approximate velocity of the spaceship, in the earth’s frame is
(a) 0.2 c (b) 0.3c (c) 0.4c (d) 0.5c

9. A charge q is at the centre of two concentric spheres. The outward electric flux through the
inner sphere is φ, while that through the outer sphere is 2φ. The amount of charge contained in
the region between the two spheres is
(a) 2q (b) q (c) −q (d) −2 q
Solved Paper 2015 ●
5

10. At room temperature, the speed of sound in air is 340 m/s. An organ pipe with both ends open
has a length L = 29 cm. An extra hole is created at the position L /2. The lowest frequency of
sound produced is
(a) 293 Hz (b) 586 Hz (c) 1172 Hz (d) 2344 Hz

11. A system comprises of three electrons. There are three single particle energy levels accessible
to each of these electrons. The number of possible configurations for this system is
(a) 1 (b) 3 (c) 6 (d) 7

12. A rigid and thermally isolated tank is divided into two compartments of equal volume V,
separated by a thin membrane. One compartment contains one mole of an ideal gas A and
the other compartment contains one mole of a different ideal gas B. The two gases are in
thermal equilibrium at a temperature T. If the membrane ruptures, the two gases mix. Assume
that the gases are chemically inert. The change in the total entropy of the gases on mixing is
3
(a) 0 (b) R ln 2 (c) R ln 2 (d) 2 R ln 2
2

13. A Zener regulator has an input voltage in the range 15 V-20 V RS


V0
and a load current in the range of 5 mA-20 mA . If the Zener
voltage is 6.8 V, the value of the series resistor should be +
15-20 V 6.8 V
(a) 390 Ω (b) 420 Ω –

(c) 440 Ω (d) 460 Ω

14. The variation of binding energy per nucleon with respect to the mass number of nuclei is
shown in the figure.
9 Si206 Xe124 Xe134 126
65 Al25
Mo114 He Pt123
O16 13 Fe54 Cu
Mo25
So114
8 P11 Cl U235
Average binding energy per nucleon (MeV)

Xe115 Mo114
C12 F19 W183
N14 Pt206
7 U216
B11

6 Be7

Li7
5 Li6

2 H1
He2
1

0 H1

0 20 40 60 80 100 120 140 160 180 200 220 240


-* Number of nucleons in nucleus, A

Consider the following reactions:


(i) 238 206
92 U → 82 Pb + 10p + 22 n (ii) 238 206
92 U → 82 Pb +8 4
2 He + 6e −
238
Which one of the following statements is true for the given decay modes of U?
92
(a) Both (i) and (ii) are allowed (b) Both (i) and (ii) are forbidden
(c) (i) is forbidden and (ii) is allowed (d) (i) is allowed and (ii) is forbidden
6 IIT JAM Physics Solved Papers & Practice Sets

15. A rigid triangular molecule consists of three non-collinear atoms joined by rigid rods. The
constant pressure molar specific heat (C p ) of an ideal gas consisting of such molecules is
(a) 6R (b) 5R (c) 4R (d) 3R

16. A satellite moves around the earth in a circular orbit of radius R centered at the earth. A
second satellite moves in an elliptic orbit of major axis 8R, with the earth at one of the foci.
If the former takes 1 day to complete a revolution, the latter would take
(a) 21.6 days (b) 8 days (c) 3 hours (d) 1.1 hour

17. A positively charged particle with a charge q, enters a region in which there is a uniform
electric field E and a uniform magnetic field B, both directed parallel to the positive Y-axis.
At t = 0, the particle is at the origin and has a speed v 0 directed along the positive X-axis.
The orbit of the particle, projected on the xz-plane, is a circle. Let T be the time taken to
complete one revolution of this circle. The y-coordinate of the particle at t = T is given by
π2 m E 2 π2 m E π2 m E v0π m 2 πmv 0
(a)
2
(b)
2
(c)
2
+ (d)
2q B qB qB qB qB

18. Vibrations of diatomic molecules can be represented as those of harmonic oscillators. Two
halogen molecules X 2 and Y 2 have fundamental vibrational frequencies νX = 16.7 × 1012 Hz
and νY = 26.8 × 1012 Hz, respectively. The respective force constants are K X = 325 N/m and
KY = 446 N/m. The atomic masses of F, Cl and Br are 19.0, 35.5 and 79.9 atomic mass unit,
respectively. The halogen molecules X 2 and Y 2 are
(a) X2 = F2 and Y2 = Cl2 (b) X2 = Cl2 and Y2 = F2
(c) X2 = Br2 and Y2 = F2 (d) X2 = F2 and Y2 = Br2

19. A hollow conducting spherical shell of inner radius R1 and outer radius
R 2 encloses a charge q inside, which is located at a distance d (< R1 )
R1
from the centre of the spheres. The potential at the centre of the shell is
q
1 q d
(a) zero (b) R2
4πε 0 d
1 q q 1 q q q 
(c)  −  (d)  − + 
4πε 0  d R1  4πε 0  d R1 R2 

20. Doppler effect can be used to measure the speed of blood through vessels. Sound of
frequency 1.0522 MHz is sent through the vessels along the direction of blood flow. The
reflected sound generates a beat signal of frequency 100 Hz. The speed of sound in blood is
1545 m/s. The speed of blood through the vessel, (in m/s) is
(a) 14.68 (b) 1.468 (c) 0.1468 (d) 0.01468

21. Which of the following circuits represents the Boolean expression?


S = P + QR + Q P
P P
(a) S (b) S
Q Q

P
(c) P (d)
Q Q
S S
R R
Solved Paper 2015 ●
7

22. A conducting wire is in the shape of a regular hexagon, which is inscribed


I
inside an imaginary circle of radius R, as shown in the figure. A current I flows R

through the wire. The magnitude of the magnetic field at the centre of the circle C
is
3 µ 0I µ 0I 3 µ 0I 3 µ 0I
(a) (b) (c) (d)
2π R 2 3 πR πR 2π R

23. An observer is located on a horizontal, circular turntable which rotates about a vertical axis
passing through its centre, with a uniform angular speed of 2 rad/s. A mass of 10 g is sliding
without friction on the turntable. At an instant, when the mass is at a distance of 8 cm from the
axis, it is observed to move towards the centre with a speed of 6 cm/s. The net force on the
mass, as seen by the observer at that instant is
(a) 0.0024 N (b) 0.0032 N (c) 0.004 N (d) 0.006 N

24. Miller indices of a plane in cubic structure that contains all the directions [100], [011] and [111]
are
(a) (011) (b) (101) (c) (100) (d) (110)

25. Seven uniform discs, each of mass m and radius r, are inscribed inside a
regular hexagon, as shown in the figure. The moment of inertia of this system
of seven discs, about an axis passing through the central disc and
perpendicular to the plane of the discs, is
7 13 2 55 2
(a) mr2 (b) 7 mr2 (c) mr (d) mr
2 2 2

26. A nucleus has a size of 10 −15 m. Consider an electron bound within a nucleus. The estimated
energy of this electron is of the order of
(a) 1 MeV (b) 10 2 MeV (c) 104 MeV (d) 106 MeV

27. Consider a vector field, F = y $i + xz 3 $j − zy k$


Let C be the circle x 2 + y 2 = 4 on the plane z = 2, oriented counter-clockwise. The value of
the contour integral ∫ F ⋅ d r is
c
(a) 28π (b) 4π (c) −4π (d) −28π

dy y2
28. Consider the equation = with the boundary condition y (1) = 1. Out of the following, the
dx x
range of x in which y is real and finite is
(a) −∞ ≤ x ≤ − 3 (b) −3 ≤ x ≤ 0 (c) 0 ≤ x ≤ 3 (d) 3 ≤ x ≤ ∞

29. The Fourier series for an arbitrary periodic function with period 2L f (x)
∞ ∞ 1
a0 n πx n πx
is given by f (x ) = + ∑ a n cos + ∑ b n sin .
2 n =1 L n =1 L 1/2

For the particular periodic function as shown in the figure, the


x
value of a 0 is –2 –1 0 1 2
(a) 0 (b) 0.5 (c) 1 (d) 2
30. The phase of the complex number (1 + i ) i in the polar representation is
π π 3π 5π
(a) (b) (c) (d)
4 2 4 4
Section B Multiple Select Questions (MSQs)
In these type of questions, each question has four choices (a), (b), (c) and (d) out of which only
one or more than one is/are correct options.
31. For an electromagnetic wave travelling in free space, the electric field is given by
E = 100 cos (10 8 t + kx ) $j V/m.
Which of the following statements are true?
(a) The wavelength of the wave in metre is 6π
(b) The corresponding magnetic field is directed along the positive z-direction
(c) The Poynting vector is directed along the positive z-direction
(d) The wave is linearly polarised

32. In an ideal op-amp circuit shown alongside, Vi +


R1 = 3 kΩ, R 2 = 1 kΩ and Vi = 0.5 sin ωt (in volt). Vo

Which of the following statements are true?
P
(a) The current through R1 = The current through R2 R1

(b) The potential at P is Vo ( R2 / R1 ) R2

(c) The amplitude of Vo is 2 V


(d) The output voltage Vo is in phase with Vi

33. A particle of mass m is moving in XY-plane. At any given time t, its position vector is given by
r (t ) = A cos ωt i$ + B sin ωt $j,
where A, B and ω are constants with A ≠ B. Which of the following statements are true?
(a) Orbit of the particle is an ellipse
(b) Speed of the particle is constant
(c) At any given time t, the particle experiences a force towards origin
(d) The angular momentum of the particle is mω AB k$

34. A rod is hanging vertically from a pivot. A particle, travelling in horizontal direction, collides
with the rod as shown in the figure. For the rod particle system, consider the linear
momentum and the angular momentum about the pivot. Which of the following statements
are not true?

(a) Both linear momentum and angular momentum are conserved


(b) Linear momentum is conserved but angular momentum is not
(c) Linear momentum is not conserved but angular momentum is conserved
(d) Neither linear momentum nor angular momentum are conserved
Solved Paper 2015 ●
9

35. A particle is moving in a two-dimensional potential well


V (x, y ) = 0, 0 ≤ x ≤ L, 0 ≤ y ≤ 2 L
= ∞, elsewhere
Which of the following statements about the ground state energy E 1 and ground state eigen
function φ0 are true?
h2 π2 5h2 π2
(a) E1 = (b) E1 =
mL2 8mL2
2 πx πy 2 πx πy
(c) φ 0 = sin sin (d) φ 0 = cos cos `
L L 2L L L 2L

36. Consider the circuit, consisting of an AC function generator V (t ) = V0 sin 2πνt with V 0 = 5 V,
an inductor L = 8.0 mH, resistor R = 5 Ω and a capacitor C = 100 µF. Which of the following
statements are true if we vary the frequency?
L

R C

(a) The current in the circuit would be maximum at ν = 178 Hz


(b) The capacitive reactance increases with frequency
(c) At resonance, the impedance of the circuit is equal to the resistance in the circuit
(d) At resonance, the current in the circuit is out of phase with the source voltage

37. Muons are elementary particles produced in the upper atmosphere. They have a life time of
2.2 µs. Consider muons which are travelling vertically towards the earth’s surface at a speed of
0.998c. For an observer on earth, the height of the atmosphere above the surface of the earth
is 10.4 km. Which of the following statements are true?
(a) The muons can never reach earth’s surface
(b) The apparent thickness of earth’s atmosphere in muon’s frame of reference is 0.96 km
(c) The lifetime of muons in earth’s frame of reference is 34.8 µs
(d) Muons travelling at a speed greater than 0.998c reach the earth’s surface

38. As shown in the p-V diagram, AB and CD are two isotherms at temperatures T1 and T2 ,
respectively (T1 > T2 ). AC and BD are two reversible adiabats. In this Carnot cycle, which of
the following statements are true?
A
Q1
B
p T1
C
Q2 D T2

V
Q Q
(a) 1 = 2 (b) The entropy of the source decreases
T1 T2
(c) The entropy of the system increases (d) Work done by the system, W = Q1 − Q2
10 IIT JAM Physics Solved Papers & Practice Sets

39. The following figure shows a double slit Fraunhofer diffraction pattern produced by two slits,
each of width a, separated by a distance b, a < b . Which of the following statements are
correct?
Secondary maxima

Primary maxima

(a) Reducing a increases the separation between consecutive primary maxima


(b) Reducing a increases the separation between consecutive secondary maxima
(c) Reducing b increases the separation between consecutive primary maxima
(d) Reducing b increases the separation between consecutive secondary maxima

40. A unit cube made of a dielectric material has a polarisation P = 3 i$ + 4 $j units. The edges of the
cube are parallel to the cartesian axes. Which of the following statements are true?
(a) The cube carries a volume bound charge of magnitude 5 units
(b) There is a charge of magnitude 3 units on both the surfaces parallel to the yz-plane
(c) There is a charge of magnitude 4 units on both the surfaces parallel to the xz-plane
(d) There is a net non-zero induced charge on the cube

Section C Numerical Answer Type Questions (NATQs)


This section contains Numerical Answer Type (NAT) questions. For these NAT type questions, the
answer is a real number.

41. The power radiated by sun is 3.8 × 10 26 W and its radius is 7 × 105 km. The magnitude of the
Poynting vector (in W/cm 2 ) at the surface of the sun is_______.

42. A particle is in a state which is a superposition of the ground state φ0 and the first excited state
φ1 of a one-dimensional quantum harmonic oscillator. The state is given by
1 2
φ= φ0 + φ1. The expectation value of the energy of the particle in this state (in units of
5 5
hω, ω being the frequency of the oscillator) is ________.

43. In an experiment on charging of an initially uncharged capacitor, an R-C circuit is made with
the resistance R = 10 kΩ and the capacitor C = 1000 µF along with a voltage source of 6 V.
The magnitude of the displacement current through the capacitor (in µA), 5 s after the charging
has started is _______.

44. In the given circuit, V CC = 10 V and β = 100 for the n-p-n transistor. The collector voltage VC
(in volts) is ________.
VCC

1Ω

100 Ω VC

+
5V

Solved Paper 2015 ●
11

45. Unpolarised light is incident on a calcite plate at an angle of incidence 50°, as shown in the
figure. Take, n 0 = 1.6584 and ne = 1.4864 for calcite. The angular separation (in degrees)
between the two emerging rays within the plate is _________.

Air
50°
O

Calcite
pt
ic
ax
is

46. In the hydrogen atom spectrum, the ratio of the longest wavelength in the Lyman series (final
state n = 1) to that in the Balmer series (final state n = 2) is _______.

47. A rod is moving with a speed of 0.8c in a direction at 60° to its own length. The percentage
contraction in the length of the rod is _________.

48. X-rays of wavelength 0.24 nm are Compton scattered and the scattered beam is observed at
an angle of 60° relative to the incident beam. The Compton wavelength of the electron is
0.00243 nm. The kinetic energy of scattered electrons in eV is _______.

49. A diode at room temperature (kT = 0.025 eV) with a current of 1µA has a forward bias voltage
VF = 0.4 V. For VF = 0.5 V, the value of the diode current (in µA) is _______.

50. Ga-As has a diamond structure. The number of Ga-As bonds per atom which have to be
broken to fracture the crystal in the (001) plane is _________.

51. In the thermodynamic cycle shown in the figure, one mole of a monoatomic ideal gas is taken
through a cycle. AB is a reversible isothermal expansion at a temperature of 800 K in which
the volume of the gas is doubled. BC is an isobaric contraction to the original volume in which
the temperature is reduced to 300 K. CA is a constant volume process in which the pressure
and temperature return to their initial values. The net amount of heat (in joule) absorbed by the
gas in one complete cycle is __________.
Pressure

p1 A

p2 C
B

V 2V
Volume

52. In a region of space, a time dependent magnetic field B(t ) = 0.4t Tesla points vertically
upwards. Consider a horizontal, circular loop of radius 2 cm in this region. The magnitude of
the electric field (in mV/m) induced in the loop is ________.

53. A plane electromagnetic wave of frequency 5 × 1014 Hz and amplitude 10 3 V/m, travelling in a
homogeneous dielectric medium of dielectric constant 1.69, is incident normally at the interface
with a second dielectric medium of dielectric constant 2.25. The ratio of the amplitude of the
transmitted wave to that of the incident wave is ________.
12 IIT JAM Physics Solved Papers & Practice Sets

54. For the arrangement given in the following figure, the coherent light sources A, B and C have
individual intensities of 2 mW/m 2 , 2 mW/m 2 and 5 mW/m 2 , respectively at point P. The
wavelength of each of the sources is 600 nm. The resultant intensity at point P (in mW/m 2 ) is
_________.
P

15 mm
A
3.22 mm
B 1m

2.04 mm
C

55. One gram of ice at 0°C is melted and heated to water at 39°C. Assume that the specific heat
remains constant over the entire process. The latent heat of fusion of ice is 80 cal/g. The
entropy change in the process (in calories per degree) is _______.

56. A uniform disc of mass m and radius R rolls, without slipping, down a fixed plane inclined at an
angle 30° to the horizontal. The linear acceleration of the disc (in m/s 2 ) is _______.

57. A nozzle is in the shape of a truncated cone, as shown in the figure. The
area at the wide end is 25 cm 2 and the narrow end has an area of
1 cm 2 . Water enters the wider end at a rate of 500 g/s. The height of the
50 cm
nozzle is 50 cm and it is kept vertical with the wider end at the
bottom. The magnitude of the pressure difference in kPa (1 kPa
= 10 3 N / m 2 ) between the two ends of the nozzle is ________.

58. A block of mass 2 kg is at rest on a horizontal table. The coefficient of friction between the
block and the table is 0.1. A horizontal force 3 N is applied to the block. The speed of the
block (in m/s) after it has moved a distance 10 m is ________.

59. A homogeneous semi-circular plate of radius R = 3 m is shown in the figure. The distance of
the centre of mass of the plate (in metre) from the point O is _________.

O
3m

60. Consider a 20 µm diameter p-n junction fabricated in silicon. The donor density is
1016 per cm 3 . The charge developed on the n-side is 1.6 × 10 −13 C. Then, the width (in µm)
of the depletion region on the n-side of the p-n junction is _________.

Answers
1. (c) 2. (a) 3. (c) 4. (b) 5. (c) 6. (a) 7. (a) 8. (d) 9. (b) 10. (c)
11. (d) 12. (d) 13. (a) 14. (c) 15. (c) 16. (b) 17. (b) 18. (b) 19. (d) 20. (c)
21. (b) 22. (c) 23. (c) 24. (*) 25. (d) 26. (b) 27. (a) 28. (d) 29. (c) 30. (c)
Answers with Explanations
Section A Multiple Choice Questions
1. (c) For a system of identical fermions in thermal equilibrium, Potential barrier with particle of energy E < V0 incident
the average number of particles in a single quantum from the left.
V (x)
state of energy E is given by the Fermi occupation factor
1
F (E ) =
(E − µF ) / kT +1 V0
e
Reflecting particle
where, µF is a characteristic energy for the system.
According to given problem, the average number of
particle with energy E is equal to Incoming particle
X
N N O
(E1−E1− ε) +1
=
e− ε/ The above figure is illustrated what we would expect to
kT
+1
e kT happen if a classical particle of energy E < V0 were
2. (a) Consider the block of mass m as point mass (particle). incident on the barrier, it would simply bounce back as it
has insufficient energy to cross over to region x > 0,
At any point, the displacement x of the particle is equal
Given that the potential is as given above, the
to the extension or compression of the spring from its
Schrödinger equation comes in two parts.
natural length and the restoring force will be F = − kx ,
h2d 2ψ
where, k is the spring constant ma = − kx . − = Eψ, x < 0
2 m dx 2
d 2x k
⇒ =− x h2d 2ψ
dt 2  m − + V0ψ = Eψ, x > 0
2 m dx 2
d 2x  k
⇒ + ω 2x = 0 Q ω2 = where, E is the total energy of the particle.
dt 2 
 m 
We can rewrite these equations in the following ways.
Hence, the particle will execute simple harmonic motion.
d 2ψ 2 mE
We can write the equation of motion as + ψ = 0, x < 0 ... (i)
dx dx 2 h2
= ω A 2 − x 2, [where, A is the amplitude] d 2ψ 2m
dt − 2 (V0 − E )ψ = 0, x > 0 ... (ii)
x dx t dx 2 h
⇒ ∫x 0 A 2 − x 2 = ∫0ω. dt If we put k =
2 mE
h
x
 −1 x  d 2ψ
⇒ sin  A   = ωt Then, the Eq. (i) becomes + k 2ψ = 0, x < 0
 x 0 dx 2
x x  which has the general solution ψ = Aei kx + Be − i kx .
⇒ sin− 1   − sin− 1  0  = ωt
 A  A where, A and B are unknown constants.
x In the region x > 0, we write
⇒ sin− 1   = ωt [Qx 0 = 0]
 A
2m (V0 − E )
x k = >0
= sin ωt h
A
So, that the Schrödinger equation becomes
⇒ x (t ) = A sin ωt ... (i)
d 2ψ
d d − k 2ψ = 0, x > 0
(x ) = ( A sin ωt ) dx 2
dt dt
which has solution ψ = Ae − kx + Bekx ... (iii)
⇒ v (t ) = A ω cos ω t = B cosωt
where, A and B are constants.
where, B = A ω
The problem here is that the exp (kx ) solution grows
3. (c) The Schrödinger wave equation for the scattering form exponentially with x, and we do not want wave functions
of potential barrier. that become infinite, it would essentially mean that the
In this case, the potential will be of the form. particle is forever to be found at x = ∞, which does not
 V (x ) = 0, x < 0  make physical sense. So, we must put B = 0 in Eq. (iii).
 = V0, x > 0 Thus, we get ψ = Ae − kx , x > 0

14 IIT JAM Physics Solved Papers & Practice Sets

4. (b) The electric field is given by From the given expression, we can say that resistivity of
metal increases with temperature and remains almost
  π 
E = E0 $i sin(ωt − kz ) + $j sin ωt − kz −   constant at very low temperature.
  4 
8. (d) Velocity of the proton, v p = 0.99 c
Both x and y-components of the electric field are having
save amplitude, therefore the electric field is circularly Velocity of the proton w.r.t. the spaceship
polarised. v ps = 0.97c
The wave is propagating along positive z-direction which Velocity of the spaceship, in the earth’s frame is
can be found by right handed system rule ( $i × $j = k$ ). v p − v ps
vse =
Therefore, the wave is right handed circularly polarised. v p v ps
1−
x +y x −y c2
5. (c) Given, x ′ = and y ′ = 0.99c − 0.97c
2 2 =
( 0.99c )( 0.97c )
As , dx ′ dy ′ = J dx dy 1−
c2
 ∂ ∂  0.99 − 0.97
(x ′ ) (x ′ ) =
 ∂x ∂ y  c
⇒ J =   1− ( 0.99 )( 0.97)
∂ ∂
 (y ′ ) (y ′ ) 0.02c
 ∂x ∂y  ∴ vse = ≈ 0.5c
0.039
1/ 2 + 0 0 + 1/ 2 
or J=   9. (b) As, φs1 = φ
1/ 2 + 0 0 − 1 2 
1/ 2 1/ 2 
∴ J =  
1/ 2 − 1/ 2  q1

1 1 1  q
J = 1 − 1
2  
S1 S2
∴ Value of J is
1 1 1 
=
2 1 − 1
q
⇒ φ= ... (i)
2 ε0
 1
J =   ( − 1 − 1) As, φs2 = 2φ
 2
q + q1
1 ⇒ 2φ =
J = × − 2 = −1 ε0
2 2q q + q1
∴ =
6. (a) Suppose other eigen value is λ 2. ε0 ε0
Q Sum of the eigen values = trace of matrix or q1 = q
∴ λ1 + λ 2 = 4 10. (c) If an extra hole is created at the position L/2, then the
∴ λ 2 + 2 (1+ i ) = 4 middle point of the organ pipe will be a displacement
⇒ λ 2 = 2 (1− i ) …(i) antinode.
L
Q Product of the eigen values = determinant of the matrix
Q 2 ( λ 2 )(1+ i ) = 8
or λ 2 (1+ i ) = 4 …(ii)
From Eqs. (i) and (ii), we get
A N A N A
2 (1− i )(1+ i ) = 4
Hole
or 2 (1− i ) 2 (1+ i ) = 8
or λ1 λ 2 = 8 The diagram shows the wave pattern corresponding of
∴ λ 2 = 2 (1− i ). lowest frequency
λ λ λ
7. (a) Resistivity (ρ) of a metal depends upon temperature (T ) ∴ L= + + =λ
4 2 4
as v
or L=
ρ = ρ 0 [1+ α (T − T0 )] ν
where, ρ 0 and α are constants. T0 is a reference v 340
or ν= = = 1172 Hz
temperature. L 29 × 10−2
Solved Paper 2015 ●
15

238
11. (d) Possible seven configurations are 14. (c) U92 is one of the heaviest nucleus, the binding energy
States 1 2 3 of nucleus is about 7.5 MeV per nucleon.
238
(i) ↑ ↑ ↑ ∴ U92 undergoes α-decay the following steps are
(ii) ↑↓ ↑ – included.

(iii) ↑ ↑↓ – 238 (α) 238 (β)– 234 (β)– 234


U Th Pa U
92 90 91 92
(iv) ↑↓ – ↑
(α)
(v) ↑ – ↑↓
(vi) – ↑ ↑↓ 218 (α) 222 (α) 226 (α) 230
Po Rn Ra Th
(vii) – ↑↓ ↑ 84 86 88 92

(α)
12. (d) The entropy of the given system is the sum of entropies
of the two parts. S system = S gas A + S gas B , when the
214 (β)– 214 (β)– 214 (α) 210
partition is taken away (or the membrane ruptures) the Pb Bi Po Pb
82 83 84 82
gases (A and B) are free to diffuse throughout the
volume. For an ideal gas, the energy is not a function of (β)–
volume and for each gas, there is no change in
temperature. 210 (β)– 210
Po Bi
84 83
The energy of the overall system is unchanged, the two
gases were at the same temperature initially, so the final (α)
temperature is the same as the initial temperature. The
entropy change of each gas is thus, the same as that for 206 Stable
Pb
a reversible isothermal expansion from the initial specific 82 Nucleus

volume Vi to the final volumeVf . For n mole of ideal gas,


V  Total 8 α-decay and 6 β- decay
the entropy change is ∆S = nR In  f  , the entropy 238 206
 Vi  ∴ U92 → Pb82 + 8 α + 6 β −
change of the system is
15. (c) In general, the number of degrees of mechanical system
∆S system = ∆S gas A + ∆S gas B is equal to the total number of coordinates required to
V  V  specify the positions of all the constituent particles
∆S system = n A R ln  f  + n B R ln  f 
 Vi   Vi  minus the number of independent relations between the
Vf = 2V , Vi = V , n A = nB = 1 constituent particles.
If A be the number of particles in a system and R the
⇒ ∆S system = R ln 2 + R ln 2
number of independent relations between them, then the
∆S system = 2R ln 2 number f degree of freedom is given by f = 3A − R.
13. (a) According to the question voltage of the Zener diode is In the given question, A = 3 and R = 3
VZ = 6.8 V ⇒ f = ( 3 × 3) − 3 = 6
Rs Relation between f and Cp
f  6 
Cp =  + 1 R ⇒ Cp =  + 1 R = 4 R
V VZ 2  2 

16. (b) According to Kepler’s third law,


V − Vz
Current in the circuit can be written as I = T 2 ∝ a3
Rs
where, T = time period of revolution of satellite.
Vmax − Vz
I max = … (i) a = semi-major axis
Rs
Vmin − Vz For the first case, T12 ∝ R 3
I min = … (ii) 3
Rs  8R 
For the second case, T22 ∝  
 2
Given, Vmax = 20 V
2 3 3
Vmin = 15V  T1   2R   1 1 T1 1
∴   =   =   = ⇒ =
I max = 20 mA and I min = 5mA  T2   8R   4 64 T2 8
Solving above equations, we can find Rs ~
− 390 Ω or T2 = 8 (T1) = 8 (1day ) = 8 days.
16 IIT JAM Physics Solved Papers & Practice Sets

17. (b) When the charge particle is projected along the positive 325 × 10−24
mX2 = kg
X-axis. It will experience a magnetic force FB = qv 0 B 16.7 × 16.7 × 4π 2
which will provide the centripetal force, hence the particle
1 amu = 1.67 × 10−27 kg
will move in circular path in X Z - plane, it will also
1
experience the force due to electric field (Fe = qE ) in ∴ 1 kg = × 1027 amu
1.67
Y-direction. Hence, the particle will move in helical path.
325 × 10−24 1027
Y ⇒ mX2 = 2
× amu
16.7 × 16.7 × 4π 1.67
mX2 = 17.7 amu (approx)
⇒ X ⇒ 17.7 × 2 = 35.4 ~ − 35.5 amu
E B
(X → chlorine)
Similarly,
X 446 1 1027
q v0 m Y2 = × × amu
( 26.8 × 1012 )2 4π 2 1.67
mY = 9.43 amu (approx)
2
Z y ⇒ 9.43 × 2 ~ − 18.86 ~− 19
mv 02 ∴ (Y→flourine)
FB = [X 2 = Cl2, Y2 = F2]
r
mv 02 19. (d) Charge ( − q ) is induced on the inner surface of the
⇒ qv 0 B =
r shell and ( + q ) will be induced on the outer surface of
⇒ r =
mv 0 the shell.
qB
where, r is the radius of circular path of the charge R2
particle. q
q
Time period of one revolution,
q
2πr 2π mv 0
T = = d R1
v0 qBv 0
2πm
⇒ T =
qB
Calculation for Y -coordinate of the particle
The potential at the centre of the shell is given by
The motion of the charge particle along Y-axis is
uniformly accelerated motion. q q q
V = − +
F qE 4πε 0d 4πε 0R1 4πε 0R2
ay = = ,
m m 1 q q q
or V =  − + 
Initial velocity of the charged particle along Y-axis, u y = 0 4πε 0  d R1 R2 
1
Using, s y = u yt + a yt 2, t = T 20. (c) Let v = velocity of sound = 1545 m/s
2
1 qE 2 vb = velocity of blood
⇒ y = . T
2 m According to Doppler’s effect
1 qE 4π 2m 2 v − vb 
⇒ y = × 2 2 f′ =  f
2 m q B  v  0
2π 2mE where, f0 = frequency of sound
∴ y = 2 f ′ = changed frequency
qB
f0 − f ′ = beat signal
1 k
18. (b) As, f = f0 − f ′ = 100 Hz
2π m
∴ f ′ = f0 − 100 Hz
1 k
⇒ f2 = ⋅ f ′ = 1.0522 MHz − 100 Hz
4π 2 m
f ′ = 1.0521MHz
k
∴ m= ∴ vf ′ = (v − vb )f0
4π 2f 2 or vf ′ = vf0 − vbf0
325 vf ′ − vf0
∴ For X 2, mX2 = or = − vb
(16.7 × 1012 )2 × 4π 2 f0
Solved Paper 2015 ●
17

vf0 − vf ′ Centrifugal force, Fcf = mω 2r


or vb =
f0 (10) 8
= ×2×2×
 f − f ′ 1000 100
or vb = v  0 
 f0  = 32 × 10− 4 N (in radial direction)
1545 × 100
or vb = = 0.1468 m/s ∴ Total force, F = Fc2 + Fr 2
1.0522 × 106
∴ Velocity of blood = 0.1468 ms−1 = ( −24 × 10− 4 )2 + ( 32 × 10− 4 )2

21. (b) S = P + QR + Q P = 242 + 322 × 10− 4


= 40 × 10− 4
S = P + QR ⋅ Q P (by de-Morgan’s law)
= 0.004 N
S = (P + QR ) ⋅ (QP )
S = PQ P + QQ RP 24. (*) Planes of the cubic structures are [100], [011] and [111]
Intercepts of the plane [100] are 1, 0 and 0.
S = PQ + 0 (QQQ = 0)
Reciprocal of these intercepts are 1, ∞ and ∞.
= PQ
P Specifying the intercepts in fractional coordinates, we
PQ
can write them as 1, ∞, ∞.
Q Reciprocal of these are 1, 0, 0
22. (c) Similarly, for the plane [011] it is [011] and for [111] it is
B [111]

60°
25. (d)
R 60° A
√3 R
30°

2
R
30°

The moment of inertia of a disc about its centre is


1
I = mr 2
2
The magnitude of magnetic field B T at centre is Q I parallel = I centre + 4mr 2
6 × magnetic field due to AB. 1
AB is a finite straight conductor, carrying current and =mr 2 + 4mr 2
2
making an angle 30° each from centre to AB.
∴ Total moment of inertia,
µ I
B = 0 (sin φ1 + sin φ 2 ) 1 2 1 
4 πr I= mr + 6 mr 2 + 4mr 2
2  2 
3
φ1 = φ 2 = 30°, r = R  mr 2 
2 1 2
I= mr + 6 9 
µ 0I 2  2 
∴ B= (sin 30° + sin 30° )
 3  (1 + 54) 55
4π  R ∴ I= mr 2 = mr 2
 2  2 2
µ 0I  1 1 µ 0I
B=  +  = 26. (b) Size of nucleus = 10−15 m
2π 3R  2 2 2 3 πR
Using Heisenberg’s uncertainty principle
∴ Total magnetic field h
µ 0I µ I ∆x ∆p ≥
BT = 6 × B = 6 × = 3 0  4π
2 3 πR  πR 
h
∆p ≥
23. (c) Two forces are acting on the particle, 4π∆x
i.e. Coriolis and centrifugal. Moment energy ∆pc ≥
hc
Coriolis force, Fc = − 2 m (ω × v ) 4π∆x
10  6  6.6 × 10−34 × 3 × 108
=−2× 2 ×  E≥ J
1000  100 4 × π × 10−15
Fc = − 24 × 10− 4 N (tangentially) E ≥ 1.57 × 10−11 J
18 IIT JAM Physics Solved Papers & Practice Sets

Rest mass energy, E0 = m0 c 2 y− 2+1


⇒ = ln x + c
E0 = 9.1 × 10−31 × ( 3 × 108 )2 J −2+1
1
E0 = 8.19 × 10−14 J or − = ln x + c
y
∴ ET = E02 + ( ∆pc )2 [QE0 << E] Boundary condition, y(1) = 1
−11 ∴ x = 1, y = 1
ET = 1.57 × 10 J
− 1 = ln 1 + c
1.57
ET ~
− × 10−11 × 1013 MeV c = −1
1.6
1 1
[Q 1J = × 1013 MeV] ∴ − = ln x + ( − 1)
1.6 y
or ET = 0.99 × 102 MeV 1
or 1 − = ln x
y
Order = 102 MeV  1
1 − 
27. (a) Vector field, F = y$i + xz 3 $j − zy k$ or e
 y
=x
C is circle x 2 + y 2 = 4 For real and finite y
Radius (r ) of circle = 2 x = e(1 − 1/ y)
∴ r = 2(cos θ )$i + 2(sin θ )$j x ∈ [ 3, ∞]
∴ d r = ( − 2 sin θ$i + 2 cos θ$j ) dθ 29. (c)
Putting z = 2 and hence,
1 f(x)
π
∫c F ⋅ dr = 2 ∫ F ⋅ d r
0
π 1/2
= 2∫ (y$i + 8x$j − 2y k$ ) ⋅ (d r )
0
π
= 2∫ ( 2 sin θ$i + 16 cos θ$j − 2 sin θk$ ) ⋅ (d r )
0 x
–2 –1 0 1 2
π
= 2∫ (( 2 sin θ )( − 2 sin θ ) + 32 cos2 θ )) dθ
0 ∞ ∞
a0 nπ x nπ x
π 2 2 f (x ) = + ∑ an cos + ∑ bn sin
= 2∫ [ − 4 sin θ + 32 cos θ] dθ 2 n =1 L L
0 n =1
π
= 2∫ [ − 4 + 36 cos2 θ] dθ Period, 2L
0
We can rewrite the function as,
π   1 + cos 2θ   − x for − 1 ≤ x ≤ 0
= 2∫ − 4 + 36   dθ
0  2  f (x ) = 
x for 0 ≤ x ≤ 1
π
= 2∫ ( − 4 + 18 + 36 cos 2θ ) dθ ∴ Period, 2 L → [ − 1, 1]
0
π 1
= 2∫ [14 + 36 cos 2θ] dθ
0
⇒ a0 = ∫ f (x ) dx
π −1
 36 sin 2θ  0 1
= 2 14 θ +
 2  0 or a0 = ∫ f (x ) dx + ∫ f (x ) dx
 36 sin 2π  −1 0
= 2 14π + − ( 0 + 0) 0 1
 2 
or a0 = ∫ ( − x ) dx + ∫ (x ) dx
= 28π −1 0

dy y 2
 x 2 0
x 2 
1
28. (d) = or a 0 = −  + 
dx x
 2 − 1  2 0
dy dx
⇒ =  1 1
y2 x = 0 − −  +
 2 2
Integrating both sides,
1 1
dy dx or a0 = + ⇒ a0 = 1
∫ y2 = ∫ x 2 2
Solved Paper 2015 ●
19

30. (c) z = (1 + i )i It is a non-inverting voltage feedback amplifier,


z =i + i 2
=i −1  R2 
Vi = Vp =   Vo ⇒ Vp = B Vo
or z = − 1+ i  R1 + R2 
∴ | z| = 2 [Q x 2 + y 2 ] R2
where, B =
R1 + R2
Polar representation
Vo and Vi will be in same phase;
z = r [cos θ + i sin θ]
1  R + R2 
z = 2 cos θ + 2 i sin θ ⇒ Vo = Vi ⇒ Vo = Vi ×  1 
B  R2 
−1 1
⇒ cos θ = , sin θ =  3 + 1
2 2 Vo = 0.5 ×   V = 0.5 × 4 = 2V
 1 
∴ tan θ = − 1 Vo − Vp 2 − BVo
Imaginary axis IR1 = =
R1 R1
1
45° 2− ×2
θ 90° = 4
3
Real axis 1.5 1
= = = 0.5 mA
3 2
θ Phase angle Vp 0.5
IR 2 = = = 0.5 mA
R2 1k Ω
⇒ θ = π + π /4
3π ∴ IR1 = IR 2 = 0.5 mA
θ=
4 It is a non-inverting amplifier, so Vo and Vi will be in same
3π phase.
∴ Phase of (1 + i ) i is
4 ∴ Only (b) is incorrect.
33. (a, c, d) Given, r(t ) = A cos ωt $i + B sin ωt $j
dr(t )
Section B Multiple Select Questions ⇒ v= = − Aω sin ωt $i + Bω cos ωt $j
dt
31. (a, d) E = 100 cos (108t + kx )$j V/m Speed = | v| = A 2ω 2 sin2 ωt + B 2 ω 2 cos2 ωt
E = 100 cos(ωt + kx )$j
| v| = ω A 2 sin2 ωt + B 2 cos2 ωt
ω = 108 rad/s ⇒ 2πf = 1008 rad/s
c B2
or 2π = 1008 | v| = ωA (1 − cos2 ωt ) + cos2 ωt
λ A2

or λ = 8 × 3 × 108 = 6π m  B 2 − A 2
10 | v | = ωA 1 +   cos2 ωt
 A2 
Corresponding magnetic field ⇒ ( − z ) direction
Poynting vector, S = E × H | v| depends on time.
As A ≠ B
⇒ Direction = $j × ( − k$ ) = − $i in negative X-direction.
∴ r(t ) is elliptical in nature
Wave is linearly polarised, because it moves in
dv
one-dimension.. a (t ) = = − A ω 2 cos ωt $i + B ω 2( − sin ωt )$j
dt
32. (a, c, d)
= − ω 2( A cos ωt$i + B sin ωt $j )
R1 = 3 k Ω, R2 = 1k Ω
∴ a (t ) = − ω 2r(t )
Vi = 0.5 sin ωt
⇒ F (t ) = m a (t ) = − mω 2 r(t )
+
Vi v2 mv 2 $

Vo ⇒ −m 2
⋅ r(t ) = −
r
r r
Which is towards the centre.
P Angular momentum = m(r × v )
R2 R1
V2 = m( ABω cos2 ωt k$ + ABω sin2 ωt k$ )
= mAB ω k$
20 IIT JAM Physics Solved Papers & Practice Sets

34. (a, b, d) 37. (c, d) Life time, τ = 2.2 µs, u = 0.998 c


For the given particle rod system, the net force acting is
u2
zero γ = 1− = 1 − ( 0 . 998)2
⇒ Angular momentum = constant. c2
Q d L=0 or γ = 1 − 0.996004
L = constant or γ = 0.003996
∴ Angular momentum is conserved. or γ = 0.0632
But the linear momentum is being changed for particle τ
∴ Muon's life time in earth's frame, T =
rod system. γ
35. (b, c) V (x , y ) = 0 0 ≤ x ≤ L, 1 ≤ y ≤ 2L 2.2 µs ~
= − 34.8 µs
= ∞ elsewhere 0.0632
 πx   πy  Muons travelling at speed greater than 0.998 c reach the
ΨA = A sin n x  sin n y  earth's surface.
 L  2L 
Muons reach the surface of earth because ; the
 πx   πy 
n=0 Ψ = A sin   sin   apparent thickness of atmosphere;
 L  2L 
L′ = L0γ ⇒ L′ = 10.4 × 0.0632
Now to find A.
L 2L L′ = 0.657 km
Normalisation condition : ∫ ∫0 | Ψn xn y (x , y )|2 dxdy = 1
0 ∴ Time taken to reach earth,
L  nxπ  2L  nyπ  L′ 0.657 × 103
⇒ A2 ∫0 sin
2
  dx ∫0 sin2   dy = 1 t= = s
 L   2L  v 0.998 × 3 × 108
L 2L L2 = 0.2194 × 10− 5
or A2 . . = 1 ⇒ A2 × =1
2 2 2 ∴ t = 2.19 µs i.e.just before the life time.
2 2
or A2 = 2 ⇒ A= 38. (a, b, d) During adiabatic process, no addition of heat takes
L L
place.
2  πx   πy 
⇒ Ψ0 = sin   sin  
L  L  2L  A

h 2π 2  n x2 n y 
2 Q1
En xn y = + 2 B
( 2m )  L2x Ly 
h 2π 2  1 1  D Q2
⇒ E1 = × 2 + 
C
2 m L ( 2L)2 
h 2π 2  4 + 1 5 h 2π 2 ∴ Change in entropy during this,
or E1 = =
2 m  4L2  8 mL2 Q1 Q 2
− = 0
T1 T2
36. (a, c) Given, V (t ) = V0 sin 2πνt Q1 Q 2
⇒ =
Given, V0 = 5 V, L = 8 mH, R = 5 Ω and C = 100 µF. T1 T2
Current is maximum at resonant frequency,
The entropy change when gas changes volume at
1 1
ν= = constant temperature.
2π LC 2π 8 × 10−3 × 100 × 10−6 Sf V2 pdV

1 104
As pV = nRT , ∫Si dS = ∫V1 T
or ν= = RT
2π 8 × 10−7 4 20π ⇒ p= [for 1 mole]
V
104 V2 
or ν= = 178 Hz Sf V2 RT dV
8 5π ∴ ∫Si dS = ∫V1 V T ⇒ ∆S = R loge  V1 
1 1
Capacitive reactance, Xc = ⇒ Xc ∝ = V2 < V1 ⇒ ∆S decreases.
ωC ω
At resonance, Z = R Work done by the system = Change in heat
 X − XC  As, ∆Q = ∆W + ∆U
φ = tan−1 L 
 R  ⇒ Q 2 − Q1 = − W + 0
⇒ φ = 0 at resonance No change in internal energy.
Thus, at resonance current is in phase with voltage. ⇒ W = Q1 − Q 2
Solved Paper 2015 ●
21

39. (a, d) The Fraunhofer diffraction pattern, is produced by a According to formula,


slit and the intensity distribution consisted of maxima V −t/τ
I (t ) =
(e )
and minima. It is produced by two parallel slits each of R
width a separated by a distance b. We know that, the where, τ = time constant
resultant intensity distribution is a product of the single τ = RC
slit diffraction pattern and the inteference pattern τ = 10 × 103 × 1000 × 10−6 s
produced by two point sources separated by a distance τ = 107 × 10−6 s = 10 s
b. Based on the result, we concluded that, on reducing a V
increases the separation between consecutive primary ∴ I (t ) = (e −t / 10 ) A
R
maxima and on reducing b increases the separation V −5 / 10
between consecutive secondary maxima. At t = 5 s, I (5) = (e )A
R
40. (b, c) Since, P = 3$i + 4$j 6 6 × 10−4
I= (e −0.5 ) A = A
Option (a) is wrong 10 × 10 3
e 0.5
ρb = ∇ − P = 0
or I = 3.639 × 10−4 A= 363.9 × 10−6 A
Option (b) is correct
∴ I~
− 364 µA
At x = 0, σb = P . n$ = ( 3$i + 4$j ) . ( − $i ) = − 3
44. 5 .5 to 5 . 8
At x = 1, σ = P . x$ = ( 3$i + 4$j ) . ( $i ) = 3
b
VCC (10V)
Option (c) is correct
At y = 0, σb = P . x$ = ( 3$i + 4$j ) ( − $j ) = − 4 1kΩ

At y = 1, σ = P . x$ = ( 3$i + 4$j ) . ( $j ) = 4
b 100 kΩ C VC
B
Option (d) is wrong.
+ E
41. 6170 to 6175 5V VBE
Poynting vector represents the directional energy flux –
density (the rate of energy transfer per unit area) on an
electromagnetic field. The SI unit of the Poynting vector
is watt per square metre ( W /m2 ). As, VBB − VBE = IBRB
Power radiated by sun ∴ IBRB = (5 − 0.7) [QVBE = 0.7 V]
Magnitude of Poynting vector = 4.3
4πr 2 ⇒ IB = A
5 5 5 100 × 103
where, r = 7 × 10 km = 7 × 10 × 10 cm
43
3.8 × 1026 or IB = 6 A ~ − 43 µA
= 20 10
4 × 3.14 × 7 × 7 × 10 IC
3.8 As, β = = amplification factor
= × 106 IB
4 × 3.14 × 49
Here, β = 100, IC = β × 43 µA = 4.3 mA
= 0.006174 × 106 = 6174 W / cm2
42. 1. 2 to 1.5 For output voltage,
1 VCC = 10 V and RL = 1kΩ
As, En = n + hω As, VCC − ICRL = VC
2
 hω  1  3hω  4 ∴ VC = 10V − 4.3 × 10−3 × 1.0 × 103 V
P   = ,P   =
 2 5  2  5 or VC = 10V − 4.3 × 10 V
hω 1 3hω 4 or VC = 5.7 V
<E > = × + ×
2 5 2 5 45. 3 .5 to 3 . 6
hω 12hω 13hω
= + = = 1.3 hω Given, n 0 = 16584
. , ne = 1.4864 and θ = 50°
10 10 10
in terms of hω = 13
. 50°

43. 360 to 365


R C
Axis Calcite
10 k Ω 1000 µF
sin 50° = 0.76604
+ – sin i
The refractive index, n 0 =
6V
sin r
22 IIT JAM Physics Solved Papers & Practice Sets

Let r1 is the first angle of refraction, The rod is moving withv = 0.8 c at 60°.
sin θ ∴ Horizontal component of velocity,
∴ sin r1 =
n0 v cos θ = 0.4 c
 0.76604 1
or r1 = sin− 1   {Q| vH | = | v| cos θ = | v| cos 60° = 0.8c × = 0.4 c}
 1.6584  2
Length contraction
or r1 = sin− 1 ( 0.4619 )
or r1 = ( 27.51)0 = 27°30′ 36′′ v2
l′ = l0 1 − [by formula]
sin i c2
The refractive index , ne =
sin r2 l′ ( 0.4c )2
0.76604 ∴ = 1−
∴ sin r2 = l0 c2
.
14864
l′
or r2 = 31. 022° = 1 − 0.16 = 0.84
l0
r2 = 31° 20° 31′′
Angular separation, l0 − l ′
∴ % change in length = × 100%
(r2 − r1) = 31.022 − 27.51~ − 3.512° approx l0

46. 0 .15 to 0 . 20  l′


∴ % change = 1 −  100%
1  1 1  l0 
As, = R 2 − 2 
λ n1 n 2  = (1 − 0.84 ) × 100
For longest wavelengths = ( 0.83) × 100% = 8.3%
n 2 = n1 + 1 ∴ Change in length = 8.3%
Lyman series, n1 = 1
1 1 1
48. 21. 2 to 21.5
∴ = R 2 − 2  Compton shift is given by
λl 1 n2 
h
n 2 = 2 for longest wavelength λ ′− λ = (1− cos θ )
mec
1  1 4 where, λ′ = wavelength after scattering.
∴ = R 1− ⇒ λl =
λl  4  3R h
= Compton wavelength = 0 . 00243 × 10−9m
Balmer series, n1 = 2 mec
1  1 1 θ = scattering angle = 60°
= R 2 − 2  λ = initial wavelength = 0.24 × 10−9 m
λb 2 n2 
n 2 = 3 for longest wavelength, ∴ λ′−0. 24 × 10−9 = 0.00243 × 10−9 (1− cos 60° )
1  1 1 5  = 0.001215 × 10−9
=R − =R
λb  4 9   36  or λ′ = ( 0. 24 + 0.001215) ≈ 0. 241 nm.
36 The kinetic energy of scattered electrons
λb =
5R  1 1
∴ Ratio of λ l to λ b , ∆K = hc  − 
 λ λ′ 
4 36
λ l : λ b :: :  1 1 
3R 5R = hc  − 
λ l : λ b :: 5 : 27  0. 24 nm 0. 241nm
λl  1
∴ = 0.18 1 
λb = hc × 109  − 
 0. 24 0. 241 
47. 8 .1 to 8 .5 = 6.626 × 109 [ 0.0172] × 3 × 108 × 10−34
= 6.626 × 109 × 172 × 3 × 104 × 10−34
l0
= 3419 × 10−21 J
3419
= × 10−2 eV
60° 1.6
v= 0.8 c = 2136. 8 × 10−2 eV
= 21. 368 eV = 21.4 eV
.
Solved Paper 2015 ●
23

49. 54 .5 to 56 51. 450 to 460


−2 AB is isothermal expansion,
As, kT = 0.025 eV = 2.5 × 10 eV
As we know the diode current and saturation ∴ Work done from A to B,
current are related to each other as V 
WAB = nRT ln 2 
 qV   V1 
− f 
 kT 
Is = ID e  2V 
= 1 × 8.31 × 800 × ln  
V 
where, q = e charge of e −
 − eVf 
−
WAB = 1 × 6648 × ln( 2)

 kT  = 1 × 6648 × 0.6931 J
Is = ID e
= 4608.0 J
Vf = 0.4V, IS = 1µA and ID = ?
 1.6 × 0.4 × 10− 19  ∴ Heat absorbed ( ∆QAB ) from A to B is 4608.0 J
−  Heat absorbed ( ∆QBC ) = Cp ⋅ ∆T
 
−6  2.5 × 10− 2 eV 
⇒ 1 × 10 = ID × e 5
 
QCp for monoatomic gas = R
−  −
0.4 eV  2
− 2 
or 10− 6 = ID × e
 2.5 × 10 eV  5
∴ ∆QBC = − R × ( 800 − 300)
 0.4  2
− − × 102 
or 10− 6 = ID × e
 2.5  = − 2.5 × 8.31 × 500 J = − 10387.5 J
Work done from C to A, constant volume process
or 10− 6 = ID × e + 16 ⇒ W =0 [ QW = p∆V ]
10− 6 Heat absorbed ( ∆QCA ) = CV × ∆T
or ID =
e + 16 3
QCV for monoatomic gas = R
Now, for Vf = 0.5 V 2
 0.5 V 
 2.5 × 10− 2 V
 3
I = ID × e  ∴ Heat ( ∆QCA ) = × 8.31 × ( 800 − 300)
2
10− 6 = 1.5 × 8.31 × 500 = 6232.5 J
⇒ I= × e + 20
e + 16 ∴ Net heat absorbed = ∆QAB + ∆QBC + ∆QCA
or I = 10− 6 × e 4 = 4608 + ( −10387.5 ) + 6232 − 5
or I = 54.598 × 10− 6 A = 4535

I = 54.60 µA approx. 52. Magnetic field, B (t ) = 0.4t Tesla


Radius of the circular loop, r = 2 × 10− 2 m.
50. The diamond lattice structure is very common in
semiconductor materials like Si, Ge, GaAs and GaP
has a zinc blende lattice structure which is similar to
the diamond lattice structure.
Because of the two types of atoms in GaAs, the zinc
blende structure has a lower symmetry than the
diamond structure.
This can lead to interesting phenomena in the
heteroepitax of zinc blende materials and diamond
substrates.
Magnitude of the induced or induced emf,
(001)
plane
d φB
e=
dt
As, φB = B . A = BA cos 0°
= 0.4t × πr 2
The 001 plane is shown in the diagram. In the GaAs = 0.4 t × 3.14 × (2 × 10− 2 )2
structure, the two FCC sublattices are made of two = 5.02 × 10− 4 t
different types of atoms.
d
Therefore, to fracture the crystal in the (001) plane, ∴ e = (5.02 ×10− 4 t )
dt
two bonds have to be broken per atom.
= 5.02 × 10− 4 V/m
24 IIT JAM Physics Solved Papers & Practice Sets

53. 0.9 to 0.95 ∆Q= mc∆t


∆Q= 1 × 1 × 39
Here, ν = 5 × 1014 Hz, A = 103 V/m, k1 = 1.69 and k 2 = 2.25 ∆Q= 39 cal
 2 k1  Total heat gained= Qf + ∆Q
As, A t =  Ai
 k1 + k 2  = 80 + 39 cal = 119 cal
Now, change in entropy
At 2 k1 2 × 1.3 2.6 119
∴ = = = = 0.93 ∆S = cal per degree
Ai 1.69 + 2.25 1.3 + 1.5 2.8 273.15
or ∆S = 0.44
54. 5 to 5
The intensity of light sources A, B and C are 56. 3 . 25 to 3 . 28
2 mW/m2, 2 mW/m2 and 5 mW/m2, respectively. Rolling without slipping
R
λ = 600 nm m
From the figure,
D = 1.0 m (distance of screen) 30°
3
or D = 10 mm
Disc of radius R, mass m
For A and B, d = 3.22 mm 1
y = 15 mm ∴ I = mR 2
2
yd
∴ Path difference = ∆x = Acceleration of a body rolling down on inclined plane is,
D g sin θ
15 × 3.22 a=
∴ ∆x = mm  I 
103 1 + 
 mR 2 
15 × 3.22 × 106
or ∆x = nm Here, θ = 30°
103  
 
or ∆x = 15 × 3.22 × 103 nm g sin 30 °
a= 
Compare it with mλ  mR 2 
1+ 
mλ = ∆x  2mR 2 
15 × 3.22 × 103 g × 1/ 2
m= ⇒ m = 80.5 or a= = g/ 3
600 nm 3/2
161 g
m= ∴ a=
2 3
( 2 × 80 + 1) 9.81
⇒ It can be written as m = or a= ms−2 = 3.27 ms−2
2 3
λ 57. 17.3 to 17.5
∴ ∆x = ( 2n + 1) for n = 80
2 A2=1 cm2
It is destructive interference which leads to net intensity
I = ( I1 − I 2 )2
500 g/s 50 cm
I =0
2 Water
Since, I1 = I 2 = 2 mW/m .
∴Only intensity of light C will be the net intensity A1= 25 cm2
2
∴ Intensity at P = 5 mW/m .
Rate of water flow, 500 g/s
55. 0. 4 to 0 .45 Area = 25 cm2
1 g of ice at 0°C is melted ⇒ Qf = mL Density of water 1 kg/1000 cm3
where, L is latent heat of fusion. ρ = 1g/cm3
L = 80 cal/g ⇒ 500 g in 500 cm3
∴ Qf = 1g × 80 cal/g ∴ Flow of water = 500 cm3 /s
Qf = 80 cal 500
or v1 = cm/s
Now, rising of temperature from 0°C to 39°C. A1
∴ Change in temperature ( ∆t ) = 39°C 500
Specified heat of water, c = 1cal/g/°C or v1 = cm/s = 20 cm/s or 0.2 ms−1
25
Solved Paper 2015 ●
25

∴ Equation of continuity, 59. According to formula,


A 1v1 = A2 v 2 4R
Centre of mass of semi-circular disc from its centre is
⇒ 25 × 0.2 ms−1 = 1 × v 2 3π
v 2 = 5 ms−1 4×3 4
⇒ Ycm = m= m
Now, according to Bernoulli’s theorem, 3π π
p1 1 2 p 1 4 14
+ v1 + gh1 = 2 + gh2 + v 22 = ×7m = m
ρ 2 ρ 2 22 11
1 1 ∴ Ycm = 12.7 m (approx).
or ( p1 − p 2 ) = g (h2 − h1) + (v 22 − v12 )
ρ 2
60. 0.3 to 0.35
1 1
or ( p1 − p 2 ) = 9.8 [ 0.5 − 0] + [v 22 − v12] Depletion layer
ρ 2
1  1 20 µm
or ( p1 − p 2 ) = 9.8 ( 0.5 − 0) + ( 25 − 0.04)
10 3 
 2 
x
or p1 − p 2 = 1000[ 4.9 + 12.48]
⇒ ∆p = 17.38 × 1000 The radius, r = 10 µm
= 17.38 kPa ∴Area of fabrication = π(10 × 10−6 )2 m2
∴ Pressure difference = 17.38 kPa or A = 10−10 π m2
58. 3 to 3.3 Let, x is the width of depletion layer,
m = 2 kg R = 20 N ∴Volume of depletion layer = Ax
∴ Vd = (10−10 π . x ) m3 …(i)
It is given that, density of donor (ρ )
g = 10 ms–2
ρ = 1016/cm3
∴ W = 2 × 10 N= 20 N
⇒ R = 20 N ⇒ ρ = 1016 × 106/m3 = 1022/m3
Frictional force = µR = 20 × 0.1= 2 N Total charge on n-side developed = 1.6 × 10−13 C
Applied force = 3 N ∴ Number of electron transferred,
∴ Net unbalanced force = 1N 1.6 × 10−13
Distance (s ) = 10 m = 106
F 1. 6 × 10−19
a=
106
m ∴ Volume required = m3 = 10−16 m3 …(ii)
⇒ v = 2as 1022
F ∴ From Eqs. (i) and (ii), we get
or v = 2× ×s
m 10−16 = 10−10 πx
1 10−6
or v = 2 × × 10 ms−1  1 1
2 ⇒ x =  m= µm
 π π
or v = 10 ms−1
⇒ x = 0.32 µm (approx)
or v = 3.1 ms−1 (approx) Width of depletion layer ~
− 0.32 µm.
Solved Paper
2014
IITJAM Physics
MM : 100 Time : 3 hrs

n This question paper has a total of 43 questions carrying 100 marks of 3 hours duration.
n
Questions 1 to 35 objective type questions. The first 10 questions carry one mark each and the rest 25
questions carry two marks each and questions 36 to 43 subjective type questions carry five marks each.
n
Each objective question has 4 choices for its answer: (a), (b), (c) and (d). Only one of the four choices is
correct.
n
There will be negative marks for wrong answers. For each 1 mark question the negative mark will be 1/3
and for each 2 marks question it will be 2/3.
n
For subjective type questions answers must be written in blue/black/blue-black ink only. The use of sketch
pen, pencil or ink of any other colour is not permitted.
n Do not write more than one answer for the same question. In case you attempt a subjective question
more than once, please cancel the answer(s) you consider wrong. Otherwise, the answer appearing last
only will be evaluated.
n Clip board, log tables, slide rule, calculator, cellular phone and electronic gadgets in any form are not
allowed.

1. For vectors a = $j + k$ , b = 2 i$ + 3 $j − 5 k$ and c = $j − k$ , the vector product a × (b × c) is


(a) in the same direction as c
(b) in the direction opposite to c
(c) in the same direction as b
(d) in the direction opposite to b

2. A particle of mass m carrying charge q is moving in a circle in a magnetic field B. According to


Bohr’s model, the energy of the particle in the nth level is
1  hqB  hqB
(a)   (b) n  
n 2  πm   πm 
 hqB   hqB 
(c) n   (d) n  
 2πm   4πm 
Solved Paper 2014 ●
27

3. A conducting slab of copper PQRS is kept on the XY-plane in a uniform magnetic field along
X-axis as indicated in the figure. A steady current I flows through the cross-section of the slab
along the Y-axis. The direction of the electric field inside the slab, arising due to the applied
magnetic field is along the
Z

S R
Q
I
P Y
B
X
(a) negative Y- direction (b) positive Y- direction (c) negative Z- direction (d) positive Z- direction

4. A collimated beam of light of diameter 1 mm is propagating along the X-axis. The beam is to be
expanded to a collimated beam of diameter 10 mm using a combination of two convex lenses. A
lens of focal length of 50 mm and another lens with focal length F are to be kept at a distance d
between them. The values of F and d, respectively are
(a) 450 mm and 10 mm (b) 400 mm and 500 mm
(c) 550 mm and 600 mm (d) 500 mm and 550 mm

5. Octal equivalent of decimal number 478 10 is


(a) 736 8 (b) 6738 (c) 637 8 (d) 367 8
6. A spherical ball of ice has radius R 0 and is rotating with an angular speed ω about an axis
passing through its centre. At time t = 0, it starts acquiring mass because the moisture (at rest)
around it starts to freeze on it uniformly. As a result, its radius increases as R (t ) = R 0 + αt,
where, α is a constant. The curve which best describes its angular speed with time is

ω ω ω ω
(a) (b) (c) (d)

t t t t

7. In one-dimension, an ensemble of N classical particles has energy of the form


p x2 1
E = + kx 2 . The average internal energy of the system at temperature T is
2m 2
3 1
(a) NkBT (b) NkBT (c) 3NkBT (d) NkBT
2 2

8. In a photoelectric effect experiment, ultraviolet light of wavelength 320 nm falls on the


photocathode with work function of 2.1 eV. The stopping potential should be close to
(a) 1.8 V (b) 1.6 V (c) 2.2 V (d) 2.4 V

9. In an ideal operational amplifier depicted below, the potential at node A is


25 kΩ

5 kΩ

A 5V
1V +

(a) 1 V (b) 0 V (c) 5 V (d) 25 V


28 IIT JAM Physics Solved Papers & Practice Sets

10. To operate a n-p-n transistor in active region, its emitter-base and collector-base junction
respectively, should be
(a) forward biased and reversed biased (b) forward biased and forward biased
(c) reversed biased and forward biased (d) reversed biased and reversed biased

π
11. The value of ∑ r n sin (n θ) for r = 0.5 and θ =
3
is
n=0

1 2 3
(a) (b) (c) (d) 3
3 3 2

12. In a parallel plate capacitor the distance between the plates is 10 cm. Two dielectric slabs of
thickness 5 cm each and dielectric constants K 1 = 2 and K 2 = 4 respectively, are inserted
between the plates. A potential of 100 V is applied across the capacitor as shown in the figure.
The value of the net bound surface charge density at the interface of the two dielectrics is

K2 =4
10 cm 100 V
K1 =2

2000 1000 2000


(a) − ε0 (b) − ε0 (c) −250 ε 0 (d) ε0
3 3 3

13. The electric fields of two light sources with nearby frequencies ω 1 and ω 2 and wave vectors k1
and k 2 , are expressed as E1 = E 10 i$e −i (k1z − ω1t ) and E 2 = E 20 $i e −i (k 2 z − ω 2t ) , respectively. The
interference pattern on the screen is photographed at t = t 0 , denotes (k1 − k 2 ) z − (ω 1 − ω 2 ) t 0
by θ. For this pattern
(a) a bright fringe will be obtained for cos θ = − 1
(b) a bright fringe intensity is given by ( E10 )2 + ( E20 )2
(c) a dark fringe will be obtained for cos θ = 1
(d) a dark fringe intensity is given by ( E10 − E20 )2

14. A solid metallic cube of heat capacity S is at temperature 300 K. It is brought in contact with a
reservoir at 600 K. If the heat transfer takes place only between the reservoir and the cube,
the entropy change of the universe after reaching the thermal equilibrium is
(a) 0.69S (b) 0.54S (c) 0.27 S (d) 0.19S

15. If the surface integral of the field A (x, y , z ) = 2 αx i$ + βy $j − 3 γz k$ over the closed surface of
an arbitrary unit sphere is to be zero, then the relationship between α, β and γ is
β α β γ α γ 2 1 3
(a) α + −γ=0 (b) + − =0 (c) +β − = 0 (d) + − =0
6 3 6 2 2 3 α β γ

16. The moment of inertia of a disc about one of its diameters is IM . The mass per unit area of the
disc is proportional to the distance from its centre. If the radius of the disc is R and its mass is
M, then the value of IM is
1 2 3 3
(a) MR 2 (b) MR 2 (c) MR 2 (d) MR 2
2 5 10 5
Solved Paper 2014 ●
29

17. A rigid uniform horizontal wire PQ of mass M, pivoted at P, carries a constant current I. It
rotates with a constant angular speed in a uniform vertical magnetic field B. If the current was
switched off, the angular acceleration of the wire, in terms of B, M and I would be

P Q

2 BI 3BI BI
(a) 0 (b) (c) (d)
3M 2M M

18. Two points N and S are located in the Northern and Southern hemisphere, respectively, on the
same longitude. Projectiles P and Q are fired from N and S, respectively, towards each other.
Which of the following options is correct for the projectiles as they approach the equator?
(a) Both P and Q will move towards the East
(b) Both P and Q will move towards the West
(c) P will move towards the East and Q towards the West
(d) P will move towards the West and Q towards the East

19. Two particles A and B of mass m and one particle C of mass M are kept on the X-axis in the
order ABC. Particle A is given a velocity v $i. Consequently, there are two collisions, both of
7
which are completely inelastic. If the net energy loss because of these collisions is of the
8
initial energy, the value of M is (ignore frictional losses)
(a) 8 m (b) 6 m (c) 4 m (d) 2 m
1
20. The line integral ∫ A ⋅ d l of a vector field A (x, y ) = 2
( − y $i + x $j ), where r 2 = x 2 + y 2 , is taken
r
around a square (as shown in the figure) of side unit length and centered at (x 0 , y 0 ) with
1 1
| x 0 | > and | y 0 | > . If the value of the integral is L, then
2 2
Y

(x 0, y 0)

(a) L depends on ( x0, y0 ) (b) L is independent of ( x0, y0 ) and its value is −1


(c) L is independent of ( x0, y0 ) and its value is 0 (d) L is independent of ( x0, y0 ) and its value is 2

21. Diamond lattice can be considered as a combination of two f cc lattices displaced along the
body diagonal by one quarter of its length. There are eight atoms per unit cell. The packing
fraction of the diamond structure is
(a) 0.48 (b) 0.74 (c) 0.34 (d) 0.68

22. Thermal neutrons (energy = 300kB = 0.025 eV) and sometimes used for structural
determination of materials. The typical lattice spacing of a material for which these can be
used is
(a) 0.01 nm (b) 0.05 nm (c) 0.1 nm (d) 0.15 nm
30 IIT JAM Physics Solved Papers & Practice Sets

23. What is the maximum height above the dashed line attained by the water stream coming out
at B from a thin tube of the water tank assembly shown in the figure? Assume, h = 10 m,
L = 2 m, and θ = 30°.
A

h
L B
θ
y=0

(a) 10 m (b) 2 m (c) 1.2 m (d) 3.2 m

24. A steady current in a straight conducting wire produces a surface charge on it. Let E out and
E in be the magnitudes of the electric fields just outside and just inside the wire, respectively.
Which of the following statements is true for these fields?
(a) Eout is always greater than Ein
(b) Eout is always smaller than Ein
(c) Eout could be greater or smaller than Ein
(d) Eout is equal to Ein

25. A small charged spherical shell of radius 0.01 m is at a potential of 30 V. The electrostatic
energy of the shell is
(a) 10−10 J (b) 5 × 10−10 J (c) 5 × 10−9 J (d) 10−9 J

26. At an instant shown, three point masses m, 2 m and 3m rest on a horizontal surface and are at
the vertices of an equilateral triangle of unit side length. Assuming that G is the gravitational
constant, the magnitude and direction of the torque on the mass 3m, about the point O, at that
instant is
3m

m 2m
O
3
(a) zero (b) G 3 m 2, going into the paper
2
3
(c) 3G 3 m 2, coming out of the paper (d) G 3 m 2, going into the paper
4

27. A sine wave of 5 V amplitude is applied at the input of the circuit shown in the figure. Which of
the following waveforms represents the output most closely?
Vin Vout
1kΩ

3V

5V 5V 3V 3V

(a) (b) (c) (d)

–5 V –3 V –3 V –5 V
Solved Paper 2014 ●
31

28. 1011 binary input have been applied at X 3 X 2X 1X 0 input in the shown logic circuit made of
XOR gates. The binary output Y 3Y 2Y1Y 0 of the circuit will be
X3 Y3

X2 Y2

X1 Y1

Y0
X0

(a) 1101 (b) 1010 (c) 1111 (d) 0101

29. A ring of radius R carries a linear charge density λ. It is rotating with angular speed ω. The
magnetic field at its centre is
3 µ 0λω µ 0λω µ 0λω
(a) (b) (c) (d) µ 0λω
2 2 π

30. A stationary source (as shown in figure below) emits sound waves of frequency f towards a
wall. If an observer moving with speed u in a direction perpendicular to the wall, measures a
9
frequency f ′ = f at the instant shown, then u is related to the speed of sound v s as
8
Source Wall
30°

30°
Observer u
(a) v s (b) v s / 2 (c) v s / 4 (d) v s / 8

31. A real gas has specific volume V at temperature T. Its coefficient of volume expansion and
isothermal compressibility are α and kT , respectively. Its molar specific heat at constant
pressure C p and molar specific heat at constant volume CV are related as
T Vα T Vα 2
(a) Cp = CV + R (b) Cp = CV + (c) Cp = CV + (d) Cp = CV
kT kT

32. Two frames, O and O′ are in relative motion as shown in the figure O′ is moving with speed
c / 2, where c is the speed of light. In frame O, two separate events occur at (x 1, t1 ) and
(x 2 , t 2 ). In frame O′, these events occur simultaneously. The value of (x 2 − x 1 ) / (t 2 − t1 ) is
c/2

x
O O′

(a) c / 4 (b) c / 2 (c) 2 c (d) c

33. White light is incident on a grating G1 with groove density 600 lines/mm and width 50 mm. A
small portion of the diffracted light is incident on another grating G 2 with groove density
1800 lines/mm and width 15 mm. The resolving power of the combined system is
(a) 3 × 103 (b) 57 × 103 (c) 81 × 107 (d) 108 × 105
32 IIT JAM Physics Solved Papers & Practice Sets

34. Four particles of mass m each are inside a two-dimensional square box of side L. If each state
obtained from the solution of the Schrödinger equation is occupied by only one particle, the
h2
minimum energy of the system in units of is
mL2
5 11 25
(a) 2 (b) (c) (d)
2 2 4

35. At atmospheric pressure (= 105 Pa), aluminium melts at 550 K. As it melts, its density
decreases from 3 × 10 3 kg / m 3 to 2.9 × 10 3 kg / m 3 . Latent heat of fusion of aluminium is
24 × 10 3 J/kg. The melting point of aluminium at a pressure of 10 7 Pa is closest to
(a) 551.3 K (b) 552.6 K (c) 558.7 K (d) 547.4 K
2
d y dy
36. Find the solution of differential equation 2
+5 = 0, with the boundary condition y ( 0) = 2
dx dx
dy 

and = 2, giving all steps clearly. Find the value of x, where y = 0.
dx x = 0

37. The electric field in an electromagnetic (EM) wave is E = 6 π i$ sin [2π (10 6 z − 3 × 1014 t )].
What is the intensity of the EM wave and the number of photons per second falling on the unit
area of a perfectly reflecting screen kept perpendicular to the direction of propagation? When a
photon in this beam is reflected from the screen, what is the impulse it imparts to the screen?
Use this to find the pressure exerted by the EM wave on the screen.
O
38. A uniform rod of mass m and length l is hinged at one of its ends O and is hanging
vertically. It is hit at its mid-point with a very short duration impulse J, so that it starts J
rotating about O. Find the magnitude and direction of the horizontal impulse that O
applies on the rod, when it is hit.

39. An easy derivation of pV γ = constant for an ideal gas undergoing an adiabatic process.
Consider p and V as the basic variables of an ideal gas and write the heat exchanged dQ in
terms of dV and dp. Next, using the definition of C p and CV in the expression for dQ, obtain a
differential equation relating p and V for an adiabatic process and solve it to get the desired
relationship. Derivation should not use the first law of thermodynamics. [For a function f (x, y )
 ∂f   ∂f 
the differential, df =   dx +   dy ].
 ∂x  y  ∂y  x

40. As shown in the figure alongside an unpolarised beam of light of wavelength 500 nm is
incident on a linear polariser at AF with vertical polarisation. The light beam then passes
through a wave plate BE (half-wave or quarter-wave plate) of thickness 1.00125 mm and gets
reflected from a mirror CD. The reflected light is indicated by the dashed line (DEF ) in the
diagram. The ordinary and extraordinary refractive indices for the material of the wave plate
are 1.658 and 1.558, respectively. Light is incident normally on all surfaces.
A B C

F E D
Linear Wave plate Plane mirror
polariser

(i) What is the polarisation of the beam at C?


(ii) What is the polarisation of the beam at E and F?
Solved Paper 2014 ●
33

41. A standing wave of light is formed between two mirrors and a beam of atoms is incident on it
normally as shown in the figure from the left. On the right side, atoms are detected in the
direction of the beam and also at an angle θ (as shown in the figure). This is due to material
waves of atoms diffracted by the standing wave that acts like a grating, the slit width of this
grating is given by the distance between two maxima of the light intensity. If the atomic beam
is made of atoms of mass m moving with speed v and the light wave has wavelength λ L ,
find the smallest angle θ by using the diffraction condition.

Atomic beam θ

42. According to Wien’s theory of black body radiation, the spectral energy density in a black body
cavity at temperature T is given as
α
u T ( λ ) dλ = 3 5
e −β / λT d λ
c λ
where, α and β are constants and c is the speed of light. Further, the intensity of radiation
u c ∞
coming out of the cavity is T , where u T = ∫ u T ( λ ) dλ is the total energy density of
4 0

radiation. Given that, Stefan-Boltzmann constant, σ = 5.67 × 10 −8 Wm −2 K −4 and



λ maxT = 2.90 × 10 −3 mK, find the values of α and β. The value of integral ∫ x 3 e − x dx = 6.
0

43. A horizontal rod of proper length L moves with a uniform speed v > 0 along the X-axis of a
coordinate frame. A ground observer measures the position coordinates of its two ends at two
different times with time difference ∆t > 0. The observer finds that the difference between the
two coordinates is L. Calculate ∆t in terms of L, v and the speed of light c. If measured
correctly, what would have been the length of the rod in the ground frame?

Answers
1. (a) 2. (d) 3. (d) 4. (d) 5. (a) 6. (b) 7. (d) 8. (a) 9. (b) 10. (a)
11. (a) 12. (a) 13. (d) 14. (d) 15. (b) 16. (c) 17. (c) 18. (b) 19. (b) 20. (c)
21. (c) 22. (d) 23. (d) 24. (a) 25. (b) 26. (d) 27. (d) 28. (a) 29. (b) 30. (c)
31. (c) 32. (c) 33. (c) 34. (b) 35. (b)
Answers with Explanations
1. (a) a = $j + k$ , b = 2 i$ + 3$j − 5 k$ and c = $j − k$ 4. (d)
B
$i $j 0.5 mm A
k$ 5 mm 10 mm
1 mm
b ×c= 2 3 −5 F C2
C1 F
0 1 −1 50 mm

3 −5 2 −5 2 3 d
= $i − $j + k$
1 −1 0 −1 0 1 AC1 BC2
From the figure, = [Q ∆ AC1F ~ ∆ BC2F ]
50 F
= $i ( − 3 + 5) − $j ( − 2) + k$ ( 2) 50 × 5
⇒ F = = 500 mm
⇒ b × c = 2$i + 2$j + 2k$ 0.5
∴ d = 500 + 50 = 550 mm.
i$ $j k$
a × (b × c) = 0 1 1 5. (a) Decimal to octal conversion Any decimal number can
2 2 2 be converted into octal number system by dividing it by
the largest possible power of 8. The remainder then
1 1 0 1 0 1 again divided by the largest possible power of 8, this
= $i − $j + k$
2 2 2 2 2 2 method is repeated until the power of 8 is 1 and the
remainder obtained is less than 8. The result is
= $i ( 2 − 2) − $j ( − 2) + k$ ( − 2) obtained by arranging the obtained quotient as they are
a × (b × c) = 2$j − 2k$ = 2( $j − k$ ) acquired.
a × (b × c) = 2c 8 478 6
Hence, a × (b × c) in the same direction as c. 8 59 3
mv 7
2. (d) As, radius of circular orbit, r =
qB
⇒ ( 478)10 = ( 736)8
Angular momentum of particle, L = mvr
m 2v 2 6. (b) Radius is given by
= R(t ) = R0 + αt …(i)
qB
According to Bohr’s second postulate, Here, angular momentum of system remains constant
nh ∴ Iω = constant
L = mvr =
2π 2
or MR 2 × ω = constant
m 2v 2 nh 5
or =
qB 2π 2 4
or × ρ × πR 5 × ω = constant
nhqB 5 3
⇒ mv 2 =
2 πm where, ρ is density
1 2 nhqB 1
or mv = ⇒ ω∝ …(ii)
2 4 πm R5
∴ Energy of the particle in nth level Thus, from Eqs. (i) and (ii), we get as radius increases,
 hqB  and and ω decreases non-linearly.
= n 
 4πm 
7. (d) In case of one particle, the energy is given by
3. (d) Force on positive charge is in p x2 1
E= + kx 2
$j × B ≡ $j × $i = − k$ direction, 2m 2
i.e. negative Z-axis. The average internal energy of one particle is
− βE
Therefore, positive charge will be collected on the lower ∫ e Ed qd p
f f

surface of the slab. Thus, the direction of induced U′ = − βE f


∫ e d qd p
f
electric field will be from lower plate to upper plate, i.e. in
positive Z-direction. where, f = degree of freedom = 1[In 1D, for one particle]
Solved Paper 2014 ●
35

 p2 
+ ∞+ ∞
1
−  x + kx 2  10.(a) The base-emitter junction of a n - p -n transistor in
 2m 2   p x2 1 
∫ ∫ e 
 2m 2
+ kx 2 dp x dx

the active region will be forward biased, while the
∴ U′ =
−∞ −∞ collector-base junction will be reversed biased. In
 p2 
+ ∞+ ∞
1
− β  x + kx 2  other words,
 2m 2 
∫ ∫ e dp x dx VB − VE = VBE ≈ 0.7V
−∞ −∞ and VC − VB = VCB > 0 V
1 1
= kT + kT = kT
2 2
VCB > 0 n C
According to equipartition theorem, iC
1 1 p B VCE > 0.7V
U ′ = kT + kT = kT iB
2 2 VBE ≈ 0.7 V iE n
E
[since, the contribution of each quadratic term in hamiltonian
1
to average thermal energy is kT ]
2
In active mode, the electric field existing between
Therefore, average energy of N-particle system is
base and collector (caused byVCE) will cause the
U = NU ′ = NkT , where k = kB = Boltzmann constant.
majority of electrons to cross the upper p - n
8. (a) Given, λ = 320 nm = 320 × 10− 9 m junction into the collector to form the collector
Work function, φ = 2.1 eV current.
Using Einstein’s photoelectric equation The remainder of the electrons recombine with
hc holes, the majority carriers in the base, making a
= φ + K max
λ current through the base connection to form base
where, K max = maximum KE of photoelectron current IB .
hc 1242 eV nm ∞
⇒ K max = −φ= − 2.1 eV
λ 320 nm 11. (a) ∑ r n sin (n θ )
n=0
K max = (3.8 − 2.1) eV = 1.78 eV
= r 0. sin ( 0) + r 1 sin(θ ) + r 2 sin ( 2θ ) r 3 sin( 3θ ) + L
If V0 is the stopping potential, then
1 π
eV0 = K max We have, r = 0.5 = and θ =
2 3
1.78 eV ∞ 2
⇒ V0 = = 1.78 V ≈ 18
. V 1  π   1  2π 
e ∴ ∑ r n sin (n θ ) = × sin   +   sin  
2  3   2  3
n=0
9. (b) The characteristics of an ideal amplifier without regard to 3 4
what is inside the package are  1  3π   1  4π 
+   sin   +   sin  
1. infinite voltage gain  2  3   2  3
2. infinite input impedance 5 6
 1  5π   1  6π 
3. zero output impedance +   sin   +   sin   + L
 2  3   2  3
4. infinite bandwidth 2 4
5. zero input offset voltage  1 3  1 3  1  3
=  × +  × + 0 +   − 
 2 2  2 2  2  2 
25 kΩ 5
5 kΩ  1  3
– +   −  + 0+L
A 5V
 2  2 
+
 1 3  1 4  3 
1V =   ⋅ +   ⋅ −  + L +
 2 2  2  2 
 
 1  3   1 5 
2
3 
  ⋅   +   ⋅ −
 2  2 
 + L
Let the voltage at A be VA .  2  2  
Applying Kirchhoff’s current rule, Both are infinite GP series and the sum of an
1 − VA V − 5
=− A  infinite GP series is given by
5  25  a
5 S =
⇒ 1 − VA = (5 − VA ) 1− r
25
where, a is first term and r is common ratio.
or 5 (1 − VA ) = 5 − VA ⇒ VA = 0
36 IIT JAM Physics Solved Papers & Practice Sets

2
1 3  1 3 14. (d)
∞ ⋅   ⋅ 600 K Reservoir
2 2  2 2
∴ ∑ r sin (n θ ) =
n
  1 3 
+
  1 3 
n=0
1 − −    1 − −   
   
 2   2 
1 3 8 1 3 8 300 K Solid metallic
= ⋅ ⋅ + ⋅ ⋅ cube
2 2 9 4 2 9
1
= As ( ∆S )universe = ( ∆S )system + ( ∆S )surrounding
3
12. (a) The given system can be treated as two capacitors So, ( ∆S )surrounding =
∫ dQ = ∫ S dT
connected in series, as the charge on both the T T
capacitors will be same, when the dielectrics K1 and K 2 − S ( 600 − 300) S
= =−
are inserted. 600 2
T2
S dT T
and ( ∆S )system = ∫ dS = ∫ = S ln 2
K2 = 4 T1
T T1
10 cm 100 V
K1 = 2 1
= − S ln = S ln 2
2
S
K1A K A ε0 ∴ ( ∆S )universe = S ln 2 −
C1 = ε 0 and C2 = 2 2
d /2 d /2
= 0.693 S − 0.5 S
The equivalent capacitance of the combination
= 0.19 S
1 1 1 C1C2
= + ⇒ Ceq = 15. (b) Surface integral is a generalisation of multiple integrals
C C1 C2 C1 + C2
to integration over surface. It can be thought as the
2K1K 2 Aε 0
= double integral analog of the line integral. Given a
(K1 + K 2 ) d surface, one may integrate over its scalar fields and
8Aε 0 vector fields given, A (x , y , z ) = 2αx i$ + βy$j − 3γz k$
Ceq = [putting, K1 = 2 and K 2 = 4]
3d ∂A ∂( 2αx ) $ ∂(βy ) $ ∂( 3γz ) $
Total charge on the capacitor, Q = CeqV = i+ j− k
∂x ⋅ ∂y ⋅ ∂z ∂x ∂y ∂z
8Aε 0 800 Aε 0
Q= × 100 = = 2α + β − 3γ
3d 3d According to given question, 2α + β − 3γ = 0
As, the capacitors, in series with the same charge Q will 2α + β − 3γ
be at the interface of the two dielectrics. Surface charge ⇒ =0
6
Q
density at the interface = α β γ
A ⇒ + − =0
3 6 2
800 Aε 0 800 ε 0
= =−
3d × A 3d 16. (c) Moment of inertia of disc about diameter is
Q − 2000 IM = ∫ r 2 sin2 θ dm
⇒ = ε0
A 3
or IM = ∫ ∫r
2
sin2 θ ⋅ σ ⋅ r dr ⋅ dθ
13. (d) The problem is related to beats, where bright fringe is
obtained, when Here, σ = kr
θ 2π
kR 5
R
cos =1
2 ∴ IM = k ∫ r 4 dr ∫ sin θ dθ =
2
⋅π
5
θ 0 0
⇒ = nπ
2 Now, M = ∫ dm = ∫ ∫ kr ⋅ rdr ⋅ dθ
θ
The dark fringe is obtained, when cos = 0 R3
2 ⇒ M =k⋅ ⋅ 2π
r
From interference, we know that
3M
maximum intensity = (E10 + E20 )2 or k =
2
2 πR 3
and minimum intensity = (E10 − E20 ) kR 5 3M R 5
∴ IM = ⋅π = ⋅ ⋅π
Thus, a dark fringe intensity is given by (E10 − E20 )2 and 5 2 πR 3 5
bright fringe intensity is given by 3MR 2
=
(E10 + E20 )2 10
Solved Paper 2014 ●
37

17. (c) Here, magnetic torque is being balanced by some other tetrahedrally bonded atoms in each primitive cell,
torque ( τ 0 ). 1
separated by of the width of the unit cell in each
Therefore, τ magnetic = τ 0 4
1 dimension.
or IBL2 = τ 0 The atomic packing factor of the diamond cubic
2
structure (the proportion of space that would be filled by
When magnetic field is switched off, the other torque
spheres that are centered on the vertices of the
remains present. Therefore, angular acceleration is
structure and are as large as possible without
produced by the other torque.

τ overlapping) is ≈ 0.34
Hence, angular acceleration = 0 16
I′
ML2 22. (d) As, a sin θ = nλ
where, I′ = moment of inertia =
3 30.08
or a sin θ = n × Å
1 2 T
IBL
3 BI
= 2 2 = ⋅ Using, sin θ ≈ 1, n = 1
ML / 3 2 M 30.08
⇒ a= Å
18. (b) Both the projectiles will deviate in the direction of coriolis 300
force. Since, direction of coriolis force on both projectiles = 174
. Å ≈ 0.15 nm (nearly)
P and Q is towards the West (from v × ω). 23. (d) A
Therefore, both will deviate towards West.
7 1 (h –L sin θ)
19. (b) Given, net energy loss = × mv 2 h
8 2 L B
2 θ L sin θ
1 1  mv  7 1 y=0
or mv 2 − (M + 2m )  = × mv
2
2 2  M + 2m  8 2
1  m  7 1 Let v be the velocity of the water coming out from the
or mv 2 1 −  = 8 × 2 mv
2
point B. Choosing, y = 0 as the zero potential energy
2  M + 2m 
level and applying conservation of mechanical energy.
M + 2m − m 7 PEA = KEB + PEB
or =
M + 2m 8 1
mgh = mv 2 + mgL sin θ
M +m 7 2
or =
M + 2m 8 1
⇒ mg [h − L(sin θ)] = mv 2
or 8M + 8m = 7M + 14m 2
∴ M = 6m ⇒ v 2 = 2g [h − L(sin θ)]
1 1  1
20. (c) Given, A = 2 ( − y$i + x$j ) = 2 ( − y$i + x$j ) v 2 = 2 × g 10 − 2 ×  = 18 g
r x + y2  2
The water ejected at point B will follow the projectile
$i $j motion.
k$
∂ ∂ ∂ Maximum height the water can achieve
or ∇ × A = =0 v 2 sin2 θ
∂x ∂y ∂z H = L sin θ +
y x 2g
0 2
x2 + y2 x2 + y2  1
18g ×  
1  2
and the field is defined everywhere within and on the =2× + = 3.25 m
square of side unit length and centered at (x 0, y 0 ) with 2 2g
1 1 σ
| x 0| > and | y 0| > . 24. (a) We have, Eout − Ein = >0
2 2 ε0
Therefore, from Stoke’s theorem ⇒ Eout > Ein
L = ∫ A ⋅ d l = ∫∫ ( ∇ × A) ⋅ d S = 0 Hence, Eout is always greater than Ein.
C S
which is independent of (x 0, y 0 ).
25. (b) Electrostatic energy of a uniformly charged spherical
shell is given by
21. (c) Diamond cubic is the face centered cubic Bravais lattice. Q2
U = ... (i)
The lattice describes the repeat pattern for diamond 4πε 0R
cubic crystals this lattice is decorated with a motif of two Capacitance of spherical shell = 4πε 0R
38 IIT JAM Physics Solved Papers & Practice Sets

Charge stored (charge on the spherical shell) 28. (a) X3 Y3


Q = CV = 4πε 0R V ... (ii)
X2 Y2
From Eqs. (i) and (ii), we get
( 4πε 0RV )2
U = = 2πε 0RV 2
8πε 0R X1 Y1
U = 2 × π × 8.85 × 10− 12 × 0.01 × ( 30)2
U = 5 × 10− 10 J
Y0
26. (d) To calculate the torque on mass ( 3m ) first we have to X0
calculate the net gravitational force on the mass ( 3m )
All the three given gates are EX-OR gate.
G 3 m2
FA = = 3 m2 G i.e. Exclusive - OR gate → output is only 1, if either of
(1)2 the inputs is 1, but 0 when both the inputs are 0 or
G 6 m2 when both the input are 1.
FB = = 6 m2 G
(1)2 Boolean expression of EX-OR gateA ⊕ B = A . B + A. B
3m Inputs X 0 → 1, X1 → 1, X 2 → 0, X 3 → 1
B
From the given circuit it is clearY3 = X 3 = 1
FA
30° FB Y2 = X 3 ⊕ X 2 = X 3. X 2 + X 3. X 2
= 0.0 + 1.1 = 1
Y2 = 1
Y1 = Y2 ⊕ X1 = Y2. X1 + Y2. X1 = 0.1 + 1.0 = 0
60° Y1 = 0
m 2m
O
Y0 = Y1 ⊕ X 0 = Y1. X 0 + Y1. X 0 = 1.1 + 0.1 = 1
The vertical components of the forces (FA and FB ) will
Y0 = 1
produce zero torque as the line of action of these forces
Input X 3 X 2 X1 X 0 = 1011→ Output (Y3Y2Y1Y0 ) = 1101
are passing through the point O. Net horizontal force on
mass ( 3m ) = ( 6m 2G sin 30° − 3m 2G sin 30° ) Alternate Solution
Torque due to this horizontal force on 3m about O The given circuit is gray to binary converter. The
binary output corresponding to the input 1011 is
= ( 6m 2G − 3m 2G ) sin 30° × | OB |
Input : 1 0 1 1
1 3 3
= 3m 2G × × = G 3 m2
2 2 4 r r r r : XOR operation
and direction of the torque (is clockwise) or going into the
Output : 1 1 0 1
plane of the paper.
ω
27. (d) 29. (b) Linear charge density,
1 kΩ O Q
Vin Vout λ= + + + + + + + ++
2 πR ++ +
+ R +
When the ring is rotated about the axis, ++++ + + ++++
D
+ +
the motion of the electrons in a circular
3V orbit is equivalent to a current carrying
Input waveform,
loop.
3V Q Q  2π 
Current, I= = Time period,T =
T 2π 
 ω 
ω
–5V Qω
⇒ I= ... (i)

The equation of input voltageVin = 5 sin (ωt + φ ) Magnetic field at the centre of a current carrying
When the voltage at point O becomes more than 3V, the µ I
circular loop is given by B = 0 …(ii)
diode (D ) stops working as the potential at both the terminal 2R
of the diode will be same, (i.e. 3V). It becomes reverse µ Qω
From Eqs. (i) and (ii), we get, B = 0 .
bias, so the output waveform will be as shown below. 2R 2π
3V µ 0ω  Q   Q 
B= . , Using, λ =
2  2 πr   2πR 
µ λω
⇒ B= 0
–5V 2
Solved Paper 2014 ●
39

30. (c) This problem is based on Doppler effect, which Using Maxwell’s thermodynamic relation, we get
states that there is a change in frequency of a wave  ∂S   ∂p 
  = 
(or other periodic event) for an observer moving  ∂V  T  ∂T  V
relative to its source.  ∂S   ∂S   ∂p   ∂v 
In classical physics, where the speeds of source and ∴   −  =    …(ii)
 ∂T  p  ∂T  V  ∂T  V  ∂T  p
the receiver relative to the medium are lower than the
velocity of wave in the medium, the relationship From Eqs. (i) and (ii), we get
between observed frequency (f ) and emitted  ∂p   ∂V 
Cp − CV = T     …(iii)
frequency (f0 ) is given by  ∂T  V  ∂T  p
 c + vr 
f =  f0 Let us consider p = p(V , T )
 c + vs   ∂p   ∂p 
∴ dp =   dT +   dV
where, c is the velocity of wave is the mediumvr is the  ∂T  V  ∂V  T
velocity of the receiver relative to the medium, positive dp  ∂p   ∂p  dV
if the receiver is moving towards the source and or =  + 
dT  ∂T  V  ∂V  T dT
vice-versa.vs is the velocity of source relative to the
medium.  ∂p   ∂p   ∂p   ∂V 
or   =0=  +  ⋅ 
Speed of the observer along the speed of sound wave  ∂T  p  ∂T  V  ∂V  T  ∂T  p
= u cos 60°  ∂p   ∂p   ∂V 
Source ⇒   =−    …(iv)
 ∂T  V  ∂V  T  ∂T  p
Wall
30° From Eqs. (iii) and (iv), we get
2
 ∂p   ∂V 
0° 30°
Cp − CV = − T    
s6
 ∂V  T  ∂T  p
o
uc 60°
 ∂p   1  ∂V  
2
u = − TV 2   ×  
 ∂V  T V 2  ∂T  p 
Relative velocity of sound w.r.t. observer  
= vs − ( − u cos 60° ) TVα 2
∴ Cp − CV =
= u cos 60° + vs kT
According to given question, 1  ∂V  1  ∂V 
u cos 60° + vs vs [since, kT = −   and α =   ]
= V  ∂p  V ∂T  p
f′ f
u or Cp = CV + TVα 2/kT
+ vs
2 v
= s 32. (c) Let us suppose that two events that occur in frame O occurs
9 f
f at t′1 and t′ 2 .
8
8 u  From Lorentz transformation,
⇒  + vs  = vs  vx 
9 2  t ′1 = γ t1 − 21
 c 
v
⇒ u= s  vx 
4 and t ′ 2 = γ t 2 − 22 
 c 
 ∂Q   ∂Q 
31. (c) Cp − CV =   −  In frame O′, two events that will be simultaneous, i.e.
 ∂T  p  ∂T  V
t ′1 = t ′ 2
 ∂S   ∂S 
=T   −T   …(i) ∴
v
t 2 − t1 = 2 (x 2 − x1)
 ∂T  p  ∂T  V
c
Let us consider entropy, S = S (V , T ) t 2 − t1 v c 1 1
or = = ⋅ =
 ∂S   ∂S  x 2 − x1 c 2 2 c 2 2c
∴ dS =   dT +   dV
 ∂T  V  ∂V  T x 2 − x1
∴ = 2c
 dS   ∂S   ∂S  dV t 2 − t1
or   =  + 
 dT   ∂T  V  ∂V  T dT 33. (c) After passing from first grating, the wavelength λ and
 ∂S   ∂S   ∂S   ∂V  λ + ∆λ get resolved, where
or   =  +   
 ∂T  p  ∂T  V  ∂V  T  ∂T  p λ
= 600 × 50
 ∂S   ∂S   ∂S   ∂V  ∆λ
⇒   −  =   
 ∂T  p  ∂T  V  ∂V  T  ∂T  p [Q In first order, resolving power = number of lines]
40 IIT JAM Physics Solved Papers & Practice Sets

After passing from second grating, the wavelengths λ d 2y dy


∆λ 36. Given equation, +5 =0
and λ + dλ get resolved, where = 1800 × 15 dx 2 dx
dλ We can write the auxiliary equation
Therefore, resolving power of combined system
λ λ ∆λ m 2 + 5m = 0
= = × = 600 × 50 × 1800 × 15
dλ ∆ λ dλ ⇒ m(m + 5) = 0
= 81 × 107 ⇒ m=0
or m = −5
34. (b) Energy of a particle of mass m inside a two-dimensional Now, y (x ) = c1 + c 2e − 5x
square box of length L is given by
when x = 0, y = 2
π 2 h2 2
Enx ny = (n x + n y2 ) [where, n x , n y = 1, 2, 3, ...] ⇒ c1 + c 2 = 2 ... (i)
2mL2 dy
Again, for = − 5 c 2e − 5x
For ground state, n x = n y = 1 dx
π 2 h2 π 2 h2 when x = 0, y = 2
∴ E11 = 2
(1 + 1) = − 5c 2 = 2
2mL mL2 −2
For first excited state → (1, 2), (2, 1) ⇒ c2 =
5
For second excited state → (2, 2) and so on. 2
and c1 − = 2 [from Eq. (i)]
Since, each state is occupied by one particle, therefore, 5
minimum energy will be 12
c1 =
E = (E11 + E12 + E21 + E22 ) 5
12 2 − 5x
π 2h2 5 π 2h2 5 π 2h2 4π 2h2 Hence, y = − .e ... (ii)
= + + + 5 5
mL2 2 mL2 2 mL2 mL2 12 2 − 5x
2 2 2 when, y = 0, − e =0 [from Eq. (ii)]
10π h 5 h  h  5 5
∴ E= = Qh=
mL2 2 mL2  2π  ⇒ 12 = 2e − 5x
1
35. (b) p1 = 105 Pa and p 2 = 107 Pa e − 5x = 6 ⇒ e 5x =
6
∴ ∆p = p 2 = p1  1
loge (e 5x ) = loge  
7 5 5
∆p = 10 − 10 = 10 (100 − 1)  6
∆p = 99 × 105 Pa  1
5x = loge  
 6
p1 = 3 × 103 kg / m3
1
p 2 = 2.9 × 103 kg / m3 x = − loge ( 6)
5
For unit mass, change in volume
37. Equation of EM wave,
 1 1 0.1× 10−3 10−4
∆V = − × 10−3 = = E = 6 π i$ sin [ 2π(106 z − 3 × 1014t )]
 2.9 3  3 × 2.9 3 × 2.9
Comparing given equation with
Using Clausius-Clapeyron relation
∆p ∆HF E = E0 sin (kz − ωt )
=
∆T T∆V ⇒ E0 = 6π
where ∆p = change in pressure and ω = 2π × 3 × 1014
∆T = change in melting point
Intensity (power per unit area)
∆HF = Latent heat of fusion
1
T = temperature I = ε 0E02c
2
∆V = change in volume
 10−4  1
99 × 105 × 500 ×   or I = × 8.85 × 10− 12 ( 6π)2 × 3 × 108
∆p × T∆V  3 × 2.9  2
∴ ∆T = = K or I = 4.42 × 10− 12 × 6 × π 2 × 3 × 108
∆HF 24 × 103
= 2.41 × 10− 6 W / m2
54450 × 10
∆T = K = 2.607 K Energy of photon of the EM Wave, E = hν
24 × 3 × 2.9 × 103
E = 6.63 × 10− 34 × 3 × 1014
Final temperature or melting point
= T + ∆T = 550 + 2.60 K = 552.6 K = 19.89 × 10− 20 J
Solved Paper 2014 ●
41

Number of photons per second falling on unit area  ∂p  nR


⇒   = ≠0
Intensity  ∂T  V V − nb
=
E dQ CV p
⇒ = dT + dV
2.41 × 10− 6 T T T
= = 1.21 × 1013photons  ∂CV / T 
19.89 × 10− 20 ⇒   =0 [T = constant]
 ∂V 
U
Radiation pressure is given by, p =  nR 
c ∂ 
where, U is the energy.  (V − nb ) 
⇒ =0
 ∂T V
The pressure experienced by a perfectly reflecting planar
2 Ef
surface, p ref = cos2 α Pa, where Ef is energy flux, c is Thus, we can say that
dQ
is exchanged.
c T
the speed of light.
According to the given question, 40. Given, d = 100125
. mm,λ = 500 nm,n 0 = 1658
. and ne = 1558
.
2 Ef The path difference introduced by the plate between
p ref =
c ordinary and extraordinary rays is given by
2 × 2.4 × 10− 6 ∆ = (n 0 − ne )d
=
3 × 108 = (1658
. − 1558
. ) × 100125
. mm
= 1.6 × 10− 6 − 8Pa = 0.100125 mm = 100125 nm

p ref = 1.6 × 10− 14 Pa Now, the phase difference, φ = ×∆
λ
38. Moment of inertia of the rod about the axis through O, 2π
= × 100125
I = I CM + Mh 2 [parallel axis theorem] 500
~
− 40π
2
1 ML
I= ML2 + As the plate is a quarter wave plate, so (i) at C, the light is
12 4 elliptically polarised and (ii) at E and F the light has passed
1 2
I = ML the quarter wave plate twice, so the phase difference of π is
3 introduced, therefore, the light is linearly polarised.
Angular momentum of the system about O 41. Diffraction Diffraction is the slight bending of light as it
1 passes around the edge of an object. The amount of
Lf = Iω = ML2ω
3 bending depends on the relative size of the opening. If the
where, ω is the angular velocity of the rod after the impulse opening is much larger than the light’s wavelength, the
(J) acted on it. bending will be almost unnoticeable.
Initial angular momentum of the system about O is L1. Suppose the secondary wavelets diffracted at an angle θ
are focused wavelets start from different parts of the slit in
As no external torque acting on the system, we can apply same phases but they reach the point P in different phases.
conservation of angular momentum using 3
For secondary maxima, d sin θ = λ L
t2
J = ∫ τ ⋅ dt and τ =
dL 2
t1 dt We can divide the slit into three equal parts. The path
⇒ ∫ dL = ∫ τ dt difference between two corresponding points of the first two
λ
⇒ τ = L2 − L1 parts will be L . The wavelets from these points will
2
Change in angular momentum is equal to the angular interfere destructively. However, the wavelets from the third
impulse of the resulting torque. part of the slit will contribute to some intensity forming a
 ∂U  secondary maximum. The intensity of this maximum is
39. Using, dU =   dT much less than that of the central maximum.
 ∂T  V
Condition for first secondary maximum,
⇒ dU = CV dT 3
d sin θ1 = λ L
⇒ dQ = CV dT + pdV 2
[because the process is reversible] On either side the central maximum of decreasing intensity
 ∂CV   ∂p  at positions
⇒   =  λ
 ∂V  T  ∂T  V θ = ± ( 2n + 1) L
2d
42 IIT JAM Physics Solved Papers & Practice Sets

42. Planck’s law for the spectrum of black body radiation 43. Given that, c is the speed of light and v is the speed of earth
predicts the Wien displacement law and may be used to relative to other.
numerically evaluate the constant relating temperature and When light goes from Q to N1, distance L0 is travelled at
peak wavelength (or frequency). speed c − v and returned from N1 travels at speed c + v total
L L
According to one from that law, the black body spectral time, T1 = 0 +
radiation (power per emitting area per solid angle per unit c + v (c − v )
wavelength) is given by When the light travel from Q to N2 and back the travel time
2hc2
1 2L 0 2L 0
u λ (λ,T ) = T2 = 2 =
5
λ e hc / λkT
−1 (c − v 2 )1/ 2  v 2  1/ 2c
1− 2 
Differentiating u( λ ,T ) with respect to λ and setting the  c 
derivative equal to zero gives
 L 0  v  2
∂u  hc e hc / λkT
1 5  ∆T = T1 − T2 =    
= 2hc 2  −  =0  c  c 
∂λ  kTλ7 (ehc / λkT −1)2 λ6 ehc / λkT −1
If the rod length L0 in a rest frame (S′) makes an angle θ 0
⇒ According to given concept with the X-axis, its projection length
∞ ∞ α − β / λT ∆x ′ = L 0 cosθ 0
∫0 uT ( λ )dλ = ∫0 c 3 λ5 e dλ and ∆y ′ = L 0 sin θ 0,
α ∞ But in the frame S1 in which the rod moves at speedv along
= 3 5 ∫ e − β / λ T dλ
c λ 0 X-axis, the projection length.
Tα e − β / λT  β ∆x and ∆y are given by ∆y = ∆y′
=
βc 3λ3 ∫ λ2
dλ  
T  The length L of the rod as measured by a stationary
αT 0 −t  −β  observer in S1,L = ( ∆x )2 + ( ∆y )2
=
βc 3λ3 ∫∞ e dt Q t =


λT  = L 0(1− β 2 cos2 θ 0 )1/ 2
Solved Paper
2013
IITJAM Physics
MM : 100 Time : 3 hrs

n
In this question paper, there are total of 30 questions carrying 100 marks of 3 hours duration.
n Questions 1-10 (Objective questions) carry 2 marks each and questions 11-20 (Subjective questions) carry
three marks and questions 21-30 (Descriptive questions) carry 5 marks each.
n Each objective question has four choices for its answer i.e. (a), (b), (c) and (d). Only one of them is the
correct answer.
n There will be negative marking for wrong answers to objective questions. The following marking scheme
for objective questions shall be used:
(a) For each correct answer, you will be awarded 2 marks.
(b) For each wrong answer, you will be awarded – 0.5 (Negative 0.5) mark.
(c) Multiple answers to a question will be treated as a wrong answer.
(d) For each un-attempted question, you will be awarded 0 (Zero) mark.
(e) Negative marks for objective part will be carried over to total marks.
n Do not write more than one answer for the same question. In case you attempt a subjective question
more than once, please cancel the answer(s) you consider wrong. Otherwise, the answer appearing last
only will be evaluated.
n Clip board, log tables, slide rule, calculator, cellular phone and electronic gadgets in any form are not
allowed.

1. The inverse of the matrix


é 0 1 1ù
M = ê 0 0 1ú is
ê ú
êë 1 0 0úû
(a) M - I (b) M 2 - I
(c) I - M 2 (d) I - M

2. The value of i + -i , i = -1, is


1
(a) 0 (b) (c) 2 (d) - 2
2
44 IIT JAM Physics Solved Papers & Practice Sets

æ 2 x2ö $
3. A particle is released at x = 1in a force field F (x ) = ç - ÷ e x , x ³ 0. Which one of the
èx 2 2ø
following statements is false?
(a) F ( x ) is conservative
(b) The angular momentum of the particle about the origin is constant
(c) The particle moves towards x = 2
(d) The particle moves towards the origin

4. A travelling pulse is given by


æ 2abxt - a 2x 2 - b 2t 2 ö
f (x, t ) = A exp çç ÷÷,
è c2 ø
where A, a, b and c are positive constants of appropriate dimensions. The speed of the pulse
is
b 2b cb b
(a) (b) (c) (d)
a a a 2a

5. If the dimensions of mass, length, time and charge are M, L, T and C respectively, the
dimensions of the magnetic induction field B is
(a) [ML2T -1C -1] (b) [MT -1C -1] (c) [L2T -1C] (d) [L-1T -1C]

6. A blackbody at temperature T emits radiation at a peak wavelength l. If the temperature of the


blackbody becomes 4T, the new peak wavelength is
1 1 1 1
(a) l (b) l (c) l (d) l
256 64 16 4

7. Let N MB , N BE , N FD denote the number of ways in which two particles can be distributed in two
energy states according to Maxwell-Boltzmann, Bose-Einstein and Fermi-Dirac statistics,
respectively. Then, N MB : N BE : N FD is
(a) 4 : 3 : 1 (b) 4 : 2 : 3 (c) 4 : 3 : 3 (d) 4 : 3 : 2

8. Electric field component of an electromagnetic radiation varies with time as


E = a (cos w0t + sin wt cos w0t ),
where, a is a constant and the values of w and w0 are 1 ´ 1015 s -1 and 5 ´ 1015 s -1,
respectively. This radiation falls on a metal of work function 2 eV. The maximum kinetic energy
(in eV) of photoelectrons is
(a) 0.64 (b) 1.30 (c) 1.70 (d) 1.95

9. The fraction of volume unoccupied in the unit cell of the body centered cubic lattice is
8- 3 p 3p 6- 2 p p
(a) (b) (c) (d)
8 8 6 3 2

10. For an ideal op-amp circuit given below, the DC gain and the cut-off frequency, respectively
are 1 kΩ
+

—1 µF –

10 kΩ
1 kΩ

(a) 1 and 1 kHz (b) 1 and 100 Hz (c) 11 and 1 kHz (d) 11 and 100 Hz
Solved Paper 2013 ●
45

11. The solution of the differential equation dz(x, y ) + xz(x, y ) dx + yz(x, y ) dy = 0 is


12. Given that f (1) = 1 , f ¢ (1) = 1and f ¢ ¢ (1) = 1, the value of f (1 / 2) is

13. Two tubes A and B are connected through another tube C. A mercury manometer is connected
between B and C (see figure). The diameters of B and C are 0 .04 m and 0.01 m, respectively.
An incompressible fluid of density 1.0 ´ 10 3 kgm -3 enters A, and leaves B with a constant
speed 0.2 ms -1. If the density of mercury is 13.6 ´ 10 3 kgm -3 , the height h of the mercury
column in the manometer is (Take, acceleration due to gravity, g = 10 ms -2 )

A C B

14. The path of a particle of mass m, moving under the influence of a centre force, in plane polar
coordinates is given by r = r 0e kq , where r 0 and k are positive constants of appropriate
dimensions. The angular momentum of the particle is L and its total energy is zero. The
potential energy function V (r ), in terms of m, L and k is

15. The dimensions of thin convex lens of refractive index n are shown in the figure. The radius of
curvature of the lens in terms of n, h and d (where, d << h) is

δ
h

16. A charged particle of mass m, charge q and constant velocity v enters a uniform magnetic field
B = B 0 e$ x , (B 0 > 0), at an angle q to the direction of magnetic field. Find the angle q, if in one
revolution of the helical motion, the particle advances along the direction of the magnetic field
a distance equal to the radius of the helical path.

17. Two thermally isolated identical system have heat capacities which vary as CV = bT 3 (where,
b > 0). Initially, one system is at 300 K and the other at 400 K. The systems are then brought
into thermal contact and the combined system is allowed to reach thermal equilibrium. The
final temperature of the combined system is

18. A beam of X-rays of wavelength 0.2 nm is incident on a free electron and gets scattered in a
direction with respect to the direction of the incident radiation resulting in maximum wavelength
shift. The percentage energy loss of the incident radiation is

19. A free particle of mass m is confined to a region of length L. The de-Broglie wave associated
with the particle is sinusoidal in nature as given in the figure. The energy of the particle is

0
L/3 2L/3 L
46 IIT JAM Physics Solved Papers & Practice Sets

500 kΩ
20. A variable power supply (5 V-20 V) is connected to a Zener diode
specified by a breakdown voltage of 10 V (see figure). The ratio of the
maximum power to the minimum power dissipated across the load 5 V-20 V
1 kΩ
resistor is

21. Apply Gauss divergence theorem to the gravitational field due to a spherical object of mass M
and uniform density r located at the origin. Obtain Gauss’s law for gravitation (analogous to
the Gauss law in electrostatics) in integral and differential forms.

22. A thin annular disc of inner and outer radii a and b (a < b ), respectively has uniform mass
density s s and is placed in the xy-plane such that its axis lies along the Z-axis. Determine the
gravitational force due to the disc on a particle of mass m located on the Z-axis at a distance z
from the origin. If the particle is released at z, such that z << a , b describes the nature of motion
of the particle.

23. A concentric spherical volume of inner radius a and outer radius b is


filled with a material of finite conductivity specified by
A Vo
s (r ) = 2 , where, A is a positive constant of appropriate dimensions. a
b
r
The outer surface is grounded and the inner surface is maintained at a
potential Vo . Calculate the resistance of this configuration.

24. A conducting solid sphere of radius a, carrying a charge q is kept in a dielectric of dielectric
constant k, such that half the sphere is surrounded by the dielectric as shown in figure. Find
the surface charge densities in the upper and lower hemispherical surfaces.
q

25. A particle of mass m is subjected to a potential V (x ) = ax 2 , - ¥ < x < ¥, where a is a positive


h
constant of appropriate dimensions. Using the relation Dx Dp » , estimate the minimum
2
energy of the particle.

26. A hollow cylinder (closed at both ends) with adiabatic walls is divided into n equal cells
(C1, C 2 , K, Cn ) using discs D1, D 2 , K, Dn - 1 (see figure). The discs can slide freely without
friction. The first disc (D1 ) is adiabatic and the remaining discs are diathermal (thermally
conducting). Each cell contains one mole of ideal monoatomic gas. Let the initial pressure,
volume and temperature of each cell be P 0 , V 0 and T0 , respectively. The gas in cell C1 (first
cell) is heated slowly until the temperature of the gas in cell Cn (last cell) reaches final
equilibrium temperature 4T0 . Find the volume of the first cell in terms of the number of cells (n )
and the initial volume (V 0 ).
D1 D2 D3 Dn–1

C1 C2 C3 Cn
Solved Paper 2013 ●
47

27. A spaceship S1 leaves the Earth along the positive x-direction. Another spaceship S 2 also
leaves the Earth along the direction which makes an angle 60° with X-axis. The speeds of S1
and S 2 are measured as 0.6 c and 0.9 c, respectively by an observer on the Earth. Find the
speed of S 2 as measured by an observer in S1.
v2=0.9 c
Y
S2

O X

60°
S1 v1=0.6 c
Earth

28. For the given circuit, calculate the input impedance, output impedance and voltage gain.
26 mV
Use b = 200,V BE = 0.7 and re =
IE
20 V

12 kΩ 2.2 kΩ

10 µF
1 µF
5 kΩ
1 kΩ 3.9 kΩ
1 kΩ 20 µF

50 µV

29. The mass density of a disc of mass m and radius R varies as


æ r ö
r (r ) = r 0 ç1 - ÷ for 0 £ r £ R
è Rø
=0 for r > R

Find the moment of inertia of the disc about the axis perpendicular to the plane of the disc and
passing through the centre of the disc in terms of m and R.

30. The speed of sound propagation in air as a function of temperature T is given by v = aT ,


where, a is a constant of appropriate dimensions. Calculate the time taken for a sound wave to
travel a distance L between two points A and B, if the air temperature between the points
varies linearly from T1 to T2 .

Answers
1. (b) 2. (c) 3. (d) 4. (a) 5. (b) 6. (d) 7. (a) 8. (b) 9. (a) 10. (c)
Answers with Explanations
1. (b) According to the question, given matrix is or Y 2( -2 -( -1)) = 4i 2 = - 4
é 0 1 1ù or Y2 = + 2
M = ê 0 0 1ú or Y =± 2
ê ú
êë 1 0 0úû As Y is sum of two square roots which are positive.
Square of the matrix M is Thus, Y cannot be negative.
M 2 = M ×M \ Y = 2
é 0 1 1ù é 0 1 1ù æ 2 x2ö
= ê 0 0 1ú ê 0 0 1ú
3. (d) Given, force field is, F (x ) = ç - ÷ e$ x , x ³ 0
ê úê ú èx 2 2ø
êë 1 0 0úû êë 1 0 0úû At x = 1, force is
é 1 0 1ù 2 (1)2 1 3
| F ( 1) | = 2
- = 2 - = = 1.5 N
= ê 1 0 0ú (1) 2 2 2
ê ú
êë 0 1 1úû Thus, 1.5 N force acts on the particle along X-direction,
when released.
Cube of the matrix is
Curl of the force,
M 3 = M 2 ×M
é $i $j k$ ù é $i $j k$ ù
é 1 0 1ù é 0 1 1ù ê ¶ ¶ ú
¶ ú ê ¶ê ¶ ¶ úú
= ê 1 0 0ú ê 0 0 1ú Ñ´F=ê =
ê úê ú ê ¶x ¶y ¶z ú ê ¶x ¶y ¶z ú
êë 0 1 1úû êë 1 0 0úû ê Fx Fy Fz ú ê Fx 0 0ú
ë û ë û
é 1 1 1ù ¶ $ ¶ $
= Fx j - Fx k = 0 - 0 = 0
= ê 0 1 1ú ¶z ¶y
ê ú
êë 1 0 1úû ⇒ Ñ´F=0
é 1 1 1ù é 0 1 1ù Thus, the force F(x ) is conservative.
⇒ M 3 - M = ê 0 1 1ú - ê 0 0 1ú At x = 2,
2 2
F = - =0
ê ú ê ú
êë 1 0 1úû êë 1 0 0úû 2 2
Thus, the particle moves from x = 1to x = 2 to attain
é 1 0 0ù
equilibrium.
= ê 0 1 0ú = I
ê ú As curl ( Ñ ´ F) is zero, the angular momentum of the
êë 0 0 1úû particle is constant.
or M3 -M =I Thus, option (d) is wrong.
or M (M 2 - I ) = I 4. (a) According to the question, given that
Multiplying both sides by M -1, we have æ 2 abxt - a 2x 2 - b 2t 2 ö
çç ÷÷
MM -1(M 2 - I ) = M -1I è c2 ø
f (x , t ) = A e
or I (M 2 - I ) = M -1 where A, a, b and c are positive constants.
or M -1 = M 2 - I Differentiating both sides the given function w.r.t. t, we
have
2. (c) Given that, i = -1 æ 2 abxt - a 2x 2 - b 2t 2 ö æ 2 abx - 0 - 2b 2t ö
çç ÷÷ ´ çç ÷÷
Let Y = i + -i è c2 ø è c2 ø
f ¢ (x , t ) = A e
( i - -i ) Putting f ¢ (x , t ) = 0
= ( i + -i ) ´ [multiply by conjugate]
( i - -i ) 2abx - 2b 2t
( i )2 - ( -i )2 We have =0
= [Q(a + b )(a - b ) = a 2 - b 2] c2
i - -i or 2 b(ax - bt ) = 0
i - ( -i ) 2i or ax - bt = 0 [Qb ¹ 0]
= =
i - -i i - -i or ax = bt
⇒ Y ( i - -i ) = 2 i dx b
or =
dt a
Squaring on both sides, we have
dx b
⇒ Speed of the pulse,v = =
Y 2(i + - i - 2 -i 2 ) = 4i 2 dt a
Solved Paper 2013 ●
49

5. (b) Given, [mass] = M 8. (b) For maximum kinetic energy of photoelectrons, we have
[length] = L to consider maximum value of frequency between given
[time] = T values.
w 5 ´ 1015
[charge] = C (f0 )max = 0 =
2p 2p
[B] = ?
[Force] Work function, f0 = 2 eV
We can write, [B] =
[Current] [Length] Maximum kinetic energy of photoelectrons,
[MLT -2] K max = hf0 - f0
=
[Charge / Time] [ L] 6.626 ´ 10-34 ´ 5 ´ 1015
-2 -2 = J - 2 eV
[MT ] [MT ] 2p
= =
[Charge] / [Time] é Cù [Qh = 6.626 ´ 10-34 Js]
êë T úû
= [MC T ]-1 -1 6.626 ´ 10-34 ´ 5 ´ 1015
= eV - 2 eV
Dimensions of the magnetic induction field 2p ´ 1.6 ´ 10-19
[B] = [ MC-1T -1]. [Q 1 eV = 1.6 ´ 10-19]
-34 + 15 + 19
= 3.29 ´ 10 eV - 2 eV
6. (d) According to Wien’s displacement law,
l pT = constant ...(i) = (3.29 - 2) eV
where, l p = peak wavelength = 1.29 eV » 1.3 eV
T = temperature in kelvin
9. (a) In BCC lattice shown in the figure, we have
From Eq. (i), we can write
( l p1 ) (T1) = ( l p 2 ) (T2 ) [Q l p1 = l] 4r = 3a
⇒ ( l ) (T ) = ( l p 2 ) ( 4T ) 3
⇒ r = a
The new peak wavelength, 4
1 where, r is radius of an atom and a is side length of the
lp 2 = l cubic lattice.
4
7. (a) Maxwell-Boltzmann Statistics According to this Number of atoms per cubic lattice,
statistics, two particles P and Q can be distributed in 4 1
n = 8 ´ + 1= 2
ways as 8
PQ PQ
P or Q
Q P r
(i) (ii)

Bose-Einstein Statistics According to this statistics, we


get a total of 3 states or ways in which particles can be a
distributed two particles will be bunched is the same 4 3
state and one particle will be in two states (spin), as Volume of atoms = 2 ´ pr
3
shown below.
4 ( 3 )3 3
PP PP =2´ p ´ a
P P
3 ( 4 )3
(i) (ii) 4p 3 3 3
=2´ ´ a
3 ( 4 )3
Fermi-Dirac Statistics In this the particles are
indistringuishable. Both particles are labelled P. 3 3 3p 3
= 2p ´ a = a
1 3 16 8
Fermion have half-integer spin , , etc.
2 2 Unoccupied volume,
The state of the system is shown below. 3p 3
V ¢ = a3 - a
P P 8
(i) (ii) Fraction of unoccupied volume,
We get a total of 1 state of two system as a whole. V¢ 3p 8- 3p
= 1- =
Hence, required ratio is N MB : N BE : N FD = 4 : 3 : 1 a3 8 8
50 ●
IIT JAM Physics Solved Papers & Practice Sets

10. (c) Consider the op-amp circuit From Eqs. (ii) and (iii), we get
1
Rf = 10kΩ 2 ´ + q = 1⇒ q = 0
2
R1 = 1kΩ From Eq. (i),
– 1 1
+ 0 + c =1 ⇒ c =
R = 1kΩ 2 2
+
1 1 1 1
\ f (x ) = x 2 + 0 + = x 2 +
C=
1 µF 2 2 2 2
2π 1
Putting x =
2
The given configuration is non-inverting. æ 1ö 1 æ 1ö
2
1 1 1 1+ 4 5
The gain of non-inverting op-amp configuration is ⇒ f ç ÷= ç ÷ + = + = =
è 2ø 2 è 2ø 2 8 2 8 8
R
A = 1+ f
R1 13. Applying equation of continuity of B and C,
AC vC = AB vB
where, Rf = feedback resistance = 10 kW
R1 = 1kW C
10 k W A B
⇒ A = 1+ = 11
1k W
Cut-off frequency is given by h
1
fc =
2pRC
1
=
1 ⇒ rC2 ´ vC = rB2vB
2 p ´ (1k W ) ´ mF
2p æ 0.01ö
2
æ 0.04 ö
2
3
= 10 Hz = 1kHz or ç ÷ ´ vC = ç ÷ ´ ( 0.2)
è 2 ø è 2 ø
11. The given differential equation is æ 0.04 ö
2

dz (x , y ) + xz (x , y ) dx + yz (x , y ) dy = 0 or vC = ç ÷ ´ 0.2
è 0.01ø
Applying variable separable method, we can write
= (4)2 ´ 0.2 = (16) (0.2)
dz (x , y ) + z (x , y ) [x dx + y dy ] = 0
Speed of the fluid at C is,
or dz (x , y ) = - z (x , y ) [x dx + y dy ]
vC = 3.2 m/s
dz
or - = x dx + y dy Applying Bernoulli’s equation between B and C,
z (x , y )
1
or
dz
= - d (xy ) pC - pB = - rf (vB2 - vC2 ) = rmgh
z (x , y ) 2
1
Integrating on both sides, we have ⇒ rf (vC2 - vB2 ) = rmgh
2
dz
or ò z (x , y ) = - ò d (x , y ) or h=ç C
æv 2 - vB2 ö æ rf ö
÷ = ç- ÷
è 2g ø è rm ø
or ln z = - xy + C
or z = e - xy + C ¢ where, rf = density of the fluid
rm = density of mercury
where, C and C ¢ are constants.
Height of the mercury column,
12. Let f (x ) = px 2 + qx + c where p, q and c are constants. ( 3.2)2 - ( 0.2)2 æ 103 ö
h= ç ÷
Putting x = 1 2 ´ 10 3
è 13.6 ´ 10 ø
⇒ f (1) = p + q + c = 1 ...(i)
(3.2 + 0.2) (3.2 - 0.2) 1
Differentiating f (x ) w.r.t x, we have = ´
2 ´ 10 13.6
f ¢ (x ) = 2px + q (3.4) (3) 1
= ´
⇒ f ¢ (1) = 2p + q = 1 ...(ii) 20 13.6
Differentiating f ¢(x ) w.r.t. x, we have 3 ´ 34
= = 0.0375 m
f ¢ ¢ (x ) = 2p = 1 20 ´ 136
1 » 3.8 cm
⇒ p= ...(iii)
2
Solved Paper 2013 ●
51

14. Since, the particle is moving in a plane polar coordinate We can write from the geometry of the figure,
system (r , q), Lagrangian can be written as d´n
OQ = R -
1 • • 2
L= m (r 2 + r 2 q 2 ) - V (r ) …(i) We have to multiply by n because we are considering
2
Equation of motion are equivalent value of d in air.
From right angle triangle POQ,
d æç ¶L ö÷ ¶L
- =0 (PQ )2 = (OP )2 + (OQ )2
dt çè ¶ r• ÷ø ¶r
2
æ dn ö
d æç ¶L ö÷ ¶L or R 2 = (nh )2 + çR - ÷
and - =0 è 2ø
dt çè ¶ q• ÷ø ¶q
where, OP = nh = equivalent of h in air.
Putting the values of derivatives from Eq. (i), we have d2n 2
or R 2 = n 2h 2 + R 2 + - Rdn
d • •2 ¶V (r ) 4
(m r ) - (mr q ) + =0
dt ¶r As d2 is small, we can neglect it.
and
d •
(mr 2 q) = 0
⇒ R 2 » n 2h 2 + R 2 - Rdn
dt or R dn = n 2h 2
• l
Putting q = , we get n 2h 2 nh 2
mr 2 or R= =
nd d
2
•• æ l ö - ¶V (r ) nh 2
m r - mr ç 2 ÷ = ⇒ Radius of curvature of the lens, R =
è mr ø ¶r d
•• l2 - ¶V (r )
or mr- = 16. The particle moves along a helical path.
mr 3 ¶r Parallel component of the velocity is
••-¶ é l2 ù vII = v cos q
or mr = êV (r ) + ú
¶r ë 2mr 2 û Perpendicular component of the velocity is
v ^ = v sin q
•• • -¶ é l 2 ù ¶r
Þ (m r ) r = êV (r ) + ú mv ^ mv sin q
¶r ë 2mr 2 û ¶t Radius of the path is, r = =
qB0 qB0
-¶ é l2 ù æ 2 pm ö
êV (r ) +
= ú Pitch = v||T = (v cos q) ç ÷
¶t ë 2mr 2 û è qB0 ø

¶ é 1 •2 l2 ù According to the question,


Þ ê mr + 2
+ V (r )ú = 0 Pitch = r
¶t êë 2 2mr úû
æ 2pm ö mv sin q
1 •2 l2 ⇒ (v cos q) ç ÷=
Þ mr + + V (r ) = constant è qB0 ø qB0
2 2mr 2
or 2p cos q = sin q
= E ( say ).
or tan q = 2 p
According to the question, E = 0
Thus, angle in one revolution of the helical motion
1 •2 l2
Þ mr + + V (r ) = 0 q = tan-1 ( 2p )
2 2mr 2
17. Let equilibrium temperature of the system is T.
-l 2 1 •2
Þ V (r ) = - mr Change in internal energy of the first system is
2mr 2 2
dU1 = CV dT = bT 3dT
15. Consider the geometry of the lens shown below.
Total change in the internal energy,
P T
T æT 4 ö b
R = radius of curvature DU1 = ò dU1 = ò bT 3dT = b ç ÷ = [T 4 - ( 300)4]
300 è ø 300 4
4
h
Q Similarly, change in internal energy of the second system
O T
h DU 2 = ò dU 2 = ò bT 3dT
400
b 4
δ = [T - ( 400)4]
4
52 ●
IIT JAM Physics Solved Papers & Practice Sets

For these two isolated systems, For the given wave,


DU 1 = - DU 2 n=3
⇒ T 4 - ( 300)4 = - [T 4 - ( 400)4] ( 3 )2 ´ h 2 9h 2
⇒ E= =
or 2T 4 = ( 300)4 + ( 400)4 8mL2 8mL2
= 108 [ 34 + 44] = 337 ´ 108 20. Consider the given circuit
4 8 500Ω
or T = 168.5 ´ 10 K
⇒ T = 102 ´ (168.5)1/4 K
= (102 ) (3.602) K 1kΩ VL
VZ = 10V
= 360.2 K 5V-20V
Final temperature of the combined system is
T » 360 K
Suppose voltage across the load isVL.
18. Applying formula of Compton shift, we can write shift in
Maximum voltage across the load is
wavelength,
(VL )max = VZ = 10 V
φ Minimum voltage across the load is
X-rays (Vin)min 5 ´ 1000
e (VL )min = ´ 1000 = V
500 + 1000 1500
Dl =
h
(1 - cos f ) 50 10
= V= V = 3.33 V
mec 15 3
Maximum value of the shift in the wavelength, Ratio of the maximum power to the minimum power
h 2h dissipated across the load resistor is
( Dl )max = [1 - ( -1)] = 2
(mec ) mec æ ö
2
2
(VL )max é (VL )max ù ç 10 ÷
2 ´ 6.626 ´ 10-34 Pmax
= = 2
=ê ú = ç 10 ÷ = 9
9.1 ´ 10-31 ´ 3 ´ 108 Pmin (VL )min ë (VL )min û ç ÷
è 3 ø
= 0.485 ´ 10-3 - 9
= 0.485 ´ 10-11 m 21. Gauss divergence theorem can be written as
= 0.00485 nm òò ¶V g × ds = òV Ñ × gdV
Change in energy,
where, V is the enclosed volume.
æ 1 1ö æ l - lf ö é - Dl ù
DE = hc ç - ÷ = hc ç i ÷ = hc ê ú For a spherical object of mass M, we can write
l
è f li ø è li lf ø l (
ë i i l + D l ) û ¶ æ4 3ö 2
Ñ ×(V) = - ç pR ÷ = - 4pR
Percentage change in energy, ¶r è 3 ø
é - Dl ù GM
hc ê ú Q g=
DE ë li ( li + Dl ) û R2
=
Ei 1 ⇒ g( Ñ × V ) = - g( 4pR 2 ) = - 4pGM
hc ´
li Using gauss divergence theorem
Dl 0.00485
=- =- ⇒ òò ¶V g× dS = - òV 4pGM
li + Dl 0.2 + 0.00485
0.00485 485 = - 4pG ò r dV [QM = ò r dV ]
=- =- V
0.20485 20485
⇒ òV Ñ × (g dV) = - 4pG òV r dV
485
\ Percentage energy loss = ´ 100 = 2.36% Elementary shell
20485
19. The energy of a particle confined in an one dimensional box
is given by r
dr
2 2 V0
n h
En =
8mL2 a
where, n = number of states,
m = mass of the particle, b
L = angular momentum of the particle. A
σ(r) =
r2
Solved Paper 2013 ●
53

⇒ Ñ × g = - 4pGr Force on the particle is downward


This is the differential form of Gauss’s law for gravity. é 1 1 ù
⇒ F = - ( 2pmGs s ) ê - úz
We can also write êë a 2 + z 2 b 2 + z 2 úû
òò¶V g × ds = - 4pGM For z << a, b
This is the integral form of Gauss’s law for gravity. æ 1 1ö
F = - 2pmGs s ç - ÷ z
èa b ø
Here, òò denotes a surface integral over a closed surface.
dV 2pmGs s
Q (b - a ) = constant
where, g is gravitational field, ab
G is the universal gravitational constant, \ F ∝ ( -z )
M is the total mass enclosed within the surface ¶V. Therefore, motion of the particle is SHM when released.
22. Consider a mass m placed at point P ( 0, 0, z ). 23. Consider a spherical shell of radius r and thickness dr.

m Y Area of this spherical shell is


P dA = 4pr 2
z We can write resistance of this elementary shell as
1 dr é rL ù
a dR = ´ êëQ R = A úû
b X s (r ) 4pr 2
1 dr dr
= ´ =
A 4 pr 2 4 pA
r2
Consider a concentric elementary ring of the disc of mass
Total resistance of the configuration is
dM. b dr 1
\ dM = s sdA = s s ( 2pr dr ) R = ò dR = ò = (b - a )
a 4 pA 4 pA
where, r is radius and dr is thickness of the elementary ring.
Gravitation field at P due to this elementary ring is
24. Half part of the sphere will have induced charges of
G dM z magnitude
dE = 2
(r + z 2 )3/ 2 æ 1ö a
Q p = Q ç1- ÷
Resultant field at P due to whole disc is è kø
b b GdMz Hence, charge density
E = ò dE = ò
a (r 2 + z 2 )3 / 2
a
æ 1ö
Q ç1- ÷
b Gs ( 2 pr dr ) z Qp è kø
⇒ E=ò s
sp = =
a (r 2 + z 2 )3/ 2 2 pa 2 2 pa 2
Putting r2 + z2 =y2 Another half part will have charge Q and hence charge
Q
⇒ 2 r dr = 2y dy density of s = .
2pa 2k
or r dr = y dy
25. Given, V (x ) = ax 2, - ¥ < x < ¥
Now, we can write
2
r dr y dy y dy dy \ $ = p$ + a Dx$ 2 …(i)
ò (r 2 + z 2 )3/ 2 = ò (y 2 )3/ 2 = ò y 3 = ò y 2 H
2m
1 1 Also from Heisenberg uncertainty principle
=- =- h
y r + z2
2 D x$ Dp$ = …(ii)
2
b
é 1 ù Taking average on both sides of Eq. (i), we have
⇒ E = Gs s ( 2 p z ) ê - ú
êë 2 2
r + z úû a p$ 2
$ =
H + a D x$ 2
é ù 2m
1 1
= s sG ( 2pz ) ê - + ú $ = 1 æç 1 Dp$ 2 ö÷ + (a D x$ 2)
êë b2 + z 2 2
a + z úû 2 Þ E= H …(iii)
2 èm ø
Force on the particle, Now, using Eq. (ii), we get
é 1 1 ù æ Dp ö
2
a æ hö
2
2
F = mE = s sG ( 2pz ) m ê - ú ç ÷ (a Dx$ ) = ç ÷ …(iv)
êë a 2 + z 2 b 2 + z 2 úû èmø m è 2ø
54 ●
IIT JAM Physics Solved Papers & Practice Sets

From Eq. (iii), we can say that, for E to be minimum 27. From Earth’s frame of reference
Dp$ 2 a æ hö Velocity of S1 = v1e = 0 .6c
= a Dx$ 2 = ç ÷
m m è 2ø Velocity of S 2 along x-direction
1 a æ hö a æ hö v 2e = (0.9 c ) cos 60° = 0.45 c
Þ Emin = ç ÷+ ç ÷
2 m è 2ø m è 2ø We have to find of S 2 with respect to S1 (v 2l ).
3 a æ hö v + v 2l
= ç ÷ Applying the formula, v1e = 2e
v v
2 m è 2ø 1+ 2e 2 2l
c
26. Consider a hollow cylinder which is divided into n equal 0.45 c + v 2l
Þ 0.6 c =
cells. ( 0.45 c )(v 2l )
D1 Adiabatic Dn–1 1+
c2
p0 , V0 or 0 .6 c + 0.6 ´ ( 0 .45)v 2l = 0.45 c + v 2l
T0 or 0 .6 c + ( 0. 27)v 2l = 0 .45 c + v 2l
or v 2l (1- 0 .27) = 0.15 c
Diathermic or v 2l = 0. 205c » 0.21c .
Adiabatic
28. According to the question,
One mole
monoatomic b = 200,VBE = 0.7
Final equilibrium temperature of the gas in cell 2 to n is 4T0. 26 mV
and re =
For the adiabatic expansion of the first cell, we have IE
p 0V0g = pV g ...(i) 20V

For the adiabatic expansion of remaining (n - 1) cells, we


can write 12kΩ 2.2kΩ
(n ¢ p 0 )1 - g T0g = p1 - g ( 4T0 )g
æT ö
g 10µF 1µF
or (n ¢ p 0 )1 - g ç 0 ÷ = p1 - g
è 4T0 ø IB 5kΩ
5/ 3
æ 1ö 1kΩ 3.9kΩ
or p1 - g = ç ÷ (n ¢ p 0 )1 - 5/ 3
è 4ø IE
æ 1ö
5/ 3 50µV
=ç ÷ × (n ¢ p 0 )-2/ 3 1kΩ 20µF
è 4ø
5/ 3
æ 1ö
⇒ p -2 / 3 = ç ÷ (n ¢ p 0 )-2/ 3
è 4ø
-5 / 2
æ 1ö
or p=ç ÷ (n ¢ p 0 ) Consider the loop shown in the circuit diagram, we can write
è 4ø
IB ´ ( 3.9 kW ) = 0.7 + IE ´ (1k W )
= ( 2-2 )-5/ 2 (n ¢ p 0 ) = 32p 0(n ¢ )
IE
Pressure due to gases in the remaining (n - 1) cells is ´ 3.9 ´ 103 = 0.7 + IE ´ 103
b
p = 32p 0(n - 1) [Qn ¢ = n - 1] IE
or ´ 3.9 ´ 103 = 0.7 + IE ´ 103
Now, from Eq. (i), we can write 200
p 0V0g = 32p 0(n - 1) V g or IE ´ 5 ´ 3.9 = 0.7 + IE ´ 103
V0g or IE [5 ´ 3.9 - 103] = 0.7
⇒ =V g
32(n - 1) or IE [ 19.5 - 103 ] = 0.7
V0
or V = or IE » 0.7 mA
[ 32(n - 1)]1/g
Input impedance,
V0 V0
= = Z in = R1 || R2 || b (re + RE )
[ 32(n - 1)] 3/ 5 8(n - 1)3/ 5
where, R1 = 12 kW
Volume of the gas in the first cell,
V R2 = 3.9 kW
V = 0 (n - 1)-3/ 5
8 RE = 1k W
Solved Paper 2013 ●
55

26 mV 26 mV 3m æ r ö é m ù
re = = = 37.14 W = ´ ( 2pr dr ) ç1 - ÷ êëQ r 0 = pR 2 úû
IE 0.7 mA R2 è Rø
RE + re = 1k W + 0.037 k W = 1.037 k W 6m æ r ö
= 2 rdr ´ ç1 - ÷
b (re + RE ) = 200 (1.037) k W = 207.4 k W R è Rø
Equivalent resistance of Moment of inertia of the elementary ring about the axis is
R1R2 12 ´ 3.9 dI = (dm ) (r 2 )
R1 || R2 = = = 2.94 k W
R1 + R2 15.9 6m æ r ö
= (r dr ) (r 2 ) ç1 - ÷
Equivalent resistance of R2 è Rø
R1 || R2 || b (re + RE ) = 2.94 || 207.4 k W 6m é R Rr 4dr ù
2.94 ´ 207.4 = 2 ê ò r 3dr - ò ú
= kW R ë 0 0 R
û
2.94 + 207.4
Moment of inertia of the disc is
⇒ Z in = 2.902 kW R 2m 3
R 6m R
I = ò dI = ò r dr - 3 ò r 4dr
Output impedance, 0 0 R 2
R 0
Z out = RC || RL 6m R4 6m R5
( 2.2 k W ) (5 k W ) = 2 ´ - 3 ´
= R 4 R 5
2.2 k W + 5 k W
3mR 2 6mR 2
= 1.52 k W = -
2 5
Voltage gain of the circuit is æ 3 6 ö mR 2
r R || RL = mR 2 ç - ÷ = 3
AV = C = C è 2 5ø 10
re 0.037 k W
3 2
1.52 k W ⇒ I= mR
= = 41.29 » 42 10
0.037 k W
30. Given that speed of sound,v = aT
29. Consider an elementary ring of radius r and thickness dr. Differentiating both sides with respect to t, we have
dv dT
I \ =a
dt dt
Axis
dv dT
dr or v =a
r dt dt
æ dT ö
or vdv = a ç ÷ dx
è dt ø
or vdv (dt ) = a (dT )(dx ) …(i)
Taking double integrations on both sides of Eq. (i), we get
According to the question, mass of the disc is m. v2 t T2 L

\
R
m = ò r (r )( 2prdr )
òv v dv ò0 dt = òT
1 1
adT ò dx
0
0
æv 2 öv 2
R æ r ö or ç ÷ (t ) = a (T2 - T1)(L)
= ò r 0 ç1- ÷ ( 2prdr )
0 è Rø è 2 øv
1
é R R 2 pr 2dr ù év 22 - v12 ù
= r 0 ê ò 2prdr - ò ú or ê ú (t ) = a (T2 - T1)(L) …(ii)
0 0
ë R û ë 2 û
éR2 R2 ù Putting, v1 = aT1 and v 2 = aT2 in Eq. (ii), we get
= 2pr 0 ê - ú
ë 2 3 û a 2(T22 - T12 )
(t ) = a (T2 - T1)(L)
é 1ù 2
= 2 pR 2 ´ r 0 ê ú
ë 6û a 2 (T2 + T1)(T2 - T1)
or (t ) = a (T2 - T1) L
3m 2
\ r0 =
pR 2 a 2 (T1 + T2 )
or (t ) = aL
Mass of the elementary ring, 2
æ r ö 2L
dm = r (r ) ( 2pr dr ) = r 0 ç1 - ÷ ´ ( 2pr dr ) or t=
è Rø a (T1 + T2 )
Solved Paper
2012
IITJAM Physics
MM : 300 Time : 3 hrs

n
In this question paper, there are total of 25 questions carrying 300 marks of 3 hours duration.
n
Questions 1-15 (Objective questions) carry 6 marks each and questions 16-25 (Subjective questions) carry
21 marks each.
n
Each objective question has four choices for its answer i.e. (a), (b), (c) and (d). Only one of them is the
correct answer.
n
There will be negative marking for wrong answers to objective questions. The following marking scheme
for objective questions shall be used:
(a) For each correct answer, you will be awarded 6 (Six) marks.
(b) For each wrong answer, you will be awarded –2 (Negative two) mark.
(c) Multiple answers to a question will be treated as a wrong answer.
(d) For each un-attempted question, you will be awarded 0 (Zero) mark.
n
Do not write more than one answer for the same question. In case you attempt a subjective question
more than once, please cancel the answer(s) you consider wrong. Otherwise, the answer appearing last
only will be evaluated.
n Clip board, log tables, slide rule, calculator, cellular phone and electronic gadgets in any form are not
allowed.

∂f df (x, t ) dx
1. Given a function f (x, t ) of both position x and time t, the value of (where f = ,x = )
∂x dt dt
is
∂ 2f ∂f f df
(a) (b) (c) (d)
∂x 2 ∂x x dx

2. If F is a constant vector and r is the position vector, then ∇ ( F ⋅ r ) would be


(a) (∇ ⋅ r ) F (b) F
(c) (∇ ⋅ F) r (d)| r | F
Solved Paper 2012 ●
57

3. Three masses m, 2 m and 3m are moving in XY-plane with speeds 3u , 2 u and u, respectively
as shown in the figure. The three masses collide at the same time at P and stick together.
The velocity of the resulting mass would be
Y
2m

2u
60°
m X
3u P 60°
u
3m

u u u u
(a) ( x$ + 3 y$ ) (b) ( x$ − 3 y$ ) (c) ( − x$ + 3 y$ ) (d) ( − x$ − 3 y$ )
12 12 12 12

4. The figure shows a thin square sheet of metal of uniform density along with possible choices
for a set of principal axes (indicated by dashed lines) of the moment of inertia lying in the
plane of the sheet. The correct choice(s) for the principal axes would be

p q r

(a) p, q and r (b) p and r (c) p and q (d) p only

5. A lightly damped harmonic oscillator loses energy at the rate of 1% per minute. The decrease
in amplitude of the oscillator per minute will be closest to
(a) 0.5% (b) 1% (c) 1.5% (d) 2%

6. A parallel plate air-gap capacitor is made up of two plates of area 10 cm 2 each kept at a
distance of 0.88 mm. A sine wave of amplitude 10 V and frequency 50 Hz is applied across
the capacitor as shown in the figure. The amplitude of the displacement current density
(in m A/m 2 ) between the plates will be closest to

(a) 0.03 (b) 0.30 (c) 3.00 (d) 30.00

7. A tiny dust particle of mass 1.4 × 10 −11 kg is floating in air at 300 K. Ignoring gravity, its rms
speed (in µms −1) due to random collisions with air molecules will be closest to
(a) 0.3 (b) 3 (c) 30 (d) 300

8. When the temperature of a black body is doubled, the maximum value of its spectral energy
density with respect to that at initial temperature, would become
1
(a) times (b) 8 times (c) 16 times (d) 32 times
16

9. Light takes 4 hours to cover the distance from Sun to Neptune. If you travel in a spaceship at
a speed of 0.99c (where, c is the speed of light in vacuum), the time (in minutes) required to
cover the same distance measured with a clock on the spaceship will be approximately
(a) 34 (b) 56 (c) 85 (d) 144
58 IIT JAM Physics Solved Papers & Practice Sets

60 60
10. Co is a radioactive nucleus of half-life 2 ln 2 × 10 8 s. The activity of 10g of Co in
27 27
disintegrations per second is
1 1
(a) × 1010 (b) 5 × 1010 (c) × 1014 (d) 5 × 1014
5 5

11. An X-ray beam of wavelength 1.54 Å is diffracted from the (110) planes of a solid with a cubic
lattice of lattice constant 3.08 Å. The first order Bragg diffraction occurs at
 1  1   1  1
(a) sin−1   (b) sin−1   (c) sin−1   (d) sin−1  
 4 2 2  2  2

12. The Boolean expression P + PQ, where P and Q are the inputs to a circuit, represents the
following logic gate.
(a) AND (b) NAND (c) NOT (d) OR

13. Group I contains x and y- components of the electric field and Group II contains the type of
polarisation of light.
Group I Group II
E
P. Ex = 0 cos (ωt + kz ), Ey = E0 sin (ωt + kz ) 1. Linearly polarised
2
Q. Ex = E0 sin (ωt + kz ), Ey = E0 sin (ωt + kz ) 2. Circularly polarised

R. Ex = E1 sin (ωt + kz ), Ey = E2 sin (ωt + kz ) 3. Unpolarised


 π
S. Ex = E0 sin (ωt + kz ), Ey = E0 sin ωt + kz +  4. Elliptically polarised
 4

The correct set of matches is


(a) P-4; Q-2; R-4; S-1 (b) P-1; Q-3; R-1; S-4 (c) P-4; Q-2; R-1; S-4 (d) P-3; Q-1; R-3; S-2

14. For a liquid to vapour phase transition at Ttr , which of the following plots between specific
Gibbs free energy G and temperature T is correct?
G G G G

Vapour Vapour Vapour


(a) Liquid (b) Liquid (c) Liquid (d) Liquid
Vapour
T T T T
Ttr Ttr Ttr Ttr

15. A segment of a circular wire of radius R, extending from θ = 0 to π / 2 , carries a constant linear
charge density λ. The electric field at origin O is
Y

R
θ
O X

λ λ  1 1 
(a) ( − x$ − y$ ) (b) − x$ − y$ 
4πε 0R 4πε 0R  2 2 
λ  1 1 
(c)  − x$ − y$  (d) 0
4πε 0R  2 2 
Solved Paper 2012 ●
59

16. The p-V diagram below represents an ideal monoatomic gas cycle for 1 mole of a gas.
In terms of the gas constant R, calculate the temperatures at the points J, K, L and M.
Also, calculate the heat rejected and heat absorbed during the cycle and the efficiency of
the cycle.
p (kPa)

K L
160

80
J M

O V (m3)
0.03 0.10

17. 2 kg of a liquid (specific heat = 2000 JK −1 kg −1, independent of temperature) is heated


from 200 K to 400 K by either of the following two processes P1 and P 2 :
P1 : bringing it in contact with a reservoir at 400 K.
P 2 : bringing it first in contact with a reservoir at 300 K till equilibrium is reached and then
bringing it in contact with another reservoir at 400 K.
Calculate the change in the entropy of the liquid and that of the universe in processes P1 and
P 2 . Neglect any change in volume of the liquid.

18. a. Two concentric conducting spherical shells of radii R1 and R 2 (R1 < R 2 ) are maintained
at potentials V1 and V 2 , respectively. Find the potential and electric field in the region
R1 < r < R 2 .
b. A polarised dielectric cube of side l is kept on the XY-plane as shown. If the
polarisation in the cube is P = kx x$ , where k is a positive constant, then find all the
bound surface charge densities and volume charge density.
Z

G
F
D C
P E
O Y

A B
X

19. A water cannon starts shooting a jet of water horizontally at t = 0, into a heavy trolley of mass M
placed on a horizontal ground. The nozzle diameter of the water cannon is d, the density of
water is ρ and the speed of water coming out of the nozzle is u. Find the speed of the trolley as a
function of time. Assume that all the water from the jet is collected in the trolley. Neglect all
frictional losses.

Water cannon
60 IIT JAM Physics Solved Papers & Practice Sets

20. A long straight solenoid of radius R and n turns per unit length carries a current I = αt, where α
is a constant, t is time and remains finite. The axis of the solenoid is along the Z-axis. Find the
magnetic field, electric field and the Poynting vector inside the solenoid. Show these vectors at
some instant t1 at any point (i) on the axis of the solenoid and (ii) at a distance r (< R ) from the
axis.
2 1
21. In the operational amplifier circuit shown below, input voltages V1 = V and V 2 = V are
3 2
applied.
R4=100 kΩ

R2=50 kΩ +15 V
P
V2 –
vo
Q +
V1
R1=50 kΩ –15 V
R3=50 kΩ

a. Determine the current flowing through resistance R 4 and the output voltage Vo .
b. In the above circuit, if V1 is grounded and square pulses of peak voltage 1 V and
frequency 100 Hz are applied at V 2 , determine the voltage and phase change of the
output pulses.

22. A particle of mass m is confined in a potential box of sides L x , L y and L z as shown in the
..
figure. By solving the Schrodinger equation of the particle, find its eigen functions and energy
eigen values.
Z

(0, 0, Lz)

Y
(0,0,0) (0, Ly, 0)

(Lx, 0, 0)
X

23. A particle of mass m and charge q moves in the presence of a time-independent magnetic
field B (r ). Set up Newton’s equation of motion for the particle.
Since, for a magnetic field ∇ ⋅ B = 0, one can write B = ∇ × A, where A is a function of position.
dA d
Calculate as seen by the moving particle. Show that (p + qA), where p is the momentum
dt dt
of the particle, can be written as q times the gradient of a function.

24. Consider a periodic function f (x ) with periodicity 2π,


c, 0 ≤ x < π
f (x ) = 
 0, π ≤ x < 2π
where, c is a constant.
a. Expand f (x ) in a Fourier series.
b. From the result obtained in (i), show that
π 1 1 1 1
= 1− + − + − K
4 3 5 7 9
Solved Paper 2012 ●
61

25. Two orthogonally polarised beams (each of wavelength 0.5 µm and with polarisation marked in
the figure) are incident on a two prisms assembly and emerge along x-direction as shown in
the figure. The prisms are of identical material and no and ne are the refractive indices of the
sin θ 3+1
o -ray and e -ray, respectively. Use sin φ = and no = .
3 4

Optic axis Y
X
Beam 2
φ P φ P
θ θ

30° 30° uy
Beam 1

Optic axis
(a) (b)

a. Find the value of θ and ne .


b. If the right hand side prism starts sliding down with the vertical component of the
velocity u y = 1µ ms −1, what would be the minimum time after which the state of
polarisation of the emergent beam would repeat itself?

Answers
1. (a) 2. (c) 3. (d) 4. (a) 5. (b) 6. (a) 7. (c) 8. (c) 9. (a) 10. (d)
11. (b) 12. (d) 13. (b) 14. (c) 15. (a)
Answers with Explanations
1. (a) Given, f (x , t ) is a function of position of x and t, These axes are special because when a body rotated
∂f (x , t ) ∂x about one of them, the angular momentum vector
f = ,x = becomes parallel to the angular velocity ω.
dt dt
Hence, in the given question, all the given axes in
 df 
∂  ( p, q and r) are correct for the principal axes.
∂f  dt  ∂ 2f
= =
∂x  dx  ∂x 2 5. (b) Energy (E) of a harmonic oscillator is proportional to
∂ 
 dt  square of amplitude (a)
i.e. E ∝ a2
2. (c) ∇ ( F ⋅ r ) = ( ∇ ⋅ F) r ∆E ∆a
∴ =2
3. (d) Applying conservation of linear momentum along X-axis E a
and Y-axis. ⇒ Rate of decrease of amplitude
Initial momentum of the three particles ∆a 1 ∆E 1
= × = × (1%) = 0.5%
= 3 mu x$ + 2m [ − 2u cos 60° x$ − 2u sin 60° y$ ] a 2 E 2
+ 3 m [ − u cos 60° x$ + u sin 60° y$ ]
6. (a) Displacement current density (JD ) can be given as
Y ∂D
JD =
∂t
where, D is electric displacement field
D = ε0 E
m 60°
X ∂E ∂ V   V
60° 60° ∴ JD = ε 0 = ε0 QE =
∂t d   d 
3u
∂t
u ε ∂
2m JD = 0 (V ) …(i)
d ∂f
2u 3m
Assuming V = V0 sin ω t
∂V
4mu  3 ⇒ = V0 ω cos ωt
pi = 3mu x$ + ( − x$ ) − 4mu   y$ ∂f
2  2 ε
∴ JD = 0 [V0ω cos ωt ]
3mu $ 3 3mu $ d
− x+ y
2 2 V0ω ε 0 
=  cos ωt
− mu $ 3 mu $  d 
pi = x− y
2 2 ∴ Amplitude of displacement current
Final momentum, pf = 6 m ( v x + v y ) ε ωV
( JD )max = 0 0
Using conservation principle of momentum d
pf = pi where, ε 0 = 8.85 × 10−12 C2/ N- m2
mu $ V0 = 10 V,ω = 2πf = 2π × 50 = 100π,
⇒ 6m ( v x + v y ) = − ( x + 3 y$ )
2
and d = 0.88 mm = 0.88 × 10−13m
u
or v= ( − x$ − 3 y$ ) 8.85 × 10−12 × 100π × 10
12 ⇒ ( JD )max =
0.88 × 10−3
4. (a) Principal axis of rotation Moment of inertia tensor 885
takes the form of a real symmetric three-dimensional = × 10−6 × π
88
matrix. It therefore follows from the matrix theory that ≈ 10−5 π = 3.14 × 10−2mA / m2
the moment of inertia tensor possesses three mutually
≈ 0.0314 mA / m2
orthogonal eigen vectors which are associated with
three real eigen vectors. Let the i th eigen vector be 3 kT
7. (c) Rms speed of a particle is given byv rms =
denoted by ωi , and the i th eigen value λ i . It then follows m
that where, k is the Boltzmann constant
I ω$ i = λ i ω$ i for i = 1, 3
. × 10− 23 m2 kgs−2 K −1)
(k = 138
The directions of the three mutually orthogonal unit
vectors ω$ i define the three so called principal axes of T is the temperature (in kelvin) and m is the mass of the
rotation of rigid body under investigation. particle.
Solved Paper 2012 ●
63

3 × 1.38 × 10− 23 × 300  m NA 


v rms = A=λ 
1.4 × 10− 11 kg  M 

3 × 1.38 × 3 1 10 × 6.023 × 1023


v rms = × 10− 10 = 8.87 × 10− 10 ⇒ A= 8
×
1.4 2 × 10 60
60
v rms = 2.97 × 10− 5 ~
− 30 µms−1. [Molar mass of 27 Co, M = 60 g/mole]
15
A = 0.50 × 10
8. (c) The spectral energy density per unit solid angle of
radiation is just the specific intensity IV divided by the A = 5 × 1014 disintegrations per second.
flow rate c. For isotropic radiation, the spectral energy 11. (b) Bragg’s law When X-rays are scattered from a crystal
I
density is uV = 4π V , the total radiation energy density lattice, peaks of scattered intensity are observed which
c correspond to the following conditions.

 4σ  (i) The angle of incidence = angle of scattering
u = ∫ uV dV of black body radiation is u =   T 4
 c  (ii) The path length difference is equal to an integer
0
 4σ  number of wavelength.
The quantity   ≈ 7.5657 × 10− 15 erg cm− 3 K − 4 is
 c  The condition for maximum intensity contained in
called the radiation constant. Bragg’s law above allows us to calculate details about
the crystal structure or if the crystal structure is known to
⇒ u ∝T 4
determine the wavelength of the X-rays incident upon
Hence, when the temperature of black body is doubled, the crystal
the maximum value of its spectral energy density would
⇒ nλ = 2d sin θn
become 16 times.
where, d is the lattice spacing.
9. (a) Here, t = 4 hours According to the given equation,
= 4 × 60 min = 240 min λ = 1.54 × 10− 10 m
and v = 0.99c n =1
where, c is the speed of light in vacuum.  λ 
As time dilation is given by ⇒ sin θ =   ... (i)
 2d 
t0
t= Miller indices of the given cubic lattice is 110.
v2 a
1− 2 ⇒ d =
c h + k 2 + l2
2

v2 where, a is the lattice spacing of the cubic crystal and h,


⇒ t 0 = t 1−
c2 k and l are the Miller indices.
1/ 2 a
 ( 0.99c)2  ⇒ d = ... (ii)
= 240 × 1−
 c 2  2
From Eqs. (i) and (ii), we get
= 240 × 0.1411 λ
∴ t 0 ≈ 34 min sin θ =
2a / 2
10. (d) Given, T1/ 2 = 2 (In 2) × 108 s o
2λ 2 × 1.54 A
In 2 1 = =
Decay constant, λ = = s− 1 2a o
T1/ 2 2 × 108 2 × 3.08 A
Activity The activity of a sample is the average number 1
sin θ =
of disintegrations per second, its unit is the becquerel 2 2
( Bq). One becquerel is one decay per second. It can be  1 
θ = sin−1 .
calculated by using the relation  2 2
A = λN
where, A = activity in becquerel
12. (d) The given expression is
– –
N = the number of undecay nuclei Y = P + P Q = (P + P )(P + Q )
λ = decay constant ( s− 1) = PP + PQ + PP + PQ
– –

According to the given question, – –


P + PQ + P Q = P + Q (P + P )
m NA
N=
M ⇒ Y =P +Q
23 Nuclei ∴ Expression Y represents the OR gate.
NA = 6.023 × 10
Mole
64 IIT JAM Physics Solved Papers & Practice Sets

13. (b) λ
and dE( y ) = sin θ. dθ
4πε 0R
Ex =
E0
cos (ωt + kz ) Linearly polarised
P. π /2 π /2
2 λ λ
Ey = E0 sin (ωt + kz )
E( x ) =
4πε 0R ∫ cos θ. dθ and E( y ) = 4πε 0R ∫ sin θ. dθ
0 0
λ λ
Q. Ex = E0 sin (ωt + kz ) Unpolarised light E( x ) = [sin θ]π0 / 2 and E( y ) = [ − cos θ]π0 / 2
4πε 0R 4πε 0R
Ey = E0 sin (ωt + kz ) λ λ
E( x ) = and E( y ) =
R. Ex = E1 sin (ωt + kz ) Linearly polarised 4πε 0R 4πε 0R
Ey = E2 sin (ωt + kz ) Electric field at origin, E = − E $i − E $j
x y

S. Ex = E0 sin (ωt + kz ) Elliptically polarised or E = − (Ex )x$ − E( y )y$


λ
Ey = E0 sin (ωt + kz + π /4 ) ⇒ E= ( − x$ − y$ )
4πε 0 R

16. Using ideal gas equation, pV = nRT


14. (c) Gibbs free energy
For 1 mole of the gas, n = 1
G = U − TS work (pV) to give the system final volumeV
at constant pressure. ∴ pV = RT
G Temperature at each points can be calculated, using
pV
T =
R
pV 80 × 103 × 10.03
TJ = J J =
Liquid R 8.3
Vapour 800 × 3
= = 2.892 × 102 K ≈ 290 K
8.3
pV 160 × 103( 0.03)
TK = K K =
T R 8.3
290
15. (a) Linear charge density of the segment of circular wire is λ. = × 160 = 580 K
80
Y
pV 580
TL = L L = × 0.01
R 0.03
580
= = 193.33 K
dx 3
p V 580 1
TM = M M = × × 80
R 3 160
290
= = 96.66 K ≈ 97 K
dθ 3
dE cos θ θ Work done during the cycle
X
θ O W = area ( c JKLM ) = ( 0.07)( 80 × 103 )
dE sin θ
dE = 5.60 × 103 = 5.6 × 103 = 5600J
Change in internal energy during the cycle
Charge on the element of length dx , dQ = λdx . ∆Q = ∆U + ∆W
dx ⇒ ∆Q = 0 + W = 5.65
dθ=
R Heat associated with each path can be calculated as below
⇒ dx = R d θ 3R 3R
CV ∆TJK = × (TK − TJ ) = × (580 − 290)
⇒ dQ = λR ⋅ d θ 2 2
dE ( x ) = dE cos θ 3R
∆ QKL = × 290 = 8.3 × 3 × 145 = 435 × 8.3 J
and dE ( y ) = dE sin θ 2
= 363.25 J = 3.6 kJ
dQ λR d θ λd θ
and dE = 2
= 2
= 5R 5R  580 
4πε 0R 4πε 0R 4 πε 0R Cp ∆TKL = × (TL − TK ) = × − 580
2 2  2 
λ
⇒ dE( x ) = cos θ. d θ 5R 2
4πε 0R =− × × 580 = 966.66 = − 8.02 kJ
2 3
Solved Paper 2012 ●
65

∆QLM = CV (TM − TL ) 18. a. Electric potential at point P due to shell A


3R  290 580 k × qA
= × − VA =
 r
2  3 3  B
3R  −290
= ×  = −145 × 8.3 J
2  3 
A
= −1203.55 = 1.2 kJ
∆QMJ = Cp (TJ − TM ) r
P R1
5R  290 5R 2
= ×  290 −  = × 290 ×
2  3  2 3
= 483.33 × 8.3 J = 4.03 kJ
R2
∴ Heat absorbed during the cycle
∆Q a = 3.6 kJ + 4.03 kJ = 7.63 kJ
Heat rejected during the cycle V × R1 1
=k × 1 ×
∆Qr = 1.2 kJ + 8.02 kJ = 9.22 kJ  k  r
∴Efficiency of the cycle R 
= V1 1
Work done during the cycle  r 
η=
Heat absorbed during the cycle Electric potential at point P due to shell B is VB = V2
W 5.6 × 103 ∴ Net potential at point P is
= =
∆Q a 7.6 × 103 R 
VP = VA + VB = V1 1 + V2
⇒ η = 0.736 ≈ 74%  r 

17. For an ideal liquid, change in volume, dV = 0 Similar electric field at point P
VR VR
dU EP = EA + EB = 1 2 1 + 0 = 1 2 1
Using entropy, dS = and specific heat at constant volume r r
T
dU b. Bound volume charge density
CV =
dT
ρb = − [ ∇ ⋅ p]
dT
⇒ dS = CV
T 1 ∂
=− (x 2Kx ) = − 3K [Q P = Kx ]
2 2 dT x 2 ∂x
⇒ ∫1dS = ∫1CV T
As the bound volume charge is constant, the total bound
T  volume charge in the cube is equal to product charge
⇒ S 2 − S1 = CV loge  2 
 T1  density and the volume.
For an isothermal process, change in entropy ⇒ qy 3 = − 3Kl 3
q q ⇒ σP = p ⋅ n$ = Kx ⋅ n$
S1 = rev = rev
T1 300 K 1
⇒ x ⋅ n$ = = x
q q 2
and S 2 = rev = rev 1
T2 400 K Surface charge density, σP = K x ⋅ n$ = Kl
2
Using ∆U + ∆KE + ∆PE = Q − W
1  2 3
 1 1 q surface =  Kl ( 6l ) = 3Kl
⇒ ∆S = − Q  −  2 
Th Tc 
q total = qV + q surface = − 3 Kl 3 + 3Kl 3 = 0
 1 1 
= − 2000 − =5 19. Let mass of trolley at t = 0 is M.
 400 200 
3
Volume ⇒ qV = − 3Kr Water cannon
⇒ σP = p ⋅ n$ = Kx ⋅ n$
1
⇒ x ⋅ n$ = r cos θ = x
2
1
σP = Kl The water coming out through cannon at the speed of u.
2
Density of water = ρ and diameter of cannon = d
1 
q surface =  Kr  ( 6l 2 ) = 3Kl 3  d 2
2  ∴The area of cannon outlet =  π 
q total = qV + q surface = − 3 Kl + 3Kl = 0  4
66 IIT JAM Physics Solved Papers & Practice Sets

∴ Volume of water coming out after interval of time t, 1


and induced current, I = − (µ 0πa 2n )
πd 2 R
Vt = × ut ∂φ
4
 πd 2u 
⇒ ∫ E dl = − dt
Vt =  t … (i) ⇒ E ( 2πa ) = − µ 0πa 2n
 4 
−1  dI 
∴ Mass of water ejecting out, m(t ) = ρv (t ) E= µ 0 an   φ$
2  dt 
 ρπd 2u 
mt =  t … (ii) The magnetic field at the axis of thin solenoid at a distance z,
 4 
µ I b2
dm  ρπd 2  B = 0r z$
Now, =  … (iii) 2 (b 2 + z 2 )3
dt  4 
The Poynting vector at the surface at a distance z
The momentum of water changes in striking to wall of
trolley. ( E × B)
S=
d dm du µ0
(m × u ) = u +m
dt dt dt 1  − µ 0an dI   µ 0Ir 
b2
 ( φ$ × z)
dm =   $
= +m×0 µ0  2 dt   2 2
(b + z )  2 3
dt
=u
dm 1 dI s ab 2n
=− µ 0 Ir r$
dt 4 dt (b 2 + z 2 )3/ 2
dp
∴ According to Newton's second law, F = V0 − V0
dt 21. a. According to given circuit, I 4 =
dm R4
∴ M′ a = u
dt Potential at P, VP = Potential at Q, VQ
u dm   R3  V1 (50 kΩ )
⇒ a= …(iv) ⇒ VQ = V1   =
M ′  dt   R1 + R2  (50 kΩ + 50 kΩ )
From Eqs. (iii) and (iv), we get VQ = 0.5 kΩV1
u dm  R 
a= , and V0 = V2  4  = V2 × 2 kΩ
M ′  dt   R2 
where M′ is changed mass of trolley,
VP − V0 [V1 ( 0.5 kΩ ) − 2 kΩV2]
i.e. [M ′ = M + m] Now, I4 = =
R4 1000 kΩ
u  ρ πd 2   R4 
a=  4 .u Vout =   (V1 − V2 )
M +m    R2 
2 2
u d πρ R 
a= If VP = 0, then Vout (P ) = − V2  4 
4 [M + m]  R2 
 πd 2u  ⇒ V0 (P ) = − V2 ( 2 kΩ )
From Eq. (ii), we get m = ρ t
 4  If VQ = 0, then
VR  R 
d u 2d 2π ρ t V0 (Q ) = 1 3 R2 + 4 
= v (t ) = R1 + R2  R2 
dt  ρ π d2 u 
4 M + t V1 × 0.5 kΩ
 4  or V0 (Q ) = = V1 (1. 5 kΩ )
3 kΩ
 ρπd u t 
2 2
v (t ) =  ⇒ Vout = V0 (P ) + V0 (Q )
2 
 4M + ρπd ut  = ( −2 kΩ )V2 + (1.5 kΩ )V1
∴ The speed of trolley as function of t b. Given that V1 = 0
udt R4
⇒ v (t ) = ∫
 4M  R2 = 50 kΩ
 + 1 +15 V
 ρπd 2ut  V2 VP

VQ + V0
V1 = 0 –15 V
20. Let B be the magnetic field inside the solenoid. R1 = 50 kΩ
Magnetic flux, φB = B A = B πr 2 = µ 0 nI πr 2 R3 = 50 kΩ

Induced emf, e = − B
dt
Solved Paper 2012 ●
67

Clearly from the circuit diagram Now, velocity and corresponding kinetic energy can be
VQ = 0 V resolved in three components along x , y and z-axes such
Output voltage, that
R E = Ex + Ey + Ez ...(v)
V0 = − 4 V2
R2 Applying variable separable technique, Eq. (iv) separates
100 kΩ into three independent equations.
=− × (V2 )
50 kΩ − h2  ∂ 2X (x )
  = Ex X (x )
= − 2V2 2m  ∂x 2 
∴ Magnitude of the output voltage − h2  ∂ 2Y (y )
  = EyY (y )
| V0 | = 2 | V2 | = 2 (1V ) = 2 V 2m  ∂y 2 
The output voltage is out of phase with input voltage. − h2  ∂ 2Z ( z )
and  2
 = Ez ...(vi)
22. Consider a particle of mass m and energy E constrained to 2m  ∂z 
move in a three-dimensional rectangular potential well of The solutions of these equations are
sides Lx , Ly and Lz as given.
h2k x 2
Let there be no force acting on the particle in the box. The X (x ) = Ax sin(k x x ) + Bx cos(k x x ) Ex =
2m
potential is given by 2 2
h ky
V (x , y , z ) = 0 Y (y ) = Ay sin(k y y ) + By cos(k y y ) Ey =
2m
for its position at points x , y , z such that
h2k z 2
0 < x < Lx , 0 < y < Ly , 0 < z < Lz Z ( z ) = Az sin(k z z ) + Bz cos(k z z ) Ez =
2m
and V (x , y , z ) = ∞, outside the box.
Applying boundary conditions
Total kinetic energy is
2 2 2
(A) ψ(x = 0) = 0 ⇒ Bx = 0
P ⋅ P Px + Py + Pz ψ(x = Lx ) = 0 ⇒ k x Lx = n x π ...(vii (a))
T = =
2m 2m (B) ψ(y = 0) = 0 ⇒ By = 0
h2  ∂ 2 ∂2 ∂2  h2 2 ψ(y = Ly ) = 0 ⇒ k y Ly = n y π ...(vii (b))
or T =−  2+ +  = − ∇ ...(i)
2m  ∂x ∂y 2 ∂z 2  2m (C) ψ( z = 0) = 0 ⇒ Bz = 0
ψ( z = Lz ) = 0 ⇒ k z Lz = n z π ...(vii (c))
where, ∇ 2 is Laplacian operator.
The resulting x-components of eigen functions are of the
h2 2

∇ ψ =Eψ form
2m
2  nπ x 
For the motion of particle inside the box, time independent ψn = sin   , n = 1, 2, 3, 4
L  L 
Schrödinger equation is given by
h2  ∂ 2 ∂2 ∂2  Solutions for y and z- components have same form.
−  2+ +  ψ (x , y , z ) = Eψ (x , y , z ) ...(ii)
2m  ∂x ∂y 2 ∂z 2  ∴ Total normalized eigen functions for motion of particle in
a box are
Eq. (ii) is a partial differential equation.
8
By applying variable separation technique, we have ψ(x , y , z ) = sin(k x x ) sin(k y y ) sin(k z z )
Lx Ly Lz
ψ = X (x ).Y (y ). Z ( z ) ...(iii)
As H = Hx + Hy + Hz πn x πn y πn z
kx = , ky = , kz = ...(viii)
So, [Hx + Hy + Hz ] ψ (x , y , z ) = Eψ (x , y , z ) Lx Ly Lz
On substituting for ψ, equation becomes where, n x ( = n y = n z ) = 1, 2, 3, 4 ...(ix)
2mE Eigen functions are zero outside the box.
x ′′ yz + xy ′′z + xyz ′′ + 2 × yz = 0
h The eigen values of energy E are
where x ′′, y ′′, z ′′ are ordinary derivatives as each of 2 2 2
h2(k x + y y + k z )
functions x , y , z is a function of one variable only. E=
2m
On dividing above equation by xyz, we get 2 2 2
x ′′ y ′′ z ′′ 2m h π  nx ny nz 
2 2
E=  + +  ...(x)
+ + =− 2 E ...(iv) 2m  Lx Ly Lz 
2 2 2
x y z h
Each term of Eq. (iv) depends on different variable x , y or z These eigen values of energy E are called as energy levels
and three variables x , y and z are independent. of particle. They form discrete energy spectrum. The
2mE integers n x , n y and n z are called as the quantum number,
Here, = k 2 is a constant for a particular value of T . which specify each stationary state.
h2
68 IIT JAM Physics Solved Papers & Practice Sets

23. We assume that magnetic field B. that exists at the origin in the So, equation of motion under these approximations
guiding centre coordinate system is in z-direction. can be written as
B ( 0, 0, 0) = B0 = B0 z$ m
dv
= q( v × B0 ) + q v( 0) × [r( 0) ⋅ ∇ B] ...(ix)
Let r be momentary position vector of the particle. dt
Magnetic field (near the origin) can be expressed by Taylor’s The second term of Eq. (ix) corresponds to force
expansion about the origin. term such that
dv
B (r ) = B0 + r.( ∇B) + ... ...(i) m = q ( v × B) + F …(x)
dt
We also have
 ∂B  Here, F is not constant as it depends on the
Bx (x1, 0, 0) = Bx ( 0, 0, 0) +  x  x1 instantaneous position of particle.
 ∂x 
 ∂By  Therefore, small oscillation occurs during one
By ( 0, y1, 0) = By ( 0, 0, 0) +   y1 gyration period because of its dependency on r.
 ∂y 
However, for smooth motion of guiding centre, we
where, derivatives of B are to be calculated at the origin. shall eliminate small oscillations by averaging force
Since, we are assuming spatial variation of B over a distance of term over one gyration period.
≈ rc << B, higher order terms in the expansion of Eq. (i) can be F× B
neglected, such that Using VF = in Eq. (ix), we get
qB 2
r ⋅ ( ∇B) << B0 ...(ii) dv
m = q( v × B0 ) + qv × [r ⋅ ( ∇ B)]
The first order terms can be written as dt
r ⋅ ( ∇B) = (r ⋅ B) B = v ( 0) × B0 + v (1) × B0
 ∂ ∂ ∂ For unifrom field B,
= ( x$ x + y$ y + z$ z ) ⋅  x$ + y$ + z$ B
 ∂x ∂y ∂z  v (1) << v ( 0)
 ∂ ∂ ∂ $
= x +y +z  ( xBx + y$ By + z$ Bz ) dv ( 0)
 ∂x ∂y ∂z  ∴ m = q ( v( 0) × B0 )
dt
 ∂B ∂Bx ∂Bx  $  ∂By ∂By ∂By 
= x x + y +z  x + x +y +z  y$ 24. a. Given periodic function if
 ∂x ∂y ∂z   ∂x ∂y ∂z 
c 0 ≤ x < π
 ∂Bz ∂Bz ∂Bz  $ f (x ) = 
+ x +y +z z ...(iii) 0 π ≤ x < 2 π
 ∂x ∂y ∂z 
where, c is a constant.
where, partial derivatives are to be calculated at origin.
Now, we can write
As equation of motion is given by
1 2π
dv a 0 = ∫ f (x ) dx
m = q( E + v × B) π 0
dt 1 π 2π
We get for E = 0 = [ ∫ f (x ) dx + ∫ f (x ) dx ]
π 0 π
dv 1 π 2π
m = q[( v × ( B0 + r.( ∇B))] [from Eq. (i)] = [ ∫ c dx + ∫ 0 dx ]
dt π 0 π
dv =c + 0 =c
∴ m = q[( v × ( B0 )] + q v × [(r ⋅ ( ∇B)] ...(iv)
dt 1 2π
an = ∫ f (x ) cos nx dx
In terms of superposition, particle velocity is given by π 0
dr ( 0) dr (1) 1 π 2π
v = v ( 0) + v (1) = + ...(v) = ∫ [f (x ) cos nx dx + ∫ f (x )[cos nx dx ]
dt dt π 0 π

where, v (1) is first order perturbation. 1 π


= [ ∫ c cos nx dx + 0]
π 0
v (1) << v ( 0) ...(vi) c π
= ∫ cos nx dx
where, v 0 is solution of zero order equation. π 0
π
d v( 0) c  cos nx 
m = q( v( 0) × B0 ) ...(vii) = −
dt nπ  n  0
Now, neglecting second order terms, we get [using integration by parts]
v × [r ⋅ ( ∇B)] = [ v( 0) + v(1)] × [(r( 0) + r(1) ) ⋅ ( ∇B)] c  cos nπ + 1
an = −
nπ  
= v( 0) × [(r( 0) ⋅ ( ∇ B)] ...(viii) n
Solved Paper 2012 ●
69

1 2π Now, we can write


π ∫0
bn = f (x ) sin nx dx
x 2b = x 2 − x 4 + x 6 − x 8 + K
1 π 2π
= [ ∫ c sin nx dx + ∫ 0 sin nx dx ] Now, we can write
π 0 π
b + x 2b = 1
1
= [c ( − cos nx )] π0 1
nπ ⇒ b= = 1− x 2 + x 4 − x 6 + K
−c 1+ x 2
= [cos nπ − 1]
nπ Integrating both sides, we get
∞ 1 dx 1 1 2 1 4
a
∴ f (x ) = 0 + ∑ (an cos nx + bn sin nx )
2 n =1
⇒ ∫0 1 + x 2 = ∫0 (1) dx − ∫0x dx + ∫0x dx

∞ 1 1 1
c 1 − cos nπ = 1− + − + K
= + ∑ cos nx 3 5 7
2 n =1 n 2π
π 1 1 1

c ⇒ = 1− + − + K [from Eq. (i)]
+ ∑ nπ (1 − cos nπ ) (sin nx ) 4 3 5 7
n =1
25. a. For the incident beam, we can write
c 2c 2c 1
= + cos x + × 2 cos 3x n 0 sin θi = n1 sin θr ...(i)
2 π π 3
2c 1 where θi is the incident angle and θr is refracted angle.
+ × 2 cos 5x + K + Now, we can have largest value of θ for φ = φC = critical
π 5
angle.
2c 2c
+ sin x + sin 3x + K ∴ sin θr = sin (90°− φ ) = cos φ ...(ii)
π 3π
Now, Eq. (i) implies
d 1
b. As [tan− 1 (x )] = n 0 sin θi = n1 cos φ
dx 1+ x 2
n
dx −1 ⇒ sin θi = 1 cos φ
⇒ ∫ 1 + x 2 = tan x + c n0
When φ = φC , we have θi = φ max
 π
Since, tan ( 0) = 0 and tan   = 1 n
 4 ∴ sin φ max = 1 cos φc
n0
1 dx −1 −1
∫0 x 2 + 1 = tan (1) − tan ( 0) n
As sin φc = 2 , this gives
n1
π π n2 −n2
= −0= … (i) sin θ max = 1 2 2
4 4 n 0
Again, we can write
Putting values of n1, n 2 and n 0, we can find θ max .
1
2
= 1 − x 2 + x 4 − x 6 + x 8 +... b. If the right hand side prism starts sliding down with the
x +1
2 4 6 8
vertical component of the velocity u y = 1µms−1, we have
Let us assume that b = 1 − x + x − x + x to achieve phase matching condition.
Solved Paper
2011
IITJAM Physics
MM : 300 Time : 3 hrs

n
In this question paper, there are total of 25 questions carrying 300 marks of 3 hours duration.
n
Questions 1-15 (Objective questions) carry 6 marks each and questions 16-25 (Subjective questions) carry
21 marks each.
n
Each objective question has four choices for its answer i.e. (a), (b), (c) and (d). Only one of them is the
correct answer.
n
There will be negative marking for wrong answers to objective questions. The following marking scheme
for objective questions shall be used:
(a) For each correct answer, you will be awarded 6 (Six) marks.
(b) For each wrong answer, you will be awarded –2 (Negative two) mark.
(c) Multiple answers to a question will be treated as a wrong answer.
(d) For each un-attempted question, you will be awarded 0 (Zero) mark.
n
Do not write more than one answer for the same question. In case you attempt a subjective question
more than once, please cancel the answer(s) you consider wrong. Otherwise, the answer appearing last
only will be evaluated.
n
Clip board, log tables, slide rule, calculator, cellular phone and electronic gadgets in any form are not
allowed.

B x y
1. The line integral ∫ F ⋅ d l, where F = x$ + y$ , along the semi-circular path
2 2
x + y2
2
A
x +y
as shown in the figure below is
Y

X
A (–1, 0) B (1, 0)

(a) −2 (b) 0 (c) 2 (d) 4


Solved Paper 2011 ●
71

2. Six simple harmonic oscillations each of same frequency and equal amplitude are
superimposed. The phase difference between any two consecutive oscillations, i.e.
φn − φn − 1 = ∆φ is constant, where, φn is the phase of the nth oscillation. If the resultant
amplitude of the superposition is zero, what is the phase difference ∆φ?
π π π
(a) (b) (c) (d) 2π
6 3 2

3. The frequency of K β X-ray of a material can be expressed in terms of the frequencies of


K α and L α as
(a) ν K β = ν K α − ν Lα (b) ν K β = ν K α + ν Lα
(c) ν K β = ν K2 α + ν L2α (d) ν K β = ν K α ⋅ ν Lα

4. The band gap in germanium is ∆E = 0.68 eV. Assuming that the number of hole-electron pairs
is proportional to e −∆E / 2kT , find the percentage increase in the number of charge carriers in
pure germanium as the temperature is increased from 300 K to 320 K.
(a) 117% (b) 107% (c) 97% (d) 127%

5. Which of the following circuits does not satisfy the Boolean expression AB + AB = F ?
A A
B B
(a) F (b) F

A A
B B
(c) F (d) F

6. Light described by the equation E = (90 V/m) [sin (6.28 × 1015 s −1 ) t + sin (12.56 × 1015 s −1 ) t ]
is incident on a metal surface. The work function of the metal is 2.0 eV. Maximum kinetic
energy of the photoelectrons will be
(a) 2.14 eV (b) 4.28 eV (c) 6.28 eV (d) 12.56 eV

7. A gas of molecular mass m is at temperature T. If the gas obeys Maxwell-Boltzmann velocity


distribution, the average speed of molecules is given by
kBT 2kBT 2kBT 8kBT
(a) (b) (c) (d)
m m πm πm

8. Consider free expansion of one mole of an ideal gas in an adiabatic container from volume V1
to V 2 . The entropy change of the gas, calculated by considering a reversible process between
the original state (V1, T ) to the final state (V 2 , T ) where, T is the temperature of the system is
denoted by ∆S1. The corresponding change in the entropy of the surrounding is ∆S 2 . Which of
the following combinations is correct?
(a) ∆S 1 = R ln (V1 / V2 ), ∆S 2 = − R ln (V1 / V2 ) (b) ∆S 1 = − R ln (V1 / V2 ), ∆S 2 = R ln (V1 / V2 )
(c) ∆S 1 = R ln (V2 / V1), ∆S 2 = 0 (d) ∆S 1 = − R ln (V2 / V1), ∆S 2 = 0
72 IIT JAM Physics Solved Papers & Practice Sets

9. Equipotential surfaces corresponding to a particular charge distribution are given by


4x 2 + ( y − 2) 2 + z 2 = Vi , where, the values of Vi are constants. The electric field E at the
origin is
(a) E = 0 (b) E = 2 x$ (c) E = 4y$ (d) E = − 4y$

10. The wave function of a quantum mechanical particle is given by


3 4
ψ (x ) = φ1(x ) + φ2 (x ),
5 5
where, φ1(x ) and φ2 (x ) are eigen functions with corresponding energy eigen values −1eV and
−2 eV, respectively. The energy of the particle in the state ψ is
41 11 36 7
(a) − eV (b) − eV (c) − eV (d) − eV
25 5 25 5

11. A rain drop falling vertically under gravity gathers moisture from the atmosphere at a rate given
dm
by = kt 2 , where m is the instantaneous mass, t is time and k is a constant. The equation
dt
of motion of the rain drop is
dv dm
m +v = mg
dt dt
If the drop starts falling at t = 0, with zero initial velocity and initial mass m 0 (given , m 0 = 2 g,
k = 12 g / s 3 and g = 1000 cm / s 2 ), then velocity (v ) of the drop after one second is
(a) 250 cm/s (b) 500 cm/s (c) 750 cm/s (d) 1000 cm/s
$ such that P
12. Given two (n × n ) matrices P$ and Q $ is skew (anti)-Hermitian.
$ is Hermitian and Q
$ and Q
Which one of the following combinations of P $ is necessarily a Hermitian matrix?
$$
(a) PQ $$
(b) i PQ $ +iQ
(c) P $ $ +Q
(d) P $

13. The correct curve between intensity of magnetisation (I ) and magnetic field (H ) for a
ferromagnetic substance is given by

I I (c) I (d) I
(a) (b)
H H H H

14. A closed Gaussian surface consisting of a hemisphere and a circular disc of radius R, is
placed in a uniform electric field, E as shown in the figure. The circular disc makes an angle
θ = 30° with the vertical. The flux of the electric field vector coming out of the curved surface of
the hemisphere is
E

1 3
(a) πR 2E (b) πR 2E (c) πR 2E (d) 2 πR 2E
2 2
Solved Paper 2011 ●
73

15. In an experiment, the resistance of a rectangular slab of a semiconductor is measured as a


function of temperature. The semiconductor shows a resistance of 300 Ω at 200 K and 2 Ω at
250 K. Its energy band gap is [Given, ln (15 ) = 2.708, ln (10) = 2.303]
(a) 0.138 eV (b) 0.431 eV
(c) 0.690 eV (d) 0.862 eV

16. Consider a vector A = − 4 y x 2 x$ − 3 y 2 y$ . Y


Q (1, 1)


P (0, 1)
a. Calculate the line integral A ⋅ d l from point P → O along
the path P → Q → R → O as shown in the figure.
b. Using Stoke’s theorem appropriately, calculate ∫ A ⋅ d l for
the same path P → Q → R → O. X
O (0, 0) R (1, 0)
17. An infinitely long solid cylindrical conductor of radius r1, carries a J

uniform volume current density J. It is uniformly surrounded by another


coaxial cylinder of a linear magnetic medium with permeability µ, up to r1
Air
radius r 2 as shown in the figure.
r2
a. Determine the magnetic field H in the region, r < r1 and magnetic
induction B in the regions, r1 < r < r 2 and r > r 2 , where r is the
radial distance from the axis of the cylinder.
b. Sketch the variation of H with r in all the three regions.

18. a. Consider heat conduction in a medium. Let T (x, y , z, t ) denotes the temperature of the
medium at time t and position (x, y , z ). Consider a volume V enclosed by a surface S
∂T
inside the medium. The decrease in heat energy per unit volume per unit time is a
∂t
and outward flux of heat per unit area of the surface per unit time is b ∇T , where a
and b are material dependent constants. If there is no generation or loss of heat, then
show that T satisfies the equation
∂T b
= k∇ 2T , with = k.
∂t a
T0
b. Now, consider a thin annular shaped material enclosed between two 2r0
concentric circles of radii r 0 and 2r 0 as shown in the figure. The
temperature is 2T0 at r = r 0 and T0 at r = 2r 0 . Assuming steady state O r0
condition, find T as a function of radial distance r from the centre O,
2T0
for r 0 < r < 2r 0 .

19. An ideal gas reversible engine operates in a closed cycle. The p


p-V diagram is as shown alongside. B ( p2, V1)

a. Find out the efficiency of the reversible engine assuming Adiabatic


both specific heats, C p and CV as constants.
A C ( p1, V2)
b. Identify the thermodynamic processes and draw the corresponding ( p1, V1)
O V
T-S diagram schematically.
74 IIT JAM Physics Solved Papers & Practice Sets

20. a. A solid having a simple cubic structure at room temperature with lattice parameter a
and one valence electron per atom, is assumed to show free electron behaviour.
Calculate the magnitude of the fermi wave vector and the corresponding equivalent
temperature.
b. Find the ratio of the magnitude of the fermi wave vector to the radius of the largest
sphere that can be inscribed within the first Brillouin zone of the solid.

21. For the given circuit using an operational amplifier, the input is a sinusoidal signal of amplitude
V in = 1mV (peak to peak),
100 kΩ

1 µF 10 kΩ
Vin –
10 kΩ
+15 V +

10 kΩ V0
10 kΩ 100 µF

a. What is the lower cut-off frequency at which the gain is down by 3 dB as compared to
the gain at midband? If the bandwidth of the amplifier is 1 MHz for unity gain, what will
be the bandwidth of the given circuit?
b. What is the output voltage (V 0 ) at 15 kHz?

22. A particle of mass m and angular momentum l is moving under the Y


action of a central force f (r ) along a circular path of radius a as shown
in the figure. The force centre O lies on the orbit.
r
a. Given the orbit equation in a central field motion, θ
O X
d 2u m 1 C (a, 0)
+u =− f , where u =
d θ2 l 2u 2 r
Determine the form of the force in terms of l, m, a and r.
b. Calculate the total energy of the particle assuming that the potential energy V (r ) → 0
as r → ∞.

23. A particle of mass m moves in a potential given by Y


V (x ) = ∞ for x < 0 V (x)

= 0 for 0 < x < L


V0
= V 0 for x > L I
V=0 II
a. Write down the general solutions for wave functions in regions I
X
and II, if the energy of the particle E < V 0 . Using appropriate x=0 x=L
boundary conditions, find the equation that relates E to V 0 , m and L.
b. Now, set V 0 = 0 and assume that a beam of particles is incident on the infinite step
potential (from x > 0) with energy E (> 0). Using the general solution for the wave
function, calculate the reflection coefficient.
Solved Paper 2011 ●
75

24. A diffraction grating having N slits, each of width b and period d, is illuminated normally by a
monochromatic plane wave of wavelength λ.
a. Obtain an expression for the highest diffraction order that can be observed. What is the
phase difference between waves from first and N th slit in the highest diffraction order?
b. If alternate slits are covered with a retarder that retards the wave by π, obtain an
expression for the intensity distribution of the Fraunhofer diffraction pattern?

25. Unpolarised light is incident on an air-dielectric interface. The interface of the xy-plane and the
plane of incidence is yz-plane. The electric field of the reflected light is given by

 ik 
E = E 0 x$ exp  ( 3 y + z ) − iωt ,
2 
where, k is the propagation constant in air and ω is the angular frequency of the light. Assume
magnetic permeability µ = µ 0 .
a. Determine the dielectric constant of the second medium.
b. Determine the direction of the Poynting vector in the dielectric medium.

Answers
1. (c) 2. (*) 3. (b) 4. (d) 5. (a) 6. (c) 7. (d) 8. (c) 9. (c) 10. (a)
11. (b) 12. (c) 13. (b) 14. (b) 15. (d)
Answers with Explanations
1. (c) Line integral is an integral where the function to be 4. (d) The number of charge particles in an intrinsic
integrated is evaluated along a curve. semiconductor is doubled the number of hole electron
Given, F =
x
x$ +
y
. y$ pairs. Ni is the number of charge carriers at temperatureTi
2
x +y 2
x +y 2
2 and Nf atTf .
Ni = N0 e − ∆ / 2 i
E kT

B B x y 
∫A F. d l = ∫A  2 2
x$ + y$  (dx x$ + dy y$ )
x 2 + y 2 
and Nf = N0 e − ∆E / 2kTf
 x +y
The percentage increase in charge carriers
B x B y
=∫ dx x$ . x$ + ∫ dy y$ . y$  ∆E  1 1 
 − 
A 2
x +y 2 A
x + y2
2
Nf − Ni  2k  Ti Tf  
× 100 = 100 e − 1
=∫
B x
dx + ∫
B y Ni  
dy  
2 2
A
x +y A
x + y2
2
∆E  1 1
Let u = x2 + y2
Now,  − 
2k Ti Tf 
du x 0.68 eV
⇒ = dx  1 1 
=  − 
dx x2 + y2 2 × 8.62 × 10− 5  300 320
1 0
⇒ ∫ du + = 0 .82
∫0du = [u] − 1 + 0 = 2
1
−1
∴ Percentage increase in the number of charge carriers
2. (*) In case of superposition of two SHMs, the resulting = 100 [e 0. 82 − 1]
displacement is given by x = x1 + x 2
= 127 %
x = A1 cos ω t + A2 cos (ωt + δ ) 5. (a) Output of the given circuit is
where, δ is the phase difference between two SHMs.
A AB
According to given questions,
B AB+AB
y = a sin (ωt + θ)
For any two consecutive SHMs, phase difference is
AB
constant
⇒ yn = a sin (ωt + θn )
and yn − 1 = a sin (ωt + θn − 1) 6. (c) Equation for photoelectric effect
⇒ y = yn + yn − 1 hν = φ + KEmax …(i)
= a sin (ωt + θn ) + a sin (ωt + θn − 1)
where, ν is frequency of incident radiation
θn + θn − 1  θn − θn − 1
⇒ yn + yn − 1 = 2 a sin ωt cos   φ is work function.
2  2  KEmax is maximum kinetic energy of photoelectron.
It is given that resultant amplitude is zero According to the given question, the light consists of two
 θn − θn − 1 different frequencies.
⇒ 2a cos   =0
 2  The larger frequency will cause the photoelectrons with
larger kinetic energy
 ∆θ 
or 0 = cos  
 2 W 12.56 × 1015
⇒ ν= =
⇒ ∆ φ = ∆θ = π 2π 2π
From Eq. (i), we get
3. (b) When electrons make transitions between lower atomic
KEmax = hν − φ
energy levels in heavy elements. X-rays produced in
 12.56 × 1015 
this way have definite energies just like other line = ( 4.14 × 10− 15 eVS )   − 2 eV
spectra from atomic electrons. They are called  2π 
characteristic X-rays. KEmax = 6.28 eV
K α -rays ⇒ EK α = EK − EL 7. (d) The Maxwell - Boltzmann distribution is the distribution
K β -rays ⇒ EK β = EK − EM function for distribution of an amount of energy between
Lα X-rays ⇒ ELα = EL − EM identical but distinguishable particles
1
EK β = EK α + ELα ⇒ νK β = νK α + νLα f (E ) =
AeE /kT
Solved Paper 2011 ●
77

The speed distribution for the molecule of an ideal gas Electric field in y-direction
3/ 2
 m   − mv 2  ∂V −∂
f (v ) = 4π N   v 2 exp   Ey = i = [ 4x 2 + (y − 2)2 + z 2]
 2 π kBT   2 kBT  ∂y ∂y
where, m is the particle (molecule) mass, kB is = −2 (y − 2)
Boltzmann constant andT is temperature. Ey = −2(y − 2) y$
Maxwell speed distribution
function (f (x))

Net electric field is E = Ex + Ey = −8x x$ − 2 (y − 2) y$


At the origin E ( 0, 0) = −0 − 2( 0 − 2)y$ = 4y$
10. (a) The wave function for a given physical system contains
the measurable information about the system. To obtain
specific values for physical parameters, for energy, you
operate on the wave function with the quantum
mechanical operate on the wave function associated with
Molecular speed that parameter.
The average speed is the expected value of the speed The operator associated with energy is the Hamiltonian
distribution which is given as and the operation on the wave function is the Schrödinger
8 kBT equation.
v = .
πm Independent Schrödinger equation only for certain values
of energy and these values are called eigen values of
8. (c) Free expansion of one mole of an ideal gas in an energy.
adiabatic container from (V1,T ) to (V2 ,T ). 3 4
T is not changed whereas volume has been changed Given function ψ (x ) = φ1 (x ) + φ 2 (x )
5 5
from V1 to V2 . Energy of the particle in the state ψ is given as
During isothermal process
9 16 2
dQ = dW = pdV E = ψ 2(x ) = φ12(x ) + φ 2(x )
25 25
∴ Change in entropy is given by 9 16
dQ = ( −1eV ) + ( −2 eV )
dS1 = 25 25
T −9 16 − 41
pdV = × 1− ×2= eV
dS1 = 25 25 25
T
The total change in entropy when gas expands fromV1 11. (b) Equation of motion of rain drop is
dv v dm
to V2 m + = mg
Sf V2 pdV V2RT dt dt
∫Si dS1 = ∫V1 T ⇒ ∫V1 VT ⋅ dV [ Q pV = RT ] ⇒ mdv + vdm = mg dt

V
∆S1 = R [ln V ]V2 or d (mv ) = mg dt
1 Integrating both sides, we have
V 
∆S1 = R ln  2 
 V1  ∫ d (mv ) = ∫ mg dt
Change in entropy of the surroundings ∆S 2, Using integration by parts, we have
dm
∆S 2 = ∫
dQ ⇒ ∫ d (mv ) = gm ∫ dt − ∫ dt ∫ dt
T
[mv ]m 0, 0 = [gmt ]m 0, 0 − ∫ kt 2dt
m,v m,1
dQ  Q1 Q 2 Q Q 
As ∫ =0 ∴ − = − =0
T  T1 T2 T T  1
∴ ∆S 2 = 0  t 3
V  = gm − k 
⇒ ∆S1 = R ln  2  ; ∆S 2 = 0  3 0
 V1 
1
= mg − 12 × ×1
9. (c) Equipotential surfaces are given by 3
Vi = 4x 2 + (y − 2)2 + z 2 ⇒ mv = mg − 4 …(i)
Now, using
Electric field in x-direction is dm
= kt 2
− ∂Vi −∂ dt
Ex = = [ 4x 2 + (y − 2)2 + z 2] m 1
∂x ∂x
Ex = − 8x ⇒ Ex = −8x x$
⇒ ∫m 0
dm = ∫ kt 2 dt
0
78 IIT JAM Physics Solved Papers & Practice Sets

1 − Eg  50 
t 3  1 2  200 × 250 
or m − m0 = k ×   = 12 × × 1= 4 ⇒ = e 2kB
 3 0 3 300 −E g 1
×
or, m = 4 + m0 = 4 + 2 = 6g = e 2kB 1000
− Eg
Putting the value of m in Eq. (i), we get
1
6v = 6 × 1000 − 4 or = e 2000kB
150
⇒ 6v = 6000 − 4 ~− 6000 − Eg
6000
⇒ v~− = 1000 cm/s. or 150 = e 2000kB
6
Putting, kB = 1.38 × 10−23 J / K
12. (c) Hermitian matrix (or self adjoint matrix) is a square Eg
matrix with complex entries, i.e. equal to its own loge (150) =
2000 × 1.38 × 10−23
conjugate transpose, i.e. the element in the i-th row and
j-th column is equal to the complex conjugate of the ∴ Eg = (loge 150 + loge 10) × 2.76 × 10−20
element in the j-th row and i-th column, for all indices i = ( 2 m 08 + 2.303) × 2 .76 × 10−20
and j aij = a ji = 13.83 × 10−20 J
Given, P $ is Hermitian and Q
$ is skew Hermitian
13.83 × 10−20
∴ P + P and Q = − Q = eV
1.6 × 10−19
⇒ P + iQ = P − i Q = P − i( − Q ) = P + iQ
∴ Energy band gap, Eg = 0.8643 eV.
Therefore P + iQ is Hermitian matrix.
13. (b) The intensity of magnetisation (I) increases for 16. a. Given A = − 4 yx 2 x$ − 3y 2y$
ferromagnetic substance with the applied magnetic field Line integral ∫ A ⋅ d l = ∫ ( − 4yx 2dx − 3y 2dy )
(H) and become constant because the domains get
Along the path PQ
perfectly aligned with respect to the external magnetic 1
field. 1 1 x 3  −4
∫0 A d l = ∫0− 4 (1) x dx = − 4  3  = 3
2

14. (b) According to given figure, the angle between the area 0
vector and electric field is 30°. Along the path QR
1
The flux of the electric field vector 1 y 3 
0
∫0 A d l = ∫1− 3y dy = 3  3  = 1
2
coming out of the curved surface
of the hemisphere is equal to the θ Q
E 0

flux entering the circular disc, as Along the path RO


A
the net flux through the closed 1 0
∫0 A d l = ∫1 − 4 ( 0) x
2
dx = 0
Gaussian surface is zero
⇒ φ = E. dS = EdS cos θ 1 4 1
= E πR 2 cos 30°
⇒ ∫PO A d l = − 3 + 1 + 0 = − 3
3 b. Stoke’s theorem ∫ A . d l = ∫ ∇ × A n$ dS
= E πR 2 s
2
$i $j k$
15. (d) Conductivity (σ) of a semiconductor can be written as
∂ ∂ ∂
− Eg ∇×A=
σ = σ 0 e 2kBT ∂x ∂y ∂z
− 4x2 − 3y 2 0
where, σ 0 is constant, kB is Boltzmann constant and Eg
is energy bond gap. ∂ ∂ ∂ ∂
For two temperatures T1 and T2, we get = $i ∂y
$
∂z − j ∂x ∂z
− Eg − 3y 2 0 − 4x 2 0
σ1 = σ 0 e 2kBT1
…(i) ∂ ∂
− Eg + k$ ∂x ∂y
σ 2 = σ 0 e 2kBT2 …(ii) − 4x 2 − 3y 2
− Eg  1 1  = − 4x 2 k$

σ1 2k  T T  ∇ × A n$ = − 4x 2 k$ ( − k$ ) = 4x 2
∴ = e B  1 2
σ2 0 1
∫ ∇ × A ⋅ n$ . dS = ∫1 ∫04x
2
dxdy
Putting T1 = 200 K and T2 = 250 K
3
σ1 R2 g E  1 − 1  0 x 1 4 1 −4
= = e − 2kB  200 250  =∫ 4 ∫0 dy = 3 ∫0 dy =
σ 2 R1 1 3 3
Solved Paper 2011 ●
79

For the path O → P 18. a. Amount of heat flowing through the surface S per unit
1  −3y 3 
1
time ∫ (b ∇ T ) n$ dS and the amount of energy decreasing
∫OP ∫0
2
A ⋅ d l = − 3y dy =   = −1 per unit time is
 3 0 S
Now using Stokes theorem, we get

∫s ∇ × A ⋅ n$ ds = ∫PQROP A ⋅ d l v

=∫ A ⋅d l + ∫ A ⋅d l
PQO OP

⇒ ∫PQO A ⋅ d l = ∫s ∇ × A ⋅ $d S−
n ∫OP A ⋅ d l ∫a
∂T
dz
−4 −4 −1 ∂t
= − ( − 1) = + 1= For no loss of heat
3 3 3
dT
17. a. Case I Magnetic field inside long solid cylinder when or ∫ b ∇ T n$ dS = ∫ a ∂t
∂z
r < r1 ∂T 
b
∫  a ∇
2
or T −  dz = 0
⇒ ∫ H . d l= H∫ d l dt 
= H . 2πr b 2 ∂T
⇒ ∇T − =0
and I = J . πr 2 a dt
⇒ H . 2 πr = J πr 2 b 2 ∂T
⇒ ∇T =
Jr a ∂t
H=
2 b. At steady state, variation in temperature is zero
J ×r ∂T b
⇒ H= ∴ = 0, ⇒ ∇ 2T = 0
2 ∂t a
Case II Magnetic field inside the long cylinder when ⇒ K ∇ 2T = 0
r1 < r < r2
1 ∂  ∂T  1 ∂ 2T 1 ∂  ∂T 
∇ 2T = 0 ⇒ r .  + 2 + r .  =0
∫ H . d l = H . 2πr
∂ ∂r  ∂t  r ∂θ 2 r ∂r  ∂r 
I = J . πr12 All terms on LHS are independent terms, i.e. each term
⇒ H . 2πr = J πr12 can be individually equated to zero.
Also, consider two boundary conditionsT = 2T0 at r = r0
Jr12 Jrr12
⇒ H= = and T0 at r = 2r0
2r 2r 2 d  ∂T 
⇒ r .  =0
( J × r ) r12 dr  ∂r 
H=
2r 2 ∂T dr
⇒ r = c1 ⇒ ∫ dT = c1 ∫
Case III When r2 < r ∂r r
⇒ T = c1 log r + c 2
∫ H . dl = H . 2πr
and I = J . πr12 Applying boundary conditions and putting values of c1
and c 2, we have
H . 2πr = J . πr12
 
Jr12 J r r12  loge 2r0 
H= = ⇒ T = T0 1 −
2r 2r 2 1 
 loge 
( J × r ) r12  2 
H=
2r 2  
T0  loge 2r0 
µ ( J × r ) r12 T = log r + T0 1 −
B = µ 0 µr H =  1 1 
2r 2 loge    loge 
 2  2 
b. Variation of H and r
19. a. Perfectly reversible engine device can be operated
between same two reservoirs with same energy transfers
H (only direction reversed).
According to the given graph,
WAB = 0 (Isochoric process)
 pV 
r1 r2 r ∆Q AB = nCV ∫ dT pV = nRT , T = nR 
80 IIT JAM Physics Solved Papers & Practice Sets

∆Q AB =
nR
CV ( p 2V2 − p1V1) / R where, Ne = number of electrons in the system.
nR V = total volume of the lattice/solid
pV pV Here lattice parameter = a
TA = 1 1 and TB = 2 1
nR nR 1 valence electron per atom.
⇒ Q AB = CV ( p 2V1 − p1V1) = CVV1 ( p 2 − p1) ∴ Na → Na + number of atoms.
∆QBC = 0, ⇒ WBC = − ∆U kf = [ 3π 2Na /V ]1/ 3
2/ 3
p1V2 h2  3π 2Ne 
WBC = − nCV ∫ dT , TC =
nR
Ef = 
2m  V 
 .
− nCV
WBC = [ p1V2 − p 2V1] 21. a. Lower cut-off frequency
nR
= − CV [ p1V2 − p1V1] / R 1 1 1
f = = = Hz,
2πRC 2π × 10 × 103 × 100 × 10−6 2π
∆QCA = nCp ∫ dT = Cp ( p1V1 − p1V2 ) / R
Gain of the given circuit is
∆WCA = nRdT = nR ∫ dT = [ p1V1 − p1V2] Vout  100  15
= 1+  
 100
× 10 −
Total work done Vin  10   20  10
Efficiency, η =
Heat absorbed = (1+ 10) ( 7. 5) − 10
WAB + WBC + WCA = 11× 7. 5 − 10
=
∆Q AB + ∆Q BC + ∆Q CA = 72 . 5
As we know that,
CV C
0− [ p1V2 − p 2V1] + v [ p1V1 − p1V2] Gain × Bandwidth1 = Gain × Bandwidth2
R R
= ⇒ (1 MHz ) × 1 = 72 . 5 × ( Bandwidth)2
CV [ p 2V1 − p1V1] Cp ( p1V1 − p1V2 )
+ 0+ 1MHz
R R ∴ ( Bandwidth)2 = = 13.7 kHz.
72 . 5
b.
Change in entropy of a system is given as b. Output voltage of the circuit is
∆Q V0 = ( 72 . 5) (Vin)
∆S =
T = ( 72 . 5) × (1mV )
For A to B, = 0 . 072 V.
∆Q AB 22. a. A central force is a force that points from the particle
∆S AB =
T directly towards (or directly away from) a fixed point in
CV dT space the centre and whose magnitude only depends on
⇒ ∫ dS AB = ∫ T [taking, n = 1]
the distance of the object to the centre.
or S = CV lnT + ln k , Given, orbital equation in central force
= lnT CV + ln k1 = ln k1T CV d 2u −m
+u= 2 2f …(i)
⇒ T = keS /CV d θ2 l u
For B to C, ∆SBC = 0 Equation of circle of radius passing through the origin
1 S /C p 1
For C to A, ∫ dS = Cp ∫ dT ⇒ T = k e r = 2a cos θ and r =
u
sec θ
⇒ u=
T B 2a
du sec θ . tan θ
CV
∆S = 0 =
dθ 2a
A C
Cp d 2u sec3 θ + sec θ tan2 θ
=
dθ 2a
S
d 2u m  1
20. The energy (fermi energy Ef ) is ⇒ = −u − 2 2 f  
dθ l u u
hk 2
E (k ) =
 1 − l u  sec θ + sec θ (1 + tan θ) 
2 2 3 2
2m
⇒ f   =  
u m  2a 
This corresponds to occupying all the states with wave
vectors (k ) < kf , where kf is so called fermi wave vector. − l 2 sec3 θ 2
= u
kf = [ 3π 2 Ne /V ]1/ 3 am
Solved Paper 2011 ●
81

22
 1 − 8 a l 24. a. Highest number of orders with grating
⇒ f   = .u5
u m d sin θ = n λ
− 8 a 2l 2  1  d sin θ
or f (r ) = .  5 or n=
m r  λ

b. Total energy of the system ⇒ sin θ =
d
1 l2
E = mr 2 + + V (r ) nλ
2 2mr 2 θ = sin−1  
 d 
According to the given question,
b. Energy maximum drawn before is now a maximum and
r → ∞ and V (r ) = 0 vice-versa.
1 (bright and dark fringes have exchanged places).
⇒ E = mr 2
2  wavelength
∴Shifting of   to path difference and phase
23. a. V (x ) = ∞ for x < 0  2 
= 0 for 0 < x < L difference by π.
= V0 for x > L
25. a. Electric field associated with the reflected light is given
The Schrödinger equations for the different regions are as
d 2ψ 2 mE ik 
+ k 2ψ = 0 ; k2 = 0≤x <L E = E0 x$ exp  ( 3y + z ) − iωt 
dx 2 h2  2 
d 2ψ 2m Applying boundary condition for the dielectric-dielectric
and − k 12 ψ = 0 ; k 12 = 2 (V0 − E ) x > L
dx 2 h interface, we have
The solution of these equations are; ε1 En1 = ε 2 En2
ψ = A sin kx + B cos kx 0 <x <L where, En1 and En2 are normal components of electric
ψ = Ce −k1 x + Dek1x x >L fields perpendicular to plane of first and second dielectric
where A, B, C and D are constants. respectively putting E1 = E0, we can find E2, which gives
Applying boundary conditions, dielectric constant of the second medium.
ψ = 0 a + x = 0 and ψ → 0 a + x → ∞ b. Poynting vector is given by
 ψ = A sin kx 0 < x < L  p = E×H
ψ = Ce −k1x x > L 
 where, E is the electric field vector.
b. The reflection coefficient is defined as and H is the magnetic field vector
flux of reflected particles The direction of (p) perpendicular to both electric field (E)
R= . and magnetic field (H) is the medium.
flux of incident particles
Solved Paper
2010
IITJAM Physics
MM : 300 Time : 3 hrs

n
In this question paper, there are total of 25 questions carrying 300 marks of 3 hours duration.
n
Questions 1-15 (Objective questions) carry 6 marks each and questions 16-25 (Subjective questions) carry
21 marks each.
n
Each objective question has four choices for its answer i.e. (a), (b), (c) and (d). Only one of them is the
correct answer.
n
There will be negative marking for wrong answers to objective questions. The following marking scheme
for objective questions shall be used:
(a) For each correct answer, you will be awarded 6 (Six) marks.
(b) For each wrong answer, you will be awarded –2 (Negative two) mark.
(c) Multiple answers to a question will be treated as a wrong answer.
(d) For each un-attempted question, you will be awarded 0 (Zero) mark.
n Do not write more than one answer for the same question. In case you attempt a subjective question
more than once, please cancel the answer(s) you consider wrong. Otherwise, the answer appearing last
only will be evaluated.
n Clip board, log tables, slide rule, calculator, cellular phone and electronic gadgets in any form are not
allowed.

1 i 1
1. A matrix is given by M =   . The eigen values of M are
2 1 i 
(a) real and positive
(b) purely imaginary with modulus 1
(c) complex with modulus 1
(d) real and negative

2. The value of θ at which the first order peak in X-ray ( λ = 1.53 Å), diffraction corresponding
to (111) plane of a simple cubic structure with the lattice constant, a = 2.65 Å, is approximately
(a) 15° (b) 30°
(c) 45° (d) 60°
Solved Paper 2010 ●
83

3. The magnetic field associated with the electric field vector


E = E 0 = sin (kz − ωt ) $j is given by
E0 E0
(a) B = − sin ( kz − ω t ) $i (b) B = sin ( kz − ω t ) i$
c c
E0 E
(c) B = sin ( kz − ω t ) $j (d) B = 0 sin ( kz − ω t ) k$
c c

4. Consider the following truth table :


A B C F
0 0 0 1
0 0 1 0
0 1 0 0
0 1 1 0
1 0 0 1
1 0 1 1
1 1 0 1
1 1 1 0

The logic expression for F is


(a) AB + BC + CA (b) AB + AC + B C
(c) C A B + AB (d) C ( A + B ) + AB

5. The following are the plots of the temperature dependence of the magnetic susceptibility for
three different samples.

x(T) a x(T) b x(T) c

O T O T O T

The plots a, b and c are corresponding to


(a) ferromagnet, paramagnet and diamagnet, respectively
(b) paramagnet, diamagnet and ferromagnet, respectively
(c) ferromagnet, diamagnet and paramagnet, respectively
(d) diamagnet, paramagnet and ferromagnet, respectively

3 2 3 2
6. The equation of a surface of revolution is z = ± x + y
2 2
 2 
The unit normal to the surface at the point A  , 0, 1 is
 3 
3 $ 2 $ 3 $ 2 $
(a) i+ k (b) i− k
5 10 5 10
3 $ 2 $ 3 $ 2 $
(c) i+ k (d) i+ j
5 5 10 10
84 IIT JAM Physics Solved Papers & Practice Sets

7. A gas of molecules each having mass m is in thermal equilibrium at a temperature T. Let v x , v y ,


v z , be the cartesian components of velocity, v of a molecule. The mean value of
(v x − αv y + βv z ) 2 is
kBT kBT kBT kBT
(a) (1 + α 2 + β 2 ) (b) (1 − α 2 + β 2 ) (c) (β 2 − α 2 ) (d) (α 2 + β 2 )
m m m m

8. A circular platform is rotating with a uniform angular speed ω


counterclockwise about an axis passing through its centre and perpendicular
ω
to its plane as shown in the figure. A person of mass m walks radially
inward with a uniform speed v on the platform. The magnitude and the
direction of the Coriolis force (with respect to the direction along which the
person walks) is
(a) 2 mωv towards his left (b) 2 mωv towards his front
(c) 2 mωv towards his right (d) 2 mωv towards his back

9. A quarter wave plate is placed in between a polariser and a Direction of


rotation of
photo-detector. When the optic axis of the quarter wave quarter wave
plate
plate is kept initially parallel to the pass axis of the polariser
and perpendicular to the direction of light propagation. The
intensity of light passing through the quarter wave plate is
measured to be I 0 as shown in the figure. If the quarter
wave plate is now rotated by 45° about an axis parallel to Polaroid Quarter Photo-detector
wave plate
the light propagation, what would be the intensity of the
emergent light measured by the photo-detector?
I0 I0 I0
(a) (b) (c) (d) I0
2 2 2 2

10. A particle of mass m moving with a velocity v = v 0 ( $i + $j ), collides elastically with another
particle of mass 2m, which is at rest initially. Here, v 0 is a constant. Which of the following
statements is correct?
 $i + $j 
(a) The direction along which the centre of mass moves before collision is −  

 2 
(b) The speed of the particle of mass m before collision in the centre of mass frame is 2 v 0
2
(c) After collision the speed of the particle with mass 2m in the centre of mass frame is v0
3
(d) The speed of the particle of mass 2m before collision in the centre of mass frame is 2 v 0

11. A trapped air bubble of volume V 0 is released from a depth h measured from the water surface
in a large water tank. The volume of the bubble grows to 2V 0 as it reaches just below the
surface. The temperature of the water and the pressure above the surface of water
(105 Nm −2 ) remains constant throughout the process. If the density of water is 1000 kgm −3
and the acceleration due to gravity is 10 ms −2 , then the depth h is
(a) 1 m (b) 10 m
(c) 50 m (d) 100 m
Solved Paper 2010 ●
85

12. A particle of mass m is confined in a two-dimensional infinite square well potential of side a.
25 π 2 −h
2
The eigen energy of the particle in a given state is E = . The state is
ma 2
(a) 4-fold degenerate (b) 3-fold degenerate
(c) 2-fold degenerate (d) non-degenerate

13. For a wave in a medium the angular frequency ω and the wave vector k are related by
ω 2 = (ω 20 + c 2k 2 ), where ω 0 and c are constants. The product of group and phase velocities,
i.e. v g v p is
(a) 0.25 c 2 (b) 0.4 c 2 (c) 0.5 c 2 (d) c 2

14. Three identical non-interacting particles, each of spin 1/2 and mass m, are moving in a
one-dimensional infinite potential given by
 0 for 0 < x < a
V (x ) = 
 ∞ for x ≤ 0 and x ≥ a
The energy of the lowest energy state of the system is
π 2 h2 2 π 2 h2 3π 2 h 2 5π 2 h 2
(a) 2
(b) 2
(c) 2
(d)
ma ma ma 2 ma 2

15. Assume that, Z = 0 plane is the interface between two linear and Z
homogeneous dielectrics as shown in the figure. The relative εr = 5
permittivities are εr = 5 for Z > 0 and εr = 4 for Z < 0. The electric Z=0
field in the region Z > 0 is E1 = ( 3 i$ − 5 $j + 4k$ ) kVm −1. If there are no
εr = 4
free charges on the interface, then electric field in the region z < 0 is
given by
3 5 
(a) E 2 =  $i − $j + k$  kVm −1 (b) E 2 = ( 3$i − 5$j + k$ ) kVm −1
4 4 
(c) E = ( 3i$ − 5$j − 5 k$ ) kVm −1
2 (d) E 2 = ( 3i$ − 5$j + 5 k$ ) kVm −1

16. A particle of mass 2/3 kg is subjected to a potential energy function V (x ) = ( 3x 2 − 2 x 3 ) J,


where, x ≥ 0 and expressed in metres.
a. Sketch V (x ) with respect to x in the range 0 to 2 m. Mark the positions of all the
maxima and minima. What is the maximum value of the potential energy?
b. Supposing the particle is released at x = 4 / 3 m, find its velocity when it reaches x = 3 / 2 m.

17. A vector field is given by


α (x$j − y i$ ) for (x 2 + y 2 ) ≤ r 02 (region I)

F (r ) =  2 (x$j − y $i ) 2 2 2
αr 0 (x 2 + y 2 ) for (x + y ) > r 0 (region II)

Y
Here, α and r 0 are two constants.
a. Find the curl of this field in both the regions.
2r0
b. Find the line integral ∫ F ⋅ d l along the closed semicircular r0
X
path of radius 2 r 0 as shown in the figure alongside.
86 IIT JAM Physics Solved Papers & Practice Sets

18. A solid cylinder C of mass 10 kg rolls without slipping in an inclined plane P


which offers friction. The angle of the inclined plane with the horizontal is 30°
as shown in the figure alongside. A massless inextensible string is wrapped
around the cylinder and it passes over a frictionless pulley P. The other end C
B
of the string holds a block B of mass 2 kg as shown in the figure. (Take
g = 10 ms −2 ). 30°

a. Draw the free body diagram of the cylinder and the block. If the block B moves down by
0.01 m, how much distance does C move along the incline?
b. If the block B moves with an acceleration of 0.05 g upward, then find the magnitude of
the frictional force.

19. a. A photon of initial momentum p 0 collides with an electron of rest mass m 0 moving with
relativistic momentum p and energy E. The change in wavelength of the photon after
scattering by an angle θ is given by
p +p θ
∆λ = 2cλ 0 0 sin 2 ,
E − cp 2
where, c is the speed of light and λ 0 is the wavelength of the incident photon before
scattering. What will be the value of ∆λ, when the electron is moving in a direction
opposite to that of the incident photon with momentum p and energy E ? Show that the
value of ∆λ becomes independent of the wavelength of the incident photon, when the
electron is at rest before collision.
b. In a Compton experiment, the ultraviolet light of wavelength 2000 Å is scattered from
an electron at rest. What should be the minimum resolving power of an optical
instrument to measure the Compton shift, if the observation is made at 90° with
respect to the direction of the incident light?

20. A conducting spherical shell of radius R1 carries a total charge Q. A


spherical layer of a linear, homogeneous and isotropic dielectric of dielectric
constant K and outer radius R 2 (> R1 ) covers the shell as shown in the R2
R1
figure.
a. Find the electric field and the polarisation vector P inside the
dielectric. From this P, calculate the surface bound charge density
σ b , on the outer surface of the dielectric layer and the volume bound charge density
ρb , inside the dielectric.
b. Calculate the electrostatic energy stored in the region R1 ≤ r ≤ R 2 .

21. Two spaceships A and B of proper length 50 m each are moving with relativistic speeds 0.8 c
and 0.6 c respectively, in the same direction with respect to a stationary observer S. Here, c is
the speed of light. At t = 0, the spaceship A is just behind the spaceship B as shown
alongside.

a. Find the time taken by the spaceship A to completely overtake the spaceship B (i.e.
the back of A is in line with the front of B ) as seen by the observer S.
b. Find the time taken by the spaceship A to completely overtake the spaceship B as
seen by an observer in the spaceship B.
Solved Paper 2010 ●
87

V (x)
22. A free particle of mass m with energy V 0 / 2 is incident from left on a
step potential of height V 0 as shown in the figure. Writing down the
V0
time independent Schrödinger equation in both the regions, obtain
the corresponding general solutions. Apply the boundary conditions x
Region I O Region II
to find the wave functions in both the regions. Determine the
reflection coefficient R. What is the transmission coefficient T ?

23. For the transistor as shown in the figure, the DC current gain β DC = 50 and V BE = 0.7 V. The
switch S is initially open.
a. Calculate the voltage at point A. If the switch S is now closed, what would be the voltage at
point A?
b. Draw the DC load line and find the Q-point of the circuit with the switch S remaining
closed.
+5V

10 kΩ 500 Ω
S
A
B
E
10 k Ω 260 Ω

24. Consider a capacitor placed in free space, consisting of two concentric circular parallel plates
of radii r. The separation z between the plates oscillates with a constant frequency ω, i.e.
z(t ) = z 0 + z 1 cos ωt. Here, z 0 and z 1 (< z 0 ) are constants. The separation z(t ) (<< r ) is varied
in such a way that the voltage V 0 across the capacitor remains constant.
a. Calculate the displacement current density and the displacement current between the
plates through a concentric circle of radius r / 2 .
b. Calculate the magnetic field vector ( H) between the plates at a distance r / 2 from the
axis of the capacitor.

25. One mole of an ideal monoatomic gas in an initial state a with pressure p i and volume Vi is to
be taken to a final state d with p f = B 2 p i and Vf = Vi / B through the path a → b → c → d as
shown in the figure below, for a particular value of B (> 1. ) Here, a → b and c → d are
adiabatic paths while b → c is an isotherm with temperature T0 . State b and c corresponding to
( p 1, V1 ) and ( p 2 , V 2 ), respectively.
p
d ( pf , Vf)
b ( p1, V1) T=T0

c ( p2, V2)

a ( pi, Vi)
V
V
Find the ratio 2 and the total work done by the gas in terms of p i , Vi , T0 and B.
V1

Answers
1. (c) 2. (b) 3. (a) 4. (d) 5. (b) 6. (b) 7. (a) 8. (a) 9. (b) 10. (c)
11. (b) 12. (b) 13. (d) 14. (c) 15. (d)
Answers with Explanations
1. (c) Eigen values are a special set of scalar associated with a 4. (d) From the given truth table,
linear system of equations, (i.e. a matrix equation). If A F = AB C + AB C + AB C + ABC
be a n × n square matrix with eigen value λ, then = B C + AB C + ABC
| A − λI | = 0
= B (C + AC ) + ABC
where, I is the identity matrix.
1 i 1 = B [(C + A )(C + C )] + ABC
Given, matrix M =
2 1 i  = B [(C + A )] + ABC
 1 1  = B C + B A + ABC
i−λ
 2 2  = C [B + AB ] + B A
⇒ | M − λI | = 0 ⇒  =0
1 i = C [(B + A ) ⋅ (B + B )] + B A
 − λ
 2 2  = C ( A + B ) + AB
 1 0 
where, I =  
0 1 5. (b) Paramagnetic Material
2 2 χ(T)
 1   1
⇒  i − λ −   =0 (Biotite)
 2   2
 1 1  1 1
⇒  i−λ+  i−λ−  =0
 2 2   2 2
1 1
⇒ λ= (1 + i ), λ = − (1 − i )
2 2 T
Hence, λ is complex with modulus 1. Hyperbolic curve
C
2. (b) When X-rays are scattered from a crystal lattice, peaks According to the Curie’s law, K =
of scattered intensity are observed which corresponds to T
nλ = 2d sin θ where, C = proportionality constant
a T = absolute temperature.
where, d =
h 2 + k 2 + l2 Diamagnetic Material
χ(T)
In the given problem, h = 1, k = 1, l = 1and a = 2.65 Å
For the first order peak, n = 1
⇒ λ = 2d sin θ
a
1.53 Å = 2 × ⋅ sin θ
h 2 + k 2 + l2 O
T
2 × 2.65 Å
⇒ 1.53 Å = sin θ
12 + 12 + 12 Diamagnetic material susceptibility as a function of
temperature.
sin θ = 0.500 Ferromagnetic Material
⇒ θ = 30°
χ(T)
3. (a) If E and B are the electric field and magnetic field
associated with an electromagnetic wave, then E × B
gives the direction of propagation of electromagnetic wave
E = E sin(kz − ωt ) $j
0
According to the given equation, electric field is along
Y-axis and the electromagnetic wave is propagating T
along positive Z-axis. Behaviour of ferromagnetic material susceptibility as the
E material approaches it Curie temperature.
Using, c = 0 , where c is the speed of light.
B0 6. (b) A surface of revolution is a surface in Euclidean space
E 
⇒ B0 =  0  created by rotating a curve (the generatrix) around a
c 
straight line in its plane (the axis).
The magnetic field associated with the wave
E 3 2 3 2
B = − 0 sin(kz − ωt )$i Given, z = ± x + y (squaring on both sides)
c 2 2
Solved Paper 2010 ●
89

3 2 3 2
z2 = x + y 9. (b) Malus’s Law According to Malus, when completely plane
2 2 polarised light is incident on the analyser, the intensity I of
3 2 3 2
⇒ x + y − z2 = 0 the light transmitted by the analyser is directly
2 2
proportional to the square of the cosine of angle between
3 2 3 2
⇒ Let f (x , y , z ) = x + y − z 2 = 0 the transmission axes of the analyser and the polariser.
2 2
⇒ I ∝ cos2 θ
The normal to the surface f (x , y , z ) is equal to the ∇f
⇒ I = I 0 cos2 θ
⇒ ∇f = 3x $i + 3y $j − 2z k$
According to the given question, I = I 0 cos2 45°
 2$ 
∇f( = 3 × i − 2 × 1k$  2
2 / 3, 0, 1)
 3   1 I
I = I0   = 0
 2 2
The unit normal to the surface
2$ 10. (c) Velocity of centre of mass before the collision
3 i − 2 k$
3  mv 0 + 2 m × 0 $  mv 0 + 2m × 0 $
= vx =   i and v y =   j
 2
2  m + 2m  m + 2m 
2
 3  + ( − 2) v
 3 v CM = v x $i + v y $j = 0 ( $i + $j )
3
2$ Velocity of the particle of mass m in (w.r.t.) centre of
3 i − 2 k$
= 3 v
mass frame before collision = v 0( $i + $j ) − 0 ( $i + $j )
10 3
7. (a) Mean value of (v x − αv y + βv z )2 is given by 2v 0 $ $
v1 = ( i + j)
3
(v x − αv y + βv z )2 = v x2 + α 2 v y2 + β 2 v z2 …(i)
2 2
Magnitude of v1 = v 0 ms−1
The mean kinetic energy allows the root mean square 3
speed v rms of the gas particle to be calculated as Now, applying conservation of linear momentum
3kBT m v1 + 2 m × 0 = mv1′ + 2 mv 2 …(i)
v rms = v 2 = …(ii)
m v 2 is the velocity of mass 2m after collision w.r.t. centre
2
v rms of mass frame
⇒ v x2 = v y2 = v z2 =
3 v − v1′ v −v ′
and e= 2 ⇒ 1= 2 1
From Eqs. (i) and (ii), we get v1 v1
⇒ v1 =v 2 −v1′ …(ii)
 3k T   3k T   3k T 
(v x − αv y + βv z )2 =  B  + α 2  B  + β 2  B  From Eqs. (i) and (ii), we get
 3m   3m   3m 
−v
k T v 2 = 0 ( i + $j )
= (1 + α 2 + β 2 ) B 3
m 2v0
⇒ |v 2 | = = speed of particle 2 m after the collision.
8. (a) The Coriolis effect is a deflection of moving objects, 3
when the motion is described relative to a rotating 11. (b) Density, ρ = 1000 kgm−3
reference frame. In the reference frame with clockwise
As, the temperature is constant we can apply Boyle’s
rotation, the deflection is to the left of the motion of the
law, i.e. pV = constant
object. In one with counter clockwise rotation, the
p0 (atm pressure)
deflection is to the right. The vector formula for the
magnitude and direction of the Coriolis acceleration
ac = − 2 ω × v
2V0
where, ac is the acceleration of the particle in the rotating
system, v is the velocity of the particle w.r.t. the rotating h
system and ω is the angular velocity vector, which has
magnitude equal to the rotation rate ω and is directed V0
along the axis of rotation of the rotating reference frame.
According to the given problem ⇒ ( p 0 + ρgh )V0 = p 0( 2V0 )
Coriolis force, F = − 2m (ω × v ) ⇒ p 0V0 + ρghV0 = 2p 0V0
ω = ωk$ and v = −v$i ρghV0 = p 0V0
⇒ F = 2m ωv ( − $j ) p 105
10 h = 0 = 3 = 10 m
Therefore coriolis force is 2m ωv towards his left. ρg 10 × 10
90 IIT JAM Physics Solved Papers & Practice Sets

12. (b) To obtain specific values of energy, you operate on the As, we know that energy of particle in a box is given by
wave function with quantum mechanical operator n 2 π 2 h2
E=
associated with energy, which is called the Hamiltonian. 2 ma 2
The operation of the Hamiltonian on the wave function is (1)2 π 2 h2 ( 2 ) 2 π 2 h2
the Schrödinger equation. So, here E = 2 × 2
+ 1 ×
2 ma 2 ma 2
Solution exists for the time independent Schrödinger
equation only for certain values of energy and these Corresponding to Corresponding to
values are called eigen values of energy. The energy 2 particles 1 particle (↑ in n = 2 state)
eigen value of two dimensional infinite square well (↑↓ in n = 1state)
potential of side a is given by
3 π 2 h2
∴ E=
n12π 2h2 n 22π 2h2 ma 2
2
+ 2
2ma 2ma
15. (d) Boundary Conditions for the Electric Field
n1 π h + n 22π 2h2 25π 2h2
2 2 2
⇒ = We know that the curl of an electrostatic electric field is
2ma 2 ma 2 always zero. Hence, using the same argument, we used
2 2
⇒ n1 + n 2 = 50 …(i) to investigate the parallel component of D (electric
n1 = 1, n 2 = 7  displacement flux) we can say that

This is possible, if n1 = 5, n 2 = 5  3 fold degeneration ∫ E ⋅ d l = (E2, || − E1, || ) ∫ = ∫ ( ∇ × E ) ⋅ d s = 0
c s
n1 = 1 and n 2 = 7 ⇒ E2, || = E1, ||
13. (d) The group velocity of a wave is the velocity with which Hence, at the boundary, tangent components remain
the overall shape of the waves, amplitude known as the same, but normal component will be changed for making
modulation or envelope of the wave propagates through flux conserved.
 5 
space. ⇒ E 2 =  3$i − 5$j + 4 × k$  kV m−1
 4 
The group velocityvg is defined by the equation
∂ω E 2 = ( 3 i − 5 j + 5 k) kV m−1
$ $ $
vg =
∂k
16. a. Given, V (x ) = 3x 2 − 2x 3
The phase velocity of a wave is the rate at which the
dV
phase of the wave propagates in space. This is the = 6x − 6x 2
velocity at which the phase of anyone frequency dx
component of the wave travels. dV
For maxima or minima, =0
ω dx
vp =
k ⇒ 6x − 6x 2 = 0
Given, ω 2 = ω 20 + c 2k 2 ⇒ 6x (1 − x ) = 0
∂ω ∂ω ∂ ⇒ x = 0 and x = 1
vg = = 2ω = (ω 20 + c 2k 2 )
∂k ∂k ∂k d 2V
> 0 at x = 0, hence at x = 0, V will be minimum
∂ω dx 2
⇒ 2ω = 0 + 2c 2k
∂k d 2V
 ω   ∂ω  2 and < 0 at x = 1, hence at x = 1, V will be maximum
⇒    =c dx 2
 k   ∂k 
Vmax = 3(1)2 − 2(1)3 = 1 J
⇒ v p ⋅v g = c 2
and V (x ) = 0
1 ⇒ 3x 2 − 2x 3 = 0
14. (c) Since, three non-interacting particles each of spin and
2 3
mass m are moving in a one-dimensional infinite ⇒V will be 0, at x = 0 and x =
2
potential well given by
Plot of V (x ) with respect to x
 0 for 0 < x < a V (x)
V (x ) = 
∞ for x ≤ 0 and x ≥ a
Therefore, only possible configuration is two particles 1
present in ground state and one particle in excited state
as shown in figure. 3/2
x
0 1
n=2 First excited state

n=1 Ground state


Solved Paper 2010 ●
91

4  ∂  x  ∂  y 
b. When the particle is released from x = m, its velocity is ∇ × F|x 2 + y 2 ≥ r 2 = α r02   2 +
3 2  2 2 
0
 ∂ x  x + y  ∂ y  x + y 
4
zero, hence KE of the particle at x = m is zero.  r −x2 2
x −y 
2 2
3 = αr 20  2 + 2 =0
4 (x + y )
2 2
(x + y 2 )2 
Total energy of the particle at x = m
3 b. Stoke’s Theorem
= PE + KE Let S be an oriented smooth surface that is bounded
  4 2 by a simple, closed, smooth boundary curve C with
 4 
3
= 3  − 2   + 0 positive orientation. Also, let F be a vector field, then
   3
 3 
144 − 128 16
∫C F ⋅ d r = ∫∫ curl F ⋅ d S
= = J S
27 27 In this theorem note that the surface S can actually be
Let v be the velocity of the particle, when it reaches any surface, so long as its boundary curve is given by
3 3 1 C. This is something that can be used to our
x = m, KE of the particle at x = m is mv 2 advantage to simplify the surface integral on occasion.
2 2 2
3 Now, ∫ F ⋅ d l = ∫∫ ( ∇ × F) ⋅ d S
Total energy of the particle at x =
2
  3 2  3  1
3 = ∫∫ ( ∇ × F)d S + ∫∫ ( ∇ ⋅ F) ⋅ d S
= PE + KE = 3  − 2   + mv 2 2
+ y 2
≤ r 02 2
+ y 2 ≥ r 02
   2 x x
 2  2
1 2  2 
= ∫∫ 2α d S + 0 = 2α ∫∫ d S
= 0 + × v2 m = kg x 2 + y 2 ≤ r 02 x 2 + y 2 ≤ r 02
2 3  3 

=
v 2 ⇒ ∫ F⋅ d l = 2 α ( πr02 )
3 18. a. Free body diagram of the block
Now, applying conservation of mechanical energy T
2
v 16
⇒ =
3 27
(m=2 kg)
16 4
v2 = ⇒ v = ms−1
9 3
mg
17. a. We define the curl of a vector field F , as r ∇ × F as the
Free body diagram of the cylinder
cross product of ( ∇ ) with F. So, if F = < P , Q , R >, then
N
$i $j k$ f
∂ ∂ ∂
Curl F = ∇ × F = °
∂x ∂y ∂z 30
s in
P Q R mg mg cos 30° m=10 kg
30°
According to given question,
mg
F(r ) = α(x $j − y $i ) for x 2 + y 2 ≤ r02 (region I)
When the block moves down by 0.01 m, then the
(x $j − y $i )
F(r ) = αr02 2 for (x 2 + y 2 ) ≥ r02 (region II) cylinder translational displacement along the incline
x + y2 0.01
will be m = 0.005 m.
$i $j k$ 2
∂ ∂ ∂ b. From the free body diagram, we can T
Now, ∇ × F | x 2 + y 2 ≤ r 2 = = 2 α k$
0 ∂x ∂y ∂z write the equation,T − mg = ma
− αy αx 0
where, T is the tension in the string
$i $j ⇒ T = ma + mg = 105 . mg = 21N F
k$ (m=2 kg)
∂ ∂ ∂ For the translational motion of the cylinder B
and ∇ × F | x 2 + y 2 ≥ r 2 = mg sin θ − (T + f ) = ma mg
0 ∂x ∂y ∂z
− αr02y αr02x ⇒ f = mg sin θ − T − ma
0 or f = 10 × 10 × sin 30° − 21 − 10 × 0.05 × 10
x2 + y2 x2 + y2
or f = 24 N
92 IIT JAM Physics Solved Papers & Practice Sets

19. a. Given that change in the wavelength after scattering by Polarisation vector, p = χε 0E
an angle θ, ⇒
Q
p = χε 0
p +p θ 4πεR 2
∆λ = 2cλ 0 0 sin2 …(i) Now, surface bound charge density, σb = p ⋅ n$
E − cp 2
χε 0Q − χε 0Q
The value of ∆λ = ∆λ ′ when the electron is moving in a = at the outer surface and at the inner
direction opposite to that of the incident photon with 4πεR22 4πεR12
momentum p. surface.
p − p θ
∆λ ′ = 2cλ 0 0 sin2 …(ii) b. Electrostatic energy is given by
E +cp 2 1
U = εE 2
If the electron is at rest ⇒ p = 0 2
and E = m0c 2 …(iii) Inside the dielectric, E =
Q
2 2 4πεR 2
p c + m02c 4= m0c 2
Total energy is given by ∫ U ⋅ dV
p0 θ
⇒ ∆λ = 2cλ 0 sin2 [from Eqs. (i) and (iii)]
m0c 2 2 dV = small volume
λ p R2 1 εQ 2
⇒ ∆λ = 0 0 [1 − cos θ] =∫ 4πR 2dR [dV = 4πR 2 ⋅ dR ]
R1 2 ( 4 πε ⋅ R )2
m0c
R
∆λ =
h
[1 − cos θ] …(iv) Q2 2 1
4πε R∫ R 2
⇒ dR
m0c
 h  1

p 0 = λ de- Broglie equation Q2 1 1


 0  Electrostatic energy =
4πε R − R  [Q ε = ε 0εr ]
 1 2
From Eq. (iv), we can say that, ∆λ is independent from
the wavelength of incident photon. Q2 1 1
= R − R 
h 4πε 0εr  1 2
b. Using, ∆λ = (1 − cos θ )
m0c
21. a. Using relativistic velocity transformation, when the
According to the given question, θ = 90° observer is located in S, then relative speed of A with
h respect to B in observer frame S,
⇒ ∆λ = (1 − cos 90° )
m0c 0.8 c − 0.6 c 0.2c
v AS = = ms−1
⇒ ∆λ =
h 0.8 c × 0.6 c 1.48
1+
m0c c2
Resolving power of optical instrument The length to be covered
λ 2000 × 10−10
= = = 50 1− ( 0.8)2 + 50 1− ( 0 . 6)2
∆λ h
= 70m
m0c
70 70 × 1.48
2000 × 10−10 × 9.1 × 10−31 × 3 × 108 Time taken = = = 1.726 × 10−6 s
= 0. 2 c 0 . 2 × 3 × 108
6.62 × 10−34 1.48
2 × 9.1 × 3 × 10
= = 83 b. Speed of A in relative to observer stationed
6.62
vA − vB 0.8 c − 0.6 c
20. a. In a homogeneous linear and isotropic dielectric A to B v BA = =
v A vB
1− 2 0.48 c 2
medium, the polarisation is aligned with and proportional 1−
to the electric field E ⇒ p ∝ E ⇒ p = χε 0E
c c2
where, χ is the electric susceptibility of the medium. = 0 . 3846 c m/s
Using Maxwell’s relation to find the electric field Length covered to complete by overtake by spaceship
∇ .D = ρ
⇒ ( ∇ ⋅ D) dV = ρdV (v BA )2
A = 50 + 50 1−
c2
⇒ ∫ ( ∇ ⋅ D)dV = ∫ ρ dV = ∫ (D ⋅ d S) = Q
or D ⋅ 4 πR 2 = Q = 50 + 50 1− ( 0 .3846)2
Q
⇒ D= , (where, R1 < R < R2) = 96.153 ms−1
4 πR 2
Q r$ 96.153
⇒ E= R1 < R < R2 Total time taken = = 8.3 × 10−7 s.
4πεR 2 0.3846 c
Solved Paper 2010 ●
93

22. In region I The Schrödinger equation is given by The voltage at point A is


2 2 Vin × 10 kΩ 5 × 10
− h d ψ Ι V0 VA = = V = 2 .5 V
= ψΙ 10 k Ω + 10 k Ω 20
2m dx 2 2
d 2ψ Ι mV0 When switch is closed,
⇒ + 2 ψΙ = 0 …(i)
dx 2 h +5V +5V +5V
The solution of Eq. (i), we get
10 kΩ 500 Ω 500 Ω
 mV 
ψ Ι (x ) = Aeikx + Be −ikx …(ii) k = 2 0  C
 h 
A Convert in
In region II B
E standard RB
2 2 form
− h d ψ ΙΙ V 10 kΩ 260 Ω 260 Ω
+ V0ψ = 0 ψ ΙΙ
2m dx 2 2
2
d ψ ΙΙ mV0
⇒ − 2 ψ ΙΙ = 0 …(iii)
dx 2 h 10 × 10 100
The solution of Eq. (iii), we get Here, R B = = = 5 kΩ
10 + 10 20
ψ ΙΙ (x ) = Ce −kx + Dekx +5V

By the property of wave function, it should be finite 500 Ω IC


IB
everywhere
C
⇒ D = 0, ⇒ ψ ΙΙ (x ) = Ce −kx RB = 5 kΩ B
2.5 V β = 50
At x = 0, ψ Ι ( x = 0) = ψ ΙΙ (x = 0) A
E
VBE =
⇒ A+B =C …(iv) IE = IB + IC
0.7V 260 Ω
dψ Ι dψ ΙΙ
Also, =
dx x =0 dx x =0

⇒ i ( A − B ) = − C ⇒ A − B = iC .…(v) Applying KVL equation, we get


Solving Eqs.(iv) and (v), we get 260
−2 . 5 + IB × 5 + 0 .7 + (IB + IC ) × =0
1000
2A = (1+ i ) C
Here, IC = βIB = 50 × IB
2
or C= A ∴ − 2.5 + IB × 5 + 0 .7 + (IB + 50IB ) 0.26 = 0
1+ i
∴ − 2.5 + IB × 5 + 0 .7 + (IB + 50IB ) 0.26 = 0
Also, A+B =C
B C 2 ⇒ −1. 8 + 5 I B + 13 . 26 IB = 0
or 1+ = = ⇒ −1.8 + 15.06 IB = 0
A A 1+ i
B 2 1− i ⇒ 15.06 IB = 1.8
or = −1=
A 1+ i 1+ i ∴ IB = 0 .12 mA
2 2
 | B |
2 As, IC = βIB
B B
∴ Reflection coefficient, R =   = =  ⇒ IC = 50 × 0 .12 = 6 mA
 A A  | A |
2 2
Now, IE = IB + IC = 0 .12 + 6 = 6 .12 mA
1+ ( −1) − 2 i  2i
= = − =1  260 
 ∴ VA = 0 .7V + IE ×  V = 0 .7 + ( 6 .12 × 0 . 26)
 1− 1+ 2 i  2i  1000
∴ Transmission coefficient,T = 1− R = 1− 1 = 0. ⇒ VA = 2.3V.
23. a. Here, β DC = 50,V BE = 0.7V, Vin = 5 V b. To draw DC load line, consider the expression
VCE = VCC − IC (RC + RE ) …(i)
When switch is open,
+5V To locate point A on VCE-axis, put I C = 0 in Eq. (i), then
IC
10 kΩ
6.57 mA B
+5V 10 kΩ
A VA

3.07mA Q
10 kΩ 10 kΩ
Switch
separates
A
VCE
O 2.66 V 5V
94 IIT JAM Physics Solved Papers & Practice Sets

VCE = VCC = + 5V r


⇒ B ⋅ 2π   = ∫ Jd ⋅ d S
Hence, OA = 5V  2 S

To locate point B on I C -axis, putVCE = 0 in Eq. (i), we get πε 0 ω z1V0 sin ωt r 2


=
0 = VCC − IC (R C + R E ) 4( z 0 + z1 cos ωt )2
VCC  ε ω z V sin ωt 
IC = B= 0 1 0 r
RC + RE  4 ( z 0 + z1 cos ωt ) 
5
= 25. According to given p -V curve, a → b is adiabatic process.
500 + 260
γ
∴ I C = 6 . 57 mA V  p
∴ p1V1γ = piVi γ ⇒  1 = i
Join points A and B to obtain DC load line as shown in Vi  p1
the figure. Cp 5 / 2R 5
V × 10 5 ⇒ γ= = =
Now, VB , i.e. V2 = CC = × 10 = 2 . 5 V CV 3 / 2R 3
10 + 10 20
5/ 3
On applying base-emitter loop equation, V1 pi
⇒   = …(i)
V2 = VBE + I E RE Vi  p1
V −V 2.5 − 0.7 b → c is the isothermal process
⇒ IE = 2 BE = = 3 .07 mA
RE 260 p1V1 = p 2 V2
and IC ≅ IE = 3.07 mA V2 p1
=
So, VCE = VCC − IC (RC + R E ) [from Eq. (i)] V1 p 2
∴ VCE = 5 − 3.07 mA(500 + 260) Ω c → d is the adiabatic process,
= 5 − 2 .33
⇒ pf Vf 5/ 3 = p 2 V25/ 3
⇒ VCE = 2 .66 V
5/ 3
Thus, operating point, i.e. Q-point is 2 .66 V, 3 .07 mA. p 2  Vf 
= 
pf V2 
24. a. Displacement current is a quantity appearing in
γ
Maxwell’s equations, i.e. defined in terms of the rate of Vi /B  p2
change of electric displacement field. ⇒   = 2
 V2  B pi
∂E
Using displacement current, Jd = ε 0 5/ 3
∂t  Vi   p 
⇒   =  1/ 32  …(ii)
V0 V0 V2   B pi 
and E= ⇒ E=
d z 0 + z1 cos ωt Multiplying Eqs. (i) and (ii), we get
∂  V0  5/ 3
Jd = ε 0   V1 Vi  pi p2 p 1
∂ t  z 0 + z1 cos ωt  ⇒   = ⋅ = 2 1/ 3
Vi V2  p1 B1/ 3pi p1 B
εV0 z1ω sin ωt
Jd = 5/ 3
( z 0 + z1 cos ωt )2  V1  V  1
⇒   =  1  1/ 3 [from Eq. (i)]
Displacement current between the plates through V2  V2  B
r  V1  1
concentric circle of radius ⇒
2   = 2
V2  B
2
r
Id = Jd ⋅ π   Total work done, W = Wa→b + W b→c + Wc →d
 2
ε 0z1ω V0 sin ωt πr 2 piVi − p1V1 V p V − pf Vf
Id = = + RT0 In 2 + 2 2
( z 0 + z1 cos ωt )2 4 2/ 3 V1 2/ 3
b. The magnetic field vector along the circumference of 3
= [ p V − p1V1 + p 2 V2 − pf Vf ] + R T0 ln B 2
r 2 i i
circle of radius remains same using Stoke’s theorem
2 3
= [ p V − pf V ] + 2R T0 ln B [Q p1V1 = p 2 V2]
∫ ⋅ d l = ∫ ( ∇ × B)d S and
B 2 i i
3
= [ p V − B pi Vi ] + 2R T0 ln B
∫ ( ∇ × B) .d S = J + Jd = ∫ Jd ⋅ d S (Q J = 0) 2 i i
3
⇒ W = p V (1− B ) + 2RT0 In B .
∫ B⋅ d l = ∫ Jd ⋅ d S 2 i i
4

Solved Paper
2009
IITJAM Physics
MM : 300 Time : 3 hrs

n
In this question paper, there are total of 25 questions carrying 300 marks of 3 hours duration.
n
Questions 1-15 (Objective questions) carry 6 marks each and questions 16-25 (Subjective questions) carry
21 marks each.
n
Each objective question has four choices for its answer i.e. (a), (b), (c) and (d). Only one of them is the
correct answer.
n
There will be negative marking for wrong answers to objective questions. The following marking scheme
for objective questions shall be used:
(a) For each correct answer, you will be awarded 6 (Six) marks.
(b) For each wrong answer, you will be awarded –2 (Negative two) mark.
(c) Multiple answers to a question will be treated as a wrong answer.
(d) For each un-attempted question, you will be awarded 0 (Zero) mark.
n
Do not write more than one answer for the same question. In case you attempt a subjective question
more than once, please cancel the answer(s) you consider wrong. Otherwise, the answer appearing last
only will be evaluated.
n Clip board, log tables, slide rule, calculator, cellular phone and electronic gadgets in any form are not
allowed.

1  ∂V 
1. Isothermal compressibility K T of a substance is defined as K T = −   . Its value for
V  ∂p  T
n moles of an ideal gas will be
1 n 1 n
(a) (b) (c) − (d) −
p p p p

2. A space crew has a life support system that can last only for 1000 hours. What minimum
speed would be required for safe travel of the crew between two space stations separated by
a fix distance of 1.08 × 1012 km?
c c c c
(a) (b) (c) (d)
3 2 2 5
96 IIT JAM Physics Solved Papers & Practice Sets

3. An oscillating voltage V (t ) = V0 cos ωt is applied across a parallel plate V(t)=V0 cos ω t


capacitor having a plate separation d. The displacement current density
through the capacitor is
ε 0 ωV0 cos ωt ε 0 µ 0V0 cos ωt d
(a) (b)
d d
ε 0 µ 0ωV0 sin ωt ε 0 ωV0 sin ωt
(c) (d) −
d d

4. Two spherical nuclei have mass numbers 216 and 64 with their radii R1 and R 2 , respectively.
R1
The ratio is
R2
(a) 1.0 (b) 1.5 (c) 2.0 (d) 2.5

5. In the Fourier series of the periodic function as shown in the figure below:

π 2π


f (x ) = | sin x | = ∑ [α n cos nx + β n sin nx ]
n=0

Which of the following coefficients is non-zero?


(a) α n for odd n (b) α n for even n (c) β n for odd n (d) β n for even n

6. A particle is moving in space with O as the origin. Some possible expressions for its position,
velocity and acceleration in cylindrical coordinates (ρ, φ, z ) are given below. Which one of these
is correct?
dρ dφ $ dz
(a) Position vector, r = ρρ& + ρφφ& + zz& and velocity, v = ρ$ + ρ φ+ z$
dt dt dt
dρ dφ $ dz d 2ρ d  dφ $  d 2 z
(b) Velocity, v = ρ$ + ρ φ+ z$ and acceleration, a = 2 ρ$ + ρ φ + z$
dt dt dt dt dt  dt  dt 2
dρ dφ $ dz
(c) Position vector, &r = ρρ& + zz& and velocity v = ρ$ + ρ φ+ z$
dt dt dt
dρ d dz
(d) Position vector, r = ρρ$ + ρφφ$ + z z$ and velocity, v = ρ$ + (ρφφ$ ) + z$
dt dt dt

7. Which one of the following is an incorrect Boolean expression?


(a) P Q + PQ = Q (b) ( P + Q ) ( P + Q ) = P
(c) P( P + Q ) = Q (d) ( P QR + PQR + PQR + PQR ) = Q

8. Monochromatic X-rays of wavelength 1Å are incident on a simple cubic crystal. The first order
Bragg reflection from (311) plane occurs at an angle of 30° from the plane. The lattice
parameter of the crystal (in Å) is
11
(a) 1 (b) 3 (c) (d) 11
2
1 $
9. An electric field E (r ) =
(αr + β sin θ cos φφ$ ) exists in space. What will be the total charge
r
enclosed in a sphere of unit radius centered at origin?
(a) 4πε 0α (b) 4πε 0(α + β ) (c) 4πε 0(α − β ) (d) 4πε 0β
Solved Paper 2009 ●
97

10. A thin massless S rod of length 2l has equal point masses m attached at its ω

ends (as shown in figure). The rod is rotating about an axis passing through d m
its centre and making angle θ with it. The magnitude of the rate of change of θ l
 dL 
its angular momentum  is
 dt 
(a) 2 ml 2ω 2 sin θ cos θ (b) 2 ml 2ω 2 sin θ
(c) 2 ml 2ω 2 sin2 θ (d) 2 ml 2ω 2 cos 2 θ

11. A battery with a constant emf E and internal resistance r provides power to an external circuit
with a load resistance made up by combining resistance RL and 2RL in parallel. For what
value of RL will the power delivered to the load be maximum?
r r 2 3
(a) RL = (b) RL = (c) RL = r (d) RL = r
4 2 3 2

12. Among the following displacement versus time plots, which ones may represent an
overdamped oscillation?
P Q R S

x(t) x(t) x(t)


x(t) t

t t t

(a) P and Q (b) P and R (c) P and S (d) P, R and S

13. A wave packet in a certain medium is constructed by superposing waves of frequency ω to


around ω 0 = 100 and the corresponding wave number k with k 0 = 10 as given in the table
below:
ω k
81.00 9.0
90.25 9.5
100.00 10.0
110.25 10.5
121.00 11.0

Find the ratio v g /v p of the group velocity v g and the phase velocity v p .
1 3
(a) (b) 1 (c) (d) 2
2 2

14. A box containing 2 moles of a diatomic ideal gas at temperature T0 is connected to another
identical box containing 2 moles of a monoatomic ideal gas at temperature 5T0 . There are no
thermal losses and the heat capacity of the boxes is negligible. Find the final temperature of
the mixture of gases (ignore the vibrational degrees of freedom for the diatomic molecules).
(a) T0 (b) 1.5T0 (c) 2.5T0 (d) 3T0

15. Moment of inertia of a solid cylinder of mass M, height h and radius r


r
about an axis (shown in the figure by dashed line) passing through its
centre of mass and perpendicular to its symmetry axis is
h
1 1 1 1
(a) Mr 2 + Mh2 (b) Mr 2 + Mh2
4 12 2 8
1 1 1 2 1
(c) Mr 2 + Mh2 (d) Mr + Mh2
2 12 2 4
98 IIT JAM Physics Solved Papers & Practice Sets

16. A parallel beam of light of diameter 1.8 cm contains two wavelengths 4999.75 Å and
5000.25 Å. The light is incident perpendicularly on a large diffraction grating with 5000 lines per
centimetre.
a. Using Rayleigh criterion, find the least order at which the two wavelengths are
resolved.
b. What will be the angular separation (in radians) of the two wavelengths at order
n = 2?

17. A block of mass M is free to slide on a frictionless horizontal floor. The block has a cylindrical
cavity of radius R in the middle of it. The Centre of Mass (CM) of the block lies on the dashed
line passing through the centre of the cavity (as shown in the figure). Initially, the CM of the
block is at a horizontal distance X 1 from the origin. Now a point particle of mass m is released
from point A into the cavity. There is negligible friction between the particle and the cavity
surface. When the particle reaches at point B, the CM of the block is at a distance X 2 from the
origin. Find the value of (X 2 − X 1 ).

A B
R

X1 X2

18. The electric field of an electromagnetic wave propagating through vacuum is given by
E (r, t ) = E 0 cos (100 3 πx − 100πy − ωt ) z$
a. What is the wave vector k? Hence, find the value of ω.
b. At the time t = 0, there is a point charge q with velocity v = v 0 x$ at the origin. What is
the instantaneous Lorentz force acting on the particle?

19. Consider two infinitely long wires parallel to the Z-axis carrying the
Y
same current I. One wire passes through the point L with
coordinates ( −1, 1) and the other through M with coordinates ( −1, − 1) L
x
(–1, 1)
in the XY- plane as shown in the figure. The direction of the current (–1,–1) N
X
in both the wires is in the positive Z-direction. M

a. Find the value of ∫ B ⋅ d l along the semi-circular closed path


of radius 2 units shown in the figure.
b. A third long wire carrying current I and also perpendicular to the XY- plane is placed at
a point N with coordinates (x, 0), so that the magnetic field at the origin is doubled.
Find the value of X and the direction of the current in the third wire.

20. A particle of mass m is confined in an one-dimensional box of unit length. At time t = 0, the
wave function of the particle is
Ψ (x, 0) = A sin 2πx cos πx, where A is the normalisation constant.
a. Write the wave function Ψ (x, t ) at a later time t.
b. Find the expectation values of momentum and energy at t = 0.
Solved Paper 2009 ●
99

21. For the transistor circuit shown in the figure β DC = 100 and V BE = 0.7 V. Determine the base
current IB , the collector-emitter voltage V CE , the emitter voltage VE , the base voltage VB and
the saturation current IC sat .
+ 20 V

2 kΩ
450 kΩ
V0
Vi 10 µF
10 µF βDC=100
1 kΩ

 a 
22. The equation of state of one mole of a van der Waals’ gas is  p +  (V − b ) = RT and its
 V 2
a
internal energy U (T , V ) is given by U (T , V ) = U 0 + CV T − , where, U 0 and CV can be taken as
V
constants.
a. Prove that in a reversible adiabatic process, the temperature and volume satisfy the
equation T (V − b )R / CV = constant.
b. Calculate the change in entropy of the gas, when it undergoes a reversible isothermal
expansion from volume V 0 to 2V 0 .
0 1
23. a. Find the normalised eigen vector a of the matrix, M =  , corresponding to its positive
 1 0
eigen value.
0 −i 
b. The normalised eigen vectors of the matrix N =   are β 1 and β 2 with the eigen
 i 0
values λ 1 and λ 2 , respectively and λ 1 > λ 2 . If the eigen vector a obtained in part (i) is
expressed as a = P β 1 + Q β 2 , find the constants P and Q.

24. A particle of mass m is thrown vertically up from the ground with initial speed v 0 . As it moves,
it experiences a drag force | Fdrag | = kv 2 , where v is the speed of the particle and k is a
constant. Up to what height does the particle go and what is its speed, when it reaches the
ground again?

25. Consider the radioactive transformation A → B → C with decay constant λ A and λ B for
elements A and B, C is a stable element. Assume that at t = 0, N A = N 0 , NB = 0 and NC = 0,
where, N A , NB and NC are the number of atoms of A, B and C, respectively.
a. Show that at any later time t the number of atoms NB of elements B will be
N 0λ A −λ t −λ t
NB (t ) = [e A − e B ].
λB − λ A
b. Sketch qualitatively the variation of N A , NB and NC with time on three separate plots.

Answers
1. (a) 2. (b) 3. (d) 4. (b) 5. (b) 6. (a) 7. (c) 8. (d) 9. (a) 10. (a)
11. (d) 12. (c) 13. (d) 14. (c) 15. (a)
Answers with Explanations
1. (a) Isothermal compressibility is the change in volume of a 4. (b) Nuclear size is defined by nuclear radius also called rms
system as the pressure changes while temperature charge radius. It can be measured by the scattering of
remains constant. electrons by the nucleus.
For ideal gas, pV = nRT Relation between nuclear radius and the mass number
nRT A for heavier nuclear ( A > 20) is
⇒ V =
p R = R0A1/ 3 …(i)
1  ∂V  1 nRT where, R0 is an empirical constant R0 = 12
. Fm
KT = −   =−
V  ∂p  T V ( −p 2 ) R1  R0 A1 
1/ 3

pV 1 ⇒ = 
= 2 = R2  R0 A2 
pV p 1/ 3
R1  216 6
3 ⇒ =  =
2. (b) Time of travel between the two stations, t = 10 h R2  64  4
t = 103 × 3600 s ⇒ R1 : R2 = 1.5
If v be the speed of space crew, then the 5. (b) Mathematical expression for Fourier series
separation between both space stations should ∞
 α 
be equal tovt …(i) A0 + ∑ [ An cos(nx ) + Bn sin (nx )]  A0 = 0 
 2
But the separation also depends on velocity of crew n =1

according to the relation The period of given function f (x ) = | sin x | is π


2
v 1 π 1 π 2
L = Lo 1− …(ii) α 0 = ∫ | sin x | dx = ∫ sin x dx = ≠ 0
c2 π 0 π 0 π
From Eqs. (i) and (ii), we get 2 π 2 π
A n = ∫ | sin x | cos nx dx = ∫ sin x cos nx dx
π 0 π 0
v2
Lo 1− = 3600 × 103 × v 1 π
c2 = ∫ [sin(n + 1)x − sin (n − 1)x ] dx
π 0
v2 π
. × 10(12 + 3) 1−
⇒ 108 = 3.6 × 106 v 1  cos (n − 1)x cos (n + 1)x 
c2 = −
π  n −1 n + 1  0
 3.6 × 106 × 3 × 108 
⇒ c 2 −v 2 =  v 1 2 2 
 . × 1015
108  = − if n = even
π n − 1 n + 1
or c 2 −v 2 = v 2 1
or 2v 2 = c 2 = [ 0 − 0], if n is odd.
π
or v =
c ⇒ An = 0 for n = odd.
2 2 π

3. (d) In electromagnetism, displacement current is a quantity


and Bn =
π ∫0 f (x ) sin nx dx
π
appearing in Maxwell’s equations that is defined in terms 1  sin (n − 1)x sin (n + 1)x 
=  n −1 −
of the rate of change of electric displacement field. π  n + 1  0
Displacement current has the unit of electric current
density and it has an associated magnetic field just as ⇒ Bn = 0 for all n.
∂E 6. (a) The position vector of a point in coordinate system
actual current does. It is given by Jd = ε 0
∂t
r = x $i + y $j + z k$
V
Electric field inside a capacitor, E = A cylindrical coordinate system is a three-dimensional
d
V0 cos ωt coordinate system that specifies point positions by the
E= distance from a chosen reference axis, the direction
d
from the axis relative to a chosen reference direction
∂ V0 cos ωt 
⇒ Jd = ε 0 and the distance from a chosen reference plane
dt  d  perpendicular to the axis.
− ε 0 ω V0 sin ωt
= ⇒ r = ρ cos φ $i + ρ sin φ $j + z k$
d
Solved Paper 2009 ●
101

∂r α β  α − β sin φ
= cos φ $i + sin φ $j = e$ ρ …(i) =  2 − 2 sin φ =
∂p r r  r2
∂r

= − ρ sin φ $i + ρ cos φ $j ⇒ Q enclosed = ε 0 ∫ ( ∇. E)dφ
α − β sin φ 2
⇒ e$ φ = − sin φ $i + cos φ $j …(ii) = ε0 ∫ r2
. r sin θ dr dθ dφ
∂r ∂r = ε 0 ∫ (α − β sin φ ) sin θ dr dθ dφ
= k ⇒ e$ z =
∂z ∂z
∂r $ Q enclosed = 4π ε 0α
=k …(iii)
∂z 10. (a) Angular momentum ( L) It is related to the rotation or
From Eqs. (i), (ii) and (iii), we get revolution of matter. It is in fact, a measure of the
r = x $i + y $j + z k$ quantity of rotation of a system of matter.
= ρ cos φ [cos φ e$ ρ − sin φ e$ φ ] ⇒ L = r × P = r × (m v )
⇒ | L | = l sin θ(mlω sin θ )
+ ρ sin φ [sin φ e$ e + cos φ e$ φ ] + z e$ z
⇒ | L | = mωl 2 sin2 θ
r = ρe$ ρ + ze$ z
∂r dρ $ d | L|  dθ 
= eρ + ρ
d $
eρ +
dz $ = mωl 2. 2 sin θ cos θ  
ez dt  dt 
dt dt dt dt
ω
v = ρ e$ ρ + ρφ e$ φ + z e$ z
r=l sin θ
dv ..
= (ρ − ρφ& 2 ) e$ ρ + (ρφ&& + 2ρ& φ& ) e$ φ + z&& e$ z m
dt
θ
d 2ρ d  ρ dφ $  d 2 z $ l
= 2 e$ ρ +  φ + ez
dt dt  dt  dt 2
7. (c) Option (c) is incorrect.
P (P + Q ) = P ⋅ P + PQ
= P + PQ = P (1+ Q ) = P dL
⇒ = 2ml 2ω 2 sin θ cos θ
dt
8. (d) Using Bragg’s law, 2d sin θ = nλ …(i)
According to question,
11. (d) According to the given question,
h = 3, k = 1, and l = 1 RL
a
⇒ d =
h + k 2 + l2
2
2RL
a a
⇒ d = =
( 3)2 + (1)2 + (1)2 11
r
and n = 1 E
Putting the value of n and d in Eq. (i), we get
a
2× sin 30° = 1 × 10−10m
11 2RL/3
⇒ a = 11 A °

9. (a) Using Gauss’s law,


α r
ε0 ∫ τ ∇ E d τ = ∫ ρ dτ = Q enclosed = ε 0φ r 2 E
1 1 1
c
= +
1 ∂ 2 1 ∂ Req RL 2RL
∇. E = 2 (r . Er ) + (sin θ Eθ )
r ∂r r sin θ ∂θ 1 2+1
1 ∂ =
+ (Eφ ) Req 2RL
r sin θ ∂φ
α β sin θ cos φ 2RL
⇒ E = r$ + φ Req =
r r 3
R
1 ∂ 1 ∂ 1
∴ ∇. E = 2 (αr ) + +
r ∂r r sin θ ∂θ r sin θ
∂ β sin cos φ  r
V
∂φ  r 
102 IIT JAM Physics Solved Papers & Practice Sets

V 14. (c) Let T be the temperature of the mixture, then


Current in the circuit, i =
R+r
Heat gain = Heat loss
Power dissipated in load R = i 2 R 5 3
2 × × R (T − T0 ) = 2 × R (5T0 − T )
V2 2 2
= R 3
(R + r )2 [using, Q = nCV ∆T , CV for monoatomic gas, R and for
2
V2 5
P = diatomic gas R]
r 2  2
 + 2r + R
R  T ( 3 + 5) = (5 + 15)T0
For maximum power delivered to the load R 20
⇒ T = T0 = 2.5 T0
d r 2  8
 + 2r + R = 0 ⇒ r = R
dR  R  15. (a) Moment of inertia of a solid cylinder about the axis
Using the result, r = R passing through its centre and perpendicular to its
We can find the condition for maximum power delivered symmetry axis.
to the load for the given circuit M
Mass per unit length of the cylinder =
2 RL 3 a
h
⇒ r = Req ⇒ r = ⇒ RL = r
3 2 x
h
12. (c) Equation for damped oscillations,
d 2x dx C
2
+ 2K + ω 20x = 0 …(i)
dt dt
Critical damping provides the quickest approach to zero a′
amplitude for a damped oscillator. But for overdamping
Now consider a disc of thickness dx at a distance x from
of a damped oscillator will cause it to approach zero the axis passing through the centre of mass and hence
amplitude more slowly than for the case of critical moment of inertia of this disc about axis aa′ is
damping. The damping coefficient is greater than the 1
undamped resonant frequency. dI = dm r 2 + dm x 2
4
For overdamping case, oscillator dies down in infinite [using, parallel axes theorem]
times. For overdamped oscillator, the two possible M 
solutions are dm =   . dx
h
( −r + r 2 − ω 20 )t ( −r − r 2 − ω 20 )t
x = A1e + A2 e 1M 2 M 2
⇒ dI = r dx + x dx
Hence, from the given graphs, P and S are representing 4 h h
overdamped oscillation. Total MI of the cylinder

13. (d) The group velocity of a wave is the velocity with which I = ∫ dI
the overall shape of the waves amplitudes known as 1 h /2 M M h /2
∫−h / 2 h r ∫−h / 2x
2 2
modulation or envelope of the wave propagates through ⇒ I= dx + dx
4 h
space. It is defined by equation +h / 2
1M 2 M x 3 
∂ω I= r [dx ] +−hh//22 +
vg = where, ω is the wave’s angular frequency and k 4 h h 3
∂k   −h / 2
is the angular wave number. 1 Mr 2 M 2 h3
I= h+
The phase velocity of a wave is the rate at which the 4 h h 3 8
ω 1 1
phase of the wave propagates in spacev p = 2
I = Mr + Mh 2
k 4 12
According to the table given in question,
16. a. The Rayleigh criterion is the generally accepted criterion
∂ω ∂ [110.25 − 90.25] for the minimum resolvable detail. The imaging process
vg = = = 20
∂k ∂ [10.5 − 9.5] is said to be diffraction limited, when the first diffraction
ω 100 minimum of the image of one source point coincides
vp = 0 = = 10
k0 10 λ
with the maximum of another. For single slit, sin θ =
vg 20 d
⇒ = =2 λ
v p 10 and for circular aperture, sin θ = 1.22 .
d
Solved Paper 2009 ●
103

The two wavelengths are just resolved, when  3$i − $j 


⇒ k$ =  
maximum of one lies at the first minimum of other, 
 2 
λ
which leads R = = nN 1
∆λ k × E = [ − 3E0 cos (100 3πx − 100πy − ωt )$j
λ 5000 2
⇒ n= = =2
∆λ ⋅ N 0.5 × 5000 − E0 cos (100 3πx − 100πy − ωt )$i]
− 3 $j − 1 $i 
b. Now using, d sin θ = nλ ⇒ k × E = E0 cos (100 3πx − 100πy − ωt ) 
 2 2 
For small θ ⇒ sin θ ≈ θ
d θ = nλ E  3 $j − 1 $i 
⇒ B = 0 cos[100 3πx − 100πy − ωt ] −
nλ 2 × 5000 × 10−8 c  2 2 
θ= =
d 1.8 According to Lorentz formula,
θ = 5.5 × 10−5 rad F = q[ E + v × B]
F = qE0 cos (100 3πx − 100πy − ωt )
17. When the particle at the point at the first end, then the
centre of mass of the system (particle + block)  3v 0 
⇒ F = qE0 1 −  cos (100 3πx
M X + M 2(X1 − R )  2c 
XC = 1 1
M1 + M 2  3 v0 $
− 100πy − ωt ) k$ − k
MX1 + m(X1 − R )  2 v 
=
M +m
19. a. Using Ampere’s circuital law,
When the particle reaches to the other end of the block,
the coordinate of centre of mass. ∫ B ⋅ d l = µ 0 ∫ I enclosed
MX 2 + m(X 2 + R ) Y
XC′ =
M +m
As, there is no external force acting on the system, the L
(–1, 1)
centre of mass in horizontal direction of the system will X
remain unchanged ⇒ XC = XC′ M N
(–1, 1)
⇒ M1X1 + m(X1 − R ) = MX 2 + m(X 2 + R )
⇒ MX1 + mX1 − mR = MX 2 + mX 2 + mR
− 2mR
⇒ X 2 − X1 = ⇒ ∫ B ⋅ d l = 2 µ 0I
M +m
b. Magnetic field at O due to conductor L,
Hence, the block will move toward left.
18. a. Given, ∫ B ⋅ dl = µ 0 ∫ I enclosed
E (r, t ) = E0 cos (100 3πx − 100πy − ωt )z ⇒ B × 2π ( 2 ) = µ 0I
⇒ E (r, t ) = E0 cos[(100 3π$i − 100π$j ) µ 0I
⇒ B=
2π 2
Comparing this equation with
E(r, t ) = E0 cos (k ⋅ r − ωt )z µ 0I  $i + $j 
In vector form, BL =  
2π 2  2 
⇒ k = 100 3π$i − 100π$j
Similarly, magnetic field due to conductor at M,
⇒ | k | = (100 3π )2 + (100)2 = 200π
B × 2π ( 2 ) = µ 0I
ω
Now using, v p =  − $i + $j 
k µ 0I
⇒ BM =  
ω 2π 2  2 
⇒ C= ⇒ ω = KC  
K
Net magnetic field at O,
ω = 200π × 3 × 108 = 6π × 1010 rad/s
B = BL + BM
1 $
b. Using, B = (k × E) µ 0I  i + j 
$ $ µ 0I  − $i + $j  µ I
ω ⇒   +   = 0 ( 2$j )
 k$ × E 2π 2  2  2π 2  2  4π

⇒ B=  µ 0I $
 c  ⇒ B= j

104 IIT JAM Physics Solved Papers & Practice Sets

If third wire is placed at M (x , 0) such that magnetic field ⇒<p> = 0


at O, (i.e. B) is doubled, then 1

µ I < E > = ∫ ψ *Hψ dx


⇒ | BM | = | BL + BM | = 0 0

1
µ 0I µ 0I − h2 d2
⇒ =
2 πx 2 π
⇒ x = 1m =
2π ∫ (sin 3πx + sin πx ) dx (sin 3πx + sin πx )
0
And the direction of current in the third wire should flow h2 1
∫0 (sin 3πx + sin πx ) (9 π
2
in negative Z-axis (− k). $ sin 3πx + π 2 sin πx ) dx
2m
20. a. Wave function of the particle at time t h2  2 1 π 2  5 π 2 h2
⇒ <E>= 9 π , 2 + 2  = 2m
ψ (x , 0) = A sin 2πx cos πx 2m  
Now according to Schrödinger equation, 5π 2h2
∞ ∴ <E>=
2m
∫ | ψ (x , 0)|
2
dx = 1
−∞ 21. Short circuiting the capacitors, the circuit diagram is shown
1
below.
∫ ( sin 2πx ⋅ cos πx ) dx = 1
2 2
⇒ A VCC = 20 V
0
2 1
A
∫ [sin 3πx + sin πx ]
2
⇒ dx = 1 450 kΩ
2 kΩ
4 0
Vo
Vo
2 1 1
A A2 Vi
∫ ∫ sin πx dx
2
⇒ sin 3πxdx + VB IB Outer path
4 0
4 0
1 1 kΩ
A2
+
4 ∫ 2 sin 3πx sin πx = 1
0
VCC − VB
A2
1 1  Base current is, IB = …(i)
⇒ + +0 =0 ⇒ A=2 450 kΩ
4 2 2 
VB = VBE + IE × 1 kΩ
Hence, normalised Schrödinger equation is
⇒ VB = 0.7 + IE × 1 kΩ …(ii)
ψ (x , 0) = 2 sin 2πx ⋅ cos πx = sin 3πx + sin πx
From Eqs. (i) and (ii), we get
2  nπ x  V − [ 0.7 + IE × (1kΩ )]
General form of ψ (x , 0) = sin   IB = CC
L  L 
450 kΩ
1  2  nπ x   1  2  nπ x   IE
⇒ ψ (x , 0) =  sin   +  sin   ⇒ × 450 × 10 = 20 − 0.7 − IE × 103
3
2 L  L   2 L  L  β
 450 × 103 
2  nπ x  ⇒ IE  + 103  = 19.3
If φn(x ) = sin   100
L  L   
1 1 or IE [ 4500 + 103] = 19.3
⇒ ψ (x , 0) = φ 3(x ) + φ1(x )
2 2 19.3 19.3
or IE = 3
= mA = 3.5mA
1
− iE 3t
1 −
iE1t 4500 + 10 5.5
∴ ψ (x , t ) = φ 3(x )e h + φ1(x )e h
I 3.5 mA
2 2 ∴ Base current, IB = E = = 0.035 mA
β 100
n 2 h2 n 2 π 2 h2
En = = For the outer path, we have
8mL2 2mL2
VCC = IE × 2 × 103 + VCE + IE × 103 [Q IC = IE ]
1  − 3 
− iE1t iE t
3
ψ (x , t ) = φ1(x )e h + φ 3(x )e h  = 3 × 10 × IE + VCE
2 
 ⇒ 20 − 3 × 103 × 3.5 × 10−3 = VCE
1
h ∂ ∴ VCE = ( 20 − 10.5) V = 9.5 V
b. < p > = ∫ ψ *pψ dx ⇒ p =
0
i ∂x The emitter voltage
1 VE = IE RE = 3.5 × 10−3 × 1× 103 = 3.5 V
h  ∂ 
<p> = ∫
i 0
( sin 3πx + sin πx )
 ∂x
(sin 3πx + sin πx ) dx
 Base voltage, VB = 0.7 + IE × 103 [from Eq. (i)]
−3 3
1 = 0.7 + 3.5 × 10 × 10
h (sin 3πx + sin πx )2  h
=   = [ 0 + 0] = 0 = 3.5 + 0.7 = 4.2 V
i  2 0 i IC sat = IE = 3.5 mA
Solved Paper 2009 ●
105

22. a. In thermodynamics, a process involving change without 23. a It is sometimes convenient to use a set of normalised
any increase or decrease of entropy. Since, the eigen vectors, which means that, for each eigen vector,
entropy always increases in a spontaneous process, one the sum of the squares of its elements is equal to 1. An
must consider reversible or quasistatic processes. eigen vector may be normalised if we multiply it by (plus
During a reversible process, the quantity of heat or minus) the reciprocal of the square root of the sum of
transferred is directly proportional to the system’s the square of its element.
entropy change. Systems which are thermally insulated 0 1
Given matrix, M =  
from their surrounding undergo process without heat  1 0
transfer such processes are called adiabatic. | M − λI | = 0 ⇒ λ2 − 1 = 0, λ = ± 1
Thus, during an isentropic process there are no For λ = 1(positive eigen value)
dissipative effects and the system neither absorbs nor
gives off heat. For this reason, the isentropic process is − 1 1  a1  a1 
 1 − 1 a  = 0 where, a  be the eigen vector.
called reversible adiabatic process.    2  2
van der Waals equation for one mole ⇒ a1 − a 2 = 0 ⇒ a1 = a 2
 a 1 1
 p + 2  (V − b ) = RT Normalised eigen vector, a = 
 V  2 1
 RT a 0 − i 
⇒ p= −  b. For matrix N = 
V − b V 2  , the characteristic equation is
i 0 
From the first law of thermodynamics, dQ = dU + dW | N − λI | = 0 ⇒ | λ2 − 1| = 0 ⇒ λ = ± 1,
a
⇒ U = U 0 + CVT − a 
V If  1 be the eigen vectors corresponds to λ = 1, then
a a 2
⇒ dU = CV dT + 2 dV
V − 1 − i  a1  0
a  i − 1 a  = 0
dQ = CV dT + 2 dV + pdV    2  
V ⇒ a1 + ia 2 = 0
a RT a
⇒ dQ = CV dT + 2 dV + dV − 2 dV ⇒ a1 = 1 and a 2 = − i,
V V −b V
 a1 1 1 1
= CV dT +
RT ⇒   =   ⇒ β1 = 
dV a 2  i  2 i
V −b
For adiabatic process, dQ = 0 b 
Similarly, if  1 be eigen vectors corresponds to λ = − 1,
RT b2
⇒ CV dT + dV = 0
V −b then
1 − i  b1  0

dT  R 
+  dV = 0 i 1  b  = 0
CV    2  
T V − b 
⇒ b1 = i and b2 = 1
dT R dV
⇒ ∫ T
+∫
CV (V − b )
=0
⇒ β2 =
1 i

2 1
R
loge T + loge (V − b ) = C Now according to question, a = P β1 + Q β 2,
CV
(integration constant) 1 1 P 1 Q i
⇒ T (V − b )R /CV = constant ⇒  =  + 
2  1 2  i  2 1
b. From first law of thermodynamics, ⇒ P + iQ = 1 or 2P = 1 − i
RT
dQ = dU + dW = CV dT + dV 1− i 1− i
V −b iP + Q = 1, P =   and Q =  
 2   2 
dQ dT  R 
⇒ dS( entropy) = = CV +  dV 24. For upward motion of the body,
T T V − b 
ma = − mg − kv 2
Change in entropy in an isothermal process [dT = 0], dv
2V0 ⇒ m = − mg − kv 2
RdV dt
∫ dS = 0 + ∫ V −b
v
V0

 2V − b  F = kv 2
∆S = R loge  0 
 V0 − b  mg
106 IIT JAM Physics Solved Papers & Practice Sets

dv k 25. a. Radioactivity is a spontaneous and self destructive


= −g − v2
dt m activity exhibited by a number of heavy elements
dv ds k occurring in nature.
. = −g − v2
ds dt m − dN
vdv k Rate of disintegration of the element, R =
⇒ = −g − v2 dt
ds m According to question, A → B → C
vdv
⇒ = − ds At t = 0, NA = N0 and NB = NC = 0
k
g + v2 At any time t, for element A
m dNA
0 H = − λ ANA
vdv
⇒ ∫ k 2
= − ∫ ds
For element NB ,
dt
V0 g + v 0
m dNB
where, H is the maximum height achieved by the object = λ ANA − λ BNB
dt
(body). dNA dNA
m  mg  0
v ⇒ = − λ ANA ⇒ = − λ Adt
⇒ H= logv 2 +  dt NA
2k  k 0 NA t
dNA
H=
m v 2 + mg / k 
log i 
⇒ ∫ NA
= − ∫ λ Adt
N0 0
2k  mg / k 
N 
m  kv 2  ⇒ loge  A  = − λ A t
H= log1 + 0   N0 
2k  mg 
⇒ NA = N0e − λ At
For downward motion of the object,
dNB
ma = mg − kv 2 Also, = − λ BNB + λ ANA
dt
F = kv 2 dNB
⇒ + λ BNB = λ AN0e − λ At
dt
t
v ⇒ NB e λ Bt = ∫ λ AN0e( λ B − λ A )t
mg 0
 dv  e( λ B − λ A )t 
t
m   = mg − kv 2
 dt  ⇒ NB e λ Bt = λ AN0  
 ( λB − λ A ) 0
dv k
v =g − v2 λ AN0
ds m ⇒ NB (t ) = [e − λ At − e − λ Bt ]
V H λB − λ A
vdv
⇒ ∫ k 2 ∫
= ds
Now,
dNC
= λ BNB
0g − v 0 dt
m
dNC λBλ A
 k 2 
V
 = N0[e − λ At − e − λ Bt ]
1 g − v  m  g  dt λB − λ A
H = log m  = log  NC t
2  − k  2k k
g − v 2 λBλ A − λ At
 ⇒ ∫ dNC = λB − λ A N0 ∫ (e − e − λ Bt ) dt
m 0  m 
0 0
   e − λ At e − λ Bt 
   λBλ A
m kv 02  m g NC (t ) = N0  +
⇒ log1 +  = log  λB − λ A  − λA λ B 
2k  mg  2k k
g − v 2
 m  b. Variation of NA , NB and NC with time on three separate
kv 2 g k 2 1 plots are shown below.
1+ 0 = ⇒ 1− v =
mg g − k v 2 mg kv 2
1+ 0 NC
m mg NA
NB
N
Velocity of the particle, when it reaches the ground,
v0
v = 1/ 2
 kv 2 
1 + 0 
 mg 
t
Solved Paper
2008
IITJAM Physics
MM : 300 Time : 3 hrs

n
In this question paper, there are total of 25 questions carrying 300 marks of 3 hours duration.
n
Questions 1-15 (Objective questions) carry 6 marks each and questions 16-25 (Subjective questions) carry
21 marks each.
n
Each objective question has four choices for its answer i.e. (a), (b), (c) and (d). Only one of them is the
correct answer.
n
There will be negative marking for wrong answers to objective questions. The following marking scheme
for objective questions shall be used:
(a) For each correct answer, you will be awarded 6 (Six) marks.
(b) For each wrong answer, you will be awarded –2 (Negative two) mark.
(c) Multiple answers to a question will be treated as a wrong answer.
(d) For each un-attempted question, you will be awarded 0 (Zero) mark.
n
Do not write more than one answer for the same question. In case you attempt a subjective question
more than once, please cancel the answer(s) you consider wrong. Otherwise, the answer appearing last
only will be evaluated.
n
Clip board, log tables, slide rule, calculator, cellular phone and electronic gadgets in any form are not
allowed.

1. The product PQ of any two real, symmetric matrices P and Q is


(a) symmetric for all P and Q
(b) never symmetric
(c) symmetric if PQ = QP
(d) antisymmetric for all P and Q

2. The work done by a force in moving a particle of mass m from any point (x, y ) to a neighbouring
point (x + dx, y + dy ) is given by dW = 2 xy dx + x 2dy . The work done for a complete cycle
around a unit circle is
(a) 0 (b) 1
(c) 3 (d) 2π
108 IIT JAM Physics Solved Papers & Practice Sets

3. EFGH is a thin square plate of uniform density σ and side 4a. Four point masses, each of mass
m, are placed on the plate as shown in the figure. In the moment of inertia, matrix I of the
composite system,
Y
E F
(– a,a,0) (– a,a,0)
m m

O X
(–
a ,a
m ,0
)
(– a, a, 0) m
H G

(a) only I XY is zero


(b) Both I XZ and IYZ are zero
(c) all the product of inertia terms are zero
(d) None of the product of inertia terms is zero

4. The chemical potential of an ideal Bose gas at any temperature is


(a) necessarily negative (b) either zero or negative
(c) necessarily positive (d) either zero or positive

5. If the electrostatic potential at a point (x, y ) is given by V = ( 2 x + 4 y ) V, the electrostatic


energy density at that point (in J/ m 3 ) is
(a) 5ε 0 (b) 10 ε 0
1
(c) 20 ε 0 (d) ε 0 (2 x + 4 y )2
2

6. In an inertial frame S, a stationary rod makes an angle θ with the X-axis. Another inertial frame
S′ moves with a velocity v with respect to S along the common XX ′-axis. As observed from S′,
the angle made by the rod with the X ′-axis is θ′. Which of the following statements is correct?
(a) θ ′ < θ
(b) θ ′ > θ
(c) θ ′ < θ, if v is negative and θ ′ > θ if v is positive
(d) θ ′ > θ, if v is negative and θ ′ < θ if v is positive

7. Consider a doped semiconductor having the electron and the hole mobilities µ n and µ p ,
respectively. Its intrinsic carrier density is ni . The hole concentration p for which the
conductivity is minimum at a given temperature is
µn µp
(a) ni (b) nh
µp µn

µp µn
(c) ni (d) nh
µn µp

8. Two coherent plane waves of light of equal amplitude and each of wavelength 20π × 10 −8 m,
π
propagating at an angle of rad with respect to each other, fall almost normally on a
1080
screen. The fringe width (in mm) on the screen is
(a) 0.108 (b) 0.216 (c) 1.080 (d) 2.160
Solved Paper 2008 ●
109

9. A circular disc (in the horizontal XY-plane) is spinning about a vertical axis through its centre O
with a constant angular velocity ω. As viewed from the reference frame of the disc, a particle is
observed to execute uniform circular motion, in the anti-clockwise sense, centered at P. When
the particle is at the point Q, which of the following figures correctly represents the directions of
the Coriolis force Fωr and the centrifugal force Fck .

Fω r, Fck P Fω r P
Q Q Fck
(a) O
(b) O

ω ω

Fω r Fck Fω r
Fck P P
Q Q

(c) O (d) O

ω ω

10. The instantaneous position x(t ) of a small block performing one-dimensional damped oscillation
is x(t ) = Ae −γ t cos (ωt + α ). Here, ω is the angular frequency, γ the damping coefficient, A the
dx 
initial amplitude and α the initial phase. x | x, t = 0 and = v , the values of A and α
dt t = 0
(with n = 0, 1, 2, ...) are
2v (2 n + 1) π v
(a) A = ,α = (b) A = , α = nπ
ω 2 2ω
v (2 n + 1) π 2v (2 n + 1) π
(c) A = ,α = (d) A = ,α =
2ω 2 2ω 2

11. A photon of wavelength λ is incident on a free electron at rest and is scattered in the
backward direction. The fractional shift in its wavelength in terms of the Compton wavelength
λ c of the electron is
λc 2λc 3λ c 2λ c
(a) (b) (c) (d)
2λ 3λ 2λ λ

12. The logic expression for the output Y of the following circuit is
P

Y
Q

R
S

(a) P + Q + QR + S (b) P + Q + QR + S (c) P + Q + QR + S (d) P + Q + QR + S


110 IIT JAM Physics Solved Papers & Practice Sets

13. The activity of a radioactive sample is decreased to 75% of the initial value after 30 days. The
half life (in days) of the sample is approximately [You may use ln 3 ≈ 1.1, ln 4 ≈ 1.4]
(a) 38 (b) 45 (c) 59 (d) 69

14. The ratio of the second-neighbour distance to the nearest neighbour distance in an fcc lattice is
(a) 2 2 (b) 2 (c) 3 (d) 2

15. A thermodynamic system is maintained at constant temperature and pressure. In


thermodynamic equilibrium, its
(a) Gibbs free energy is minimum (b) enthalpy is maximum
(c) Helmholtz free energy is minimum (d) internal energy is zero

16. A thin hollow cylinder of radius and length both equal to L is closed at P
L
the bottom. A disc of radius L/2 is removed from the bottom as shown in
L
the figure. This object carries a uniform surface charge density σ.
Calculate the electrostatic potential at the point P on the axis of the L/2 L
cylinder as shown in the figure.
 dx 
You may use ∫ = ln (x + x 2 + a 2 ) 
 x2 + a2 
 

17. A particle of mass 1 kg is moving in a central force field given by


3 1
F1(r ) = −  2 +  r$
r r
a. Assuming that the particle is moving in a circular orbit with angular momentum 2 J-s,
find out the radius of the orbit.
b. At t = 0, an additional force F2 = − λv, where v is the instantaneous velocity of the
particle is switched ON. Show that the magnitude of its angular momentum after a time
1 2
second is J-s.
λ e

18. An incompressible fluid is enclosed between two horizontal surfaces located at z = 0 and
z = d . The fluid motion is two-dimensional and the velocity field v (x, z, t ) is given by
v (x, z, t ) = u (x, z, t ) x$ + w (x, z, t ) z$
where, u (x, z, t ) are periodic functions of the horizontal coordinate x with wave number k.
 πz 
a. If the vertical velocity w (x, z, t ) = A(t ) cos (kx ) sin   , find the horizontal velocity
d 
u (x, z, t ) using the equation of continuity. What is the vorticity field Ω = ∇ × v?
Y

Z
L

d/2
X
λ
d
b. Find the net fluid flux from a parallel pipe of size λ × L × as shown in the figure,
2π 2
where λ = .
k
Solved Paper 2008 ●
111

19. The wave function ψ n (x ) of a particle confined to a one-dimensional box of length L with rigid
walls is given by
2  nπ x 
ψ n (x ) = sin   , n = 1, 2, 3 K
L  L 

a. Determine the energy eigen values. Also, determine the eigen values and the eigen
functions of the momentum operator.
b. Show that the energy eigen functions are not the momentum operator.

20. A mass and spring system consists of two blocks of mass M and one block of mass m (< M ).
These blocks are connected with two identical springs of spring constant k as shown in the
figure. The system is constrained to move along a straight line on a frictionless horizontal
surface. The spring follows Hooke’s law. Find the angular frequencies of the independent
oscillations (normal modes).
k k
M m M

Now, the masses M and m are interchanged and the new arrangement is shown in the
following figure.
k k
m M m

The ratio of the frequency of the new arrangement to that of the old arrangement, when the
middle block remains stationary is 2. Find the ratio of the frequencies in the two
arrangements, when the middle block oscillates.

21. A half-wave plate and a quarter-wave plate are x


placed between a polariser P1 and an analyser P 2 .
All of these are parallel to each other and z
y
perpendicular to the direction of propagation of
unpolarised incident light as shown in the figure. The
P1 Half-wave Quarter-wave P2
optic axis of the half-wave plate makes an angle of plate plate
30° with respect to the pass-axis of P1 and that of
the quarter-wave plate is parallel to the pass-axis of P1.
a. Determine the state of polarisation for the light after passing through (a) the half-wave
plate and (b) the quarter-wave plate.
b. What should be the orientation of the pass-axis of P 2 with respect to that of P1 such
that the intensity of the light emerging from P 2 is maximum?
1
22. Consider a system of N non-interacting distinguishable spin − particles each of magnetic
2
moment µ. The system is at an equilibrium temperature T in a magnetic field B such that
n-particles have their magnetic moments aligned parallel to B.
a. Find the energy E and the entropy S of the system.
n
b. Using the relationship between E and S, find T. Hence, determine the ratio in terms
N
of µ, B and T. [Use ln N ! = N ln N − N ]
112 IIT JAM Physics Solved Papers & Practice Sets

23. a. An ideal gas kept in contact with a heat reservoir undergoes a quasistatic process in
which its pressure gets doubled. Obtain the Maxwell relation from the differential form
dF = − S dT − p dV and evaluate the expression for the change in entropy of n moles
of the gas.
b. Using S = S(T , V ), derive a general expression for ( ∂U / ∂V )T , where U (S, V ) is the
internal energy. Evaluate it for the ideal gas as considered in part first (a). Justify that
the outcome is consistent with the expression for the average energy known from the
kinetic energy.
1
24. Consider an ideal Fermi gas consisting of N non-relativistic spin × particles confined to a
2
length L in one dimension at 0 K.
a. Find an expression for the density of states and hence calculate the Fermi energy of
the gas.
b. Find the mean energy per particle in terms of the Fermi energy.

25. A square loop of side L and mass M is made of a wire of cross-sectional area A and
resistance R. The loop moving with a constant velocity v 0 $i in the horizontal XY-plane, enters a
$
region 0 ≤ x ≤ 2 L having constant magnetic field B k.
a b

v0 i

d c
L

B
x=0 x=2L
Y

X
Z

a. Find an expression for the x-component of the force F acting on the loop in terms of
its velocity v (t ), B, L and R.
b. Find the speed of the loop as its side ad exits the field region at x = 2 L and sketch its
variation with x.

Answers
1. (c) 2. (a) 3. (b) 4. (b) 5. (b) 6. (a) 7. (a) 8. (b) 9. (a) 10. (d)
11. (d) 12. (a) 13. (d) 14. (d) 15. (a)
Answers with Explanations
1. (c) We know a symmetric matrix is a square matrix that is 5. (b) Electrostatic field E is defined as
equal to its transpose. −∂V
E=
⇒ A = AT (for matrix A) ∂r
According to the question, P and Q are symmetric According to the question given,V = ( 2x + 4y )
matrices − ∂V ∂
⇒ Ex = =− ( 2x + 4y ) = − 2
⇒ P T = P and Q T = Q ∂x ∂x
Now, (PQ )T = Q T P T = QP − ∂V − ∂
and Ey = = ( 2x + 4y ) = − 4
⇒ Symmetric if PQ = QP ∂y ∂y

2. (a) dW = 2xy dx + x 2dy (given) ⇒ E = Ex $i + Ey $j = − 2$i − 4$j



⇒ ( 2xy ) = 2x | E | = ( − 2)2 + ( − 4)2 = 20
∂y
1
∂(x 2 ) The energy density,UE = ε 0E 2
and = 2x 2
∂x
⇒ 2xydx + x 2dy is exact differential. 1
= ε 0( 20 )2
2
[An implicit first order ordinary differential equation of the
form I (x , y ) dx + J (x , y ) dy = 0 is called an exact UE = 10ε 0
differential equation if there exists a continuously 6. (a)
differentiable function F called the potential function] Frame S Frame S′
⇒ ∫ dW = 0, hence the work done for a complete cycle S
Y
S′
Y′
Rod Rod
around a unit circle is zero. L L
Y Y′
3. (b) As the square plate is thin (lamina) and the bodies of
X X′
mass m lies in the XY-plane therefore the product of X X′
inertia terms IXY and IYZ are zero.
Y Z Z′
E F Inertial frame S′ moves with a velocityv with respect to S
m m along the common XX′-axis.
(–a, a, 0) (–a, a, 0)
For the observer in S′ frame, the rod placed in frame S,
X the observer will observe a contraction in length along
(–a, a, 0) X-axis.
m
(–a, a, 0) m Y Y′
⇒ tan θ = and tan θ′ =
H G X X′
tan θ′
⇒ tan θ =
4. (b) An ideal Bose gas is a quantum mechanical version of a v2
1− 2
classical ideal gas. It is composed of bosons, which c
have an integer value of spin and obey Bose-Einstein Thus, θ > θ′.
statistics. 7. (a) Conductivity of a doped semiconductor in terms of
Chemical potential of Bose gas number density of electrons and holes is given as
 ∂σ  σ = nee µe + nne µn
µ =−τ 
 ∂n  E , V ⇒ σ = e[neµe + nh µh ] …(i)
 ∂σ  For intrinsic semiconductor, ni2 = nenh
⇒ µ = − KT  
 ∂n  E , V
ni2
⇒ ne = …(ii)
where, σ = loge V nh
and V is the volume of the reservoir. Therefore, From Eqs. (i) and (ii), we get
chemical potential of an ideal Bose gas at any n 2 
temperature is either zero or negative. σ = e  i µe + nhµh 
 nh 
114 IIT JAM Physics Solved Papers & Practice Sets

For the conductivity to be minimum − c ± c 2 − 4mk


⇒ λ=
∂σ d ni2   − ni2  2m
=e  µe + nhµh  = e  2 µe + µh  = 0
σnh dnh n n For overdamped, c 2 − 4mk > 0
 h   h 
µe According to given question,
⇒ nh = ni x (t ) = Ae −γt cos (ω t + α ) ...(i)
µh
xt =0 = 0
According to the question, µe = µn and µh = µ p .
⇒ 0 = Ae − γt cos(ωt + α )
µn
⇒ nh = ni A cos α = 0 ⇒ cos α = 0 [A ≠ 0]
µp
 1
α = n +  π
π  2
8. (b) Given, λ = 20π × 10−8 m, φ = rad
1080 π
⇒ α = ( 2n + 1) for n = 0, 1, 2, 3,...
Dλ 2
Fringe width is given by β =
d x (t ) = Ae −γt cos (ωt + α )
where, D is the distance of screen and d is distance dx (t )
between the two slits. = Ae −γt [ − sin(ωt + α ) ⋅ ω] + A cos(ωt + α )e − γt ( − γ )
dt
λ  d dx (t )
⇒ β= Qφ= = − Ae −γt [ω sin (ωt + α ) + γ cos (ωt + α )]
φ  D  dt
20π × 10−8 × 1080 V(t = 0) = − A [ω sin α + γ cos α ] ⇒ ω =
|V |
or β=
π A
or β = 216 × 10−6 m = 0.216 mm 11. (d) Compton scattering is the inelastic scattering of a
photon by a charged particle, usually an electron. It
9. (a) Coriolis force In non-vector form at a given rate of
results in a decrease in energy (increase in wavelength)
rotation of the observer, the magnitude of the coriolis
of the photon (which may be an X-ray or gamma ray
acceleration of the object is proportional to the velocity
photon) called the Compton effect. Part of the energy of
of the object and also to the sine of the angle between
the photon is transferred to the recoiling electron. The
the direction of movement of the object and the axis of
shift of the wavelength increased with scattering angle
rotation. The vector formula for the magnitude and
h
direction of coriolis acceleration is according to the Compton formula, ∆λ = (1 − cos θ )
mec
ac = − 2 (ω × v )
According to given question,
and coriolis force Fωr = − 2m(ω × v ) h
where, v is the velocity of the particle with respect to the ∆λ = λ c (1 − cos θ ) ⇒ λ c =
mec
rotating system and ω is the angular velocity vector.
As, θ = 180°
⇒ Fωr = − 2m[ω(k$ ) × ( − $j )] ⇒ ∆λ = λ c (1 − cos 180° ) = 2λ c
F = − 2mω$i …(i) ∆λ 2λ c
ωr ⇒ =
and direction of centrifugal force is given by λ λ
Fck = − mω × (ω × r ) 12. (a) Outputs of different gates are shown below:
= − m [ω × ωk$ × r ( − $j )]
P P+Q + QR P+Q + QR +S
P P+Q
Fck = − mω 2r$i …(ii)
Y
From Eqs. (i) and (ii), we can say that Fωr and Fck both Q
are directed toward negative x-axis. Hence, figure of P+Q + QR +S
R
option (a) is correct. QR

10. (d) We know for damped harmonic oscillator, the equation S

of motion is ma + cv + kx = 0 13. (d) The following formula can be used to calculate


d 2x  dx  the activity of a radioactive material at any point of time.
m 2 + c   + kx = 0
dt  dt  A = A0e − λt
This is the form of a homogeneous second order where, A = activity remaining in the radioactive material
differential equation. after time (t ).
Auxiliary equation, mλ2 + cλ + k = 0 A0 = initial activity of the radioactive material
where, x = e λt λ = decay constant
Solved Paper 2008 ●
115

According to the given question, Consider a ring of radius r and thickness dr as shown in
A = 75% of A0 figure.
3 Potential at the point P due to this elementary ring is
A = A0
4 1 dq
dV1 =
Using, A = A0e − λt 4πε 0 PQ
3 Also, dq = ( 2πrdr ) σ
⇒ A0 = A0e − λt
4 and PQ = r 2 + 4L2
 3 ( 2πr dr ) σ
⇒ loge   = − λt ⇒ dV1 =
 4
4πε 0 r 2 + 4L2
loge 4 − loge 3 14 . − 11
. 0.3
⇒ λ= = = = 0.01 s−1 1
L
r dr σ
30 30
t
0.693 0.693
⇒ V1 = ∫
4πε 0 L/ 2 r + 4L
2 2
=
2ε 0
[(r 2 + 4L2 )1/ 2]LL/ 2
Half-life, T1/ 2 = = = 69.3 days
λ 0.01 σL  17 
V1 =  5− 
14. (d) The face centred cubic unit cell is a cube (all sides of the 2ε 0  2 
same length and all face perpendicular to each other)
Calculation of Potential due to Curved Part of
with an atom at each corner of the unit cell and an atom
Hollow Cylinder
situated in the middle of each face of the unit cell.
Consider a ring of thickness dx at a distance x from the
Z
(0, 0, a) point P,
P

(a /2, 0, a /2)
Y x
O
(a /2, a /2, 0)
(a, 0, 0) X dx

The second nearest neighbour will be at (a, 0, 0)


a a 
and first nearest neighbour will be at  , , 0
2 2 
Potential due to this ring
Distance of second nearest neighbour
1 dq
d ll = (a − 0)2 + ( 0 − 0)2 + ( 0 − 0)2 = a dV =
4πε 0 x 2 + L2
Distance of the first nearest neighbour
dq = ( σ 2πL) dx
2 2
a  a  a Total potential due to curved part of the cylinder
d ll =  − 0 +  − 0 + ( 0 − 0)2 =
2  2  2 2L
σL dx
d ll
=
a
=
2 V2 =
2ε 0 ∫ x + L2
2
L
dl a/ 2 1
σL
= [loge (x + x 2 + L2 )] L2L
15. (a) Gibbs energy is the chemical potential that is minimised 2ε 0
when a system reaches equilibrium at constant pressure σL  2 + 5
⇒ V2 = loge  
and temperature. Its derivative with respect to the reaction 2ε 0  1+ 2 
coordinate of the system vanishes at the equilibrium point.
Total potential at point P, VP = V1 + V2
As such, it is a convenient criterion for the spontaneity of
processes with constant pressure and temperature. σL  17  σL  2 + 5
V =  5−  + loge  
16. Net potential at point P will be due to bottom part (disc) 2ε 0  2  2ε 0  1+ 2 
and the curved part of hollow cylinder. 17. In classic mechanics, a central force on an object is a
Calculation of Potential due to Bottom Part force whose magnitude only depends on the distance r
P of the object from the origin and is directed along the
line joining them
F = F (r ) =| F (r )| r$
2L
where, F is the force, r is the position vector | r | is
r
its length and r$ = is the corresponding unit
L /2 |r |
r dr
vector.
116 IIT JAM Physics Solved Papers & Practice Sets

According to given question, In case that q is a conserved quantity that cannot be


3 1 created or destroyes (such as energy) σ = 0 and the
| F1 (r )| = 2 +
r r equation becomes
∂ρ
mv 2 + ∇⋅ j = 0
a. For circular motion of the particle, | F (r )| = ∂t
r
3 1 mv 2 a. In the given question,
⇒ + = ∂ρ
r2 r r ∇⋅ V +=0 …(i)
∂t
1 3 
or v 2 =  + 1 ∂ρ
m r  For an incompressible liquid, =0 …(ii)
∂t
1/ 2
1 3  Now from Eqs. (i) and (ii), we get
⇒ L = m  + 1  . r (L = mvr )
m r  ∂u ∂w
+ =0
Now, putting m = 1kg and L = 2 J-s ∂x ∂z
1/ 2
 3  ∂w ∂   πz  
⇒ 2 =  + 1  ⋅ r ⇒ = A(t ) cos kx sin   
 r  ∂z ∂z   d 
3  ∂w π πz
⇒ 4 =  + 1 ⋅ r 2 = A(t ) cos kx cos …(iii)
r  ∂z d d
r 2 + 3r − 4 = 0 ∂u ∂
and = [B(t ) sin ax ]
∂x ∂x
⇒ (r − 1)(r + 4) = 0
∂u
r = 1m (r ≠ − 4) ⇒ = aB(t ) cos ax cos bz …(iv)
∂x
b. Net force, F = F1 + F2 π πz
 3 1 A(t ) cos kx cos + aB(t ) cos ax cos bz = 0
⇒ F = −  2 +  r$ − λv …(i) d d
r r  ∂u ∂w 
Q + =0
dL  ∂x ∂z 
Using, τ= …(ii) π
dt a = k and b =
d
From Eqs. (i) and (ii), we have
π
dL   3 1  ⇒ aB(t ) = − A(t )
= r × −  2 +  r$ − λv  [Q τ = r × F ] d
dt  r r  π
B(t ) = − A(t )
dL −λ −λ kd
⇒ = (r × mv ) = L −π πz
dt m m ⇒ u(x , z , t ) = A(t ) sin kx cos
dL −λ kd d
⇒ = dt
L m $i $j $k
L 1/ λ
dL −λ ∂ ∂ ∂
∫ L
=
m ∫ dt Vorticity field = ∇ × V =
∂x ∂y ∂z
2 0
u 0 w
− λ 1/ λ
⇒ [loge L]L2= [t ] 0
m ∂w $  ∂u ∂w  $ ∂u
= $i + j −  −k
 L − λ 1 ∂y  ∂z ∂x  ∂y
⇒ loge   = × [m = 1]
 2 m λ ∂w
⇒ =0
 
L ∂y
⇒ loge   = − 1
 2 ∂u π πz  π
=+ A(t ) sin kx sin  
L 1 2 ∂z kd d d 
⇒ = ⇒ L = J-s
2 e e π2 πz
= 2
A(t ) sin kx sin
18. By the divergence theorem, a general continuity equation kd d
can be written in a differential form ∂w πz
= kA(t ) cos kx sin
∂ρ ∂x d
+ ∇⋅ j = σ ∂u
∂t =0
∂y
∇ is divergence; ρ is the amount of the quantity q per unit
 π2  πz
volume; j is the flux of q and σ is the generation of q per unit ⇒ Vorticity field =  2 sin kx − k cos kx  $j A(t ) sin
volume per unit time. kd  d
Solved Paper 2008 117

b. Divergence theorem states that the outward flux of a From Eqs. (ii) and (iii), we get
vector field through a closed surface is equal to the d  2 ibx   2 ibx 
volume integral of the divergence over the region inside − ih − i L e  = a − i L e 
dx    
the surface.
⇒ ∫ V ⋅ ds = ∫ ∇ ⋅ V∂τ h 2   2
⇒ − i (ib ) ⋅ eibx  = a − i 
i  L   L 
⇒ ∇⋅ V = 0
19.a. The wave function for a given physical system contains h  1 π
a = i β = hbβ ⇒ a = hl + 
the measurable information about the system. To obtain i  2 L
specific values for physical parameter for example h  1
a= l +  for l = 0, 1, 2, 3
energy, we operate on the wave function with quantum 2L  2
mechanical operator associated with that parameter. Hamiltonian energy operator is given by
The operator associated with energy is the Hamiltonian
dψ ′
and the operator on the wave function is the Schrödinger H$ ψ ′ = ih
dt
equation.
H$ ψ ′ = bψ ′
Now from Schrödinger wave equation
2m dψ ′
∇ 2ψ + 2 [E − V ] ψ = 0 [V = 0] ⇒ ih = bψ ′
h dt
ψ t
2mE dψ ′ b
⇒ ∇2ψ + 2 ψ = 0
h
…(i) or ∫ = ∫ dt
ψ ′ ih 0
ψ0
d 2ψ 2mE
⇒ + 2 ψ =0 − ibt
dx 2 h ⇒ ψ ′ = ψ 0′ e h
2  nπ x 
⇒ ψ= sin  20. Represent the displacement of masses from equilibrium
L  L 
configuration as a1, a 2 and a 3. Let’s consider two cases here.
dψ 2 nπ  nπ x 
= ⋅ cos   Case I Kinetic energy of the system
dx L L  L 
1 1
dψ − n π 2 2
2  nπ x  K = M (a12 + a 32 ) + ma 22
⇒ = sin   2 2
dx L2 L  L 
2 2 2 M 0 0   a1
d ψ −n π   
2
= 2
ψ n(x ) 2K = a1a 2a 3  0 m 0  a 2
dx L   
 0 0 M  a 3
Substituting this value in Eq. (i), we get
M 0 0 
2mEn n 2π 2  
⇒ = K = ( K ij  0 m
) 0
h2 L2  
 0 0 M
n 2 π 2 h2
⇒ En = [n = 1, 2, 3, ...] 1 1
2mL2 Potential energy,V = K (a 2 − a1)2 + K (a 3 − a 2 )2
dψ 2 2
Momentum p = − ih∇ = ih 1
dx = K (a12 + 2a 22 + a 32 − 2a1a 2 − 2a 2a 3 )
2
b. Let a be the energy eigen value of the momentum
 K −K 0   a1
operator   
dψ ⇒ 2V = (a1 a 2 a 3 )  − K 2K − K  a 2
⇒ − ih = aψ …(ii)   
dx  0 − K K  a 3
ψ x
dψ ia ψ iax  K −K 0 
⇒ ∫ ψ = h ∫ dx ⇒ loge ψ 0 = h  
ψ0 0 or V = (V )
ij  − K 2 K − K
iax  
 0 −K K 
ψ = ψ 0e h
Wave function ψ n(x ) of a particle confined to a The singular equation is |V − ω 2K | = 0
one-dimensional box of length L is given by
 K −K 0  M 0 0 
  2 
2  nπ x  2 − ibx ⇒  − K 2K − K  − ω  0 m 0 = 0
ψ n(x ) = sin   = −i e …(iii)    
L  L  L  0 −K K   0 0 M
 1 π
where, b = l +  for l = 0, 1, 2, ... [where, ω is frequency]
 2 L
118 IIT JAM Physics Solved Papers & Practice Sets

K − ω 2M −K 0  When half-wave plate was introduced between P1 and P2


  λ
or  −K 2K − ω 2m −K  = 0 the plate will introduced a path difference of or phase
 2
 0 −K K − ω 2M 
change of π.
or (K − ω 2M )[( 2K − ω 2m )(K − ω 2M ) − K 2] ⇒ x = a cos ωt , y = b sin (ωt + φ )
+ K [ − K (K − ω 2M )] = 0 y
⇒ = sin (ωt + φ )
b
This cubic equation gives the value of ω
y
⇒ ω1 = 0 = sin ωt cos φ + cos ωt sin φ
b
K
ω2 = y x2 x
M = 1 − 2 cos φ + sin φ
1/ 2 b a a
K  2M  
and ω3 =  1 +  y x x2
M  m  ⇒ − sin φ = 1 − 2 cos φ
Case II b a a
1 1 Squaring on both sides, we get
K = m(a12 + a 32 ) + ma 22
2 2 
y2 x2 2 2xy x 2
m 0 0   a1 + sin φ − sin φ =  1 −  cos2 φ
   b2 a2 ab  a2
2K = (a1 a 2 a 3 ) 0 M 0  a 2
   y 2 x 2 2xy
 0 0 m a 3 + − sin φ = cos2 φ
b2 a2 ab
m 0 0
  φ = 90° + 30° = 120°
K = (Kij ) 0 M 0
  3
0 0 m sin φ = sin 120° =
2
The potential energy will remain same 1
and cos 120° = −
⇒ |V − ω′ 2 K | = 0 2
K − ω′ 2m −K 0  y 2 x 2 2 3xy 1
  ⇒ + − =
⇒  −K 2K − ω′ 2 M −K  = 0 b2 a2 2ab 4
 0 −K K − ω′ 2 m x2 y2 3xy 1
 + − =
⇒ ω′ 2 (ω′ 2 m − K )[K (M + 2m ) − ω′ 2 mM ] = 0 a2 b2 ab 4

Normal frequencies, The light will be elliptically polarised, when quarter wave
plate introduced between P1 and P2 it will introduce a
ω1′ = 0 λ π
K path difference of and phase difference of φ =
ω 2′ = 4 2
m
x 2 y 2 2xy π π
K  2m  
1/ 2 + − sin = cos2
ω′ 3 =  1 +  a2 b2 ab 2 2
m  M  
x 2 y 2 2xy
Ratios of frequencies are ⇒ + − =0
a2 b2 ab
ω′1
=0 x y 
2
b
ω1 ⇒  −  = 0, y = x , the light will be plane
K M + 2m a b a
ω 2′ M ω′ m M polarised.
= and 3 =
ω2 m ω3 K M + 2M b. Using law of Malus, I = I 0 cos2 φ, the emergent light will
M m be of maximum intensity when the axis of P2 will be
M + 2m parallel to P1.
=
m + 2M ⇒ φ = 0, cos φ = 1
Hence, the emergent light will be of maximum intensity.
21. a. A wave plate is an optical device that alters the
polarisation state of light wave travelling through it. Two 22. a. The energy of the n-particles which are aligned parallel
common types of wave plates are the half-wave plates in magnetic field
which shift the polarisation direction of linearly polarised = − nµB cos 0° = − n µB
light and the quarter wave plate which converts linearly and that (N − n ) particle which are aligned anti-parallel to
polarised light into circular polarised light. the magnitude field
According to given question, = − (N − n )µB cos 180° = (N − n ) µB
Solved Paper 2008 ●
119

Partial function, Z = e − β [ − nµB + (N − n)µB ] V /2


dV
Z n
⇒ ∫ dS = nR ∫ V
⇒ Z = V
N!  ∂S   ∂S 
log Z = log Z n − log N ! ⇒ dS =   dT +   dV
 ∂T  T  ∂V  T
⇒ log Z = 2n µB β − N µB β − N log N + N
 ∂S   ∂S 
−∂ or T ∂S = T ⋅   dT + T   dV
⇒ E= (log Z ) = − 2n µB + N µB  ∂T  V  ∂V  T
∂β
1 or TdS = dV + pV
⇒ E = (N − 2n ) µB, also, F = − log Z
β TdS = dU + pdV …(i)
1 b. From first and second law of thermodynamics
where, β =
kBT  dS 
1 dU + pdV = nCV dT + T   dV [using Eq. (i)]
⇒ F = − [ 2n µB β − N µB β − N log N + N]  dV  T
β
 dS 
= − 2n µB + N µB + (N log N − N )kBT ⇒ dU = nCV dT + T   dV − pdV
 dV  T
= (N − 2n ) µB + (N log N − N )kBT  ∂U   ∂S 
⇒   =T   −p
 ∂F   ∂V  T  ∂T  V
b. Entropy, S = −   = N(1 − log N )kB
 ∂T  V
 ∂U   ∂P 
⇒   =  −p
As, E = ST  ∂V  T  ∂T  V
E
⇒ T =  ∂p  nR
S ⇒   = [using, pV= nRT]
 ∂T  V V
Now substituting the values of E and S, we get
 ∂U  nRT
(N − 2n ) µB ⇒   = −p = 0
or T =  ∂V  T V
N(1 − log N )kB
⇒ dU = nCV dT
or NT (1 − log N )kB = N µB − 2n µB
or NTkB − NT log NkB − NµB = − 2nµB Hence, for an ideal gas, the total energy is entirely
n µB + kBT log N − kBT kinetic and internal energy is the function of
= temperature.
N 2 µB
For denominator to be positive, log N < 1
24. a. The properties of an ideal Fermi gas are strongly
determined by the Pauli principle. We shall consider the
n µB − kBT
Neglecting kBT log N ⇒ = limit u >> kT ⇒ B µ >> 1.
N 2 µB
which defines the degenerate Fermi gas. In this limit, the
23. a. A quasistatic process is a thermodynamic process that quantum mechanical nature of the system becomes
happens infinitely slowly. Any reversible process in especially important and the system has little to do with
necessarily a quasistatic one. the classical ideal gas.
Given, dF = − SdT − pdV According to given question,
 ∂F   −∂F  P (k ) ⋅ dk
S =−  and p =  
 dT  V  dV  T P ( ε) = …(i)

 ∂S   ∂ 2F 
⇒   =−  h2k
 ∂V  T  ∂T ∂V  and ε= …(ii)
V, T 2m
 ∂p   ∂ 2F  8ε h2k
and   =−  ⇒ =
 ∂T  V  ∂T ∂V  ∂k m
V, T

 ∂S   ∂p  Lm 2
⇒   =  P ( ε) = hk …(iii)
 ∂V  T  ∂T  V 2π
The above equation is Maxwell’s relation. 2πε
From Eq. (ii), k =
 ∂p  h2
∂S =   dV
 ∂T  V Substituting this value in Eq. (iii), we get
For an ideal gas, pV = nRT m
P ( ε) =
⇒ p=
nRT 2π 2m h2 E
V
120 IIT JAM Physics Solved Papers & Practice Sets

The Fermi energy of the gas is given by −l 2v (t )B 2


⇒ F= i
kF R
E = 2∫ ε(k )ρ (k ) dk
0 −l 2v (t )B 2
L ma= i
where, ρ (k ) = and 2 is due to the spin degeneracy R

dv l 2vB 2
factor. ⇒ m⋅ =
dt R
kF h2 k 2  L
⇒ E = 2∫   dk dv l 2vB 2
0 2m  2π  m ⋅v ⋅ =
dx R
2L h2 kF
= ∫0 k 2dk dv l 2B 2
2π 2m m⋅ = 2
kF
dx R
h2L k 3 1 h2LkF3 l 2B 2
E= = mdv = 2 ⋅dx
2 πm 3 0
3 πm R
E −l 2B 2
m ∫ dv =
R ∫
b. The mean energy of the particle is given by E = dx
N
kFh2k 2  L  −l 2B 2
2∫   dk mv = +c
0 2m  2π  R
=
kF  L  [As, the loop come out the region at x = 2L, the direction
2∫   dk
0  2π  of induced emf and current reverse according to Lenz’s
k2 law]
h2 k F
2m 3 0 1  h2kF2  b. At x = 0,v = v 0
E= =   ⇒ c = mv 0
kF 3  2m 
−l 2B 2x
25. a. Motional emf is given by, e = l ⋅ ( v × B) ⇒ mv = + mv 0
R
⇒ e = −lv 0B $j l 2B 2x
v =v0 −
e −lv 0B $j mR
i= = On comparing with y = mx + c , sketch can be drawn as
R R
− l 2v 0B 2
and magnetic force, F = i ( l× B) = i V0
R
When the loop enters the region x = 0, then velocity v (X)
becomes function of time.
x 2L
Solved Paper
2007
IITJAM Physics
MM : 300 Time : 3 hrs

n
In this question paper, there are total of 25 questions carrying 300 marks of 3 hours duration.
n Questions 1-15 (Objective questions) carry 6 marks each and questions 16-25 (Subjective questions) carry
21 marks each.
n Each objective question has four choices for its answer i.e. (a), (b), (c) and (d). Only one of them is the
correct answer.
n There will be negative marking for wrong answers to objective questions. The following marking scheme
for objective questions shall be used:
(a) For each correct answer, you will be awarded 6 (Six) marks.
(b) For each wrong answer, you will be awarded –2 (Negative two) mark.
(c) Multiple answers to a question will be treated as a wrong answer.
(d) For each un-attempted question, you will be awarded 0 (Zero) mark.
n Do not write more than one answer for the same question. In case you attempt a subjective question
more than once, please cancel the answer(s) you consider wrong. Otherwise, the answer appearing last
only will be evaluated.
n Clip board, log tables, slide rule, calculator, cellular phone and electronic gadgets in any form are not
allowed.

1. Fermi energy of certain metal M 1 is 5 eV. A second metal M 2 has an electron density which is
6% higher than that of M 1. Assuming that the free electron theory is valid for both the metals,
the Fermi energy of M 2 is closest to
(a) 5.6 eV (b) 5.2 eV (c) 4.8 eV (d) 4.4 eV

2. The following histogram represents the binding energy per particle


(BE/A) in MeV as a function of the mass number A of a nucleus. 8

A nucleus with mass number A = 180, fissions into two nuclei of 6


BE
equal masses. In the process, ——
A 4
(a) 180 MeV of energy is released
2
(b) 180 MeV of energy is absorbed
(c) 360 MeV of energy is released 0
40 80 120 160 200
(d) 360 MeV of energy is absorbed A
122 IIT JAM Physics Solved Papers & Practice Sets

3. A particle is confined in a one dimensional box with impenetrable walls at x = ± a . Its energy
eigenvalue is 2 eV and the corresponding eigenfunction is as shown below.

–a 0 +a

The lowest possible energy of the particle is


(a) 4 eV (b) 2 eV (c) 1 eV (d) 0.5 eV

4. Experimental measurements of heat capacity per mole of aluminium at low temperature show
that the data can be fitted to the formula, CV = aT + bT 3 , where a = 0.00135 JK −2 mole −1,
b = 2.48 × 10 −5 JK −4 mole −1 and T is the temperature in Kelvin. The entropy of a mole of
aluminium at such temperature is given by the formula
bT 3
(a) aT + + C, where C > 0 is a constant
3
aT b 3
(b) + T + C, where C > 0 is a constant
2 4
bT 3
(c) aT +
3
aT bT 3
(d) +
2 4

5. A uniform and constant field B coming out of the plane of the paper ω B
P
exists in a rectangular region as shown in the figure.
A conducting rod PQ is rotated about O with a uniform angular speed ω
in the plane of the paper. The emf E PQ induced between P and Q is O
best represented by the graph
Q
EPQ EPQ EPQ EPQ

(a) O t (b) O t (c) O t (d) O t

6. Three polarisers P, Q and R are placed parallel to each with their planes perpendicular to the
Z-axis. Q is placed between P and R. Initially, the polarising directions of P and Q are parallel
but that of R is perpendicular to them. In this arrangement when unpolarised light of intensity I 0
is incident on P, the intensity coming out of R is zero. The polariser Q is now rotated about the
Z-axis. As a function of angle of rotation, the intensity of light coming out of R is best
represented by
I0 I0
— —
4 4
I0 I0

(a) 8 —
(b) 8
π
– π 3π 2π π
– π 3π 2π
–— –—
2 2 2 2
I0 I0
— —
4 4
(c) —
I0 (d) —
I0
8 8
π
– π 3π 2π π
– π 3π 2π
–— –—
2 2 2 2
Solved Paper 2007 ●
123

7. The black body spectrum of an object O1 is such that its radiant intensity (i.e. intensity per unit
wavelength interval) is maximum at a wavelength of 200 nm. Another object O 2 has the
maximum radiant intensity at 600 nm. The ratio of power emitted per unit area by O1 to that of
O 2 is
1 1
(a) (b) (c) 9 (d) 81
81 9

8. In terms of the basic units of mass (M ), length (L ), time (T ) and charge (Q ), the dimensions of
magnetic permeability of vacuum (µ 0 ) are
(a) [MLQ −2 ] (b) [ML2 T −1 Q −2 ] (c) [LTQ −1] (d) [LT −1 Q −1]

9. When two simple harmonic oscillations represented by Y


2
x = A0 cos (ωt + α ) and y = B 0 cos (ωt + β )
1
are superposed at right angles, the resultant is an ellipse with its
major axis along the Y-axis as shown in the figure. The conditions X
O
1 2
which correspond to this are
π π
(a) β = α + ; A0 = 2 B0 (b) β = α + ; A0 = B0
2 4
π π
(c) β = α + ; 2 A0 = B0 (d) β = α + ; A0 = B0
2 4

10. A projectile is fired from the origin O at an angle of 45° from the horizontal. Y
At the highest point P of its trajectory, the radial and transverse components
P
of its acceleration in terms of the gravitational acceleration g are
2g g −2 g −g
(a) ar = , aθ = (b) ar = , aθ =
5 5 5 5 O X
g 2g −g −2 g
(c) ar = , aθ = (d) ar = , aθ =
5 5 5 5

11. A satellite moves around a planet in a circular orbit at a distance R from its centre. The time
period of revolution of the satellite is T. If the same satellite is taken to an orbit of radius 4R
around the same planet, the time period would be
T T
(a) 8T (b) 4T (c) (d)
4 8
12. The speed of an electron, whose de-Broglie wavelength is equal to its Compton wavelength, is
(c is the speed of light)
c c c
(a) c (b) (c) (d)
2 2 3
5 −7  x 
13. (x y )     = 15
 7 3  y 
The matrix equation above represents
(a) a circle of radius 15 (b) an ellipse of semi-major axis 5
(c) an ellipse of semi-major axis 5 (d) a hyperbola

14. Figures (i) and (ii) represent, respectively

(i) (ii)

(a) NOR, NOR (b) NOR, NAND (c) NAND, NAND (d) OR, NAND
124 IIT JAM Physics Solved Papers & Practice Sets

15. f (x ) is a periodic function of x with a period of 2π. In the interval − π < x < π, f (x ) is given by
 0, −π < x < 0
f (x ) = 
 sin x, 0 < x < π
In the expansion of f (x ) as a Fourier series of sine and cosine functions, the coefficient of
cos ( 2x ) is
2 1 2
(a) (b) (c) 0 (d) −
3π π 3π

16. If the total surface area (including the area of the top and bottom ends) of a cylinder is to be
kept fixed ( = A), what is its maximum possible volume?
For such cylinders of fixed total area, plot in the axes shown below their volume (V ) versus the
radius (R ) clearly indicating the values of R for which the volume is maximum and zero.

A
V L

R
R

17. A horizontal square platform of mass m and side a is free to rotate B


about a vertical axis passing through its centre O. The platform is
O
stationary and a person of the same mass (m ) as the platform is
A
standing on it at point A. The speed v of the person with respect to
the platform is constant. Find the time the person takes to reach B.
Also, find his distance r (t ) from O as a function of time. Further find
the angle through which the platform has rotated by the time the
person reaches B.

18. Two identical parallel plate capacitors are connected across y


terminals A and B as shown. Each of the capacitors is made of A
l
square plates of side l with a distance d between them. A dielectric
slab (relative permittivity K) of thickness d is kept between the
plates. The slab covers only half of the length of the plates in each
of the capacitors as shown. Find the total capacitance of the B
d
assembly. The capacitors are charged by a battery and then battery
is disconnected. If the slab is now displaced slightly by a distance
y [( y / l ) << 1], show that it will perform simple harmonic oscillations.

19. For the circuit shown below, calculate the output voltage Vo . What would be Vo if the polarity of
2 V battery is reversed at terminal B ? (Assume the operational amplifier to be ideal).
30 kΩ

10 kΩ +12 V
+4 V –
A
Vo
B +
–12 V
2V
Solved Paper 2007 ●
125

20. A beam of light of wavelength 400 nm and power 1.55 mW is directed at the cathode of a
photoelectric cell. (Given, hc = 1240 eV nm, e = 1.6 × 10 −19 ). If only 10% of the incident
photons effectively produce photoelectrons, find the current due to these electrons. If the
wavelength of light is now reduced to 200 nm, keeping its power the same, the kinetic energy
of the electrons is found to increase by a factor of 5. What are the values of the stopping
potentials for the two wavelengths?

21. Two thin lenses L1 and L 2 of focal lengths 15 cm and 10 cm respectively, are kept 15 cm apart
from each other. Their axes are separated by 0.5 cm as shown in figure (not to scale). If a
point object P is placed on the axis of L1 to its left at a distance of 30 cm, find the x and
y-coordinates (origin O) of the image formed by the combination.
Y

P 0.5 cm
X
O
15 cm
30 cm L2
L1
S1
22. The circuit shown consists of two identical capacitance C each and an
inductor of inductance L. Initially, both switches are open and capacitor 1 is 1
charged with a charge Q 0 while the second capacitor has no charge. Switches S2 L
S1 and S 2 are closed simultaneously at t = 0. The circuit now becomes
oscillatory. 2

a. Calculate the maximum current in the circuit.


b. Obtain expressions for the charge on the capacitors 1 and 2 as a function of time.

23. A particle travels along the diameter of the earth at a relativistic speed. It crosses the earth in a
time 3 × 10 −2 s in its own frame. An observer, located on the earth, measures the same time
interval to be 5 × 10 −2 s. Find the speed of the particle with respect to the earth and the
diameter of the earth.
b
p
24. 1 m 3 of an ideal gas with γ = C p / Cv = 1.5 is at a pressure of 100 kPa
and a temperature of 300 K. Initially the state of the gas is at the point a of a c
the pV diagram shown. The gas is taken through a reversible cycle
a → b → c → a . The pressure at point b is 200 kPa and the line ba, when
O
extended, passes through the origin. V

a. Calculate the work done by the gas in each of the steps a → b , b → c and c → a .
b. Calculate the change in entropy of the gas in each of the three steps above.

25. How much work is done when an object moves from Y


O → P → Q → R → O in a force field given by
F (x, y ) = (x 2 − y 2 ) $i + 2xy $j P
Q (a , b )

along the rectangular path shown. Find the answer by evaluating the
line integral and also by using the Stoke’s theorem. R X
O
(0, 0)

Answers
1. (b) 2. (c) 3. (d) 4. (a) 5. (c) 6. (b) 7. (d) 8. (a) 9. (c) 10. (d)
11. (a) 12. (b) 13. (b) 14. (c) 15. (d)
Answers with Explanations
1. (b) Fermi energy can be given as 3. (d) Consider the eigenfunction given in the question. It is
h2 clear from the function that n = 2 that is the particle is in
EF = ( 3π 2n )2/ 3 ...(i)
8π 2m first excited state. Expression for energy of the particle
where, h = Planck’s constant confined in the box can be given as
m = mass of electron n 2π 2h 2
En =
n = state of electron 8mL2
From Eq. (i), we can write where, L = length of the box = 2a
EF ∝ (n )2/ 3 m = mass of the particle
h = Planck’s constant
Let fermi energy corresponding to metals M1 and M 2 are
EF1 and EF2 , respectively. Now, we can write
We can write,
2/ 3 +a
EF1 n  n1 –a O
=  1 = 2/ 3
EF2  n2    6 
( 2)2 π 2h 2 π 2h 2
n1 1 + 100  E2 = =
  8m( 2a )2 8ma 2
  6 
For the lowest energy, we have to put n = 1
Q n 2 = n1 1 + 100 
  π 2h 2 π 2h 2
1 E1 = = ...(i)
= 2/ 3
8m( 2a )2 32 ma 2
 6 
1 +  According to the question,
 100
π 2h 2
5 eV 1 E2 = 2 eV = ...(ii)
or = 8ma 2
EF2 (1 + 0.06)2/ 3
Dividing Eqs. (i) by (ii), we have
Using binomial approximation, we can write E1 π 2h 2 8ma 2 1
= × =
5 eV 1 E2 32ma 2 π 2h 2 4
= [Q(1 + n )x ≈ 1 + nx ]
EF2 2
1 + × ( 0.06) ∴ Lowest possible energy of the particle,
3 E 2 eV
1 1 1 E1 = 2 = = 0.5 eV
= = = 4 4
1 + 2 × 0.02 1 + 0.04 1.04
4. (a) We can write the change of heat energy associated with
∴ Fermi energy of M 2 is
the system as, ∆Q = nCV ∆T = CV ∆T
EF2 = (1.04) (5 eV ) = 5.2 eV As temperature is low, i.e. ∆T → 0
2. (c) According to the given histogram, binding energy of A , ⇒ ∆Q → dQ = CV dT = (aT + bT 3 ) dT
( BE)A = 180 × 4 = 720 MeV The entropy of the system can be written as
Suppose the nucleus A, fissions into B and C, T dQ T  aT + bT 3 
each of mass number 90. Binding energy of B and ∫ dS = ∫0 T = ∫0  T  dT
C are
( BE)B = 90 × 6 = 540 MeV Assuming entropy of the system as positive constant (C )
at 0 K. We can write,
( BE)C = 90 × 6 = 540 MeV S T
Total binding energy of the output product is ∫C dS = ∫ (a + bT 2 ) dT
0
( BE)O = ( BE)B + ( BE)C bT 3 
T

= (540 + 540) MeV or S − C = [aT ]T0 +  


 3 0
= 1080 MeV
bT 3
Thus, energy released during the fission process is or S = aT + +C
3
∆E = ( BE)O − ( BE)A
where C is a positive constant
= 1080 MeV − 720 MeV
bT 3
= 360 MeV Entropy of a mole of aluminium, S = aT + +C
3
Solved Paper 2007 ●
127

5. (c) Suppose at any instant the rod PQ makes an angle θ IR = IP cos2 φ ⋅ sin2 φ
with AB as shown in the adjacent diagram. I
= P × 4 cos2 φ sin2 φ
D C 4
IP
= × sin2 2φ
4
Q I
l ⇒ IR = 0 sin2 2φ
8
θ
O IR will be maximum, when
A l B
sin 2φ = 1
P or 2φ = 90°
The area enclosed by the arc is π
⇒ φ = 45° =
4
πr 2θ r 2θ l 2θ
A= = = [Qr = l] I0
2π 2 2 (IR )max = ; at φ = 45°
8
Flux associated with this area is
Thus, graph corresponding to IR is in accordance with
φ = B⋅ A = BA [Q θ = 0°]
that is shown in option (b).
 l 2θ 
=B  7. (d) According to the Wien’s displacement law, we have
 2
λ mT = constant
1 2
⇒ φ=
Bl θ where, λ m is wavelength corresponding to maximum
2 intensity and T is temperature.
Emf induced between the ends P and Q is For the two objects O1 and O 2, we have
−d φ −d  1 2  λ m1T1 = λ m 2T2 ...(i)
EPQ = = Bl θ
dt dt  2 
Given that λ m1 = 200 nm
B l 2  −dθ  and λ m 2 = 600 nm
=
2  dt  Now from Eq. (i), we can write
1  dθ  T1 λ m 2 600 nm
= − B l 2ω Q ω= = = =3
2  dt  T2 λ m1 200 nm
Magnitude of emf is
1 2 The ratio of power emitted per unit area by O1 to that of
| EPQ | =
B l ω = constant O 2 is given by
2 4
P1  T1 
During the motion of the rod inside the field, induced emf =   = ( 3)4 = 81
P2 T2 
remains constant. Outside the field induced emf is zero.
6. (b) Consider three polarisers P , Q and R. 8. (a) Magnetic field due to a long current carrying wire is
given by
P Q R µ 0I
B= ...(i)
2r
where, µ0 = permeability of vacuum
Unpolarised
light I= current
r = perpendicular distance of a point
Initially, polarising direction of P and Q are parallel but
that of R is perpendicular to them. Writing each terms of equation in bracket [⋅], we have
Intensity of light passing through P is [µ ] [I ]
[B] = 0
I [ 2] [r ]
IP = 0
2 Now, writing dimensions for the quantities in the bracket,
Intensity of light passing through Q, according to law of we have
Malus is  Force  [µ 0] [I ]
IQ = IP cos2 φ  Current × Length  = [ 2] [r ]
 
where, φ is the angle between polarising directions of P [ Force] [r ] [ MLT −2] [ L]
and Q. ⇒ [µ 0] = =
[I ] 2 [L] [ Q] 2 [ T −1] 2 [ L]
Intensity of light coming out of R is
∴ Dimensions of magnetic permeability of vacuum,
IR = IQ cos2 (90° − φ )
[MLT −2]
= IP cos2 φ ⋅ cos2 (90° − φ ) [µ 0] = 2 −2 = [ MLQ−2]
[Q T ]
128 IIT JAM Physics Solved Papers & Practice Sets

9. (c) Equations of two simple harmonic oscillations are u 2 2g 1


∴ tan θ = × =
x = A0 cos (ωt + α ) ...(i) 4g u 2 2
and y = B0 cos (ωt + β ) ...(ii) Acceleration of the particle is a = −g $j
According to the question, x and y are superposed at Radial component of the acceleration is
right angles.
π ar = g cos(90 − θ ) = −g sin θ
⇒ β =α ± 1 g
2 = −g × =−
Considering only positive sign, we have 5 5
π Tangential or transverse component of the acceleration
β =α +
2 is
 π 2 2g
⇒ y = B0 cos ωt + α +  a θ = −g cos θ = −g × =−
 2 5 5
or y = B0 sin (ωt + α ) ...(iii) 11. (a) According to the Kepler’s third law,
From Eqs. (i) and (iii), we can write T 2 ∝ a3
 x 
2
y 
2 where, a is radius of the orbit and T is the time period
2 2
  +   = cos (ωt + α ) + sin (ωt + α ) Now, we can write
 A0   B0  2 2
 T1  a 
x2 y2 2 2   =  1
or + =1 [Q cos θ + sin θ =1 ]  T2   a2 
A02 B02
Given that T1 = T , a1 = R and a 2 = 4R, T2 = ?
The above equation represents an ellipse with axes at 2 3 2
x 0 = A0 and y 0 = B0. T  R 1  1
⇒   =  = = 
According to the ellipse given in the question, we can  T2   4R  64  8
write T 1
x′ 0 = 1 and y ′ 0 = 2 = B0 or =
T2 8
⇒ B0 = 2A0 ∴ Time period for radius 4R is
10. (d) Let speed of projection of the particle is u. T2 = 8T
Y 12. (b) As we know that, the de-Broglie wavelength is given by
h
λd =
mv
u
P θ where, m = mass of electron
v = speed of electron
90–θ and h = Planck’s constant
Compton wavelength can be written as
h
θ 45° X λc =
O O′ m0c
where, m0 is rest mass of electron.
Range of the projectile, According to the question,
u 2 sin 2 θ λd = λc
R=
g h h
⇒ =
u 2 × sin 90° u 2 m0c mv
= =
g g m
or v =c × 0 ...(i)
R u2 m
Distance, OO ′ = = Using relativity, we can write
2 2g
m0
From ∆OO′ P m= ...(ii)
PO ′  2g  1−v 2 /c 2
tan θ = = (Hmax )  2 
OO ′ u  From Eqs. (i) and (ii), we have
where, maximum height attained by the projectile , v m0
= = 1−v 2 /c 2
c m
u 2 sin2 45° u 2
Hmax = = v2 v2
2g 4g or 2
= 1− 2
c c
Solved Paper 2007 ●
129

2v 2 where, coefficients are


or =1
c2 1 π
2π ∫− π
a0 = f (x ) dx
c2 c
or v2 = ⇒ v = 1 π
2 2 an = ∫ f (x ) cos nx dx
c π −π
∴ Speed of the electron, v = 1 π
2 bn = ∫ f (x ) sin nx dx
π −π
 5 −7 x  Coefficient of cos ( 2x ) is
13. (b) Given that, (x y )     = 15
7 3 y 
1 π
a 2 = ∫ f (x ) cos 2x dx
Now, multiplying matrices separately, we have π −π
 5 −7 1 0 1 π
P1 = (x y )   = ∫ f (x ) cos 2x dx + ∫ f (x ) cos 2x dx
7 3 π −π π 0
= (5x + 7y − 7x + 3y ) 1 0 1 π
= ∫ ( 0) cos 2x dx + ∫ sin x cos 2x dx
x  π −π π 0
Similarly, P2 = P1   1 π
y 
2π ∫0
=0+ 2 sin x cos 2x dx
x 
= (5x + 7y − 7x + 3y )   1  π π
y  = ∫ sin ( 2x + x ) dx − ∫ sin ( 2 x − x ) dx 
2π  0 0 
= 5x 2 + 7xy − 7xy + 3y 2 1  π π
sin 3x dx − ∫ sin x dx 
2π ∫0
=
= 5x 2 + 3y 2 0 
 π
According to the question, given that 1 cos 3x 
π

⇒ a2 = − + cos x  
P2 = 15 2π  3 0 0 

⇒ 5x + 3y 2 = 15
2
1  − cos 3π + cos 0 
x2 y2 = + cos π − 1
or + =1 2π  3 
3 5 1 1 + 1 
x2 y2 = + −1− 1
or + =1 2π  3 
2
( 3) ( 5 )2
1 2  1  −4 
Clearly, this is an ellipse with semi- major axis 5. = −2 =
2π  3  2π  3 
14. (c) Consider the gates as shown below. −2
⇒ a2 =
a y 3π
b 16. Consider the cylinder given in the question.
Here, we have assumed the inputs as a and b and the Surface area of the cylinder,
output as y. A = πR 2 + 2πRL + πR 2
Now, we can write
= 2πR 2 + 2πRL
y = ab = a + b [by De-Morgan’s law] L
= 2πR (R + L)
Clearly, it is the output of a NAND gate.
Volume of the cylinder,
Similarly, consider the other gate as shown below.
 A − 2 πR 2 
a V = πR 2L = πR 2   R
b
y
 2 πR 
R
We can write, = [ A − 2 πR 2 ]
2
y = a + b = ab [by De-Morgan’s law] where, A = surface area
This output is also same as that of NAND gates. Thus, Differentiating volume V w.r.t. R, we have
both the given gates are NAND gates. dV 1
= [ A − 2π × 3 × R 2]
15. (d) Given that, dR 2
 0, −π < x < 0 dV
f (x ) =  Putting =0
sin , 0 <x < π dR
 x
⇒ A = 2π × 3R 2
Considering the Fourier series as
F (x ) = a 0 + Σ an cos (nx ) + bn sin (nx ) or R=
A

130 IIT JAM Physics Solved Papers & Practice Sets

Again, differentiating
dV
w.r.t. R, we have = OA cos 45° ( − $j ) + OA sin 45° ( $i ) + (vt )( $j )
dR
2a
d 2V 1 = [ sin 45° $i − cos 45° $j ] + (vt )( $j )
= [ 0 − 12πR] = − 6πR = Negative value 2
dR 2 2
1  $i $j 
$
Thus, volume is maximum at ⇒ r (t ) = a −  + (vt )( j )
2  2 2 
A
R= = 0.23 A 2 2
6π a  a
∴ Distance r (t ) = | r (t )| =   + vt − 
 A  1 A  A   2  2
V R −  = A − 2π ×
 6π  2 6π   6π  Angular momentum conservation gives
1 A  A I ω = mv r (t )
= A−
2 6π  3  2 2
ma 2 a  a
⇒ × ω = mv   + vt − 
∴ Maximum possible volume, 6  2  2
1 A  2A  A A a2 dθ
2 2
Vmax = = a  a
2 6π  3  3 6π ⇒ × =v   + vt − 
6 dt  2  2
2 2
6v a  a
V ⇒ dθ =   + vt −  dt
Vmax a2  2  2
2 2
θ 6v t0 a  a
= A
3
A

⇒ ∫0 d θ = a 2 ∫0   + vt −  dt
 2  2
V=0 2 2
6v a /v a  a
R = 0.23 A
R
R = 0.4 A ⇒ θ=
a2 ∫0   + vt −  dt
 2  2
Putting V = 0, we have Putting z = vt −
a
R 2
[ A − 2 πR 2 ] = 0 a /v
2
6 a2
or A − 2 πR 2 = 0 Gives, θ = 2
log z + z 2 +
a 4
A 0
or R2 = a /v
2π 6 a2
A ⇒ θ = 2 log z z 2 +
or R= = 0.399 A ≈ 0.4 A a 4
2π 0

AR 6  a2 a2 a 
Q V (R ) = = πR 3 =  log
a
+ + − log .
2 a2 v 2
4 2 

v
∴ Plot of volume (V ) versus the radius (R ) can be drawn as 
shown above. 18. Consider the two capacitors shown in the figure.
Y
17. The person is moving with constant speedv with respect to l
A
the platform. Time taken to reach the end B is
Z Dielectric
Y l
B

B d

Initially, half portion of the capacitors are filled with the


X dielectric.
O
P Capacitance of the first capacitor,
vt
A ε l 2 Kε 0l 2 ε 0l 2
C1 = 0 + = (1 + K )
2 2 2d
AB a d d
t0 = = .
v v Q Both capacitors are identical.
Position vector of the person at any time ‘t ’ to origin ∴ C1 = C2
‘O’ is OP = OA + AP Q Both capacitors are in series.
Solved Paper 2007 ●
131

∴ Total capacitance of the assembly is ⇒ Force on the dielectric,


C ε l2 ∂U e2 ∂
C = 1 = 0 (1 + K ) F =− =− × Ceq
2 4d ∂Y 2 ∂Y
ε0  2 l 
Suppose emf of the battery is e. =− K ( −2Y ) + 2K  + Y
d (K + 1)  2 
Hence, total energy can be written as l  
+ 2K  − Y  ( −1) − 2Y 
1 1 2  
U = Ceq e 2 = Ce 2
2 2 ε0
= [ −2YK 2 + Kl + 2KY − Kl + 2KY − 2Y ]
2 d (K + 1)
1 ε 0l (1 + K )
= × e 2 = constant − ε 0Y
2 4d = [ 2K + 2K − 2K 2 − 2 ]
d (K + 1)
If the slab is now displaced slightly by a distance y, then
−Y [ 4K − 2K 2 −2 ] ε 0
capacitance of the first capacitor is ⇒ F =
d (K + 1)
l  l 
Kε 0l  + Y  ε 0l  − Y  ∴ F ∝ ( −Y )
2  2 
C1 = + i.e. Force ∝ − (displacement)
d d
Thus, motion of the dielectric is simple harmonic.
ε 0l Kl l 
= + KY + − Y
d  2 2  19. The given op-amp circuit is
I
εl  l  l  30kΩ
= 0 K  + Y +  − Y 
d  2  2 
+12V
+4V –
Similarly, capacitance of the second capacitor is 10kΩ A
Vo
l  l  B +
Kε 0l  − Y  ε 0l  + Y  –12V
2  2 
C2 = + 2V
d d
ε 0l  l  l 
= K  2 − Y  +  2 + Y   Suppose the current through upper network is I.
d  
Applying Kirchhoff’s current rule at point A, we can write
Now, we can write
4 − VA VA − Vo
ε 0l ε l2 = ...(i)
C1 + C2 = [K (l ) + l] = 0 [K + I ] 10 kΩ 30 kΩ
d d
4 − 2 VA − Vo
2 ⇒ = [Q| VA | = | VB | = 2 V]
 ε l  l  l  10 30
Now, C1C2 =  0  K  + Y +  − Y 
d   2  2  ⇒ 6 = VA − Vo = 2 − Vo
 l  l  ⇒ Vo = 2 − 6 = − 4 V
× K  − Y  +  + Y  
  2   2 
When polarity of the battery is reversed, we can write
 ε l 
2 2
l2  l  VA = − 2 V
=  0  K 2  − Y 2 + K  + Y 
d  4  2  Now, Eq. (i) gives

4 − ( −2) −2 − Vo
l 
2
l2  =
+ K  − Y + −Y 2 10 k Ω 30 k Ω
2  4  6 −2 − Vo
Equivalent capacitance, ⇒ =
10 30
Ceq =
C1C2 ⇒ 18 = − 2 − Vo
C1 + C2 ⇒ Vo = − 20 V
d  ε l  2  2  l 2 2
= 0
  K  − Y  20. Given that, λ 1 = 400 nm, P1 = 1.55 mW
2
ε 0l (K + 1)  d  4 
hc = 1240 eV-nm, e = 1.6 × 10−19 C
l 
2
l  l2
2 Energy of the incident beam of light,
+ K  + Y + K  − Y + −Y 2
2  2  4 hc 1240 eV × 10−9
 E1 = =
1 λ1 400 × 10−9
∴ Total energy, U = Ceq e 2
2 = 310 × 10−2 eV = 3.10 eV
132 IIT JAM Physics Solved Papers & Practice Sets

Energy which effectively producing photons is 1 1 1 1


or = − = cm
10 v1 15 30 30
E′1 = ( 3.10 eV ) × = 0.310 eV
100 or v1 = 30 cm
Number of photoelectrons emitted per second is For lens L2, we have
1.55 × 10−3 W u 2 = 30 − 15 = 15 cm, f2 = 10 cm
N1 =
0.31 eV Applying lens formula for L2, we can write
Current due to these electrons, 1 1 1
− =
I1 = N1e v 2 u 2 f2
1.55 × 10−3 1.6 × 10−19 or
1

1
=
1
= × ≈ 5 mA
0.31 1.6 × 10−19 v 2 15 10
For the first wavelength, 1 1 1 25 1
or = + = =
v 2 15 10 150 6
(K max ) eV0 = 3.1 eV − φ ...(i)
where, V0 is stopping potential, φ is work function of the or v 2 = 6 cm
material of the cathode and K max is maximum kinetic energy Thus, final image is formed at 6 cm from L2.
of emitted electrons. Now for the lens L2, we can write magnifying power
For the second wavelength, v Height of image
m2 = 2 =
3.1 u 2 Height of object
eV ′ 0 = K ′ max = × ( 400 − φ )
200 Magnification produced by lens L2 is
where, V′0 and K′max are corresponding values (as  v  Height of image hI
m2 =  2  = =
mentioned in the previous equation) for second wavelength.  u 2  Height of object −0.5 cm
eV′0 = 6.2 eV − φ ...(ii) 6 cm hI
⇒ =
Given that, ′
K max = 5K max 15 cm −0.5 cm
⇒ eV ′ 0 = 5eV0  6 cm  0.6
or hI = ( −0.5 cm)   =− = − 0.2 cm
or V ′0 = 5V0  15 cm 3
⇒ 5eV0 = 6.2 eV − φ ...(iii) Thus, y-coordinate of the image is
From Eqs. (i) and (iii), we get yI = 0.5 cm − 0.2 cm = 0.3 cm
4eV0 = ( 6.2 − 3.1) eV Thus, position of the image is
∴ Stopping potential for the first wavelength, ≡ (21 cm, 0.3 cm)
6.2 − 3.1
V0 = volt 22. a. Consider the circuit diagram shown below. Initially the
4 capacitor 1 has charge Q 0 while capacitor 2 is
3.1 uncharged. When both switches are closed
= volt = 0.775 V = 775 mV
4 simultaneously, applying Kirchhoff’s loop rule, we have
∴ Stopping potential for the second wavelength, Potential drop across the inductor
V ′0 = 5V0 = 3.875 volt
−Ldi S1
21. Consider the combination of lenses L1 and L2 as shown VL =
dt
below. q
q −Ldi
⇒ = 1 C
Ceq dt S2
L
q −Ld dq   dq 
0.5cm or = Qi =
C /2 dt  dt   dt 
q
O I′ 2 C
I
6 cm 2q −Ld 2q
or =
30 cm 15 cm 15 cm C dt 2
2
L1 Ld q 2q
or + =0
L2 dt 2 C
For lens L1, applying lens formula, we can write d 2q 2q
or + =0 …(i)
1 1 1 dt 2 LC
− =
v1 u1 f1 Comparing with standard equation
1  1  1 d 2q
⇒ −  = + ω 2q = 0 …(ii)
v1  −30 15 dt 2
Solved Paper 2007 ●
133

2 2 5 1
ω2 =
⇒ ω= ⇒ =
LC LC 3 v2
1−
Standard solution of Eq. (ii) can be written as c2
q = Q 0 (cos ωt + φ ) 3 v2
At t = 0, q = Q 0 ⇒ φ = 0 or = 1− 2
5 c
∴ q = Q 0 cosωt
9 v2
dq or = 1− 2
Current in the circuit, i = = (Q 0 ω )( − sin ωt ) 25 c
dt
v2 9 16
∴ Maximum current, imax = Q 0 ω or = 1− =
c2 25 25
2
or imax = Q 0 4
LC or v = c
5
b. Current in the circuit is Also, if DE be the diameter of the earth in its own frame,
−dq then diameter of the earth in the frame of the particle is
i= = + Q 0 ω sin ωt
dt
v2
[Q charge decreases on capacitor 1] Dp = D0 1 −
c2
Q t =t
⇒ ∫Q 0
dq = + Q 0 ω ∫
t =0
sin ωtdt
⇒ D0 ×
3
= Dp = vt p
t
5
 − cos ωt  4
or Q − Q 0 = + Q 0ω × = c × 3 × 10−2
 ω 
0 5
−Q 0ω or D0 = 4 × 3 × 108 × 10−2
= [cos ωt − 1]
ω = 12 × 106 cm
= −Q 0 cosωt + Q 0
∴ Diameter of the earth, D0 = 12000 km
or Q = 2Q 0 − Q 0 cosωt = Q 0 [ 2 − cos ωt ]
Q Charge on capacitor 1 24. a. For the path a → b
Q1 = Q 0 [ 2 − cos ωt ] p ∝V
For capacitor 2 p
dq b
i= = −Q 0ω sin ωt
dt
Q t =t
⇒ ∫0 dq = ∫t =0 −Q 0ω sin ωt dt a
t
c
 − cosωt 
or Q = −Q 0ω
 ω 
0
+ Q 0ω V
= [cos ωt − 1] = Q 0 [cos ωt − 1] O a′ c′
ω
∴ Charge on capacitor 2, p a pb
Q 2 = Q 0 [ cos ωt − 1] ⇒ =
Va Vb
23. Suppose the time in the frame of the particle is t p and in (100 kPa ) 200 kPa
the frame of the earth is te . We can write or =
1 m3 Vb
tp
te = ...(i) or Vb = 2 m3
v2
1− 2 Comparing with the process pV − x = constant
c
where, v = speed of the particle Work done during the process ab
c = speed of light. p V − pbVb
Wab = a a
According to the question, given that x −1
t p = 3 × 10−2 s and te = 5 × 10−2 s (100 kPa)(1) − ( 200 kPa )( 2)
=
−1 − 1
From Eq. (i), we can write
100 × 103
3 × 10−2 =− = 50 × 10 3 J = 50 kJ
5 × 10−2 = −2
v2 During process bc , ∆V = 0
1−
c2 Work done, ∆Wbc = p ∆V = 0
134 IIT JAM Physics Solved Papers & Practice Sets

Work done during isobaric process ca −2 −2


∆Sbc = × 103 ln 2 = ln 2 kJ/K .
∆Wca = p∆V 3 3
= p a (Va − Vc ) = p a (Va − Vb ) In the process c → a
= p a (1m3 − 2 m3 ) dQca = nCp dT
= −100 × 10 3 (1m3 ) = 100 kJ Change in entropy
dQ dT
dS = nCp
b. For the process ab T T
∆Q ab = ∆U ab + ∆Wab Ta dT T 
∆Sca = ∫ nCp = nCp ln  a 
⇒ dQ ab = nCV dT + pdV Tc T  Tc 
Change in entropy during as V 
dQ dT pdV = n [R + CV ] ln  a 
dS = = nCV + Vc 
T T T
 R  V 
dT dT = n R +  ln  a 
= nCV + nR  γ − 1 Vb 
T T
Tb dT Tb dT  1   1
⇒ ∆S ab = nCV ∫ + nR ∫ = nR 1+  ln  2 
Ta T Ta T  γ − 1  
T  T  pV  1
= nCV ln  b  + nR ln  b  = a a [1 + 2] ln  
 Ta   Ta  Ta  2
CV p aVa pV  pV pV  100 × 103 × 1  1
= ln  b b  + a a ln  b b  = × 3 ln  
RTa  p aVa  Ta  p aVa  300  2

 pV   pV pV  = −103 ln 2 = − ln 2 kJ/K.
=  ln b b   a a + a a 
 p aVa  (γ − 1)Ta Ta  25. According to the question, given that
 200 × 10 × 2 
3
p aVa  1  F (x , y ) = (x 2 − y 2 ) $i + 2xy $j
⇒ ∆S ab = ln  3  × T 1+ γ − 1
 100 × 10 × 1  a   The object moves along the path OPQR.
100 × 103 × 1  1  Let d r = dx $i + dy $j + dz k$
= 1+ 1.5 − 1 × ln ( 4) Y
3  
103
= [ 3] × ln 4 P Q
3
= 2000 (ln 2)
= 2 ln 2 kJ/K.
X
In the proces b → c O R
dQ = dU + dW = dU + 0 [QdW = 0] Work done by the force F during differential displacement
⇒ dQ = nCV dT dr is
Change in entropy dW = F ⋅ dr = (x 2 − y 2 ) dx + 2xy dy
dQ nCV dT Work done during displacement OP is
dS = = x = 0, y = b
T T WOP = ∫ dW
c dT x = 0, y = 0
∆Sbc = ∫ dS = nCV ∫ x = 0, y = b
=∫
b T
[(x 2 − y 2 ) dx + 2xy dy ]
x = 0, y = 0
T 
= nCV ln  c 
Tb  ⇒ WOP = 0 + 0 = 0 [Qdx = 0]
p  Similarly, work done during PQ is
= nCV ln  c  x = a, y = b
 pb  WPQ = ∫ [(x 2 − y 2 ) dx + ( 2xy ) dy ]
x = 0, y = b
nR p  x = a, y = b
= ln  d  =∫ (x 2 − y 2 ) dx + 0 [Qdy = 0]
γ − 1  pb  x = 0, y = b
x =a
p aVa R  1 =∫ (x 2 − b 2 ) dx
= × ln
RTa γ − 1  2  x =0
a
100 × 103 × 1  1 x 3 1
= × 2 ln   =   − (b 2x )0a = a 3 − b 2a
300  2  3 0 3
Solved Paper 2007 ●
135

Work done during QR is From Stoke’s theorem, we can write

WQR = ∫
x = a, y = 0
[(x 2 − y 2 ) dx + 2xy dy ] ∫ ( ∇ × F) ⋅ n$ dS = ∫ F ⋅ d r
x = a, y = b
Now, curl of the force F, ( ∇ × F) can be written as
x = a, y = 0
=∫ ( 0 + 2xy dy ) [Qdx =0 ]  $i $j k$ 
x = a, y = b
 ∂ ∂ ∂ 
y =0 ∇ × F =
= 2a ∫y = b y dy [Qx = a]
 ∂ x ∂ y ∂ z

0  Fx Fy Fz 
y 2 
= 2a   = a( 0 − b 2 ) = − b 2a  $i $j k$ 
 2 b  ∂ ∂ ∂ 
= 
Work done during RO is
 2 ∂x 2 ∂y ∂z 
x − y
x = 0, y = 0 2xy 0 
WRO = ∫ [(x 2 − y 2 ) dx + 2xy dy ]
x = a, y = 0
x = 0, y = 0 = i ( 0 − 0) − $j ( 0 − 0) + k$ ( 2y + 2y )
$
or WRO = ∫ (x 2 − y 2 ) dx + 0 [Qdy = 0]
x = a, y = 0 = 4y k$
x =0
=∫ x dx2
[Qy = 0 ] ⇒ ∫ ( ∇ × F) ⋅ n$ dS = ∫ 4y k$ ⋅ k$ dy dx
x =a
0 = − 4 ∫ y dy dx
x 3 1
=   = − a3 x =a b
 3 a 3 = −4 ∫ dx ∫0 y dy
x =0

Total work done during O → P → Q → R → O b2


= − 4a ×
WOPQRO = WOP + WPQ + WQR + WRO 2
= − 2 ab 2
1  1 
= 0 + a 3 − b 2a + ( −b 2a ) +  − a 3 Thus, we can write
3  3 
= − 2b a 2 ∫OPQRO F ⋅ d r = ∫ ( ∇ × F) ⋅ n$ dS = − 2 ab 2
Solved Paper
2006
IITJAM Physics
MM : 300 Time : 3 hrs

n
In this question paper, there are total of 25 questions carrying 300 marks of 3 hours duration.
n Questions 1-15 (Objective questions) carry 6 marks each and questions 16-25 (Subjective questions) carry
21 marks each.
n
Each objective question has four choices for its answer i.e. (a), (b), (c) and (d). Only one of them is the
correct answer.
n
There will be negative marking for wrong answers to objective questions. The following marking scheme
for objective questions shall be used:
(a) For each correct answer, you will be awarded 6 (Six) marks.
(b) For each wrong answer, you will be awarded –2 (Negative two) mark.
(c) Multiple answers to a question will be treated as a wrong answer.
(d) For each un-attempted question, you will be awarded 0 (Zero) mark.
n
Do not write more than one answer for the same question. In case you attempt a subjective question
more than once, please cancel the answer(s) you consider wrong. Otherwise, the answer appearing last
only will be evaluated.
n Clip board, log tables, slide rule, calculator, cellular phone and electronic gadgets in any form are not
allowed.

1. In a crystalline solid, the energy band structure (E-k relation for an electron of mass m is given
h 2k ( 2 k − 3 )
by E = . The effective mass of the electron in the crystal is
2m
2
(a) m (b) m
3
m
(c) (d) 2 m
2

2. Two electric dipoles p 1 and p 2 are placed at (0, 0, 0) and (1, 0, 0) respectively, with both of
them pointing in the positive z-direction. Without changing the orientations of the dipoles, p 2 is
moved to (0, 2, 0). The ratio of the electrostatic potential energy of the dipoles after moving to
that before moving is
1 1 1 1
(a) (b) (c) (d)
16 2 4 8
Solved Paper 2006 ●
137

3. The truth table for the given circuit is


J

(a) J K Q (b) J K Q
0 0 1 0 0 1
0 1 0 0 1 0
1 0 1 1 0 0
1 1 0 0 1 1

(c) J K Q (d) J K Q
0 0 0 0 0 0
0 1 1 0 1 1
1 0 0 1 0 1
1 1 1 1 1 0

a iθ
4. At a given point in space the total light wave is composed of three phasors P1 = a , P2 = e
2
a −i θ
and P 3 = e . The intensity of light at this point is
2
 θ  θ
(a) 4a 2 cos 2   (b) 4a 2 cos 4   (c) a 2 cos 2 θ (d) 4a 2 cos 2(2 θ )
2 2

5. A small magnetic dipole is kept at the xy-plane. One wire L1 is located at z = − a in the
xz-plane with a current I flowing in the positive X-direction. Another wire L 2 is at z = + a in
yz-plane with the same current I as in L1, flowing in the positive Y-direction. The angle φ made
by the magnetic dipole with respect to the positive X-axis is
(a) 225° (b) 120° (c) 45° (d) 270°

6. The ratio of the inner radii of two glass tubes of same length is 2. A fluid of viscosity 8.0cρ
flows through the first tube and another fluid of viscosity 0.8 cρ flows through the second one
when equal pressure difference is applied across both of them. The ratio of the flow rate in the
first tube to that in the second tube is
(a) 1.6 (b) 10 2 (c) 2 (d) 0.4

7. The relation between angular frequency ω and wave number k for given type of waves is
ω 2 = αk + βk 3 . The wave number k 0 for which the phase velocity equals the group velocity is
α  1 α α  1 α
(a) 3 (b)   (c) (d)  
β  3 β β 2 β

8. A neutron of mass mn = 10 −27 kg is moving inside a nucleus. Assume the nucleus to be a


cubical box of size 10 −14 m with impenetrable walls. Take, h ~
− 10 −34 Js and 1 MeV ~
− 10 −13 J.
An estimate of the energy (in MeV) of the neutron is
1 1
(a) 80 (b) (c) 8 (d)
8 80
138 IIT JAM Physics Solved Papers & Practice Sets

9. A spring mass system has undamped natural angular frequency ω 0 = 100 rad s −1. The
solution x(t ) at critical damping is given by x(t ) = x 0 (1 + ω 0t ) exp ( − ω 0t ), where x 0 is a constant.
The system experiences the maximum damping force at time
(a) 0.01 s (b) 0.1 s (c) 0.01 π s (d) 0.1 π s

10. In an intrinsic semiconductor, the free carrier concentration n


36.5
(in cm −3 ) varies with temperature T (in Kelvin) as shown in the
35.5
figure alongside. The band gap of the semiconductor is 34.5

In (n)
(use, Boltzmann constant, kB = 8.625 × 10 −5 eVK −1 ) 33.5
(a) 1.44 eV (b) 0.72 eV 32.5
(c) 1.38 eV (d) 0.69 eV
2 3 1000/T

11. E (x, y , z, t ) = ( 3 i$ + 4 $j ) exp [i (ωt − kz )] represents an electromagnetic wave.


Possible directions of the fast axis of a quarter wave plate converts this wave into a
circular wave are
1 1 1 1
(a) [7 $i + $j ]and [ − $i + $j ] (b) [ 3i$ − 4$j ]and [ 4i$ − 3$j ]
2 2 2 2
1 1 $ 1 1 $
(c) [ 3i$ − 4$j ]and [ 4i + 3$j ] (d) [7 $i − $j ]and [ i + 7 $j ]
2 2 2 2

12. A particle of rest mass m 0 is moving uniformly in a straight line with relativistic velocity βc,
where, c is the velocity of light in vacuum and 0 < β < 1. The phase velocity of the de-Broglie
wave associated with the particle is
c c
(a) βc (b) (c) c (d)
β β2

13. Electrons of energy E coming in from x = − ∞ impinge upon a potential V

barrier of width 2a and height V 0 centered at the origin with V 0 > E , as

2 m(V 0 − E ) V0
shown in the figure alongside. Let K = . In the region
h
–a a x
−a ≤ x ≤ a, the wave function for the electrons is a linear combination of
(a) e Kx and e − Kx (b) e iKx and e − Kx
(c) e iKx and e − iKx (d) e − iKx and e Kx

14. A solid melt into a liquid via first order phase transition. The relationship between the
pressure p and the temperature T of the phase transition is p = − 2T + P 0 , where P 0 is a
constant. The entropy change associated with the phase transition is 1.0 J mol −1 K −1.
 dp 
The Clausius-Clapeyron equation for the latent heat is L = T   ∆V . Here,
 dT 
∆V = V liquid − V solid is the change in molar volume at the phase transition. The correct
statement relating the values of the volumes is
(a) Vliquid = Vsolid (b) Vliquid = Vsolid − 1
1
(c) Vliquid = Vsolid − (d) Vliquid = V + 2
2
Solved Paper 2006 ●
139

a 
15. The symmetric part of P =   (a − 2 b ) is
b 
 a 2 − 2 ba − 1  a( −2 ) b   a( a − 1) b( a − 1)  a( a − 2 ) b( a − 1)
(a)   (b)   (c)   (d)  
 ba − 1 b 2 − 2  b b 2  b( a − 1) b2   b( a − 1) b2 

16. Consider a Body Centered Cubic (BCC) crystal with lattice constant A. Determine
a. the miller indices for the (1, 0, 0) plane.
b. the number of atoms per unit area in the (1, 1, 1) plane.

 a 2
17. The equation of state of a gas is p V −  = nRT , while during an adiabatic process the gas
 V 
γ
obeys pV = K , where a and K are positive constants. All other symbols have their usual
meaning. Find the work done by the gas when it is expanded first isothermally from ( p , V ) to
p 
( p 1, 2V ) and then adiabatically from ( p 1, 2V ) to  1 , V1 , where p 1 < p .
 2 

18. A conducting sphere of radius R A has a charge Q. It is surrounded by a


dielectric spherical shell of inner radius R A and outer radius RB (as shown RA
in the figure alongside) having electrical permittivity ε (r ) = ε 0r .
B
a. Find the surface bound charge density at r = R A . A

b. Find the total electrostatic energy stored in the dielectric RB

(region B).

19. For the transistor circuit shown below, evaluate VE , RB and RC , given, IC = 1mA, V CE = 3.8 V,
V BE = 0.7 V and V CC = 10 V. Use the approximation IC ≈ IE
VCC=10 V
RC
RB
VC

VE
24 kΩ
1.3 kΩ

20. For the vector field, V = xz 2 i$ − yz 2 $j + z(x 2 − y 2 ) k$ ,


a. Calculate the volume integral of the divergence of V over the region defined by
−a ≤ x ≤ a, −b ≤ y ≤ b, and 0 ≤ z ≤ c.
b. Calculate the flux of V out of the region through the surface at z = c. Hence, deduce
the net flux through the rest of the boundary of the region.

21. The spherical surface of a plano-convex lens of radius of curvature R = 1m is gently placed on
a flat plate. The space between them is filled with a transparent liquid of refractive index 1.55.
The refractive indices of the lens and the flat plate are 1.5 and 1.6, respectively. The radius of
the sixteenth dark Newton’s ring in the reflected light of wavelength λ is found to be 5 mm.
a. Determine the wavelength λ (in microns) of the light.
b. Now, the transparent liquid is completely removed from the space between the lens
and the flat plate. Find the radius (in mm) of the twenteenth dark ring in the reflected
light after this change.
140 IIT JAM Physics Solved Papers & Practice Sets

22. A resistor of 1kΩ and an inductor of 5 mH are connected in series with a battery of emf 4V
through a switch. The switch is closed at time t = 0. In the following, you may use e 3 ≈ 20.
a. Find the current flowing in the circuit at t = 15 microsecond.
b. Find the heat dissipated through the resistor during the first 15 microsecond.

23. A photon of energy E ph collides with an electron at rest and gets scattered at an angle 60°
with respect to the direction of the incident photon. The ratio of the relativistic kinetic energy T
of the recoiled electron and the incident photon energy E ph is 0.05.
a. Determine the wavelength of the incident photon in terms of the Compton wavelength,
 h 
λ c =  , where h, me , c are Planck’s constant, electron rest mass and velocity of
 me c 
light, respectively.
b. What is the total energy E e of the recoiled electron in units of its rest mass?
3
24. A particle moves in a plane with velocity v = v r r + v θ θ, such that v r = v θ . The time
4
dependence of the magnitude of the velocity | v | = 5t. It is given that r = 1, θ = 0 and v r > 0 at
t = 0. (In the following, you may use e 3 ≈ 20.)
a. Determine the trajectory r (θ) of the particle.
b. At what time will θ become 4 rad?

25. A body of mass 1 kg moves under the influence of a central force, with a potential energy
exp ( −3 / 2)
function V (r ) = − Joule, where r is in metre. It is found to move in a circular orbit of
5r 2
radius r = 2 m. (In the following, you may use e 3 ≈ 20).
a. Find its angular momentum L and total energy E.
b. A piece of mass m1 = 0.5 kg breaks off suddenly from the body and begins to fall
radially inwards with velocity v = 10 cm s −1. What are the values of angular momentum
L 2 and total energy E 2 of the remaining piece, assuming that the potential energy
function remains the same?

Answers
1. (c) 2. (d) 3. (c) 4. (b) 5. (d) 6. (d) 7. (c) 8. (b) 9. (a) 10. (d)
11. (c) 12. (b) 13. (a) 14. (c) 15. (d)
Answers with Explanations
h 2k ( 2k − 3) 3. (c) The given circuit is shown below:
1. (c) Given that, E = ...(i)
2m J JK
A
Effective mass of the electron can be given by
Q
h2 C
me = 2 ...(ii)
∂ E / ∂k 2 JK
B
Now, from Eq. (i), we get K

2h 2k 2 3h 2k In above diagram, A and B are AND gates and C is


E= − OR gate, then outputs of A and B are indicated in the
2m 2m
circuit diagram.
On differentiation w.r.t. k, we get
We can write,
∂E ∂  2k 2h 2 3h 2k  4h 2k 3h 2
= − = − Q = JK + J K = K ( J + J ) = K [Q J + J = 1]
∂k ∂k  2 m 2 m  2 m 2m
Thus, the correct truth table for the output is shown below:
∂E h 2
= [ 4k − 3] J K Q =K
∂k 2 m
Again, differentiation w.r.t. k, we get 0 0 0
2
∂ E h 2
2h 2 0 1 1
⇒ 2
= [ 4] = 1 0 0
∂k 2 m m
2 1 1 1
∂ E
Put the value of in Eq. (ii) , we get
∂k 2
4. (b) According to the question, given that
h2 m a iθ a
me = = P1 = a, P2 = e and P3 = e −i θ
2 h 2/m 2 2 2
2. (d) Consider the two dipoles. The given dipoles are parallel Resultant amplitude can be written as
to each other. P = P1 + P2 + P3
Y a a
= a + ei θ + e −i θ
(0, 2, 0) p1 2 2
p2 a iθ a
(0, 0, 0)
r2 = a + [e + e −i θ ] = a + [ 2 cos θ]
X 2 2
r1
(1, 0, 0)  ei θ + e −i θ 
= a + a cos θ Q cosθ = 
Z
 2 
= a(1 + cos θ )
Potential energy of the two dipole systems is
1 Thus, intensity is
U p1p 2 ∝ 3
r I = (P )2 [QP = P1 + P2 + P3]
(U p1p 2 )1 1 I = [a(1 + cos θ )] 2
⇒ = × r23
(U p1p 2 )2 r13 = a 2(1 + cos θ )2 = a 2( 2 cos2 θ / 2)2
where, r is the separation between the dipoles. = 4a 2 cos4 θ / 2
3
(U p1p 2 )1 r 
⇒ =  2 5. (d) The magnetic fields due to wires L1 and L2 are of same
(U p1p 2 )2  r1 
magnitude.
3 Z
 ( 0 − 0 )2 + ( 2 − 0 )2 + ( 0 − 0 )2 
= 
 
 (1− 0)2 + ( 0 − 0)2 + ( 0 − 0)2  z = +a
I
L2
3/ 2
 0 + 4 + 0 O
Y
=  = ( 2 2 )3 / 2 = 2 3 = 8 Magnetic
 1+ 0 + 0  dipole
z=–a
(U p1p 2 )2 1 X
⇒ = I
(U p1p 2 )1 8 L1
142 IIT JAM Physics Solved Papers & Practice Sets

Since, both are placed at equal distance from the 8. (b) Given that,
magnetic dipole. L = 10−14 m, mn = 10−27 kg
Same amount of current flow in both the wires. The
h = 10−34 Js and 1 MeV = 1.6 × 10−13 J
magnetic field due to wire L1 is leftward and due to
L2 it is along xy-plane. Hence, the dipole will be Using Heisenberg’s uncertainty principle, we can write
located along Y-axis. ∆x ∆p = h
Thus, angle made by the dipole with respect to the h 10−34
⇒ ∆p = = −14 = 10−20
positive X-axis is ∆x 10
θ = 360° − 90° = 270° Estimated energy of the neutron is
6. (d) Rate of flow of liquid is given by ∆p 2 (10−20 )2
E= =
πρr 4 2mn 2 × 10−27
v =
8ηl 10−40 + 27 1
= = × 10−13
where, symbols have their usual meanings. 2 2
For the first pipe, 0.5 MeV 5 1
= = MeV ≈ MeV
πρr14 1.6 16 3
v1 =
8 × 8c ρ × l Closest option is (b).
For the second pipe, 9. (a) According to the question, given that
πρr24 x (t ) = x 0(1 + ω 0t ) e − ω 0t
v2 =
8 × ( 0.8 c ρ ) × l On differentiation w.r.t.t, we get
Now, we can write dx
= x 0 [e − ω 0t × ( −ω 0 ) + ω 0t e − ω 0t ( −ω 0 ) + e − ω 0t × ω 0]
4 dt
v1  r1  1
=   × × 0.8 = x 0 [ −ω 02t e − ω 0t ]
v 2  r2  8
d dV dU
1 [using product rule (U .V ) = U +V ]
= ( 2 )4 × dx dx dx
10
v1 4 Again, differentiation w.r.t. t, we get
= = 0.4 d 2x
v 2 10 d 
⇒ = x 0( −ω 20 ) (te − ω 0 t )
dt 2 
dt 
7. (c) According to the question, given that
= − ω 20x 0 [te − ω 0t ( −ω 0 ) + e − ω 0t ]
ω 2 = αk + βk 3 ...(i)
= − ω 20x 0 × e − ω 0t [1 − ω 0 t ]
Phase velocity is given by
ω Force on the mass,
vp =
d 2x
k F = m 2 = − mω 20x 0e − ω 0t [1 − ω 0 t ]
Group velocity can be given by dt
dω The force will be maximum if
vg =
dk d 2x
=0
On differentiation Eq. (i) w.r.t. k, we get dt 2
dω ⇒ − ω 20x 0e − ω 0t [1 − ω 0 t ] = 0
2ω = α + 3 βk 2
dk or 1 − ω 0t = 0
d ω α + 3βk 02 ω 1 1
⇒ = = or t= = = 0.01 s
dk 2ω k0 ω 0 100
(given)
10. (d) Intrinsic carrier concentration can be expressed as
⇒ k 0α + 3 βk 03 = 2ω 2 −Eg / 2kT
ni = (Nc Nv )1/ 2 e
= 2 αk 0 + 2 βk 03 −Eg /kT
⇒ ni2 = Nc Nv e
⇒ k 0α = βk 03
where, symbols have their usual meanings.
α
or k 02 = Now, we can write
β −Eg /kT1
ni21 = Nc Nv e
α
or k0 = ni22 = Nc Nv e
−Eg /kT 2
β
Solved Paper 2006 ●
143

 1 1 
ni21 e
−Eg /kT1 Eg  −  13. (a) The general Schrödinger equation is
 kT 2 kT1 
⇒ = −Eg /kT 2
=e d 2ψ 2 m
ni22 e − 2 (V0 − E ) ψ = 0 ...(i)
2 dx 2 h
n  Eg  1 1 where, symbols have their usual meanings.
or ln  i 1  =  − 
 ni 2  k T2 T1  According to the question,
 n  Eg  1 1  2 m(V0 − E )
or 2 In  i 1  = − K =
 ni 2  k T2 T1  h
2 2 m(V0 − E )
Eg  1 1  ⇒ K = ...(ii)
or 2 In ni 1 − In ni 2 = − h2
k T2 T1 
From Eqs. (i) and (ii), we have
Considering the given figure in the question, we can
d 2ψ
write − K 2ψ = 0
dx 2
Eg 1000 1000 
2 [ 36.5 − 32.5] = − d 2ψ
1000k  T2 T1  or = K 2ψ ...(iii)
dx 2
Eg
= [ 3 − 2] Solution of Eq. (iii) can be found as
1000k ψ = aeKx + be −Kx
⇒ Eg = 2 × 4 × 1000 × 8.625 × 10−5 eV
where, a and b are constants.
= 8 × 8.625 × 10−2 eV
14. (c) Given, p = − 2T + P0 ...(i)
= 69 × 10−2 eV = 0.69 eV
On differentiation w.r.t. T, we get
11. (c) The quarter wave plate will introduce a path difference dp
= −2 ...(i)
λ π dT
of that is a phase difference of . The combination of
4 2  dp 
L = T   ∆V
circularly polarised waves should have equal amplitude  dT 
as the given electromagnetic wave. One of the waves = T [ −2] [ ∆V ] [from Eq. (i)]
will be clockwise and other should be anti-clockwise.
⇒ L = − 2T ∆V ...(ii)
These conditions are satisfied by the waves in the
option (c). As we know that change in entropy,
∆Q
12. (b) Wavelength associated with the particle, ∆S =
T
h
λ= ...(i) where, ∆Q = mL, during phase transition
mv
mL L
2π 2πmv ⇒ ∆S = = [Qm = 1]
⇒ k = = ...(ii) T T
λ h
1
Energy (E ) of the particle can be expressed as = [ −2T ∆V ] [from Eq.(ii)]
T
hc ω
E= = hν = h × ⇒ ∆S = − 2 ∆V
λ 2π
2 πE Given, ∆S = 1J mol−1 K −1
⇒ ω= ...(iii)
h 1 = − 2∆V
Also, using Einstein’s relation, we can write −1
or ∆V =
E = mc 2 2
Again, from the question
2πmc 2
⇒ ω= [from Eq. (iii)] ∆V = V liquid − Vsolid
h
Now, phase velocity (v p ) can be written as 1 1
or Vliquid −Vsolid = − ⇒ V liquid = Vsolid −
ω 2πmc 2
2πmc 2 2 2
vp = = = ×h [from Eq. (ii)]
k hk h × 2πmv 15. (d) According to the question,
c 2 a  a(a − 2) ab 
= P =   [a − 2 b] =  2
v b  b(a − 2) b 
From the question, v = βc Transpose of the matrix,
c2 c a(a − 2) b(a − 2)
⇒ vp = = PT = 
βc β  ab b 2 
144 IIT JAM Physics Solved Papers & Practice Sets

Now, symmetric part of the matrix is 17. Equation of state is given as


1  a2
Ps = [P + P T ] p V −  = nRT
2
 V
1  2a(a − 2) ab + b(a − 2)
=   For an isothermal process,T = constant
2 b(a − 2) + ab 2b2  nRT
⇒ p=
1 2 a(a − 2) 2ab − 2 b  a2
= V −
2  2ab − 2b 2 b 2  V
Work done by the gas,
a(a − 2) ab − b  a(a − 2) b(a − 1)  
Ps =  =
 ab − b b 2   b(a − 1 b 2   nRT 
W = ∫ p dV = ∫   dV
16. a. Consider a (1, 0, 0) plane.  a2 
V − 
 V 
Intercept of the given plane on X, Y and Z-axes
2V dV V dV
respectively are 1, ∞ and ∞. = nRT ∫ = nRT ∫
Thus, miller indices are
V a2 V 2 − a2
V −
1 1 1 V
h = = 1, k = = 0, l = = 0 Put 2
z =V − a 2
1 ∞ ∞
Z ⇒ dz = 2V dV
dz / 2
⇒ W = nRT ∫ z
(1, 0, 0)
nRT dz
2 ∫
=
X z
nRT nRT
= (ln z )V2V = [ln (V 2 − a 2 )]V2V
Y 2 2
nRT
b. Consider the (1, 1, 1) plane. = [ln ( 4V 2 − a 2 ) − ln (V 2 − a 2 )]
2
Clearly, from the adjacent figure
nRT  4V 2 − a 2 
AB = BC = CA = 2 A = ln
2  V 2 − a 2 
Z
nRT  4V 2 − a 2 
A (0, 0, 1) ⇒ W = ln  2 
2  V − a2 
Work done during an adiabatic process,
Y p V − p 2V2
C (0, 1, 0) W = 11
D γ −1
B (1, 0, 0) p
X p1 × ( 2V ) − 1 × V1
= 2
1 γ −1
Area of ∆ABC = × BC × AD pV
2 2p1V − 1 1
1 = 2
= × 2 A × AB × cos 30° γ −1
2
4p V − p1V1
1  3 = 1
= ( 2 A) × ( 2 A)   2( γ − 1)
2  2
18. a. Field produced at the surface of the conductor due to
3 2 −ρ
= A = 0.866A 2 bounded charge is Es = .
2 E0
As the crystal is BCC, we can write, Dielectric
Number of atoms in ∆ABC
1
= 1+ × 3 = 2 O
3
RB RB
∴ The number of atoms per unit area is Conductor
2 2.31
= ≈ 2
0.866A 2 A
where, P is the polarisation of the dielectric medium.
Solved Paper 2006 ●
145

Also, field at the surface of the conductor due to internal 19. Consider the circuit shown below.
ρ Given, IC = IE = 1mA
dipoles is, Ei =
3E0 VCE = 3.8 V
Net electric field at the surface is VBE = 0.7 V
−ρ ρ −2 ρ VCC = 10 V
E = Es + Ei = + =
E0 3E0 3E0
VE = ?, RB = ? and RC = ?

⇒ |E | = ...(i) For the outer path, we can write
3E0
VCC = 10 V
Field at the surface of the conductor due to its own
RC
charge Q is 1 Q
RB
VC
|E | = ...(ii)
4πE0 RA2 Outer path
From Eqs. (i) and (ii), we have 24 k Ω VE
2ρ 1 Q Inner 1.3 kΩ
= path
3E0 4πE0 R A2
3Q (Assume grounded)
⇒ Surface bound charge density, ρ =
8πRA2
VCC = ICRC + VCE + IE RE
b. Consider a spherical shell of radius r and thickness dr is
the dielectric. ⇒ 10 = 1 × 10−3 × RC + 3.8 + 1 × 10−3 × 1. 3 × 103
⇒ 10 = RC × 10−3 + 3.8 + 1. 3
Density
⇒ RC × 10−3 = 10 − 5.1 = 4. 9
⇒ RC = 4.9 kΩ
r dr
O Consider the left most inner path, for which we can write
as I1 × 24 × 103 = VBE + IE × 13
. × 103 [QIB ≈ 0]
Shell
⇒ I1 × 24 × 103 = 0.7 + 1. 3 × 10−3 × 103
1 Q = 0.7 + 1.3 = 2
Electric field at r can be given as, E =
4πε r 2 2 1
⇒ I1 = × 10−3 = mA
Energy density of the field at this position is 24 12
1 2 1  1 Q
2 Again for the inner path, we have
U = εE = ε ×  
2 2  4πε r 2  VCC = I1RB + I1 × 24 × 103
Electrostatic energy stored in this shell of thickness dr is 1 1
1 ⇒ 10 = × RB × 10−3 + × 10−3 × 24 × 103
dU = εE 2( 4πr 2dr ) 12 12
2
RB × 10−3
Total energy stored in the dielectric is ⇒ 10 = +2
RA 1
12
U = ∫ du = ∫ εE 2( 4πr 2dr ) ⇒ 8 × 12 = RB × 10−3
RB 2
2
RA 1  1 Q ⇒ RB = 96 k Ω
=∫ ×ε × 2
 4πr dr
RB 2  4πε r 2  20. a. Given, vector field is
RA E Q2 1
=∫ × 4 × × 4πr 2dr V = xz 2$i − yz 2$j + z (x 2 − y 2 ) k$
RB 2 r ( 4 π )2 ε 2
Divergence of V is
RA Q 2 1 1 Q2 RA dr
=∫ ∂ ∂ ∂
RB 2r 2
×
4πε
× dr =
2 4π ∫R r 2 × ε 0r ∇⋅ V = Vx + Vy + Vz
B
∂x ∂y ∂z
1 Q2 R A dr Q2 RA = ( z 2 − z 2 ) + (x 2 − y 2 )
= ∫R 3
= ∫R r −3dr
2 4πε 0 B r 8πε 0 B
=x2 −y2
RA −2 R A
Q 2  r −3 + 1  Q 2 r Volume integral of the divergence of V is
=   =
8πε 0  −3 + 1 R 8πε 0  −2 R a b c
B B
∫ ( ∇ ⋅ V ) dV = ∫
x = −a ∫y = −b ∫z = 0 dx dy dz
−Q 2  1 1 Q2  1 1
=  2 − 2  = 16πε  2 − 2 a b c
16πε 0 RB RA  0  A
R RB
=∫ ∫y = −b ∫z = 0 (x 2 − y 2 ) dx dy dz
x = −a
146 IIT JAM Physics Solved Papers & Practice Sets

=∫
a b c
From the geometry of the figure, we have
x = −a
x 2dx ∫y = −b dy ∫z = 0 dz R 2 = (R − t )2 + rn2
a b c
−∫ dx ∫y = −b y 2dy ∫z = 0 dz where, rn is radius of nth ring
x = −a
a b ⇒ R 2 = R 2 + t 2 − 2Rt + rn2
x 3  y 3 
=   [ y ]b−b [ z ]c0 − [x ] a− a   [ z ]c0 ⇒ 2Rt = t 2 + rn2 ≈ rn2 [Q t is very small]
 3 −a  3  −b
1 [b 3 − ( −b )3] rn2
= [a 3 − ( −a )3] ( 2 b ) (c ) − ( 2 a ) ×c ⇒ 2t = ...(ii)
3 3 R
2a3 1 From Eqs. (i) and (ii), we get
= × 2 b × c − 2a × × ( 2 b 3 ) (c )
3 3 rn2 λ
µl × = ( 2n − 1)
4 3 4 R 2
= [a bc − ab c ] = abc (a 2 − b 2 )
3
3 3 For sixteenth dark fringe n = 16, because central ring is
b. Flux of V, bright.
a b ( 5 × 10−3 )2 λ
ψ V = ∫ V ⋅ dS = ∫ ∫y = −b V ⋅ k dx dy ⇒ 1. 55 × = ( 2 × 16 − 1)
x = −a 1 2
a b 31 λ
=∫ ∫y = −b z (x 2 − y 2 ) dx dy ⇒ 1. 55 × 5 × 10−6 =
x = −a 2
= c ∫ y 2dx dy  [Q z = c ] 2 × 1. 55 × 5 × 10−6
a b a b

 − a ∫−b x 2dx dy − ∫
−a ∫−b  ⇒ λ=
31
m

= c ∫ x 2dx y 2dy  3.1 × 5 × 10−6


a b a b

 − a ∫−b dy − ∫
−a
dx ∫−b  = m
31
 x 3  a y 3 
b
⇒ λ = 5 × 10−7 = 5000 Å
= c   ( 2 b ) − ( 2 a )   
 3  − a  3  −b 
 b. When the transparent liquid is removed from the space
 4a b 4ab 3
4 3 between the lens and the flat plate an additional path
2 2
=c  −  = 3 abc (a − b ) difference of λ / 2 is introduced.
 3 3 
Thus, condition for maxima and minima gets reversed.
Using Gauss divergence theorem, we can write
Now, we can write
∫ ( ∇ ⋅ V ) d V = ∫∫ V ⋅ d S rn2 = 2Rt ...(iii)
⇒ ∫∫ V ⋅ d S = ∫ ( ∇ ⋅ V ) d V and 2 t = nλ ...(iv)
From Eqs. (iii) and (iv), we get
Rest surface z =c
rn2 = R × (nλ )
⇒ ∫∫ V ⋅ d S = ∫ ( ∇ ⋅ V ) d V − ∫∫ V ⋅ d S
Rest surface z =c = 20 × 1 × 5 × 10−7 [Qn = 20]
4 4 = 10 −5
= abc (a 2 `− b 2 ) − abc (a 2 − b 2 )
3 3
⇒ rn = 10−5m
=0
= 10 mm = 3.16 mm
21. a. For the formation of dark fringe, we can write
λ 22. a. When the switch is closed at t = 0. 1 kΩ 5 mH
2 µl t = ( 2n − 1) ...(i)
2 The circuit acts as charging R-L
O
circuit.
4V
Instantaneous current flowing
R–t R through the circuit can be given St = 0
Liquid as t

i = i0(1 − e L /R )
rn
R
t − t
⇒ i = i0 (1 − e L ) ...(i)

where, t = thickness of the liquid, where, i0 = steady state current


λ = wavelength of the light, 4 4
= = = 4 mA
µl = refractive index of the liquid. 1k Ω 1000
Solved Paper 2006 ●
147

Putting t = 15 × 10−6 s, L = 5 × 10−3 H and R = 1k Ω in Eq. 23. a. Consider the diagram in which the incident photon gets
(i), we have scattered at an angle φp and electron through φe .
−10 3 −6 Energy of the incident photon,
× 15 × 10
−3
i = 4 × 10−3 (1 − e 5 × 10 ) E0 = h ν 0 =
hc
= Eph (given)
−3 −3 λ0
= 4 × 10 (1 − e )
 1  1 Scattered
= 4 × 10−3 1 − 3  = 4 × 10−3 1 −  photon
 e   20 
E =hν
19 φp
= 4 × 10−3 × = 3.8 mA φe
20 Photon
(incident)
b. Heat dissipated through the resistor during first t Electron
seconds is recoiled
t
H = ∫ i 2R dt Change in energy of the photon,
0
hc hc

R ∆E = Eph − E = −
t
= ∫ [i0(1 − e L
t
)] 2 R dt λ0 λ
0
hc hc

Rt −
=∫
t
i02R (1 − e L )2 ∆E λ 0 λ λ λ − λ0 ∆λ
0
dt ⇒ = = 1− 0 = =
Eph hc λ λ λ 0 + ∆λ
E02  t 
2Rt
− −
Rt
λ0
= ∫ (1 + e L − 2e L ) dt 
R 

0
 Now, the Compton shift can be expressed as
h
E02  t t −
2Rt
t −
tR  ∆λ = (1 − cos φp )
R ∫0 ∫0 e
=  dt + L dt − 2 ∫ e L dt  mec
0
  ⇒ ∆λ = λ c (1 − cos φp )
  − 2Rt 
t
 −Rt  
t
Given, φp = 60°
2  e L  e L  
E λc
= 0 t +   −2×   ⇒ ∆λ = λ c (1 − cos 60° ) =
R   − 2R   −R   2
  L 0  L  0 According to the question, given that

 2Rt Rt  ∆E
E02 L − 2L − L = 0.05
= t − × (e L − 1) + (e − 1) Eph
R  2R R 
 ∆λ λc / 2
⇒ 0.05 = =
Putting t = 15 × 10−6 s, we get λ 0 + ∆λ λ 0 + λ c / 2
 − 2 × 10 3 × 15 × 10 −6 λc λc 100λ c
16  5 × 10 −3
5 × 10 −3 ⇒ λ0 + = = = 10λ c
H= 15 × 10−6 − (e − 1) 2 2 × 0 .05 10
1 × 103  2 × 103
 λc
⇒ λ 0 = 10λ c − = 9 . 5λ c
3
10 × 15 × 10 −6
 2

2 × 5 × 10−3 ∆E
+ (e 5 × 10 −3
− 1) b. Given that, = 0.05
103  Eph

∆K
= 16 × 10−3 [15 × 10−6 − 2 . 5 × 10−6 (e −6 − 1) ⇒ = 0.05 [Q ∆E = ∆K ]
Eph
−6 −3
+ 10 × 10 (e − 1)] hc
⇒ ∆K = 0.05 ×
  1   1  λ0
= 16 × 10−9 15 − 2 . 5  400 − 1 + 10  20 − 1 
  19 λ c
Also, from previous result, λ 0 =
= 16 × 10−9 [15 − 2 . 5 × ( 0.00025 − 1) + 10 ( 0.05 − 1)] 2
hc hc
= 16 × 10−9 [15 − 2.5 × ( −0 . 99975) + 10 ( −0.95)] ⇒ ∆K = 0.05 × = 0.05 × 2 ×
19 λ c 19 λ c
= 16 × 10−9 [15 + 2.4993 − 9.5] 2
= 16 × 10−9 × 7.999 hc  h 
= 0.1 mec Q λc =
19 × h  mec 
= 127.99 × 10−9 J
1
H = 12.799 × 10−10 J ≈ 12.8 × 10−10 J = × mec 2
190
148 IIT JAM Physics Solved Papers & Practice Sets

3
From relativistic expression, we can write 1 −2 r  3 2
2 | F| = − e  − 2 − 3
∆K = (m − me ) c 5  2r r 
1 Putting r = 2, we get
or mc 2 − mec 2 = mec 2
190 1  3 2 1  5
 1  = − e −3 − − = − e −3 −
or mc 2 =  + 1 mec 2 5  8 8  5  8 
 190 
1 1 1 1
191 = e −3 = × =
or Ee = mc 2 = mec 2 8 8 20 160
190
For circular motion,
24. a. Time dependent velocity is given as mv 2 1
| F| = =
| v | = 5t ...(i) r 160
We can write, 1 1
2 ⇒ v2 = ⇒v =
3   3  80 80
| v | = vr2 + v θ2 =  v θ  + v θ2 Qvr = v θ
4   4  Thus, angular momentum of the body,
9  5 L = mvr = (1) (v ) ( 2)
=vθ 1+ = (v θ )  
16  4 1 2 1
= 2v = 2 × = ×
5 80 4 5
⇒ v θ = 5t [from Eq. (i)]
4 1 1 1
dθ 4 = × = J- s
or r = 5t × 2 5 2 5
dt 5 1
or d θ = 4t dt [Qr = 1] Total energy, E = KE + PE = mv 2 + V (r )
2
or ∫ d θ = ∫ 4t dt  − × 2
 e 2 
3
1 1 1 e −3
4t 2 = × 1× + − = −
or θ= + c = 2t2 + c 2 80  5 × 4  160 20
2  
At t = 0, θ = 0
1 1 1 1 3
⇒ c = 0 ⇒ θ = 2t2 = − = − = J
160 20 e 3 160 400 800
b. Given that, θ = 4 rad ⇒ 4 = 2 t 2
b. Conserving angular momentum of the system about
or t 2 = 2 or t = 2 = 1. 414 s centre, we have
3 Li = Lf
Again, vr = v θ
4 1
dr 3 d θ dr 3 = Lf
= r or = dθ 2 5
dt 4 d t r 4
Lf consists of angular momentum of first piece ( = 0) and
r dr 3 θ second piece.
or ∫r =1 r
= ∫
4 θ=0

1
3 Hence, Lf = L 2 = J- s
or ln r = θ or 4 ln r = 3θ 2 5
4 1
or ln r 4 = 3θ or r 4 = e 3θ L 2 = m2v 2t r =
2 5
θ/4
or r = (e 3θ )1/ 4 = (e 3 ) = ( 20)θ / 4 1 1
or ( 0.5) (v 2 ) ( 2 m ) = ⇒ v 2t = m/s
r 2 5 2 5
e − ( 3/ 2)
25. a. Potential energy of the body is,V (r ) = − ∴ Magnitude of total velocity of the second particle,
5r 2
 − 3r   1
2
 1 
2
∂V (r ) ∂ e 2  v 2 = v 22r + v 22t =   +  
⇒ Force, F (r ) = − =−  10  2 5
∂r ∂r  5r 2 
 
1 1 1+ 5 6
= + = = m/s
∂ 1 3
− r  1∂
3
− r  100 20 100 100
=−  r −2e 2 =−  r −2e 2 
∂r  5  5  ∂r  ∴ Total energy of the remaining piece,
 
1 1 1 6 e −3
1  m2 × v 22 + v (r ) = × ×
3 3
 3 − 2 r − r E2 = −
= − r −2 −  e + e 2 ( −2r −3 ) 2 2 2 100 20
5  2
  3 1 6 −1 5 1
= − = = = J
[Q using product rule] 200 400 400 400 80
Solved Paper
2005
IITJAM Physics
MM : 300 Time : 3 hrs

n
In this question paper, there are total of 25 questions carrying 300 marks of 3 hours duration.
n Questions 1-15 (Objective questions) carry 6 marks each and questions 16-25 (Subjective questions) carry
21 marks each.
n
Each objective question has four choices for its answer i.e. (a), (b), (c) and (d). Only one of them is the
correct answer.
n
There will be negative marking for wrong answers to objective questions. The following marking scheme
for objective questions shall be used:
(a) For each correct answer, you will be awarded 6 (Six) marks.
(b) For each wrong answer, you will be awarded –2 (Negative two) mark.
(c) Multiple answers to a question will be treated as a wrong answer.
(d) For each un-attempted question, you will be awarded 0 (Zero) mark.
n
Do not write more than one answer for the same question. In case you attempt a subjective question
more than once, please cancel the answer(s) you consider wrong. Otherwise, the answer appearing last
only will be evaluated.
n Clip board, log tables, slide rule, calculator, cellular phone and electronic gadgets in any form are not
allowed.

1. A solid sphere of mass m and radius a is rolling with a linear speed v on a flat surface without
slipping. The magnitude of the angular momentum of the sphere with respect to a point along
the path of the sphere on the surface is
2 7
(a) mav (b) mav
5 5
3
(c) mav (d) mav
2

2. The susceptibility of a diamagnetic material is


(a) positive and proportional to temperature
(b) negative and inversely proportional to temperature
(c) negative and independent of temperature
(d) positive and inversely proportional to temperature
150 IIT JAM Physics Solved Papers & Practice Sets

5
3. The molar specific heat of a gas as given from the kinetic theory is R. If it is not specified
2
whether it is C p or CV , one could conclude that the molecules of the gas
(a) are definitely monoatomic (b) are definitely rigid diatomic
(c) are definitely non-rigid diatomic (d) can be monoatomic or rigid diatomic

4. The value of entropy at absolute zero of temperature would be


(a) zero for all the materials (b) finite for all the materials
(c) zero for some materials and non-zero for others (d) unpredictable for any material

5. A circuit and the signal applied at its input terminals (Vi ) are shown in figure below. Which one of
the options correctly describes the output waveform (Vo )? (Assume all the devices used are
ideal).
C
Vi D Vo

+2 V

0 t

–2 V
+2 V +2 V

(a) 0 t (b) 0 t

–2 V –2 V

4V

(c) 0 t (d) 0 t

–4 V

6. Consider a beam of light of wavelength λ incident on a system of a polariser and an analyser.


The analyser is oriented at 45° to the polariser. When an optical component is introduced
between them, the output intensity becomes zero. Light is incident normally on all components.
The optical component is
(a) a full-wave plate (b) a half-wave plate
(c) a quarter-wave plate (d) an ordinary glass plate

7. A small loop of wire of area A = 0.01m 2 , N = 40 turns and resistance R = 20 Ω is initially kept
in a uniform magnetic field B in such a way that the field is normal to the plane of the loop.
When it is pulled out of the magnetic field, a total charge of Q = 2 × 10 −5 C flows through the
coil. The magnitude of the field B is
(a) 1 × 10−3 T (b) 4 × 10−3 T
(c) zero (d) unobtainable, as the data is insufficient
Solved Paper 2005 ●
151

8. If M e , M p and M H are the rest masses of electron, proton and hydrogen atom in the ground
state (with energy −13.6 eV), respectively, which of the following is exactly true? (c is the
speed of light in free space)
13.6 eV
(a) MH = Mp + Me (b) MH = Mp + Me −
c2
13.6 eV 13.6 eV
(c) MH = Mp + Me + (d) MH = Mp + Me + K , where K ≠ ± or zero
c2 c2

9. An observer is sitting on a horizontal platform which is rotating with a constant angular velocity.
He puts an object on the smooth frictionless floor of the platform, away from the axis of
rotation, with zero initial velocity with respect to him. Let the time at this instant be t = 0. In the
frame of the platform, the object would
(a) remain at rest for all t > 0
(b) accelerate purely in a radial direction outwards for all t > 0
(c) accelerate purely in a tangential direction for all t > 0
(d) accelerate radially in the outward direction at t = 0, however the direction of acceleration
changes for t > 0

 0 1
10. Which of the following is incorrect for the matrix M =  ?
 1 0
(a) It is its own inverse (b) It is its own transpose
(c) It is non-orthogonal (d) It has eigen values ± 1

11. A combination of two thin convex lenses of equal focal lengths is kept separated along the
optic axes by a distance of 20 cm between them. The combination behaves as a lens system
of infinite focal length. If an object is kept at 10 cm from the first lens, its image will be formed
on the other side at a distance x from the second lens. The value of x is
(a) 10 cm (b) 20 cm (c) 6.67 cm (d) infinite

12. Two point charges +q 1 and +q 2 are fixed with a finite distance d between them. It is desired to
put a third charge q 3 in between these two charges on the line joining them so that the charge
q 3 is in equilibrium. This is
(a) possible only if q 3 is positive (b) possible only if q 3 is negative
(c) possible irrespective of the sign of q 3 (d) not possible at all

13. A periodic function can be expressed in a Fourier series of the form,



f (x ) = ∑ (a n cos nx + b n sin nx ).
n=0

The functions f1(x ) = cos 2 x and f 2 (x ) = sin 2 x are expanded in their respective Fourier series.
If the coefficients for the first series are a n1 and b n1 , and the coefficients for the second series
are a n2 and b n2 , respectively, then which of the following is correct?
1 −1 1 −1 1 −1 1 −1
(a) a 12 = and b 22 = (b) b 12 = and a 22 = (c) a 12 = and a 22 = (d) b 12 = and b 22 =
2 2 2 2 2 2 2 2

14. Which of the following statements is correct for NaCl crystal structure?
(a) It is a simple cubic lattice with one atom basis
(b) It is a face centered cubic lattice with one atom basis
(c) It is a simple cubic lattice with two atoms basis
(d) It is a face centered cubic lattice with two atoms basis
152 IIT JAM Physics Solved Papers & Practice Sets

15. Which of the following statements is incorrect?


(a) Indistinguishable particles obey Maxwell-Boltzmann statistics
(b) All particles of an ideal Bose gas occupy a single energy state at T = 0
(c) The integral spin particles obey Bose-Einstein statistics
(d) Protons obey Fermi-Dirac statistics

16. a. Consider a constant vector field v = v 0 k$ . Find anyone of the many possible vectors u, for
which ∇ × u = v.
b. Using Stoke’s theorem, evaluate the flux associated with the field v through the curved
hemispherical surface defined by x 2 + y 2 + z 2 = r 2 , z > 0.

17. A logic circuit and the time varying logic levels applied at its A and B inputs are shown below.
Sketch the corresponding output waveform at points C, D, E, F and G in the space given
below the waveforms A and B.
C
A
E 1
B t
A0
1
G B0
D
F

Y
18. a. Determine whether the force represented by B D (1, 1, 0)
F (x, y ) = k [(x 2 + y 2 ) $i + 2 xy $j] is conservative or not.
Here, k = 1 Nm −2 .
b. Calculate the work done by this force in moving a particle A
X
from the origin O ( 0, 0, 0) to the point D (1, 1, 0) on the z = 0 O (0.5, 0, 0)
plane along the paths OABD and OD as shown in the figure,
where the coordinates are measured in metres.

19. A rod is moving with a speed of 0.4c along its length in the positive x-direction and a particle is
moving along the negative x-direction with a speed 0.8c as shown in the figure below. Both
the speeds are measured in an inertial frame S and c is the velocity of light in free space. The
length of the rod as measured in the S-frame is 3.6 m.
0.8 c
0.4 c

a. Find the relative velocity of the rod (in terms of c) in the rest frame of the particle.
b. Find (a) the time taken for the particle to cross the rod in the S-frame and in the rest
frame of the rod and (b) time taken by the rod to cross the particle in the rest frame of
the particle.

20. A particle of mass m and energy E moving in the positive x-direction, encounters a
one-dimensional potential barrier at x = 0. The barrier is defined by
V = 0 for x < 0
V = V 0 for x ≥ 0 (V 0 is positive and E > V 0 )
If the wave function of the particle in the region x < 0 is given as Ae ikx + Be −ikx ,
B
a. Find the ratio .
A
B E
b. If = 0.4, then find , and the transmission and reflection coefficients.
A V0
Solved Paper 2005 ●
153

∂U   ∂p  
21. a. Establish the equation dU =   dT + T   − p  dV ,
 ∂T  V   ∂T  V 
 ∂S   ∂p 
Given that, dU = TdS − pdV and   =  
 ∂V  T  ∂T  V
where, U, p, T, V and S are, respectively, the internal energy, pressure, temperature,
volume and entropy of the system.
b. If the specific heat is taken to be independent of T, utilise the above equation to derive
an expression for U (T , V ) for one mole of a van der Waals’ gas and then obtain the
corresponding expression for an ideal gas.
dy
22. Solve the differential equation xy = 3 y 2 + x 2 with the initial condition y = 2, when x = 1.
dx

23. A beam of light is incident normally on a diffraction grating of width 2 cm. It is found that at
30°, the nth order diffraction maximum for λ 1 = 5000 Å is superimposed on the (n + 1)th order
for λ 2 = 4000 Å. How many lines per cm do the grating have? Find out whether the first order
spectrum from such a grating can be used to resolve the wavelengths λ 3 = 5800 Å and
λ 4 = 5802 Å.

24. Consider two electromagnetic plane waves propagating in vacuum with their electric field
vectors
E1 = E 0 cos (kz − ωt ) i$
and E = E cos (kz + ωt ) i$.
2 0

a. Evaluate the magnetic field vector corresponding to the superposition of these two
waves.
b. Calculate the time averaged energy density as well as the time averaged Poynting
vector for the resultant wave. The time average is carried over one period of
oscillation.

25. A 150 Ω resistor, a 10 µF capacitor and a 0.1 H inductor are connected in series to an
AC source operating at an angular frequency ω.
a. Find the value of ω for which the combination acts as a pure resistive load.
b. The AC source is operated at a peak voltage of 300 2 V and a frequency equal to half
the resonance frequency of the circuit. Find the peak value of the current in the circuit
and the phase difference between the current and voltage. Also, find the peak voltage
across the inductor.

Answers
1. (b) 2. (c) 3. (d) 4. (a) 5. (b) 6. (c) 7. (a) 8. (b) 9. (b) 10. (c)
11. (a) 12. (c) 13. (c) 14. (d) 15. (a)
Answers with Explanations
1. (b) Consider a solid sphere of mass m and radius a which is 4. (a) Entropy measures the degree of randomness of
rolling with a linear speed v. constituents of a system. At absolute zero of
Consider a point P on the surface. temperature, all molecular motion ceases. Thus, we can
say that entropy of the system is zero.
5. (b) Consider the clamper circuit given in the question. In the
a
negative cycle of the input AC signal, the diode is
v
forward biased and conducts, charging the capacitor to
the peak negative value ofVin. During the positive cycle,
P the diode is reverse biased and thus does not conduct.
Angular momentum of the sphere about this point P can
be written as C
LP = LCM + r × (m v )
where, LCM = angular momentum of the sphere
about CM
r = position vector of the centre of the sphere 6. (c) According to the law of Malus, we know that
with respect to point P I = I 0 cos2 φ
⇒ | LP | = | LCM | + | r × m v | = I ω + mvr where, φ is the angle between the axes of analyser and
where, I = moment of inertia of the solid sphere about polariser.
Component
2
CM = ma 2
5
Considering the point P at the point of constant, we can
write
r =a
2 v  v Polariser Analyser
⇒ LP = ma 2 × + mva Qω =
5 a  a  ∴ Intensity of light obtained through the component,
2 I = I 0 cos2 θ = 0
= mva + mva
5
⇒ cos θ = 0
7
= mva π
5 or θ=
2
2. (c) Materials for which magnetic susceptibility ( χ ) is small where, θ is phase introduced due to the component
and negative are called diamagnetic materials. In such π
materials, the individual magnetic moments of the which is . This value of phase is equivalent to path
2
various atoms tend to cancel out completely. So in the λ
difference of . Thus, the introduced component is a
normal state, the atoms of such substances have no 4
magnetic moment. Examples are bismuth, copper, lead, quarter-wave plate.
sodium, etc. Magnetic susceptibility (χ) for such
7. (a) According to the question,
materials are independent of temperature.
Area, A = 0.01 m2, N = 40
3. (d) As, we know that for a monoatomic gas,
R = 20 Ω, Q = 2 × 10−5 C
3
CV = R Magnetic flux associated with the loop can be written as
2
3 5 φB = NBA
⇒ Cp = CV + R = R+R= R
2 2 Emf induced in the loop is
For a diatomic gas, d
e=− ( φB )
5 dt
CV = R d
2 = − (NBA )
5 7 dt
⇒ Cp = CV + R = R+R= R We know that,
2 2 |e | 1 d
Thus, we can conclude that the materials of the gas can I= = (NBA )
R R dt
be monoatomic or diatomic having no vibrations (rigid
[put the value of e and R ]
diatomic).
Solved Paper 2005 ● 155

dQ d  NBA  2 d
or =   or 0= −
dt dt  R  f f2
d 20
NBA QR or f = = = 10 cm
⇒ Q= or B = 2 2
R AN
f1 = f f2 = f
2 × 10−5 × 20
or B= = 1 × 10−3 T
0.01 × 40

8. (b) Mass defect can be written as


∆m = M H − (M p + Me )
Also, we know that 20 cm
∆E = ∆mc 2
∆E Now, consider the image formation by the combination.
⇒ ∆m = 2
c L1 L2
Energy of hydrogen atom in the ground state is
∆E = − 13.6 eV O I
−13.6 eV
⇒ ∆m =
c2
−13.6 eV 10cm 10 cm
or M H − M p − Me =
c2
The object (O ) is at focus of the lens L1. Thus, rays
13.6 eV
or M H = M p + Me − becomes parallel after going through L1. Now, parallel
c2
beam of light incident on the lens L2, these rays
9. (b) Observing in the frame of rotating platform. Only force converge at the focus of L2.
acting on the object is centrifugal force which is acting in Thus, final image I is formed at the focus of L2, i.e. at 10
the radially outward direction. This is the only force cm from the second lens.
between the surface of the platform and object. Thus,
the object is accelerated in radially outward direct for all 12. (c) Consider a third charge q 3, which is placed at x from q1.
t > 0. Forces acting on q 3 are
ω F1 = force due to q1 d
1 q1q 3 +q1 x q3 +q2
mω2x =
x 4πε 0 x 2
Similarly, F2 = force due to q 2
Platform
1 q 3q 2
=
4πε 0 (d − x )2
0 1 For charge q 3 to be in equilibrium.
10. (c) Given, matrix A =M = 
1 0
F1 = F2
Transpose of the matrix, 1 q1q 3 1 q 3q 2
⇒ =
0 1 4πε 0 x 2 4πε 0 (d − x )2
AT = 
1 0 (d − x )2 q 2
⇒ =
0 1 0 1 x2 q1
AAT =   
 1 0  1 0 d −x q
⇒ =± 2
0 1 x q1
=  =I
 1 0 ⇒
d q
− 1= ± 2
Thus, given matrix is orthogonal. x q1
d q
11. (a) Focal length of the combination of the two lenses can be ⇒ = 1± 2
written as x q1
1 1 1 d d
= + − ⇒ x =
feq f1 f2 f1 f2 q2

1 1 1 d q1
⇒ = + − [Qfeq = ∞]
∞ f f f2 Clearly, x is independent of sign and magnitude of q 3.
156 IIT JAM Physics Solved Papers & Practice Sets

13. (c) Coefficients of Fourier series of function f (x ) is 1 1 1 


= × 0 − ( 2π + 0)
1 π π  2 4 
an = ∫ f (x ) cos nx dx
π −π 1  π 1
= − =−
1 π 
π  2  2
bn = ∫ f (x ) sin nx dx
π −π 1 π
π ∫− π
For the first series, b22 = f2(x ) sin 2 x dx
1 π 1 π
a12 = ∫ f1(x ) cos 2 x dx
π ∫− π
= sin2 x sin 2 x dx
π −π
1 π 1 π 1 − cos 2x
= ∫ cos2 x cos 2 x dx
π ∫− π
π −π = sin 2 x dx
2
1 π  1 + cos 2x 
= ∫   cos 2 x dx 1 1 π 1 π 
π −π  2  =

π 2 ∫ − π
sin 2 x dx − ∫
4 −π
2 sin 2 x cos 2 x dx

1 1 π 1 π 
π  2 ∫−π
= cos 2 x dx + ∫ cos2 2 x dx 1  1  cos 2x 
π 
2 −π  1 π

π
= − × 
π  2
 −
2  −π 4 ∫−π sin 4x dx 
1  1  sin 2 x  1 π  1 + cos 4x   
=  ×  + ∫   dx 
π  2  2  −π 2 −π  2   1 1
π
1  cos 4x  
= − × 0 + ×   
1 1 1 π sin 4x   π  2 4 4  −π 
= (sin 4π + sin 4π ) +  2π + ∫ − 
π  4 4 −π 4   1
= [ 0 + 0] = 0
1 1 1 1  π
= × 0 +  2π − sin 4π  
π  4 4 4 
14. (d) The NaCl crystal structure consists of Na + and Cl− ions
1 1  1 arranged alternatively in a cubic pattern. The Na + ions
= 0 + × ( 2π − 0) =
π  4  2 are situated at the corners as well as at the centre of the
1 π faces of the cube. Thus, we can say that Na + ions lies
π ∫− π
b12 = f1(x ) sin 2x dx
on a face centered lattice with two atoms (Na and Cl)
1 π basis.
π ∫− π
= cos2 x sin 2x dx
15. (a) Distinguishable particles obey Maxwell- Boltzmann
1 π  1 + cos 2 x 
π ∫− π 
=   sin 2 x dx statistics. Thus, option (a) which describes about
2 
indistinguishable particles is not correct.
1 1 π 1 π 
=
 ∫
π  2 −π
sin 2 x dx + ∫
2 −π
sin 2 x cos 2 x dx
 16. a. Given, v = v k$ 0

π Suppose, u = $i u1 + $j u 2 + k$ u 3
1  1  cos 2 x  1 π 
= − × 
π  2  2
 +
 4 ∫− π sin 4x dx   $i $j k$ 
−π   ∂ ∂ ∂ 
π Now, ∇ × u =  
 1 1  cos 4x  
=
1
− (cos 2 π − cos 2 π ) −    ∂x ∂y ∂z 
 
π  4 4  4  −π  u
 1 u 2 u 3 

1  1 1 1   ∂u 3 ∂u 2  $ $  ∂ ∂ 
= − ×0− × ×0  −  i − j  u3 − u1
π  4 4 4   ∂y ∂z  ∂x ∂z 
 ∂u ∂  $
=0 + 2 − u1 k = v (given)
 ∂x ∂y 
For the second series,
= v k$
1 π
a 22 = ∫ f2(x ) cos 2 x dx
π −π Equating the coefficients of $i, $j and k$ on both sides, we
1 π have
= ∫ sin2 x cos 2 x dx ∂ ∂
π −π u3 − u2 = 0 ...(i)
1 π  1 − cos 2x  ∂y ∂z
= ∫   cos 2 x dx ∂ ∂
π −π  2  u3 − u1 = 0 ...(ii)
1 π 1 1 π  ∂x ∂z
π ∫− π 2 ∫− π
= cos 2 x dx − cos2 2 x dx ∂ ∂
2  u2 − u1 = v 0 ...(iii)
π
∂x ∂y
1  1  sin 2 x  1 π  1 + cos 4x  
=    − ∫   dx  One of the many possible values of u from Eqs. (i), (ii)
π 2  2  2 −π  2 
−π  and (iii) can bev 0 x $j.
Solved Paper 2005 ● 157

b. Flux associated with the field v is G =E + F


ψ = ∫ v ⋅ n$ (dx dy ) ...(i)
E
∇φ x $i + y $j + z k$
where, n$ = =
| ∇φ | r
F
with φ =x2 + y2 + z2 −r2 1
Now, from Eq. (i), we get 0
ψ = ∫ v 0 dx dy [Q n$ = 1and v = v 0 k$ ]
18. Given,
=v0 ∫ dx dy a. F (x , y ) = k [(x 2 + y 2 ) $i + 2xy $j ]
r r2 − x2  $i $j k$ 
=v0 ∫−r ∫y = − r2 − x2
dx dy  ∂ ∂ ∂ 
∇ × F = 
 2 ∂x 2 ∂y ∂z
r
=v0 ∫−r dx × 2 r 2 − x 2
x + y 2 xy 0


r
⇒ ψ = 2v 0 ∫−r r 2 − x 2 dx  ∂
= $i  ( 0) −
∂ 
( 2xy ) − $j
 ∂
( 0) −

(x 2 + y 2 )

r
 ∂y ∂z   ∂x ∂z 
∫0
2 2
= 4v 0 r − x dx
 ∂ ∂ 
x
r + k$  ( 2xy ) − (x 2 + y 2 )
x
2 2 2
r −x r  ∂x ∂y 
= 4v 0  + sin−1 
 2 2 r $ $ $
= i [ 0 − 0] − j [ 0 − 0] + k [ 2y − 2y ]
0
r 2  =0
= 4v 0  × sin−1(1) + 0 Since, ∇ × F = 0, force is conservative.
 2 
r2 π b. As, we know that
= 4v 0 × × = πv 0r 2 | F | = | ∇U |
2 2
⇒ ψ = ( πr 2 ) v 0 ∂U
⇒ =x2 + y2
∂x
17. From the given logic circuit, x3
C=A or U = + xy 2 + f1(y ) ...(i)
3
Thus, waveform for C is ∂U
and = 2xy ⇒ U = xy 2 + f1(x ) ...(ii)
D =B ∂y
1
From Eqs. (i) and (ii), we get
x3
0 f1(y ) = 0 and f1(x ) =
3
Thus, waveform for D is x3
1 ⇒ + xy 2
U =
3
0 As, the force is conservative, work done by the force
E = AB ∆W = ∆U = U (1, 1, 0) − U ( 0, 0, 0)
A 1 1  0 
=  + 1 −  + 0
3  3 
0
4
= = 1.335
B
3
1 19. a. Given, velocity of the rod, vr = 0.4 c
E 0 Velocity of the particle,v p = − 0.8 c
F = BA Velocity of the rod with respect to the particle is
vr − v p 0.4 c − ( − 0.8 c )
B 1 vrp = =
vrv p ( 0.4 c ) ( − 0.8 c )
1− 2 1−
0
c c2
1
1.2 c 1.2
A 0
= 2
= c
0.32 c 1.32
1 1+
c2
F 0 = 0.91 c
158 IIT JAM Physics Solved Papers & Practice Sets

b. (a) Time taken for the particle to cross the rod in the b. According to the question, given that
S-frame is B
L 3.6 m = 0.4
t= = A
Speed 0.8 × 3 × 108 E − E − V0
36 ⇒ = 0.4
= = 1.5 × 10−8 s E + E − V0
24 × 108
Relative velocity of the particle in the rest frame of the ⇒ 0.4 [ E + E − V0 ] = E − E − V0
rod is ⇒ 0.6 E = 1.4 E − V0
v p − vr
v pr = E − V0 0.6 3
v pvr ⇒ = =
1− 2 E 1.4 7
c
− 0.8c − ( 0.4c ) ⇒ 72 (E − V0 ) = 32E
=
( −0.8c ) ( 0.4c ) ⇒ 49(E − V0 ) = 9E
1−
c2 ⇒ 40 E = 49 V0
−1.2 c E 49
= ≈ − 0.91c ⇒ = = 1.225
1 + 0.32 V0 40
∴ Time taken for the particle to cross the rod in the rest Reflection coefficient,
frame of the rod is
| B |2
L 3.6 m Γ= = (0.4)2 = 0.16
t= = | A |2
| v pr | 0.91c
Transmission coefficient,
3.6 4
= = × 10−8 T = 1 − Γ = 1 − 0.16 = 0.84
0.91 × 3 × 108 3
= 1.33 × 10−8 s 21. a. According to the question,
∂U = T ∂S − p ∂V ...(i)
(b) Time taken by the rod to cross the particle in the
∂U T ∂S
rest frame of the particle ⇒ = −p
∂V ∂V
L 3.6 m
t= = ≈ 1.33 ×10−8 s  ∂U   ∂S 
| vrp | 0.91c or   =T   −p
 ∂V  T  ∂V  T
20. a. Assuming region (I) for x < 0 and region (II) for x > 0.  ∂p 
=T   −p ...(ii)
For region (I), wave function  ∂T  V
ψ I = A eikx + B e −ikx  ∂S ∂p 
Given, =
 ∂V ∂T 
2mE
where, k =
h Suppose U = U (T , V )
For region (II), we can write wave function as  ∂U   ∂U 
⇒ dU =   dT +   dV
ψ II = C eiγx  ∂T  V  ∂V  V
2m(E − V0 )  ∂U    ∂p  
where, γ = or dU =   dT + T   − p  dV
h  ∂T  V   ∂ T  V 
At the boundary, x = 0 [from Eq. (ii)]
ψ I = ψ II ⇒ A + B = C ...(i)  ∂U    ∂p  
⇒ dU =   dT + T   − p  dV
γ  ∂T  V   ∂ T  
⇒ ψ ′ I = ψ ′ II ⇒ A − B = C ...(ii) V
k Hence proved.
On dividing Eq. (i) by Eq. (ii), we get
 ∂U   ∂p 
A+B k b. As,   =T   −p ...(i)
=  ∂V  T  ∂T  V
A −B γ
B k −γ For van der Waals’ gas, we can write
⇒ =  a
A k +γ  p + 2  (V − b ) = RT
 V 
2mE − 2m (E − V0 )
= ⇒ p=
RT

a
...(ii)
2mE + 2m (E − V0 ) V −b V2
E − (E − V0 )  ∂p  R
= or   =
E + (E − V0 )  ∂T  V V − b
Solved Paper 2005 ● 159

From Eqs. (i) and (ii), we get dz dx


 ∂U  TR a
⇒ ∫ 4z = ∫ x
  = −p= 2 1
 ∂V  T V − b V ⇒ ln z = ln x + C
  ∂p   4
 ∂U 
⇒ dU =   dT + T   − p  dV ⇒ ln ( 2v 2 + 1) = 4 ln x + 4C
 ∂T  V   ∂ T  V 
a ⇒ ln ( 2y 2 / x 2 + 1) = 4 ln x + 4C [ Qv = y / x ]
= CV dT + 2 dV
V  2y 2 + x 2 
⇒ ln   = ln x 4 + 4C
a  x2 
⇒ ∫ dU = CV ∫ dT + ∫ V 2 dV
Putting the initial conditions, we have
1 1
or U 2 − U1 = CV (T2 − T1) + a  −  At x = 1, y = 2
 1
V V 2  8 + 1
⇒ ln   = ln 1 + 4C
For an ideal gas,  1 
 ∂U   ∂p  1 1
As,   =T   −p or ln 9 = 4C or C = ln 9 = ln 3
 ∂V  T  ∂T  V
4 2
For ideal gas, pV = RT  2y 2 + x 2 
⇒ ln   = ln x 4 + 2 ln 3 = ln 9x 4
 ∂p  R  x2 
⇒   =
 ∂T  V V
⇒ 2y 2 + x 2 = 9x 6
 ∂U  TR pV
⇒   = −p= −p=0 23. According to the question,
 ∂V  T V p
nλ 1 = (n + 1) λ 2
 ∂U    ∂p  
∴ dU =   dT + T   − p  dV λ2 4000 Å 4000
 ∂T  V   ∂ T  V  ⇒ n= = = =4
λ 1 − λ 2 (5000 − 4000) Å 1000
 ∂U 
= CV dT +   = CV dT
 ∂V  T Suppose, there are N lines per metre in the grating.
Therefore, we can write
∫ dU = ∫ CV dT
(nλ 1) N = sin θ
or U 2 − U1 = CV (T2 − T1)
sin θ sin 30°
22. Given, differential equation is or N= =
nλ 1 4 × 5000 × 10−10
dy
xy = 3y 2 + x 2 1 107
dx = −10
=
8 × 5000 × 10 40
dy 3y 2 + x 2
⇒ =
dx xy 1000 × 104
= = 25 × 104 lines/m
Thus, the given differential equation is homogeneous. 40
So, putting y = vx = 25 × 102 lines/cm
dy dv d sin θ
⇒ =v + x Resolving power, Pr =
dx dx λ1
dv 3v x + x 2 3v 2 + 1
2 2
2 × 10−2 × sin 30°
⇒ v +x = = =
dx vx 2 v 5800 × 10−10
dv 3v 2 + 1
⇒ x = −v 108 106
dx v = = = 1.72 × 106
5800 58
dv 3v 2 + 1 − v 2 2 v 2 + 1
⇒ x = = λ 1 5800 × 10−10
dx v v Again, =
v dv dx ∆λ λ4 − λ3
⇒ =
2v 2 + 1 x 5800 × 10−10
= = 2900
Integrating on both sides, we get 2 × 10−10
v dv dx λ1
∫ 2v 2 + 1 = ∫ x Thus, Pr ≠
∆λ
Let 2v 2 + 1 = z Hence, the first order spectrum from such a grating cannot
⇒ 4v dv = dz be used to resolve the wavelengths λ 3 and λ 4.
160 IIT JAM Physics Solved Papers & Practice Sets

24. a. According to the question, electric fields are 25. a. Consider the R-L-C circuit shown alongside.
E = E cos (kz − ωt )$i Impedance of the R-L-C circuit is
1 0

and E2 = E0 cos (kz + ωt )$i 150 Ω 10 µF 0.1 H

Net electric field,


E = E1 + E2
= E0 cos (kz − ωt ) $i + E0 cos (kz + ωt ) $i ω
= E [cos (kz − ωt ) + cos (kz + ωt )] $i 2
0
 1  2
Z =  ωL −  +R
= E0 [ 2 cos kz cos ωt ] $i  ωC 
= 2 E cos kz cos ωt $i
0 The combination acts as purely resistive circuit if
⇒ | E | = 2 E0 cos kz cos ωt 1
ω0 L − =0
ω 0C
Magnetic field, 1 1
E 2E0 ⇒ ω 20 = or ω 0 =
B= = cos kz cos ωt $j LC LC
c c
1
or ω0 =
b. Time averaged energy density, 0.1 × 10 × 10−6
T

〈U E

〉= 0
| E |2 dt = 1000 rad/s
T
b. According to the question,
∫0 dt ω ω
T ω = resonance = 0 = 500 rad/s
=
4E02 cos2 kz ∫0 cos2 ωt dt 2 2
T Impedance of the circuit,
∫0 dt Z = R 2 + (XL − XC )2
4E02 T
=
T ∫0 cos2 ωt dt cos2 kz
 1 
2
= (150)2 + ωL − 
 ωC 
4E02 T
=
T
cos2 kz ∫0 cos2 ωt dt
2
 1 
  T = (150)2 + 500 × 0.1 − 
∫ cos ωt dt 
2
 500 × 10 × 10−6 
= 4E02 cos2 kz  0
 T 
 ∫0 dt  = (150)2 + (150)2
⇒ 〈U E 〉 = 2 E02 cos2 kz = 150 2 Ω
Time averaged Poynting vector, Peak value of current,
T V0 300 2
P = 〈E × B 〉 =
∫0 | ( E × B) | dt I0 = =
Z 150 2
= 2A
T
∫0 dt Phase difference ( φ ) between current and the
voltage,
 2  
| ( 2E0 cos kz cos ωt ) $i | ×   E0 cos kz cos ωt  $j 
T
∫0   c   cos φ =
R
=
150
=
1
= T
Z 150 2 2
∫0 dt ⇒ φ = 45°
2E02 Peak voltage across the inductor,
= cos2 kz
c VL = I 0 XL = 2 × 50 = 100 V
Practice Set 1
IITJAM Physics
MM : 100 Time : 3 hrs

n
This test paper has a total of 60 questions carrying 100 marks. The entire question paper is divided into
three Sections A, B and C. All sections are compulsory. Questions in each section are of different types.
n Section A contains Multiple Choice Questions (MCQ). Each MCQ type question has four choices out of
which only one choice is the correct answer. This section has 30 Questions and carry a total of 50 marks.
Q.1 – Q.10 carry 1 mark each and questions (Q.11 – Q.30) carry 2 marks each.
n Section B contains Multiple Select Questions (MSQ). Each MSQ type question is similar to MCQ but with
a difference that there may be one or more than one choice(s) that are correct out of the four given
choices. The candidate gets full credit if he/she selects all the correct choices only and no wrong
choices. This section has 10 questions (Q. 31 – Q. 40) and carry 2 marks each with a total of 20 marks.
n Section C contains Numerical Answer Type (NAT) questions. For these NAT questions, the answer is a
real number which needs to be entered using the virtual numerical keypad on the monitor. No choices
will be shown for these type of questions. This section has 20 questions and carry a total of 30 marks.
Q.41 – Q. 50 carry 1 mark each and questions (Q.51 – Q.60) carry 2 marks each.
n
Do not write more than one answer for the same question. In case you attempt a subjective question
more than once, please cancel the answer(s) you consider wrong. Otherwise, the answer appearing last
only will be evaluated.
n
Clip board, log tables, slide rule, calculator, cellular phone and electronic gadgets in any form are not
allowed.

Section A Multiple Choice Questions (MCQs)


In these type of questions, each question has four choices (a), (b), (c) and (d) out of which only
one option is correct.

1. The Lagrangian of particle of mass m moving in a plane is given by


1
m (x 2 + y 2 ) + a (x y + y x )
L=
2
The canonical momenta are given by
(a) px = m x ; py = m y (b) px = m x + ay ; py = m y + ax
(c) px = m x − ay ; py = m y + ax (d) px = m x − ay ; py = m y − ax
π π
2. The value of A for Ψ = A cos 2 x for − < x < is
2 2
8 3 1 3
(a) (b) (c) (d)
3π 8π 2π 2π
164 IIT JAM Physics Solved Papers & Practice Sets

3. Magnetic field in a region is B = βt z$ , the induced electric field due to this time varying
magnetic field is
β β
(a) rφ (b) r ( − φ) (c) βr ( φ ) (d) βr ( − φ )
2 2

4. A sphere of radius a has a charge density which varies with distance as ρ = A r . The electric
field at a distance r < a varies with r as
(a) E ∝ r −1/ 2 (b) E ∝ r 1/ 2 (c) E ∝ r 3/ 2 (d) E ∝ r −2

4 2 101
5. For the matrix  , the eigen value corresponding to the eigen vector   is
 2 4 101
(a) 5 (b) 6 (c) 7 (d) 4

6. An observer sees two spaceships flying apart with speed 0.99c. The speed of one spaceship
as viewed by the other is
(a) + 0.8995 c (b) + 0.9995 c (c) − 0.99995 c (d) + 0.99995 c

7. If the electrostatic potential were given by φ = φ 0 (x 2 + y 2 + z 2 ), where φ 0 is constant, then


the charge density giving rise to the above potential would be
6 φ0
(a) 0 (b) − 6 φ 0 ε 0 (c) − 2 φ 0 ε 0 (d) −
ε0
8. Octal equivalent of decimal number (926 )10 is
(a) 1640 (b) 1636 (c) 1852 (d) 1836

9. In a photoelectric effect experiment, ultraviolet light of wavelength 340 nm falls on the


photocathode with the work function of 2.4eV. The stopping potential should be close to
(a) 1.6V (b) 1.4V (c) 3.2V (d) 1.2V
27 27
10. TC (critical temperature) for Al13 is T0 . TC for its isotope Al13 will be
(a) equal to T0 (b) less than T0
(c) greater than T0 (d) cannot be predicted

11. An atomic bomb consisting of U235 explodes and releases an energy of 1014 J. It is known that
each U235 which undergoes fission releases 3 neutrons and about 200 MeV of energy. Further
only 20% of the U235 atoms in the bomb undergoes fission.
The total number of neutron released is about,
(a) 4.7 × 1024 (b) 9.7 × 1024 (c) 1.9 × 1025 (d) 3.7 × 1025

12.
R = 100 kΩ

C = 0.01 µF
V1 1 kΩ
+ –
V2
+

The low frequency gain of the low pass filter shown in the given figure is
(a) 10dB (b) 20dB (c) 30dB (d) 40dB
Practice Set 1 ●
165

13. C +

Vi R D Vo


Input signal
Vi
VP

O t

A circuit is given in figure such that input signal is applied (input signal is given in figure
above), the output (Vo ) signal will be
Vo Vo
VP VP
(a) (b)

O t O t
O t O t
(c) (d)
–VP –VP
Vo Vo

14. A system of non-interacting Fermi particles with Fermi energy E F , has the density of states
proportional to E , where E is the energy of a particle. The average energy per particle at
temperature, T = 0 is
1 1 2 3
(a) EF (b) EF (c) EF (d) EF
6 5 5 5

15. For the thermodynamic process as shown in the figure, corresponding T-S diagram is
(p2, V1)
C

p Adiabatic

A B
(p1, V1) (p1, V2)
O V
S S
B C C B
(a) (b)

A A
O O
T T
S S
B C C B
(c) (d)

A A
O T O T
166 IIT JAM Physics Solved Papers & Practice Sets

16. Which of the following circuits represents the Boolean expression;

Y = A + BC + B. A

A A
(a) B Y (b) B Y
C C

A A
(c) B Y (d) B Y
C

R
17. An infinitely long hollow cylinder (conducting) with inner radius and outer radius R carries a
2
uniform current density along its length. The magnitude of the magnetic field, | B| as a function
of the radial distance r from the axis is best represented by

(a) (b)

B  B

r r
O R R O R R
2 2

(c) (d)

B  B

r r
O R R O R R
2 2

x if − π / 2 < x < π / 2
18. If f (x ) = 
 π − x if π / 2 < x < 3 π / 2
3π / 2
Then, a 0 is equal to where, a 0 = ∫ f (x ) dx
−π / 2

(a) 2π (b) − 2π
(c) 0 (d) 1
π
i
19. If Z = 6e 3 , then the value of | e iz | is
(a) e 3 3
(b) e − 3 3
(c) e − 3
(d) e −3/ 3


dx
20. ∫ 1+ x 2 is equal to
0
π π π π
(a) (b) (c) (d)
4 3 6 2
3z + 4
21. ∫ z( 2z + 1) dz, where c is the circle |z| = 1, will be equal to
c

(a) 3πi (b) 4 (c) − 4 (d) 2πi


Practice Set 1 ●
167

∂u ∂u
22. The solution of =4 is
∂t ∂x x 
k + t 
4 
(a) Ae kx + t (b) Ae k( x + t ) (c) Ae x + t (d) Ae

23. A heavy chain of uniform mass per unit length is hung from one end. If the mass of the chain
is m and length l, the tension at each point of the chain is
mgx mg mg
(a) mg at each point x (b) (c) ( L − x) (d)
l l l
where, x is the distance from the fixed end to any point of the chain.

24. A particle of mass m released from a height h falls under a gravity. Assuming that the
resistance offered by the atmosphere is mkv 2 , where k is a constant and v is the speed of the
particle. The terminal speed of the particle is
g
(a) (b) g / k (c) gk (d) k / g
k

25. A circular ring of radius a rolls without slipping in a straight line with constant velocity v . A small
pebble stuck on the ring touches the road at t = 0 . The position vector of the pebble at any
time is
(a) x = a (ωt − sin ωt ), y = a (1 − cos ωt )
(b) x = a (ωt + sin ωt ), y = − a cos ωt
(c) x = − a sin ωt , y = − a cos ωt
(d) x = − a cos ωt , y = − a (ωt − sin ωt )
[where, v = aω]

26. A block of mass m slides on a frictionless table. It moves inside a ring of radius a. Let at time
t = 0, the speed of the block be v 0 . Given that the coefficient of friction between the block and
the ring is µ, the velocity at any time t is
µv 0 t  µv t  v0
(a) v 0 1− (b) v 0 (1 − µv 0 ) (c) v 0 1− 0  (d)
l  l   µ v0 t 
1+ 
 l 
27.

Consider a circular wire of radius R moving towards a region of constant and uniform magnetic
field B perpendicular to the plane of the circular wire. It is pushed into the region of the
magnetic field with a constant speed at t = 0. The induced emf at time t when the edge of the
field region subtends an angle θ at the centre is
θ
(a) 2BRv sinθ (b) 2 BRv sin
2
(c) BRv sinθ (d) 2Brv cos θ
168 IIT JAM Physics Solved Papers & Practice Sets

28. A metal rod of length l, rotates about an end with a uniform angular velocity ω . A magnetic
field B exists perpendicular to the plane of rotation. The induced emf between the two ends of
the rod are

Bωl 2 Bωl 2 Bωl 2


(a) Bωl 2 (b) (c) (d)
2 3 4

29. Two ends of a rod are kept at 127°C and 227°C. When 2000 cal of heat flows in this rod, then
the change in entropy is
(a) 1.0 cal/K (b) 2.0 cal/K (c) 6.9 cal/K (d) 0.7 cal/K
3750
30. The sublimation curve of solid ammonia is given by ln p = 23 − and the vaporisation
T
3050
curve of the liquid ammonia is given by ln p = 19.5 − , where p is in mm of Hg and T is in
T
K. The temperature of the triple point of ammonia is
(a) 3750 K (b) 3050 K (c) 700 K (d) 200 K

Section B Multiple Select Questions (MSQs)


In these type of questions, each question has four choices (a), (b), (c) and (d) out of which only
one or more than one is/are correct options.

31. A monkey of mass m kg slides down a light rope attached to a fixed spring balance with an
acceleration a. The reading of the spring is w kg. (g = acceleration due to gravity), then
wg  g
(a) m = (b) m = w 1 +  (c) tension in rope is wg (d) None of these
g−a  a

32. Three identical cars, A, B, C are moving at the same speed on the three bridges. The car A
goes on a plane bridge, B on a bridge convex upward C goes on a bridge concave upward.
Let FA , FB and FC be the normal forces exerted by the cars on the bridge when they are at
the middle of bridge
(a) FA > FB > FC (b) FB is least (c) FC is max (d) FA = FB = FC

33. 0.71Å wavelength X-ray is diffracted with a simple cubic crystal having edge a = 2.814 Å for
the plane (1, 1, 0), then
(a) d 110 = 1.99 Å (b) d 110 = 1.59 Å
(c) for 2nd order θ = sin−1( 0.357 ) (d) for 2nd order θ = sin−1(0.5)

34. If the momentum of an electron is 10 times its rest mass times the speed light, then
(a) its speed is 2.983 × 108 m/s (b) its speed is 108 m/s
(c) its energy is 9.22 × 10−13J (d) its energy is 10−13J
Practice Set 1 ●
169

35. Consider the longitudinal wave and a transverse wave propagating a cubic crystal in the [100]
direction if C11 and C 44 are the stiffness constants, then
c 11 c 44
(a) velocity of longitudinal wave ve = (b) velocity of transverse wave vT =
ρ ρ
c 44 c 11
(c) velocity of longitudinal wave vT = (d) velocity of transverse wave vT =
ρ ρ
[where, ρ is the density of cubic crystal]

36. A block of mass m moving on a smooth horizontal plane with a velocity v0 k


m M
v 0 collides with a stationary block of mass M, at the back of which a
spring of spring constant k is attached as shown in figure below. Select the correct option ( s).
m
(a) Velocity of centre of mass v0
m+M
1  mM  2
(b) Initial kinetic energy of the system in centre of mass frame is   v0
4  m + M
mM 1
(c) Maximum compression in the spring is v 0
( M + m) k
(d) When the spring is in state of maximum compression the kinetic energy in the centre of mass
frame is zero

37. A uniform heavy disc is rotating at constant angular velocity ω about a vertical axis passing
through is centre and perpendicular to the plane of disc. Let L be its angular momentum. A
particle is dropped vertically on the disc and sticks to it. Then,
(a) ω will change (b) L will change (c) ω will not change (d) L will not change

38. A net force Fx (t ) = A + Bt 2 in the positive x-direction is applied to a boy of mass m. The force
starts at t = 0 and continues until time t.
A B 3
(a) Impulse of the force is 2Bt (b) His speed at time t is t+ t
m 3m
B 3 2B
(c) Impulse of the force is At + t (d) Speed at time t is t
3 m

39. The relaxation time of a conduction electron in copper is 3.5 × 10 −14 s. An electric field of
2.5V/cm is applied along the negative X-axis, then
(a) the increase in x-component of velocity between two collisions is 2 m/s
(b) the increase in x-component of velocity between two collisions is 1.54 m/s
(c) the average energy increased between two collisions is 2.18 × 10−30J
(d) None of the above

40. The correct matching pair of potential with the potential shape is
V
V
b r
(a) exponential potential well (b) square well potential –v0/e

–v0

V V
b r b
r
(c) yukawa well potential (d) square well potential
–v0/e
–v0
Section C Numerical Answer Type Questions (NATQs)
This section contains Numerical Answer Type (NAT) questions. For these NATQs, the answer is a
real number.

41. A uniform solid cylinder of mass 2 kg and radius 0.20 m rolls without slipping on a flat surface.
If the total energy of the cylinder be 12 J. Its rotational kinetic energy will be ..............(J).

42. Two particles execute SHM of the same amplitude and frequency along the same straight line.
Then pass one another travelling in opposite directions, whenever their displacement is half
their amplitude. The phase difference between the two is .............

43. A long straight vertical wire carries a current of 10 A directed upwards. If the horizontal
component of earth’s magnetic field at the plane is 2 × 10 −5 T, then the distance of the point
from the wire at which the net B is zero is .............. m.

44. An X-ray of energy 50 KeV strikes an electron initially at rest. The change in wavelength of the
X-ray scattered at an angle 90° is approximately ............ × 10 −12 m. (Given, h = 6.63 × 10 −34
Js, me = 9.11 × 10 −31 kg)

45. The velocity of longitudinal waves in quartz crystal is 5.46 × 10 3 ms −1. If the thickness of the
quartz crystal is 1 mm, then the frequency of the ultrasonic wave generated by it is
............MHz.

46. The amplitude of the vibration of a particle due to superposition of two SHMs.
 π
y 1 = sin ωt +  and y 2 = sin ωt, is ............
 3

47. In the ideal double slit experiment, when a glass plate ( µ = 1.5) of thickness t is introduced in
the path of one of the interfering beams (wavelength, λ), the intensity at the position where,
the central maximum occurred previously remains interchanged. The minimum thickness of the
glass plate is ........... (if λ = 600 nm)

48. A plane electromagnetic wave E = 100 cos (6 × 10 8 t + 4k ) V/m propagates in a medium of


dielectric constant..............

49. In a common base configuration, Ie = 1mA, α = 095


. , the value of base current is ............mA.

50. The ionisation energy of hydrogen atom is 13.6 eV. The ionisation energy of helium atom
would be .............. eV.

51. Bullets are fired at regular intervals of 10 seconds from an armoured car A moving with 36 km/h.
At interval will an officer seated in a car B hear the report when the car B is stationary and car A
is moving towards car B be ........... second. (velocity of sound = 340 ms −1, velocity of wind = 20
ms −1 in the direction AB)

52 Two closed organ pipes when sounded together produced 12 beats in 4 s, at a temperature of
27°C. The temperature at which number of beats produced are sixteen during the same period
.........°C.
Practice Set 1 ●
171

53. The thickness of a quarter wave plate from a doubly refracting crystal is 6.7 × 10 −5 cm for a
°
light of wavelength 4800 A. The corresponding thickness of half-wave plate for a light of
°
wavelength 6000 A is .......... cm.

54. In a diffraction grating experiment, the grating has 105 rulings. Then for λ = 5000 A° , region of
the spectrum and in the second order, the grating can resolve two lines with a wavelength
°
difference of ...........A

55. If the scattering intensity of a liquid is 6 units at a wavelength of 500 nm, then the scattering
intensity at a wavelength of 400 nm will be approximately ........... units.

56. In series L-R circuit, X L = 3 R. Now a capacitor is added in series. The ratio of new to old
power factor is ...........

57. The self-inductance of a coil which produces 5 V, when current in it changes from 3 A to 2 A
in one millisecond is ........ H.

58. A proton of energy 1 MeV moves in a uniform magnetic field along a circular path. The energy
for an α-particle to circulate along the same orbit in the same magnetic field, is ......... MeV.

59. Two condensers of capacitances 2 µF and 3 µF are charged to potentials 100 V and 200 V
respectively. They are then joined in parallel in such a way that plates which had positive
charges are connected together. The energy dissipated into heat in the process is ......... mJ.
1
60. A reversible engine converts th of heat into work. When the temperature of the sink is
6
reduced by 62 °C. The efficiency of the engine is doubled. The temperatures of source is
.......... K.

Answers
1. (b) 2. (a) 3. (b) 4. (c) 5. (b) 6. (c) 7. (b) 8. (b) 9. (d) 10. (b)
11. (b) 12. (d) 13. (c) 14. (d) 15. (a) 16. (c) 17. (d) 18. (c) 19. (b) 20. (d)
21. (a) 22. (d) 23. (c) 24. (b) 25. (a) 26. (d) 27. (b) 28. (b) 29. (b) 30. (d)
172 IIT JAM Physics Solved Papers & Practice Sets

Answers with Explanations


Section A Multiple Choice Questions 4. (c) The electric field at r can be easily found out using Gauss
law.
1. (b) As from definition, A
px =
∂L
=
∂ 1 2 2 
 m(x + y ) + a(x y + y x )
∴ ∫ E ⋅ dS = ∫ ε 0 r dr
∂x ∂x 2 
1 2Ar 3/ 2 2
= m( 2 x ) + a (y ) = mx + ay ∴ EA = ⇒ E= ⋅ r 3/ 2
2 3ε 0 3ε 0
∂L ∂ 1 2 2  or E ∝ r 3/ 2
py = =  m(x + y ) + a(x y + y x )
∂y ∂y  2 
4 2
= m y + ax 5. (b) Matrix A =  
2 4
∴ p x = m x + ay ; p y = m y + ax .
Let the given vector corresponding to eigen value λ is
π /2
∗ 101
2. (a) As we know, ∫Ψ Ψ dx = 1 X1 =  
101
− π /2
π /2 ⇒ AX1 = λX1
∫A
2
∴ cos4 x dx = 1
4 2 101 101  404 + 202  101 λ 
− π /2 ⇒     = λ  ⇒ = 
π /2 2 4 101 101 202 + 404 101 λ 
A2 ∫ cos
4
or x dx = 1 ⇒ 606 = 101 λ
− π /2 λ=6
π /2
6. (c) The speed of one spaceship as viewed by the other
2A 2 ∫ cos
4
or x dx = 1
0
would be equal to the speed of one spaceship viewed
π /2 2 from an inertial frame moving with the other spaceship.
 1 + cos 2x 
or 2A 2 ∫ 
 2
 dx = 1

u x = − 0.99 c and v = 0.99 c
u −v
0
u′x = x
π /2 2 uv
 1 cos 2x  1− x2
or 2A 2 ∫  2
+
2 
dx = 1 c
0 ( − 0.99 − 0.99 ) c − 1.98 c
π /2 u′x = =
1 + cos2 2x + 2 cos 2x ( − 0.99) (0.99 ) 1 + ( 0.99 )2

2
or 2A 2 dx = 1 1− ×c
0
2 c2
− 1.98
π /2
1 1  cos 4x + 1 v ′x =
1.9801
or 2A 2 ∫ + 
2 2 2
 + cos 2x dx = 1
 v ′x = − 0.99995 c
0
π /2
1 1 1  7. (b) E = − ∇φ (by definition)
or 2A 2 ∫  + + cos 4x + cos 2x  dx = 1
2 4 4  ⇒ E = − ∇ [ φ 0(x 2 + y 2 + z 2 )]
0
On solving, we get or E = − φ 0∇ [x 2 + y 2 + z 2 ]
16 8  ∂ 2$ ∂ 2$ ∂ 2$ 
2A 2 = ⇒A = or E = − φ0 x i+ y j+ z k
3π 3π  ∂x ∂x ∂z 
3. (b) The induced electric field lines are concentric circles, or $ $
E = − φ [ 2x i + 2y j + 2z k] $
0
∴ Along a circle of radius r , we get
or E = − 2φ 0 [x$i + y$j + zk$ ]
dφB
E. 2πr = | emf| = The volume charge density
dt
dB dB = ε 0 ∇E = − 2ε 0 φ 0 ∇ ⋅ (x$i + y$j + zk$ )
⇒ E. 2πr = πr 2 . = π(r 2 ) .
dt dt  ∂ ∂ ∂ 
= − 2ε 0 φ 0  x $i ⋅ $i + y $j ⋅ $j + z k$ ⋅ k$ 
πr 2 dB πr 2  ∂x ∂y ∂z 
or E= . = ⋅β
2πr dt 2 πr = − 2ε 0 φ 0 (1 + 1 + 1)
β = − 2ε 0 φ 0 × 3
or, E = ⋅ r ( −ϕ$ )
2 = − 6ε 0 φ 0
Practice Set 1 ●
173

8. (b) Conversion of decimal to octal 12. (d) For low pass filter,
8 926 V2  R 1 100. kΩ 1
  = = ⋅
 V1  R1. RC3 + 1 1 kΩ 100 × 10 × 10−8 + 1
3
8 115 → 6
1 ~
8 14 → 3 = 100 ⋅ −3 − 100
10 + 1
1 → 6
V2 100
∴ Low frequency gain = 20 log ⇒ 20 log
(926)10 = (1636)8 V1 1
∴ 1636 20 log |100|
9. (d) According to photoelectric effect formulation, stopping 20 × 2 = 40 dB
potential (V0 ) is related with work function ( φ ) as
13. (c) Clamping of circuit/input whenVi is positive, then diode
hc
eV0 = −φ conducts and capacitor charges quickly to peak valueVp
λ
where, λ is incident radiation wavelength. of Vi . The output voltage, V0 is V0 = Vi − Vp . When Vi drops
hc φ to zero, V0 = − Vp . In this way, the output waveform is
V0 = −
eλ e formed.
 6.6 × 10−34 × 3 × 108 2.4 × e  U
V0 =  −  14. (d) As we know, E =
. × 10−19 340 × 10−9
 16 e  N
V0 = ( 0.036 × 102 − 2.4)V where, U is energy, N is particles.
3/ 2
⇒ V0 = 3.6 − 2.4 = 1.2V π  8ma 2 
where, =   . (EF )5/ 2
5  n2 
10. (b) The relation between isotope mass and critical
3/ 2
temperature TC is π  8m 
and N =   V (EF )3/ 2
MTC = constant 3  n2 
M1TC1 = M 2TC 2 3/ 2
π  8ma 2 
M1 TC 2   EF 5/ 2
∴ = 5  n2  3 a3
M 2 TC1 ∴ E= 3/ 2
⇒ E= × . EF 5/ 2 − 3/ 2
π  8m  5 V
M1 = 27 , M 2 = 28   V EF 3/ 2
3  n2 
TC1 = T0 , TC 2 = TC iso
As, a 3 = V
27 TC iso 3
∴ = E = × EF
28 To 5
 27
∴ TC iso =  T0 15. (a)
 28 S
⇒ TC iso < T0 B C
14
11. (b) Released energy = 10 J
Per fission energy = 200 MeV
A
= 200 × 1.6 × 10−13J O
T
= 3.2 × 10−11J
During the adiabatic process BC, dQ = 0
Total energy ∆Q
∴ Number of atoms = Change in entropy, ∆S BC = = 0 ⇒ S BC = constant
Fission energy T
1014 During isobaric process AB
= × 10+11
3.2 ∆Q = C p ∆T
1 ∆Q CP ∆T
= × 1025 ∆S BA = =
3.2 T T
= 0.3125 × 1025 atom ∆Q BA = ∫ dST = area of S -T curve.
= 3.125 × 1024 atom
Similarly during the process AC area
∴Number of neutron = 3 × atom (uranium) ∆Q AC = area of S-T curve.
= 3 × 3.125 × 1024 During isochoric process ( AC ) all heat supplied is utilised
= 9.625 × 1024 in increasing internal energy (or randomness). Hence,
change in entropy (or area of S-T curve) should be more
= 9.7 × 1024
during AC.
174 IIT JAM Physics Solved Papers & Practice Sets

16. (c) Using, de-Morgan’s law, Let us first find the limit
lim f ( z )
Y = A + BC + BA R→ ∞
z
lim =0
Y = A + BC . B A R→ ∞ (1 + z 2 )
Y = ( A + BC ) ⋅ (BA ) Therefore, by theorem 2
⇒ Y = AB A + BCB A lim ∫ f ( z ) dz = 0
R→ ∞ r
Y = AB + BB CA ∞ dx
Y = AB + 0 Hence, ∫c f ( z ) dz = ∫− ∞ 1 + x 2
Y = AB
A
B
AB So, let us find∫c f ( z )dz
B By residue theorem ∫ f ( z ) dz = 2πi Res ( z + i ) since, the
17. (d) Magnetic field inside the hollow conducting cylinder is pole z = − i lines outside the contour C.
zero, i.e. Res ( z = i ) = lim ( z − i ) f ( z )
R z →i
B = 0 for r ≤ . ( z − i )  1
2 = lim  =
z → i 1+ z 2
In the solid part of cylinder,   2i
1
Hence, ∫ F ( z )dz = 2πi =π
R/2
R 2i
∞ dx ∞ dx π
So, ∫ = π. So, ∫ =
− ∞ 1+ x 2 0 1+ x 2 2

 µ  r2  3z + 4 4 5
B =  0  .I  2  21. (a) ∫ dz = ∫ dz − ∫ dz
 2 π  R  c
z ( 2z + 1) c
z c
2 z +1

 1 (by Cauchy integral formula)


Outside the cylinder, B ∝ 2 5
 r  = 4 × 2 πi − × 2 πi
π /2 3π / 2
2
1 1 = 8 πi − 10 πi = 3πi
18. (c) a0 =
π ∫ f (x ) dx + π ∫ f (x ) dx
−π / 2 π /2 22. (d) We have,
π /2 3π / 2
1 1 ∂u ∂u
=4
a0 =
π ∫ x dx + π ∫ ( π − x ) dx ∂t ∂x
… (i)
−π / 2 π /2
1 1 Let u(x , t ) = X (x ) T (t ) be solution of Eq. (i), then
⇒ a0 = ( 0) + ( 0) XT ′ = 4 XT
π π
⇒ a0 = 0 + 0 ⇒ a0 = 0 T′ X′
⇒ =4 =k (say)
 π π T X
i 6  cos + i sin 
19. (b) We have, eiz = ei 6e πi / 3 e  3 3 X′
Then, 4 =k
1 3 X
i6 +i 
3 2  kx
=e ⇒ log X = + log C
4
−3 3
= |e | = e
iz kx
⇒ X = Ce 4 …(ii)
1
20. (d) Consider, f ( z ) = T′
1+ z 2 and =k
T
f ( z ) has poles at, z = ± i
⇒ logT = kt + log C′
⇒ T = π C′ ekt …(iii)
kx
∴ u(x , t ) = XT = C′ C 4 . ekt
kx
Integrating f ( z ) over a closed contour C as shown.
R = Ae 4 . ekt
1
∫c f ( z ) dz = ∫r f ( z ) dz = ∫ (1 + x 2 ) dx = Ae
x 
k + t 
4 
−R
Practice Set 1 ●
175

23. (c) T(x ) The particular form of r′ is dictated by the initial condition
that at t = 0 the pebble touches the ground. Since, the
origin of frame S′ moves with a uniform velocity.
R=a j +v + i
T(x + dx) r = ( −a sin ωt + vt ) i − a cos ωt j + aj
δw x = a(ωt − sin ωt )
Consider a small elementary length dl between the y = a (1 − cos ωt )
points x + dx and x. We identify the forces acting on which is the parametric equation of a cycloid.
the elementary mass contained in the elementary
length. We draw the free body diagram for the 26. (d)
elementary mass.
So, T (x ) = T (x + ∂x ) + ∂w
θ
Mgdx  dT 
T (x ) = + T (x ) + ∂x 
L  dx 
dT Mg The forces acting on the block are the normal force and
or =− the frictional force. Let us draw free body diagram.
dx L
T =−
Mgx
+A mV 2
N= N
L l θ
where, A is a constant of integration dv
f = −m
At x = L ,T = 0 dt
Mg and f = µN
Thus, T = (L − x ) f
L dv µmv 2
So, m =−
24. (b) The equation of motion of the particle is dt l
dv Integrating from t = 0 to t = t and using the fact that at
mg − kmv 2 = m
dt t = 0 ,v = v 0
The terminal speed is the speed attained by the particle 1 1 µt v0
after a long time. After a long time, the speed is − = or v =
v v0 l  µv 0t 
effectively constant. It is easy to see that we need not 1 + 
 l 
solve the differential equation to first findv as a function
of time and then take the limit as t tends to infinity. After 27. (b)
a sufficiently long time, the speed is constant, therefore
the time rate of change of v is zero.
θ
dv
or g − kv 2 = =0
dt
g
So, the terminal speed,v =
k
25. (a) Let at any instant of time t, the edge of the field region
S′ makes an angle of θ at the centre of the circular wire. So,
S area of the circular wire which has gone inside the field
region is
1 1
= R 2θ − R 2 sin θ
2 2
dφ d 1 2 1 2 
emf = − = −B  R θ − R sin θ
dt dt  2 2 
Let us take two frames, S the lab frame and another
 R 2 dθ R 2 dθ 
frame S ′ that moves with velocity v, the same velocity = −B  − cos θ
that the circular ring rolls on the ground. Also, let the  2 dt 2 dt 
centre of the ring be taken as the origin of the moving d  θ d  θ
Also, v = R cos  or = R cos 
frame S′. dt  2 dt  2
Let r and r′, be the position vectors of the pebble with
R2 2v
respect to frames S and S′ respectively. Also, let the ⇒ |e | = B (1 − cos θ )
2 θ
position vector of the origin of the frame S′ be R with R sin
respect to frame S. 2
BRv 2 θ θ
Therefore, r = r′ + R = 2 sin = 2BRv sin
θ 2 2
and r′ = − (a cos ω t j + a sin ωt i) sin
2
176 IIT JAM Physics Solved Papers & Practice Sets

28. (b) When the metal rod moves with one end fixed, each FB FC
valence electron inside the rod moves along the m m
direction of velocity of the rod at that point. The B C
electron is acted by the Lorentz force.
F = evB, where v is the velocity of the electron. Bridge mg mg
As electrons pile up in one end, the other end will get
For car A, FA = mg
an excess of positive charge. Hence, an electrostatic
field will be built up. This electrostatic field will For car B, − FB + mg = centripetal force
ultimately cancel the magnetic force in equilibrium. ⇒ FB = mg − centripetal force
Hence, there exists a potential difference between the For car C, FC − mg = centripetal force
two ends. FC = mg + centripetal force
This is the emf, e = vBl as e = ∫ E dl and E = vB. Hence, FC is maximum, and FB is least.

Consider an element of length dr between r + dr and r a 2.814Å


33. (a, c) dhkl = = = 1.99Å
2 2 2
points of the rod. This points moves with a velocity h +k +l 1 + 12 + 02
2

v = ωr perpendicular to both the magnetic field and the By Bragg’s law,


length of the rod.
2 d sin θ = nλ
Hence, the emf between the ends of the elementary
2 d110 sin θ = 2λ
length dr is
For 2nd order n = 2
de = ωrB dr 2 × 0.71
The emfs of all such elementary length will add up and ⇒ sin θ = = 0.357
2 × 1.99
the emf between the ends of the rod is
⇒ θ = sin−1 ( 0.357)
ωBl 2
l
je = ∫ ωrB dr =
0
2 34. (a, c) Given, p = 10 m0c
m0v
29. (b) Change in entropy Using p=
dQ 2000 2000 1−v 2 /c 2
∆S = = = cal/K
dT 500 − 400 100 So, 10 m0 c =
m0v
∆S = 20 cal/K 1−v 2 / c 2

30. (d) At triple point temperatureTt can be found using any of 100
Solving v = c = 2.983 × 108 m/s
100
the two relation and must be unique
3050 3750 and E = m02 c 4 + p 2c 2
19.5 − = 23 −
Tt Tc Putting, m0 = 9.1 × 10−31 kg,
3050 3750 p = 10 m0 × c = 10 × 9.1 × 10−31 × 3 × 108
⇒ − + = 23 − 19.5
Tc Tc = 2.73 × 10−21 kg-m/s
700 we get, E = 9.22 × 10−13J
or = 3.5
Tc
35. (a, b) Velocity of longitudinal wave
700
or Tc = = 200K ⇒ Tc = 200K c11
3.5 ve =
ρ
Velocity of transverse wave
c 44
Section B Multiple Select Questions vT =
ρ
31. (a, c) Tension, T = wg and mg − wg = ma m ×v0 + M × 0
wg 36. (a,c,d) Velocity of centre of mass, v CM =
⇒ m= m+M
g −a mv 0
⇒ v CM =
m+M
32. (b, c) The situation is shown in the figure below :
For maximum compression, velocity of each mass
FA
become same, let it is v
1 1 m
m ⇒ mv 02 = (m + M )v 2 ⇒ v = v0
A 2 2 m+M
Bridge 1 2 1 1 m2 1 mM
mg ⇒ kx = mv 02 − v 02 − v 02
2 2 2 (m + M ) 2 (m + M )
Practice Set 1 ●
177

mM 1 42. Range, 90° to 150°


On solving,x = v 0
M + mk k a π
Given, a sin ωt = , ∴ωt =
1 2 1 2 6
Initial KE = mv cm + Mv 2cm −a 5π
2 2 and a sin (ωt + φ ) = ∴ ωt + φ =
1 m2 1 m2 2 6
= × v 2 [m + M ] =
2 0
v 02 5π π 2π
2 (m + M ) 2 (M + m ) Now, φ = − =
6 6 3
37. (a, d) Since, I increases hence, ω decreases by conservation of
43. Range 0.1 to 0.1
angular momentum.
µ 0 2I
Q External torque is zero. L is constant. B= ⋅
4π r
⇒ L will be not change. As, B = 2 × 10−5 J
38. (b, c) Given, F = A + Bt 2 10 2 × 10−6
or 2 × 10−5 = (10−7 ) × 2 × ⇒ r =
t t
Bt 3 r 2 × 10−5
Impulse, I = ∫ F dt = ∫ A + Bt
2
= At + or r = 0.1m
0 0
3
F A + Bt 2 44. Range, 2.2 to 2.4
Acceleration, a = = h
m m As, ∆λ = (1 − cos φ ) or φ = 90°
dv A + Bt 2 mc
⇒ = 6.6 × 10−34
dt m ∴ ∆λ = × (1 − 0)
v t A + Bt 2 9.1 × 10−31 × 3 × 108
⇒ ∫0 dv =∫
0
dt
6.6
m ⇒ ∆λ = × 10− 3 − 8
A B 3 9.1 × 3
⇒ v = t+ t
m 3m 6.6
⇒ ∆λ = × 10−11 m
e τEx e τEx 27.3
39. (b, c) We have , vn = − =
6.6
m m ⇒ ∆λ = × 10−12 m
where, |v x | is change in x-component of velocity between two 2.73
collisions. ⇒ ∆λ ~
− 2.4 × 10−12 m
Here, σ = 3.5 × 10−14 s
45. Range 2.73-2.73
Ex = 2.5V/cm = 2.5 × 102 V/m = 9.1× 10−31kg v 5.46 × 103
Putting these values, we get Frequency ( ν) = =
2t 2 × 1 × 10−3
1.6 × 10−193.5 × 10−14 × 2.5 × 102 = 2.73 × 106 Hz = 2.73 MHz
|vn| =
9.1× 10−31
Average energy increased 46. Range 1.73 to 1.74
π
1 1 Given, a1 = 1, a 2 = 1 and φ =
= m |vn|2 = × 9.1 × 10−31 × (1.54)2 = 2.18 × 10−30 J 3
2 2
Resultant amplitude,
40. (c) Yukawa well potential increases with distance and is
A = a12 + a 22 + 2a1a 2 cos φ
minimum ( ≈ −∞ ) at r = 0.
1
= 1 + 1 + 2 ⋅ 1⋅ 1⋅ = 3
2
Section C Numerical Answer Type Questions A = 1.732

41. Range 4 to 4 47. Range 1.2 × 10−6 m to 1.2 × 10−6 m


Total KE = KE of translation + KE of rotation Path difference due to slab should be an integral
1 1 multiple of λ.
⇒ E = mv 2 + Iω 2 i.e. ∆x = (µ − 1)t = nλ
2 2
1 11  nλ
⇒ 12 = × 2 × v 2 +  mr 2 × ω 2 ∴ t=
2 2 2  µ −1
1 3 For minimum value of t, n = 1
⇒ 12 = (rω )2 + r 2ω 2 = r 2ω 2 ⇒ ω 2 = 200
2 2 1× λ
t= = 2λ = 2 × 600 nm
1 1 1 1.5 − 1
∴ Rotational KE = Iω 2= × × 2 × ( 0.2)2 × 200 = 4 J
2 2 2 t = 1200 nm = 1.2 micron
178 IIT JAM Physics Solved Papers & Practice Sets

48. Range 4 v v
Similarly, n2 = =
8
Given, E = 100 cos ( 6 × 10 t + 4x ) λ 2 2l2

where, ω = 6 × 108 and K = 4 v  1 1


∴ n1 − n 2 =  − 
2  l1 l2 
ω 6 × 108
As, v = = = 1.5 × 108
k 4 v 27  1 1 
At 27°C, 3=  −  ...(i)
c 3 × 108 2  l1 l2 
But, v = ⇒ εr = =2
ε2 1.5 × 108 vt  1 1
In second case, 4=  −  ...(ii)
∴ εr = 4 2  l1 l2 
4 v
49. 0.05 On dividing Eq. (ii) by Eq. (i), we have =− t
Ic Ie − Ib 3 v 27
α= =
Ie Ie vt 273 + t
⇒ =
I v 27 273 + 27
⇒ α = 1− b
Ie 2
 4 273 + t
⇒   =
⇒ α = 1− b
I  3 300
Ie
⇒ t = 260.3 °C
Ib
⇒ = 1 − 0.95 53. Range 0.000167-0.000168
Ie
As, ∆ = (µ − 1)d
⇒ Ib = Ie (1 − 0.95) 4800
For quarter-wave plate, × 10−10
⇒ Ib = 1m A × 0.05 = 0.05 m A 4
50. 54.4 eV = (µ − 1) × 6.7 × 10−5 …(i)
2
13.6 × Z 6000
Ionisation energy = eV For half-wave plate, × 10−10 = (µ − 1)x …(ii)
n2 2
(Q for hydrogen, Z = 1and n = 1) By Eqs. (i) and (ii), we get
For helium, Z = 2, n = 1 x = 6.7 × 10−5 × 2.5
13.6 × ( 2)2 x = 16.75 × 10−5 cm
∴ Ionisation energy = = 54.4 eV
(1)2 ⇒ x = 0.0001675 cm

51. Range 9-10 54. Range 0.02 to 0.03


°
v + a −v0 Given, n = 2, N = 105 and λ = 5000 A
Apparent frequency = n ×
v + a − vs λ
Condition for just resolution is = nN
Let T and T ′ be the true and apparent periods, then dλ
1 1 5000
T = and T ′ = = 2 × 105
n n′ dλ
5000 °
1 1 v + a −v0 ⇒ dλ = = 0.025 A
∴ = × 2 × 105
T ′ T v + a − vs
v + a − vs 55. Range 14-15
∴ T′ =T × 4
v + a −v0 1 I λ 
Since, I ∝ ⇒ 2 =  1
340 + 20 − 10 λ4 I1  λ 2 
∴ T ′ = 10 ×
340 + 20 2 4
 500   5
350 350 ⇒ I2 =   ×6=  ×6
= 10 × = = 9.7 s  400  4
360 36
= (1.25)4 × 6
52. Range 260-270 = 2.44 × 6 = 14.64 units
12
In 1st case, n1 − n 2 = =3 56. Range 1.4 to 1.5
4
Let l1 and l2 be the length of closed organ pipes, then In series L- R circuit,
λ R R R
l1 = 1, λ 1 = 2l1 Power factor, cos θ1 = = =
2 Z 2 2
(R + XL ) R + 9R 2
2

v v 1
As, v = n1λ 1 we get, n1 = = =
R
=
λ 1 2l1
R 10 10
Practice Set 1 ●
179

New power factor, 59. Range 6 to 6


R R 1
cos θ 2 = = Energy lost = Cp (V1 − V2 )2
Z R + (XL − XC )2
2
2
R R 1 C C (V − V2 )2
= = = = 1 2 1
2
R + ( 3R − R ) 2 R 5 5 2(C1 + C2 )

cos θ 2 1 10 2 × 10−6 × 3 × 10−6 × ( 200 − 100)2


∴ = × = 2 =
cos θ1 5 1 2( 2 × 10−6 + 3 × 10−6 )
3 × 10−6 × 10000
57. Range 0.004 to 0.006 =
5
L = ?, e = 5 V,
= 6 × 10−3 J = 6 mJ
dl 2 − 3
= = − 103 As−1
dt 10−3 60. 370-375
dl 1 T1 − T2
As, e = −L Efficiency, η = = …(i)
dt 6 T1
∴ 5 = − L( − 10−3 ) 1 T1 − (T2 − 62)
η1 = = …(ii)
5 3 T1
3
∴ L = 3 H = 5 mH, [ Q1 H = 10 mH]
10 From Eq. (i), we get
58. Range 1 to 1 6T1 − 6T2 = T1
mv 2 6
Since, = Bqv ⇒ 5T1 = 6T2 or T1 = T2
r 5
6
mv p 2mE T − T2 + 62
⇒ r = = = 1 5 2
Bq Bq Bq Putting in Eq. (ii) ⇒ = …(iii)
3 6
For the same orbit, i.e. same radius and same T2
5
magnetic field, we have 3 6 3
⇒ T2 + 186 = T2 186 = T2
2mpEp 5 5 5
2mαEα
= 186 × 5
qp aα ⇒ T2 = = 310°K
2
3
Eα q α  mp From Eq. (iii), we get
⇒ =  ×
Ep q p  mα  6
⇒ T1 =   × 310 = 372 °K
2  5
q  mp
⇒ Eα = Ep  α  × ∴ T1 = 372 − 273 = 99°C
 qp  mα
and T2 = 310 − 273 = 37°C
1
= 1MeV × 4 × = 1MeV ∴ Source = 372 K.
4
Practice Set 2
IITJAM Physics
MM : 100 Time : 3 hrs

n
This test paper has a total of 60 questions carrying 100 marks. The entire question paper is divided into
three Sections A, B and C. All sections are compulsory. Questions in each section are of different types.
n Section A contains Multiple Choice Questions (MCQ). Each MCQ type question has four choices out of
which only one choice is the correct answer. This section has 30 Questions and carry a total of 50 marks.
Q.1 – Q.10 carry 1 mark each and questions (Q.11 – Q.30) carry 2 marks each.
n Section B contains Multiple Select Questions (MSQ). Each MSQ type question is similar to MCQ but with
a difference that there may be one or more than one choice(s) that are correct out of the four given
choices. The candidate gets full credit if he/she selects all the correct choices only and no wrong
choices. This section has 10 questions (Q. 31 – Q. 40) and carry 2 marks each with a total of 20 marks.
n Section C contains Numerical Answer Type (NAT) questions. For these NAT questions, the answer is a
real number which needs to be entered using the virtual numerical keypad on the monitor. No choices
will be shown for these type of questions. This section has 20 questions and carry a total of 30 marks.
Q.41 – Q. 50 carry 1 mark each and questions (Q.51 – Q.60) carry 2 marks each.
n
Do not write more than one answer for the same question. In case you attempt a subjective question
more than once, please cancel the answer(s) you consider wrong. Otherwise, the answer appearing last
only will be evaluated.
n
Clip board, log tables, slide rule, calculator, cellular phone and electronic gadgets in any form are not
allowed.

Section A Multiple Choice Questions (MCQs)


In these type of questions, each question has four choices (a), (b), (c) and (d) out of which only
one option is correct.

1. If the eccentricity of a planet’s orbit is e, the ratio of maximum to minimum speed of the planet
in its orbit is
1− e 1− e 1+ e
(a) (b) (c) (d) 1
1+ e 1+ e + e 2
1− e

2. A perpendicular bob of mass m is suspended from a string of length l attached to a car which
accelerates with an acceleration a 0 . The angle made by the string with the vertical is
a  g a0 g
(a) tan−1 0  (b) tan−1  (c) tan−1 (d) tan−1
g  a0  g a0
Practice Set 2 ●
181

3. A sphere of radius R and mass m rolls without slipping on a surface with speed v . The ratio of
translational KE and rotational KE is
(a) 1/5 (b) 2/5 (c) 5/2 (d) 5

4. Work done when a force F = ( x$ + 2y$ + 3 z$ ) N acting on a particle takes it from the point
r1 = ( x$ + y$ + z$ )m to the point r2 = ( x$ − y$ + 2z$ )m is
(a) − 3 J (b) − 1J (c) zero (d) 2 J

5. A massless spring having force constant k has masses m1 and m 2 attached at its two ends.
The frequency of oscillation is
k k( m1 + m2 ) km1 + m2 k( m1 + m2 )
(a) (b) (c) (d)
m1 + m2 m1 m2 ( m1 + m2 ) m12 m22

6. An excited nucleus of rest mass m 0 is at rest with respect to a chosen inertial frame. It emits a
γ-ray photon and goes on to a lower state whose energy is smaller by ∆E. The frequency of
the γ-ray photon is
∆E ∆E  ∆E 
(a) (b) 1 − 
h h  2 m0 c 2 

∆E − m0 c 2 ∆E  1 − m0 c 2 
(c) (d)  
h h  ∆E 

7. The correct expression for the de-Broglie wavelength λ of a particle (E is the kinetic energy) is
hc hc
(a) λ = (b) λ =
( E + 2 m0 c )
2
E( E + m0 c 2 )
hc hc
(c) λ = (d) λ =
E( E + 2 m0 c ) 2
( E + m0c 2 )

8. A rod of length l is inclined at an angle α with X-axis. An observer moving at a velocity v along
 1 
the X-axis, will measure the angle of inclination as α′,  with γ =  given by
 2 2
1− v / c 

(a) tan−1 (tan α ) (b) tan−1 ( γ tan α )
 α  γ 
(c) tan−1  tan  (d) tan−1  
 γ  tan α 

9. The kinetic energy of a body is twice its rest mass energy. What is the ratio of relativistic mass
to rest mass of the body?
1
(a) 3 (b) 1 (c) (d) 2
2

10. Two particles, each of rest mass m collide head-on and stick together. Before collision, the
speed of each mass was 0.6 times the speed of light in free space. The mass of the final
entity is
5m 5m 25 m
(a) (b) 2 m (c) (d)
4 2 8
182 IIT JAM Physics Solved Papers & Practice Sets

11. A circular arc QTS is kept in an external magnetic field B0 as shown in the figure. The arc
carries a current i. The magnetic field is directed normal and into the page, the force acting on
the arc is
T
B0
S
i Q i
R
60°

(a) 2 iB0R k$ (b) iB0Rk$ (c) − 2 iB0Rk$ (d) − iB0R k$

12. Consider a conducting loop of radius a and total loop resistance R placed in a region with a
magnetic field B thereby enclosing a flux φ 0 . The loop is connected to an electronic circuit as
shown, the capacitor being initially uncharged.
C


Vout
+

If the loop is pulled out of the region of the magnetic field at a constant speed u, the final
output voltage V out is independent of
(a) φ0 (b) u (c) R (d) C

13. The two ends of an inextensible string of length l and mass m are fastened together on a
circular loop and set spinning about the centre of the circle with a uniform angular velocity ω.
The tension in the string is
2 mω 2l mω 2l
(a) mω 2l (b) (c) mω 2l (d)
3 2 4π 2

14. After cutting off a circular portion of radius R / 2 from the centre of a uniform circular disc
of radius R, the moment of inertia about an axis passing through its centre and perpendicular
to its plane becomes I. What is the moment of inertia of the original disc about the same
axis?
(a) 2I (b) 4I
(c) ( 8 / 7 ) I (d) (16 / 15) I

15. Figure shows a hollow cylindrical conductor of radii a and b which carries a current i uniformly
spread over its cross-section. The magnetic field B for a < r < b is given by

µ 0ir µ0 i µ0 i
(a) (b) µ 0 i (c) (d)
2π πr 2 πr
Practice Set 2 ●
183

16. Consider a tightly wound solenoid (helical coil) consisting of n turns per unit length wrapped
around a cylindrical tube of radius a carrying a current i. Let the length of the solenoid be d.
The magnetic field at a point on its axis distant x from one near end is
a
x
d

µ 0ni  x+d x  µ 0ni  x+d x 


(a)  −  (b)  + 
2  a 2 + ( x + d )2 a 2 + x 2  2  a 2 + ( x + d )2 a 2 + x 2 
 

µ 0ni  x d  µ 0ni  x d 
(c)  +  (d)  − 
2  a 2 + ( x + d )2 a 2 + ( x + d )2  2  a 2 + ( x + d )2 a 2 + x 2 
 

17. The magnetic moment for the circuit shown, each side is of length a is
Z

i
X

(a) ia ( $j − k$ )
2
(b) ia ( $j + k$ )
2
(c) 2 ia 2( $j + k$ ) (d) 2 ia 2( $j − k$ )

18. The magnetic moment of a uniformly charged spherical shell of radius R, carrying charge q,
rotating with ω angular velocity is
4 3 3 4
(a) πωσR 4 (b) πωσR 2 (c) πωσR 4 (d) πωσR 2
3 4 4 3

19. Which one of the following sets of Maxwell’s equations for time independent charge density P
and current density $j is correct?
ρ ∂B ∂E
(a) ∇ ⋅ E = ∇⋅B= 0 ∇×E=− ∇ × B = µ 0ε 0
ε0 ∂t ∂t
ρ
(b) ∇ ⋅ E = ∇⋅B= 0 ∇×E=0 ∇ × B = µ 0$j
ε0
(c) ∇ ⋅ E = 0 ∇⋅B= 0 ∇×E=0 ∇ × B = µ 0$j
ρ ∂E
(d) ∇ ⋅ E = ∇ ⋅ B = µ 0$j ∇×E=0 ∇ × B = µ 0ε 0
ε0 ∂t

20. The magnetic field associated with the electric field vector E = E 0 sin (kz − ωt )$j is given by
E0 E0
(a) B = − sin( kz − ωt )i$ (b) B = sin ( kz − ωt )i$
c c
E0 E
(c) B = sin ( kz − ωt ) $j (d) B = 0 sin ( kz − ωt ) k$
c c
ω
21. A wave of frequency ω and wave vector ( $i + $j − k$ ), is propagating through a medium. The
c
phase difference between the points (0, 1, 2) and (2, 1, 0) is
4ω 6ω ω
(a) − (b) (c) zero (d)
c c c
184 IIT JAM Physics Solved Papers & Practice Sets

22. Two plane waves having the same ω, k and amplitude E and opposite circular polarisation is
superposed. The resulting wave is
(a) linearly polarised with amplitude E (b) linearly polarised with amplitude 2E
(c) linearly polarised with amplitude 4E (d) circularly polarised with amplitude E

23. Consider two plane waves in vacuum having same ω, k, amplitude E 0 but different
polarisations, one along $i and one along $j and a phase difference φ. Then, the magnitude of
the time averaged Poynting vector is
1 1
(a) c ε 0E 2 cos φ (b) c ε 0E02 (c) c ε 0E02 (d) c E0E02 cos φ
2 2

24. Which of the following wave functions can be solutions of Schrödinger’s equation for all values
of x (x > 0)?
2 2
(a) ψ = A sec x (b) ψ = A tan x (c) ψ = Ae x (d) ψ = Ae − x

25. With symbols having the usual meaning, the wavelength of electron of energy E is given by
2h h 2 2 mE
(a) 2 mhE (b) (c) (d)
mE 2 mE h

26. A particle with energy V 0 / 2 coming in from the left encounters a potential − 3V 0 / 2 at x = 0
V (x ) = 0 for x < a = − 3V 0 / 2 for x ≥ a
What is the transmission coefficient?
1 2 4 8
(a) (b) (c) (d)
2 3 9 9

27. For a particle of mass m in a one-dimensional box of length l, what is the average of
momentum p x for the ground state?
(a) zero (b) h / (2 l ) (c) h / l (d) h / (2πl )

28. A free particle with energy E whose wave function is a plane wave with wavelength λ enters a
V 
region of constant potential V > 0, where the wavelength of the particle is 2λ. The ratio   is
E 
1 2 3 4
(a) (b) (c) (d)
2 3 4 5

29. A particle of mass m is confined in the potential


V(x)

X
O

1
 mω 2x 2 , for x < 0
V (x ) =  2
 ∞, for x ≤ 0
1 1
Let the wave function of the particle be given by, ψ(x ) = − ψ0 + ψ1
5 5
where, ψ 0 and ψ 1 are the eigen functions of the ground state and the first excited state
respectively.
Practice Set 2 ●
185

The expectation value of the energy is


31 25 13 11
(a) hω (b) hω (c) hω (d) hω
10 10 10 10
pV B(T )
30. The equation of states of a dilute gas at very high temperature is described by = 1+ ,
kBT V
where, V is the volume per particle and B(T ) is a negative quantity. One can conclude that this
is a property of
(a) a van der Waal’s gas (b) an ideal Fermi gas
(c) an ideal Bose gas (d) an ideal inert gas

Section B Multiple Select Questions (MSQs)


In these type of questions, each question has four choices (a), (b), (c) and (d) out of which only
one or more than one is/are correct options.

31. A particle of mass m moving with a velocity ( 3 i$ + 2$j ) ms −1 collides with another particle of
mass M and finally moves with velocity ( − 2i$ + $j ) ms −1. Then, during collision
(a) impulse received by m is m( 5i$ + $j ) (b) impulse received by m is m( − 5i$ − $j )
(c) impulse received by M is m( − 5i$ − $j ) (d) impulse received by M is m( 5i$ + $j )

32. A uniform rod of length l and mass 2m rests on a horizontal table. A point mass m moving
horizontally at right angle to the rod with velocity v collides with one end of the rod and sticks
it. Then,
v v
(a) angular velocity of system after collision is (b) angular velocity of system after collision is
l 2l
mv 2 7 mv 2
(c) loss in KE of system is (d) loss in KE of system is
6 24

33. Consider the following statements for a particle moving in an elliptical orbit under the influence
of a central force.
(a) The radius vector covers equal area in equal time
(b) The motion takes place in a plane
(c) The angular momentum is a constant of motion
(d) The energy remains constant and negative

34. Which one of the following statements is correct?


(a) Gravitational force has an inverse square dependence on distance, whereas electromagnetic,
weak and strong forces are the short range forces
(b) Gravitational and electromagnetic force have the inverse dependence on distance, whereas
weak and strong forces are the short range forces
(c) Gravitational, electromagnetic and weak force have the inverse-square dependence on
distance whereas strong force is the short range force
(d) Gravitational, electromagnetic, weak and strong force have inverse-square dependence on
distance
186 IIT JAM Physics Solved Papers & Practice Sets

35. Mark out the correct statements.


(a) Higher binding energy per nucleon means the nucleus is more stable
(b) If the binding energy of nucleus were zero, then it would spontaneously break apart
(c) Binding energy of a nucleus can be positive
(d) Binding energy of a nucleus can be negative
4
36. An O16 nucleus is spherical and has a charge radius R and a volume V = πR 3 . According to
3
the empirical observations, the volume of 54 X128 nucleus assumed to be spherical is V ′ and
radius R ′. Then,
(a) V ′ = 8 V (b) V ′ = 2V (c) R ′ = 2 R (d) R ′ = 8 R

37. With reference to nuclear forces which of the following statements is true?
(a) Short range (b) Charge independent
(c) Velocity dependent (d) Spin independent

38. Sodium contains 2.54 × 10 22 free electrons per cm 3 (Given that, h = 6.63 × 10 −34 J-s,
m = 9.11 × 10 −31 kg, k = 1.38 × 10 −23 J/K and 1 eV = 1.6 × 10 −19 J), then
(a) Fermi energy is 4 eV (b) Fermi energy is 3.1 eV
. × 10 K
(c) Fermi temperature is 36 4
(d) Fermi temperature is 4.2 × 104 K

39. A combination of three blocks shown in the figure given below is pushed by a horizontal force
of 30 N on a frictionless surface.
A C
B
F = 20 N
2 kg 1 kg 3 kg

Which of the following are correct?


(a) Force exerted by A on B is 5 N (b) Force exerted by A on B is 20 N
(c) Force exerted by C on B is 15 N (d) Force exerted by C on B is 10 N

40. Choose the correct statements for a conservative force.


(a) Work done by the conservative force on a particle moving between two points is independent of
the path
(b) Force of gravity is a conservative force
(c) ∇ × F = 0
(d) F = − grad U

Section C Numerical Answer Type Questions (NATQs)


This section contains Numerical Answer Type (NAT) questions. For these NATQs, the answer is a
real number.

41. A string of length 2 m is fixed at both ends. If the string vibrates in its fourth mode with a
frequency of 500 Hz, then the waves would travel on it with a velocity of .......... ms −1.

42. The glancing angle in a X-ray diffraction experiment is 30° and the wavelength of X-rays used
is 20 nm. The interplanar spacing of the crystal diffracting these X-rays will be ……nm.
Practice Set 2 ●
187

43. The rest mass energy of electron is ....... MeV.


44. An unstable particle of rest energy 1000 MeV decays into µ-meson and neutrino, with a mean
life time of 10 −8 s, when at rest. The mean decay distance, in metre, when the particle has a
momentum of 1000 MeV/c is

45. An n - p - n transistor circuit has α = 0985


. . If Ic = 2 mA, then the value of Ib is .............. mA.

46. The mutual characteristic of a triode valve is shown in figure below for operating point Q
specified by V A = 200 V, VG = − 4V and I A = 10 mA. If the anode slope resistance r p = 10 kΩ,
then the amplification factor µ will be ............
IA [mA]

13

Q 10
VA = 200 V
7

VA = 100 V

0
VG = (V ) –6 –4 –2

47. In an n-p-n transistor, the collector current is 24 mA. If 84% of electrons reach collector, its
base current in mA is ...........

48. In a common base transistor circuit Ic = 097


. mA, Ib = 30 µA, then current gain, α is ............

49. Two slits, 4 mm apart, are illuminated by light of wavelength 6000 Å. The fringe width on a
screen placed 2 m from the slits is ... ... mm.

50. The velocity of light in glass, whose refractive index ( µ g ) = 1.5 is 2 × 10 8 ms −1. In certain
. × 10 8 ms −1. The refractive index ( µ w ) of that liquid
liquid, the velocity of light is found to be 26
is .......

51. Illuminance of 1.5 × 105 lumens/m 2 of a surface is produced by sunlight falling normally on the
surface of the earth. The distance of the earth from the sun is 1.5 × 10 8 km. The luminous
intensity produced is ............. × 10 27 lumens.

52. The vibration of a string can be described by the equation


y = ( 0.75 ){cos( 2π / 5 )x} sin {(60πs −1 )t}. The speed of the wave travelling in the string is
...........m/s.

53. A particle is projected at an elevation tan −1( 4 / 3 ) from a point O. The ratio of the range on the
horizontal plane through O to the greatest height ascended above O, is ..........

54. An engine blowing its whistle at a frequency of 1200 Hz, approaches a stationary observer
with a velocity of 50 ms −1. If the speed of sound is 350 ms −1, then the apparent frequency as
heard by the observer is ........... Hz.
188 IIT JAM Physics Solved Papers & Practice Sets

55. A railway engine passes by the platform at speed of 36 km h −1, blowing its whistle having a
frequency of 660 Hz. The difference in the frequencies of the whistle heard by a person
standing on the platform as the engine goes pass the persons is equal to ............

56. Two organ pipes closed at one end when blown simultaneously produced 4 beats per second.
If the length of the shorter pipe is 0.75 m, the length of other will (velocity of sound = 330
ms −1) .............. m.

57. Two travelling waves y 1 = 065


. sin ( 0.4x − 800t ) m and y 2 = − 065
. sin ( 0.4x + 800t ) m are
superimposed in a medium. For the resultant wave at the point x = 2.5 π m, the maximum
displacement is

58. The temperature in kelvin at which the average speed of H2 molecules will be same as that of
N2 molecules at 35 °C will be ...........

59. A Carnot engine takes in 3000 kcal of heat from a reservoir at 627 °C and gives it to sink at
27 °C. The work done by the engine is ..............× 10 6 J.

60. The value of current passing through the branch AB in the electric circuit given in the
figure............A.
l1 A 2Ω
4Ω l2

12V 8 Ω 10V

Answers
1. (c) 2. (a) 3. (c) 4. (b) 5. (b) 6. (c) 7. (c) 8. (b) 9. (a) 10. (c)
11. (b) 12. (a) 13. (d) 14. (c) 15. (d) 16. (d) 17. (b) 18. (a) 19. (b) 20. (a)
21. (a) 22. (b) 23. (c) 24. (d) 25. (c) 26. (c) 27. (b) 28. (c) 29. (c) 30. (a)
Answers with Explanations
Section A Multiple Choice Questions 4. (b) Work done, W = F ⋅ d
rmax 1 + e Here, F = x$ + 2y$ + 3z$ N
1. (c) We know that =
rmin 1 − e r1 = ( x$ + y$ + z$ ), r2 = ( x$ − y$ + 2z$ )
and by conservation of angular momentum ⇒ d = r2 − r1
rmaxv1 = rminv 2 = ( x$ − y$ + 2z$ ) − ( x$ + y$ + z$ )
v 2 rmax 1 + e = ( − 2y$ + z$ )
= =
v1 rmin 1 − e ∴Work done, W = ( x$ + 2y$ + 3z$ ) ⋅ ( − 2x$ + z$ )
v 2 is maximum and v1 is minimum, since rv must be a = 2( − 2) + 3(1)
constant. = − 4 + 3 = − 1J
2. (a) 5. (b) m1 k m2
θ
T x1 x2
θ
The Lagrangian for the system is simply
ma0 1 1 1
L = m1 ⋅ x12 + m2 ⋅ x 22 − k (x 2 − x1)2
2 2 2
The equations of motion are
mg
m1 x&&1 − k (x 2 − x1) = 0 ...(i)
Free body diagram for the pendulum bob in the frame and, m2 x&&2 + k (x 2 − x1) = 0 ...(ii)
of the car is shown in figure given above. Equating forces,
On subtracting Eq. (ii) from Eq. (i), we get
T cos θ = mg
 1 1
T sin θ = ma 0 x&&2 − x&&1 + k  +  (x 2 − x1) = 0
 m2 m1
a a0
tan θ = 0 ⇒ θ = tan−1 k (m1 + m2 )
g g Let, ξ = x 2 − x1 and ω 2 =
m1 m2
or T = m (a 02 + g 2 )1/ 2
ξ&& + ω 2ξ = 0
3. (c)
Which is a simple harmonic motion with eigen frequency,
R k (m1 + m2 )
ω v ω=
O m1 m2

6. (c) By momentum conservation, the nucleus must recoil with


a velocity v.
Let a sphere moves on a surface with translational hν
So, mv =
velocity v and angular velocity ω, then c
v = ωR …(i) where, ν is the frequency of the emitted photon and
1 energy conservation gives
Now, translational TKE = mv 2 …(ii)
2 ⇒ ∆E = m0c 2 − mc 2 = hν
1
Rotational RKE = Iω 2 …(iii) where, m is the mass of the nucleus after emitting the
2
photon.
On dividing Eq. (ii) by Eq. (iii), we get
h
1 7. (c) The de-Broglie wavelength, λ = …(i)
mv 2 mv
TKE 2 m v2
= 2
= m0
RKE 1/ 2 Iω I ω2 where, relativistic mass, m =
2 1−v 2 /c 2
But, I= mR 2, v = ωR
5 m0 = rest mass
TKE (ωR )2 5 v2 m2
= = ⇒ = 1 − 02
RKE 2 2 2 c 2
m
(ωR )
5
190 IIT JAM Physics Solved Papers & Practice Sets

⇒ mv = c m 2 − m02 …(ii) 11. (b) The force acting on an element of length d I is


Thus, from Eqs. (i) and (ii), we get dF = id I × B
h hc Set up the coordinate axes as shown in the figure. B is
⇒ λ= = …(iii)
c m − m0 c m 2 − m02
2 2 2 in the negative X-direction. So, d F will be inYZ -plane.
On observation, it is easily seen that only the component
Now, c 2 m 2 − m02 = c 4(m − m0 )(m + m0 ) of F along Z -axis will survive.
= {(m − m0 )c 2}{(m − m0 )c 2 + 2m0c 2}1/ 2 Z
dI
= E(E + 2m0c 2 ) [since, E = (m − m0 ), c 2]
i i
So, by Eq. (iii), we get
hc θ
λ=
2
E(E + 2 m0c )
Y
X
8. (b) Let OP is the rod of length l, then
π /6
Hence, F = i ∫ (dI × B)z = i ∫ dlB0 cos θ
−π / 6
N P π /6
= iB0R ∫ cos θ dθ
− π /6

⇒ F = iB0R k$
α
O M 12. (a) Since, the voltage developed across the circuit due to
ON the change in the magnetic flux associated with the
tan α = …(i) loop, i.e.
OM

If an observer moves with velocityv along X-axis, then Vemf =
there is a contract along X-axis. Thus, new component dt
along X-axis becomes At t = 0, = φ 0 [φ 0 = constant]
OM ′ = (1/ γ ) OM …(ii) Hence, output voltage is independent of φ 0.
OM 13. (d) Consider a elementary length of the string dl which
and so, tan α′ = …(iii)
OM ′ subtends an angle δθ at the centre. Drawing free body
ON diagram for this elementary length.
⇒ tan α ′ = γ
OM
 ON 
= γ  T T
 OM 
= γ tan α by Eq. (i) r
α ′ = tan−1( γ tan α )
δθ
9. (a) From mass energy equivalence, mc 2 = m0c 2 + EK
where, r is the radius of the circular loop.
where, EK is kinetic energy m is relativistic mass and m0 δθ
is mass at rest. 2T sin = δmω 2T
2
Given, EK = 2m0c 2
δθ mrδθω 2r
We have mc 2 = m0c 2 + 2m0c 2 So, 2T =
2 L
or mc 2 = 3m0c 2 mω 2L
or T =
So m / m0 = 3 4π 2
m m Since, l = 2πr
10. (c) m′ = =
1−v 2 /c 2  0.6c 
2
14. (c) Shaded portion → Removed portion
1−  
 c  Now, moment of inertia of the unremoved
m 5
= = m R
 3
2 4 R
1−   2
 5 M
5 5 10 5
Mass of final entity = m+ m= m= m
4 4 4 2
Practice Set 2 ●
191

Disc = Moment of inertia of shaded portion + MI of Consider ndx circular wires at a distance x from P,
unshaded portion …(i) µ i na 2dx
dB = 0
Now, if M is total mass of the disc, then mass of shaded 2 (a + x 2 )3/ 2
2
portion
M (R / 2)2 M µ 0i x +d dx
M′ = = …(ii) B= na 2 ∫
R2 4 2 x (a 2 + x 2 )3/ 2
Moment of inertia of unshaded portion Let x = a cot θ
1 1 R
2 µ 0nia 2 θ2 acosec2 θ
=
2
MR 2 − M ′  
2  2
B=−
2 ∫θ 3 3
1 a cosec θ

1 M′ R 2 µ 0ni
I= MR 2 − = (cos θ 2 − cos θ1)
2 4 2
1 M R2  M µ ni  x +d x 
I = MR 2 −   Q M ′ =  B= 0  − 
2 4  4  4 2  a 2 + (x + d )2 a 2 + x 2 

8 −1 7
I= MR 2 = MR 2 17. (b) We may think of the given circuit as made up of two
16 16
square current circuits, whose normal are along Y-axis
7 1 7 1 
= × 2 × MR 2 = ×  MR 2 and Z-axis, respectively.
16 2 8 2 
i
7
I = × (Moment of inertia of whole disc)
8
8 i
Moment of inertia of whole disc = I
7
15. (d) The current i flows in the region between the two cylinders.
We draw an Amperian loop of radius r having its centre on m = ia 2( $j + k$ ) is the required magnetic moment.
the common axis.
18. (a) Let the sphere rotates about the Z-axis.
Using Ampere’s law, ∫ B ⋅ dI = µ 0 ∫ J ⋅ d S ω
The magnetic field is circumferential, so
i π (r 2 − a 2 )
B 2 πr = µ 0
π (b 2 − a 2 )
µ 0i (r 2 − a 2 )
B= 2 2
(for a < r < b) R
(b − a ) 2π r
Consider some cases
r = a, B = 0
µ i
and r = b, B = 0
2 πb Any surface element of area dS rotates with angular
and in the region r >b velocity ω. This constitutes a current and hence has a
µ i magnetic moment which equals the current times the
B= 0 (for r > b)
2 πr area enclosed by the circle.
( σ dS ) 2
16. (d) The magnetic field due to a circular wire of radius a Thus, dm = πr
carrying current i at a point on its axis at a distance x from t
the centre is given by where,
µ i a2 σ = Surface charge density assumed constant.
B= 0 t = Time of revolution by the dS element.
2 (a + x 2 )3/ 2
2

r = Radius of the circle described by the element of


θ1 P area dS in revolution.
θ2 σπr 2
∴ m=∫ dS
For n turns, it becomes t
µ i na 2 σπ (R sin θ )2R 2 sin θ dθ dφ
B= 0 =∫ ω
2 (a + x 2 )3/ 2
2 2π
192 IIT JAM Physics Solved Papers & Practice Sets

Using, r = R sin θ 21. (a) Between two points having the position vector r1 and r2
dS = R 2 sin θ dθ dφ respectively, the phase difference of a wave having
2π wave vector k is given by
t= in spherical coordinates
ω ∆φ = k ⋅ (r1 − r2 )
r π π /2
m = ωR 4 2π ∫ sin3 θ d θ = 2πωσR 4 ∫ sin3 θ d θ ω
2 0 0 For k = ( $i + $j − k$ )
c
Γ ( 2)Γ (1/ 2)
= 2πωσR 4 r = (0, 1, 2) = ( $j + 2 k$ )
1
2Γ (5 / 2)
4 r2 = (2, 1, 0) = ( 2$i + $j )
m = πωσR 4
3 ω $ $ $ $
Hence, ∆φ = ( i + j − k) ⋅ ( j + 2 k$ − 2$i − $j )
qL c
which can also be put into the form m = , where L is
2m 4ω
=−
the angular momentum and m is its mass. c
19. (b) The Maxwell’s equations are 22. (b) We can represent the two circularly polarised waves by
ρ ∂B the following equations.
∇ ⋅ B = , ∇ ⋅ B = 0, ∇ × E + −
ε0 ∂t E1 = E [sin ( − k ⋅ r + ωt )$i + cos ( − k ⋅ r + ωt )$j]
 ∂E  (Right hand polarisation)
and ∇ × B = µ 0 J = ε0
 ∂t 
and E2 = E [ − sin (ωt − k ⋅ r ) i + cos (ωt − k ⋅ r )$j]
$
Qρ and J are time independent.
(Left hand polarisation)
∂E ∂B
Hence, = =0 Their superposition
∂t ∂t
E′ = E1 + E2
Thus, the equations become
ρ E′ = 2E cos (ωt − k ⋅ r )$j
⇒ ∇ ⋅ E = , ∇ ⋅ B = 0, ∇ × E = 0
ε0 which is a linearly polarised wave with amplitude 2E.
and ∇ × B = µ 0J 23. (c) Let the two waves be represented by
20. (a) E = E0 sin (kz − ωt )$j E1 = E0 cos (ωt − k ⋅ r )$i
Let us use the equation and E2 = E0 cos (ωt − k ⋅ r + φ )$j
+Y At fixed point say r = 0, we find the resultant electric
E field E
E = E1 + E2
k
E = E [cos ωt$i + cos (ωt + φ )$j]
0
+Z
B The Poynting vector S is given by
–X k
S= E2
∂B µ 0ω
∇×E=−
∂t The time averaged Poynting vector is
$i $j k$ 1
<s > = < E2 >
∂ ∂ ∂ ∂B µ 0ω
=−
∂x ∂y ∂z ∂t 1 2
0 E0 sin (kz − ωt ) 0 = E0 ( < cos2 ωt > + < cos2(ωt + φ ) > )
µ 0ω
∂ ∂B
− $i E0 sin (kz − ωt ) = − 1E02
∂z ∂t =
µ0 ω
$i kE cos (kz − ωt ) = + ∂B
0
∂t < s > = c ε 0E02
ikE0 It is instructive to note that there is no interference them,
or B=− sin (kz − ωt ) (after integration)
ω when the direction of the polarisation vectors of the two
ω waves are perpendicular to each other.
as =c 2
k 24. (d) Only ψ = Ae − x remains finite when x → ∞
E
∴ B = − 0 sin(kz − ωt )$i and x → − ∞, hence it is only solution.
c
Practice Set 2 ●
193

25. (c) de-Broglie law states that every moving body is 29. (c) Since, given potential is for SHO.
associated with a wave of wavelength λ given as  1
Hence, En = n +  hω
h  2
λ= …(i)
mv 1 2
Now, ψ (x ) = − ψ 0 + ψ1
where, h is Planck’s constant. 5 5
p2 (mv )2 Expectation value of energy is
Kinetic energy, K = =
2m 2m 1 4
ψ *| E | ψ = ψ 0| E0| ψ x0 > + ψ 1| E1| ψ 1x
⇒ (mv )2 = 2 mE 5 5
⇒ mv = 2 mE …(ii) 1 4
= E0 + E1
5 5
h
From Eqs. (i) and (ii), we get λ = 1 hω 4 3hω 13hω
2 mE E = × + × =
5 2 5 2 10
26. (c) x <a x ≥a 30. (a) van der Waals’ equation
V (x ) = 0, − 3V0 / 2  a
 p + 2  (V − b ) = RT
 V 
a RT
p=− +
V2 V −b
a
pV = − + RT [since, b is very small]
V
(V0 )2 V0 × 4 As, R = NkB = kB
∴ Transmission coefficient = 2
=
 3V0  9V02 Q N =1
 
 2  a
⇒ pV = − + kBT
4 V
= = 4 :9
9  a 
or pV = kBT 1 − 
27. (b) A particle will lie in a box, if length of the box is an  kBTV 
integral multiple of half wavelength pV B (T ) a
or ≈ 1+ ⇒ = BT
λ/2 kBT V kBT
As the given equation of states resembles with van der
A B Waals’ equation one can conclude that the given gas is
L a van der Waals’ gas.
i.e. L = n( λ / 2), n = 1, 2, 3, ....
2L
⇒ λ=
n Section B Multiple Select Questions
So, by D-Broglie wave equation
2L 31. (b, d) Impulse received by m = Change in momentum of m
p=
λ = m( − 2$i + $j ) − m ( 3$i + 2$j )
h nh = ( − 5$i − $j ) m
⇒ p= =
(2 L / n ) 2 L
The change in momentum of M
For ground state n = 1
= − [change in momentum m]
h
So, p= = (5$i + $j ) m
2L
28. (c) For free particle 32. (a, c)
p 2
h 2 2m
E= =
2 m λ2 2 m
L /2
h2
KE = E − V =
( 2 λ )2 2 m
Dividing Eq. (ii) by Eq. (i), we get L /2
V 3
⇒ = mv
E 4
194 IIT JAM Physics Solved Papers & Practice Sets

Applying conservation of energy Putting the value of m, h,


N
, we get
mv = (m + 2 m )v ′ V
2/ 3
⇒ v′ =
v ( 6.63 × 10−34 )2  3 × 2.54 × 1028 
3 EF = × 
2 × 9.1 × 10−31  8 × 3.14 
L = Iω
L v ( 2m ) l 2 mL2 = 5 × 10−19 J ~
− 3.1eV
⇒ ( 3m ) × = ×ω + ω
2 3 12 4 Fermi temperature,
v E 5 × 10−19
On solving, we get ω = TF = F =
L k 1. 38 × 10−23
mv 2 ⇒ TF = 3.6 × 104 K
Loss in KE will be =
6
39. (b, c) The combined acceleration
33. (a, b, c) Angular momentum J = r × mv implies that
30
angular momentum is perpendicular to r and v. a= = 5 m/ s2
2 + 1+ 3
Hence, the path of particle under the influence of a
C
central force must lie in a plane. A B
F T1 T2 T2
dJ 2 kg 1 kg 3 kg
Torque, τ= = r × F = r × r f (r ) = 0
dt
Thus, angular momentum J = constant
1 1 J Consider A,
Areal velocity = r × v = × (J = mvr )
2 2 m ma = F − T1
J ⇒ 2 × 5 = 30 − T1
= = constant
2m ⇒ T1 = 20 N
Thus, radius vector of orbit sweeps out equal area Force exerted by A on B
in equal interval of time. T1 − T2 = m2 a
34. (a, c, d) The gravitational electromagnetic and weak 20 − T2 = 1 × 5
forces are the inverse square dependence of the T2 = 15 N
distance whereas and strong forces are the short
Force exerted by C on B = 15 N
range forces.
40. (a, b, c, d)
35. (a, b, c) Higher binding energy per nucleon means the
The work done by a conservative force in displacing particle
nucleus is more stable. If binding energy of a
from one point to another depends only on the position of the
nucleus were zero, then it would spontaneously
two given points and is quite independent of the actual path
break apart. Binding energy of a nucleus is
taken between them. For conservative force, curl is zero.
positive.
Curl of gravitational force,
36. (a, c) R = R0A1/ 3  Gm1 m2r  1
∇×F=∇×  = Gm1 m2 ∇ × 3 r = 0
For O16, R = R0(16)1/ 3  r3  r
128
For 54 X , R′ = R0(128)1/ 3 Hence, gravitational force is conservative.
1/ 3
 128
⇒ R′ =   R = 2R
 16 
4 Section C Numerical Answer Type Questions
and V′ = i (R′ )3 = 8 V
3 41. Range 500 to 500
37. (a, b, c) The nuclear forces are spin dependent with 2l l
For fourth normal mode, λ = =
the singlet (nucleon having opposite spin) 4 2
interaction much weaker than the triplet (nucleon 2
∴ λ = = 1m
having parallel spin). 2
2/ 3 Now, velocity of wave v = nλ = 500 × 1 = 500 ms−1
h 2  3N 
38. (b, c) We know that, EF =  
2m  8πV  42. Range 15 to 25
Here,
N
= 2.54 × 1022/cm3 Since, 2 d sin θ = nλ
V nλ 1 × 20
⇒ d = = = 20 nm
= 2.54 × 1028/m3 2 sin θ 2 sin 30°
Practice Set 2 ●
195

43. Range 0.50-0.52 49. Range 0.15 to 0.40


Rest mass of electron = 9.1 × 10−31 kg λD
β=
⇒ E = m0c 2 d
6000 × 10−10 × 2
E = 9.1 × 10−31 × ( 3 × 108 )2 β=
4 × 10−3
or, E = 9.1 × 9 × 10−15 J
β = 3 × 10−4 m = 0.3 mm
− 15
or, E = 81. 9 × 10 J
50. Range 1.1 to 1.3
⇒ 1 eV = 1. 6 × 10−19 J
c = µv = 1. 5 × 2 × 108 = 3 × 108 ms−1
⇒ 1 MeV = 1. 6 × 10−13 J
c 3 × 108
1 µ= = = 12
.
∴ 1J= MeV v 2.6 × 108
1.6 × 10−13
1 51. Range 3.2 to 3.4
1J= × 1013 MeV
.
16 Given, I = 1.5 × 108 km lumens/cm2
81. 9
E= × 10−2 MeV r = 1.5 × 1011 m
1. 6
I = L /r2
. × 10−2 MeV ⇒ 0.511 MeV
E = 511
or L = I × r 2 = 15
. × 105 × (15
. × 1011)2
44. Range 2 to 4 L = 3.375 × 1027 lumens
Energy of 1000 MeV/c category of neutrino
1000 52. Range 150 to 150
= momentum × c = ×c  2π 
c y = ( 0.75)cos × sin( 60π )t 
= 1000 MeV  5 
Half-life for 1000 MeV neutrino = 10−8 s Comparing this with standard equation,
∴ Distance travelled in 10−8 s = 10−8 × ( 3 × 108 ) = 3 m 2π 2π
y = a cos x sin × t , we get
λ T
45. Range 0.02 to 0.04 2π 2π
= ⇒λ =5
Given, α = 0.985 and l = 2 mA 5 λ
I
α = c ⇒ Ie = c
I 2π
Q n = 30 and = 60π ⇒ 2nπ = 60π
Ie α T
2 × 10−3 v = nλ = 30 × 5 = 150 cms−1
⇒ Ie = = 2.03 × 10−3
0.985
53. Range 3 to 3
Since, Ie = Ic + Ib
Horizontal range (v 2 sin2 θ / g )
∴ Ib = Ie − Ic = ( 2.03 − 2) × 10−3 = 0.03mA = 2
Maximum height (u sin2 θ / 2g )
46. Range 14 to 16 2 u 2 sin θ cos θ 2g
−3 = × 2
δI A (13 − 7) × 10 g u sin2 θ
gm = = . × 10−3 Ω −1
= 15
δVG − 2 − ( − 6) 4 4
= = =3
µ = rp × gm = 10 × 103 × 1.5 × 10−3 = 15 J tan θ 4 / 3

47. Range 6 to 6 54. Range 1400-1400


80 Given, frequency, ν = 1200 Hz
Ic = Ie = 0. 8Ie
100 Velocity of source, vs = 50 ms−1
I 24 Velocity of sound, v = 350 ms−1
So, Ie = c = = 30 mA
0.8 0.8
 v 
Ib = Ie − Ic = 30 − 24 = 6 mA Apparent frequency, ν′ =   ×ν
v − vs 
48. Range 0.9-1.0
 350 
α=
Ic
= c
I ⇒ ν′ =   × 1200 = 1400Hz
Ie Ic + Ib  350 − 5
0.97 55. Range 41 to 42
α=
0.97 + 0.030  v 
Since, n1 =  n
0.970 v − vs 
α= = 0.97
1.00
196 IIT JAM Physics Solved Papers & Practice Sets

 v  58. Range 22 to 22
and n2 =  n
v + vs  T
Average speed v ∝
∴ Difference in frequencies of the whistle heard by a person, m
 v   v  273 + 35 T
∆n = (n1 − n 2 ) =  n −  n Since, v n2 = v H 2 =
v − vs  v + vs  28 2
n × 2 vvs 308 T
= ⇒ = ⇒ T = 22K
(v 2 − vs2 ) 28 2

 36 × 5 59. Range 8.0-9.0


660 × 2 × 320 ×  
 18  4,224,000 Here, Q1 = 3 × 106 cal
= 2
=
 36 × 5 10,2300 T1 = 627 + 273 = 900 K
( 320)2 −  
 18  T2 = 27 + 273 = 300 kcal
= 4129
. As,
Q 2 T2
=
Q1 T1
56. Range 0.77-0.80
T 300
Given, v = 330 ms−1 and l1 = 0.75 m ∴ Q 2 = 2 × Q1 = × 3 × 106 = 106cal
T1 900
v v
ν1 = and ν 2 = W = Q1 − Q 2
4l1 4l2
= 3 × 106 − 106 = 2 × 106 cal
Since, ν1 − ν 2 = 4
v v  1 1 4 × 4 = 2 × 106 × 4.2 J = 8.4 × 106
∴ − =4⇒  −  =
4l1 4l2  l1 l2  v 6 9
60. Range to
1 1 16 1 7 7
⇒ − = . − = 0.048
⇒ 133
0.75 l2 330 l2 [Hint Given, circuit applying the superposition theorem,
1 I = I1 + I 2]
⇒ = 1282
. ⇒ l2 = 0.78 m
l2  
 12  2
57. Range 1.1 to 1.4 Here, I1 =  ×
1 + 8 × 2  2 + 8
Given, y1 = 0.65 sin ( 0.4x − 800 t ) m and
 8 + 2 
y 2 = − 0.65 sin ( 0.4x − 800 t ) m
Resultant, y = y1 + y 2 12 2 24 3
= × 10 × = = A
40 + 16 10 56 7
= [ 0.65 sin ( 0.4x − 800 t )]
 
+ [ − 0.65 sin ( 0.4x + 800 t )]
 10  4
= 2 × 0.65 cos 0.4x sin ( − 800 t ) and I2 =  ×
2 + 4 × 8  4+ 8
Here, maximum displacement at 4 + 8 

x = 2.5π m is 10 4 40 5
= 2 × 0.65 cos 0.4 × 2.5π = × 12 × = = A
24 + 32 12 56 7
= 13
. cos π = − 13
. 3 5 8
∴ I= + = A
∴ Maximum displacement = 13
. m 7 7 7
Practice Set 3
IITJAM Physics
MM : 100 Time : 3 hrs

n
This test paper has a total of 60 questions carrying 100 marks. The entire question paper is divided into
three Sections A, B and C. All sections are compulsory. Questions in each section are of different types.
n Section A contains Multiple Choice Questions (MCQ). Each MCQ type question has four choices out of
which only one choice is the correct answer. This section has 30 Questions and carry a total of 50 marks.
Q.1 – Q.10 carry 1 mark each and questions (Q.11 – Q.30) carry 2 marks each.
n Section B contains Multiple Select Questions (MSQ). Each MSQ type question is similar to MCQ but with
a difference that there may be one or more than one choice(s) that are correct out of the four given
choices. The candidate gets full credit if he/she selects all the correct choices only and no wrong
choices. This section has 10 questions (Q. 31 – Q. 40) and carry 2 marks each with a total of 20 marks.
n Section C contains Numerical Answer Type (NAT) questions. For these NAT questions, the answer is a
real number which needs to be entered using the virtual numerical keypad on the monitor. No choices
will be shown for these type of questions. This section has 20 questions and carry a total of 30 marks.
Q.41 – Q. 50 carry 1 mark each and questions (Q.51 – Q.60) carry 2 marks each.
n
Do not write more than one answer for the same question. In case you attempt a subjective question
more than once, please cancel the answer(s) you consider wrong. Otherwise, the answer appearing last
only will be evaluated.
n
Clip board, log tables, slide rule, calculator, cellular phone and electronic gadgets in any form are not
allowed.

Section A Multiple Choice Questions (MCQs)


In these type of questions, each question has four choices (a), (b), (c) and (d) out of which only
one option is correct.

1. A uniform metre stick of mass m is pivoted about a horizontal axis through its lower end O.
Initially, it is held vertical and is allowed to fall freely down. Its angular velocity at the instant
when it makes an angle 60° with the vertical is
g g 3g 2g
(a) (b) (c) (d)
2l l 2l l

2. For the Lagrangian L = ax& 2 + by& 2 − kxy , the Hamiltonian H is


p2x py2 p2x py2 p2x py2 p2x + py2
(a) + + kxy (b) + − kxy (c) + + kxy (d) + kxy
2a 2b 4a 4b 4a 4b 4ab
198 ●
IIT JAM Physics Solved Papers & Practice Sets

3. In given figure, XY is an infinite line charge distribution. P and Q are points as shown in the
figure. The ratio of electric fields at P and Q is
P Q
2a
a
X Y
(a) 1 : 1 (b) 1 : 2
(c) 2 : 1 (d) 1 : 4

4. A planet moving in an elliptical orbital has its smallest and greatest speeds v min and v max
respectively. The eccentricity of orbit is
vmax + vmin vmax − vmin 2 vmax 2 vmin
(a) (b) (c) (d)
vmax − vmin vmax + vmin vmax + vmin vmax − vmin

5. The electric field between two concentric cylindrical conductors at, r = 0.01 m and r = 0.05 m is
 105 
given by E =   a$ r V/m. The energy stored in a 0.5 length (assuming free space)
 r 
(a) 0.224 J (b) 1.224 J (c) 2.224 J (d) 3.224 J
3750
6. The sublimation curve of solid ammonia is given by In p = 23 − and the vaporisation
T
3050
curve of the liquid ammonia is given by In p = 19.5 − , where p is in mm of Hg and T is
T
in K. The temperature of the triple point of ammonia is
(a) 3750 K (b) 3050 K (c) 700 K (d) 200 K

7. In the circuit having an ideal diode as shown below, the rms voltage across R is

D1
100 V R

(a) 100 V (b) 140 V (c) 70 V (d) 50 V

8. If n represents the number of eigen states of a hydrogen atom, then its discrete energy levels
are proportional to
(a) n (b) n2 (c) 1/ n (d) 1 / n2

 cos θ 0 sin θ 
9. If A =  0 1 0 

− sin θ 0 cos θ
(a) A is orthogonal (b) A is unitary
(c) A is Hermitian (d) A is skew Hermitian

10. A rigid frictionless rod rotates with a constant angular velocity ω. A bead of mass m slides
outwards with a velocity u. The coriolis force on the bead is
(a) − muω θ (b) muω θ (c) −2mωu θ (d) 2mωu θ
Practice Set 3 ●
199

11. Consider a tightly wound solenoid (helical coil) consisting of n turns per unit length wrapped
around a cylindrical tube of radius a carrying a current i. Let the length of the solenoid be d.
The magnetic field at a point on its axis distant x from one near end is
a
x
d

µ 0ni  x+d x  µ 0ni  x+d x 


(a)  −  (b)  + 
2  a 2 + ( x + d )2 a 2 + x 2  2  a 2 + ( x + d )2 a 2 + x 2 
 

µ 0ni  x d  µ 0ni  x d 
(c)  +  (d)  − 
2  a 2 + ( x + d )2 a 2 + ( x + d )2  2  a 2 + ( x + d )2 a 2 + x 2 
 

12. Which of the following expressions for a vector potential A does not represents a uniform
magnetic field of magnitude B 0 along the z-direction?
B x B y   −B y B y 
(a) A = ( 0, B0 x, 0) (b) A = ( − B0 y, 0, 0) (c) A =  0 , 0 , 0 (d) A =  0 , 0 , 0
 2 2   2 2 

13. Consider a particle in one-dimensional box between x = 0 and x = a


V (x ) = 0 for 0 < x < a = ∞ for x ≤ 0 or x ≥ a

When it is in the ground state, what is the probability that it will be found in the region
(a / 4 ) < x < ( 3a / 4 )?
(a) 1/2 (b) (1/2) + (1/π) (c) (1/4) + (2/π) (d) (1/4) + (1/π)

14. If the radius of earth were to decrease by 1%, its mass remaining the same, what would be
the approximate change in acceleration due to gravity?
(a) Decrease by 2% (b) Increase by 2% (c) Decrease by 1% (d) Increase by 1%

15. The electric field of a wave propagating in the xy - plane, with wave vector k making an angle
of 30° with the X-axis is
 3   3 1 
(a) E = E0 cos ω t − kx + ky (b) E = E0 cos ω t − kx − ky
 2   2 2 
 3 1 
(c) E = E0 cos (ω t − kx − ky ) (d) E = E0 cos ω t + kx + ky
 2 2 
Z
16. A parallel plate capacitor is being discharged. What is the direction of
the energy flow in terms of the Poynting vector in the space between
the plates? +
(a) Along the wire in the positive Z-axis – Y

(b) Radially inward (− r$)


φ
(c) Radially outward ( r$ ) X r
(d) Circumferential (φ)

17. A heat pump working on the Carnot cycle maintains the inside temperature of a house at 22°C
by supplying 450 kJ/s. If the outside temperature is 0°C, then heat taken in kJ/s, from the
outside air is approximately
(a) 487 (b) 470 (c) 467 (d) 417
200 ●
IIT JAM Physics Solved Papers & Practice Sets

18. The form factor F (q ) = ∫ exp (i q ⋅ r ) / h ) ρ(r )d 3 r of Rutherford scattering is obtained by


choosing a delta function for the charge density ρ(r ). The value of the form factor is
(a) unity (b) infinity (c) zero (d) undefined

19. A long cylindrical wire kept along the Z -axis carries a current density k = j 0r k$ , where j 0 is a
constant and r is the radial distance from the axis of the cylinder, the magnetic induction B
inside the conductor at a distance d from the axis of the cylinder is
µ 0 j 0d 1 1
(a) µ 0 j 0 φ (b) − φ (c) µ 0 j 0d 2φ (d) − µ 0 j 0 d 3φ
2 3 4
4
20. An O16 nucleus is spherical and has a charge radius R a volume V = πR 3 . According to the
3
empirical observations of the charge radii, the volume of the 128
54 Xe nucleus assumed to be
spherical is
(a) 8V (b) 2 V (c) 6.75 V (d) 1.89 V

21. A circular conducting loop C1 of radius a is located in the xy - plane with its centre at ( 0, 0, 0).
Another circular loop C 2 of the same radius is located such that the plane of C 2 is parallel to
the xy- plane and its centre at (0, 0, 2a ). Current i flows in each of the loops such that the
Z -axis lies to the left of the directions of the currents. The magnetic field B at the point ( 0, 0, 0)
neglecting mutual inductance is
µ 0i µ 0i µ 0i  1 µ 0i  1 
(a) (b) (c) 1 +  (d) 1 + 3 / 2 
2a a 2a  5 2a  5 

22. There are only three bound states for a particle of mass m in a one-dimensional potential well
of the form as shown in the figure. The depth V 0 of the potential satisfies.
V

–a/2 a/2
X

V0

2 π 2 h2 9π 2 h2 π 2 h2 2 π 2 h2 2 π 2 h2 8π 2 h2 2 π 2 h2 50π 2h2
(a) 2
< V0 < 2
(b) 2
< V0 < 2
(c) 2
< V0 < 2
(d) 2
< V0 <
ma 2 ma ma ma ma ma ma ma 2

23. The lossless circuit shown in figure has the following parameter, Vg = 10∠0° V, Z g = 50 − j 40Ω,
β = 0.25 rad/m. Determine the input impedance Z in and voltage.
100 m

Zg

Z0 = 60 Ω j 40 Ω ZL

+
Vg

At sending end is
(a) j 29.4Ω, 2.68∠102 ° V (b) j 40.3Ω, 5.75∠102° V
(c) j 29.4Ω, 575
. ∠102 ° V (d) j 40.3Ω, 2.68∠102 ° V
Practice Set 3 ●
201

24. The p-V relation for a monoatomic ideal gas undergoing an adiabatic process
(a) pV 1/ 3 = constant (b) pV 2/ 3 = constant (c) pV 4/ 3 = constant (d) pV 5/ 3 = constant

25. Referring to the figure,

A particle of mass m, charge q is fired into a region perpendicular to a constant uniform


magnetic field B. It emerges after traversing a horizontal distance, d in the region of the
magnetic field. The angle of deviation θ is given by
 dqB d d  mv 
(a) tan−1  (b) tan−1 (c) tan−1 (d) tan−1 
 mv  2 2 mv  qBd 
m v
− d2 −d
q 2B2 qB

26. A stone is dropped from a height 100 m at the equator. The eastward deflection is
(a) 0.15 cm (b) 2.15 cm (c) 3.15 cm (d) 3.50 cm

27. The registers QD , QC , QB and Q A as shown in the figure are initially in the state 1010
respectively. An input sequence SI = 0101is applied. After two clock pulses, the state of the
shift registers (in the same sequence QD QC QB Q A ) is

CLK SI 0101
QD QC QB QA

(a) 1001 (b) 0100 (c) 0110 (d) 1010

28. The electric field E (r ,t ) at a point r at time t in a metal due to the passage of electrons can be
described by the equation
1  ∂ 2 E(r, t ) 
∇ 2 E (r, t ) = 2  2
+ ω ′ 2 E(r, t )
c  ∂t 
where, ω′ is a characteristic associated with the metal and c is the speed of light in vacuum.
The dispersion relation corresponding to the plane wave solutions of the form exp [i (i.r − ωt )]
is given by
(a) ω 2 = c 2k 2 − ω ′ 2 (b) ω 2 = c 2k 2 + ω ′ 2 (c) ω = ck − ω ′ (d) ω = ck + ω ′

29. A spherical conductor of radius a is placed in a uniform electric field P


$ The potential at a point P (r , θ) for r > a , is given by
E = E 0k. r
3
E 0a θ
φ(r , θ) = constant −E 0 r cos θ + 2
cos θ O k
r
where, r is the distance of P from the centre O of the sphere and θ is
the angle, OP makes with the z-axis.
The charge density on the sphere at θ = 30° is
(a) 3 3 ε 0 E0 / 2 (b) 3ε 0 E0 / 2 (c) 3 ε 0 E0 / 2 (d) ε 0 E0 / 2
202 ●
IIT JAM Physics Solved Papers & Practice Sets

30. A metallic sphere of radius r and carrying a charge q is enclosed by a dielectric shell of
thickness δ, outer radius r 2 and relative permittivity ε. The medium elsewhere is air. The
potential V (r ) for r > r 2 is
q ( ε − 1) r + r2 q  ε r2 + δ r − r
(a) (b)  + 2 
4 π ε0 r r2 4 π ε 0  ε r22 r2r 

q  ε r2 + δ r −r q
(c)  + 2 1 (d)
4 π ε 0  ε r2
2
r2r1  4 π ε 0r

Section B Multiple Select Questions (MSQs)


In these type of questions, each question has four choices (a), (b), (c) and (d) out of which only
one or more than one is/are correct options.

31. An electron is describing a circular orbit of radius a in a plane perpendicular to the Z -axis,
along which there is a uniform magnetic field B 0 . The magnitude of
(a) angular momentum is B0 e a 2 (b) linear momentum is B0 e v 0
(c) linear momentum is zero (d) angular momentum is B0 e a

32. Given a wave with the dispersion relation, ω = ck + m for k > 0 and m > 0, which of the
following are not true?
(a) The group velocity is greater than the phase velocity
(b) The group velocity is less than the phase velocity
(c) The group velocity and the phase velocity are equal
(d) There is no definite relation between the group velocity and the phase velocity

33. The volume of a mole of liquid He 4 is 27 × 10 −6 m 3 and the mass of a He 4 atom is


6.65 × 10 −27 kg. Assuming that liquid He 4 is an ideal boson gas with zero spin, then
(a) the concentration of boson in the volume is 2.2 × 1028 m −3
(b) the concentration of boson in the volume is 4.2 × 1026 m −3
(c) the boson temperature is 6.4 K
(d) the boson temperature is 3.1 K

34. If α and β are Dirac matrices, then
1 1
(a) α x = [α x, α y , α y ] (b) α x = [α xα yα zβ, β ]
2 2
1 1
(c) α x = [α x, α y , α x ] (d) α xα yα z = [α xα y β, β α z ]
2 2

35. If ε-K relation for one dimensional crystal lattice is ε = 2K 2 + K + 1, then


4K + 1 4K
(a) velocity of electron in lattice is (b) velocity of electron in lattice is
h h
h2 h2
(c) effective mass of electron is (d) effective mass of electron is
4 2
Practice Set 3 ●
203

a b
36. The interaction energy of two particles in the field of each other is given by U(r ) = − + ,
r ra
then
1/ 8
 9b 
(a) the particle form a stable compound if r = r0 =  
 a
1/ 8
 a
(b) the particle form a stable compound if r = r0 =  
 9b 
 a 8
(c) the potential energy in stable equilibrium is −  
 r0  9
a  8
(d) the potential energy in stable equilibrium is  
r0  9

37. A reversible engine cycle is shown in the following T-S diagram,


T
2T1

T1

S1 2S1 3S1 S

1
(a) efficiency of engine is (b) heat of source is 3T1 S 1
2
1
(c) heat of source is T1 S 1 (d) efficiency of engine is
3

38. Which of the following statement (s) is/are correct?


(a) In case of harmonic oscillator, En ∝ n−2
(b) In case of hydrogen atom, En ∝ n2
(c) In case of particle in a box, En ∝ n2
 1
(d) In case of free particle in motion En ∝  n + 
 2

39. Unpolarised light falls from air to a planar air-glass interface (refractive index of glass is 1.5)
and the reflected light is observed to be plane polarised. Which of the following conditions
is/are correct?
(a) Angle of incidence (θ i ) is 50°
(b) Polarisation vector is perpendicular to plane of incidence
(c) Angle of incidence (θ i ) is 56°
(d) Polarisation vector is parallel to plane of incidence

40. The potentials (A and φ) at the position are defined by the vector r in uniform electric and
magnetic fields. Which of the following condition(s) is/are correct?
(a) φ = − E ⋅ r (b) A = B × r
1
(c) φ = E ⋅ r (d) A = (B × r )
2
Section C Numerical Answer Type Questions (NATQs)
This section contains Numerical Answer Type (NAT) questions. For these NATQs, the answer is a
real number.

41. Velocity of light in a medium for which relative permittivity and relative permeability are
respectively 3 and 2, (in 10 8 m/s) is _________.

42. A thin mica sheet of thickness 2 × 10 −6 and refractive index (µ = 1.5 ) is introduced in the path
of one of the waves. The wavelength of the wave used is 5000 Å. The central bright maximum
will shift _______ fringes.

43. 110 J of heat is added to a gaseous system and its internal energy increases by 40 J, then the
amount of work done in joule is __________.

44. Twelve equal resistances each of 2 Ω form the edges of a cube. If a 2 V battery having internal
resistance 0.1 Ω is connected between two adjacent corners of a cube, then the current (in
ampere) drawn from battery is _________.

45. In Bragg’s experiment of diffraction by crystals using X-rays of wavelength λ Å, the glancing
angle of the first order spectrum is 8°. If the lattice spacing is 2.82 Å, then using
sin 8 ° = 01392
. , we can have value of λ (in nm) in ________.

46. A common emitter amplifier is designed with n-p-n transistor (α = 099


. ). The input impedance is
1 k Ω and load is 10 k Ω. The voltage gain will be _________.

47. An open organ pipe has a fundamental frequency of 300 Hz. The first overtone of this organ
pipe is the same as the first overtone of a closed organ pipe. The length of the closed organ
pipe (in m) is ________. [Assume velocity of sound in air as 330 ms −1]

48. The value of lattice constant a (in Å) for a fcc cubic having density 6250 kg/m 3 and atomic
weight 60.2. (Avogadro’s number, N = 6.02 × 10 23 ) is ________.

49. The collision time and root mean square velocity of the electron at room temperature are
2.5 × 10 −14 s and 1 × 105 m/s respectively. The classical value of mean free path of the
electron (in nm) is ________.

50. The mass equivalent of the energy from an antenna radiating 10000 W for 24 h in (10 −9 kg) is
_________.

51. In an air filled waveguide, a TE mode operating at 6 GHz has


 2πx   πx 
E y = 15 sin  cos   sin (ωt − 12z ) V/m
 a  b 
The cut-off frequency (in GHz) is ________.

52. An electromagnetic wave whose electric field is given by,


E (r, t ) = ( $i + 2i $j − 2 k$ ) exp [i (kz + 2kx − ωt )] is incident from vacuum (z < 0) to a dielectric
interface, coinciding with xy- plane. If the reflected light is linearly polarised, then refractive
index of the dielectric is _________.
Practice Set 3 ●
205

53. A plane electromagnetic wave, E 2 = 100 cos (6 × 10 8 t + 4x ) V/m propagates in a medium of


dielectric constant _________.

54. The values of the component R 2 is k Ω is such that the frequency of the Wien bridge oscillator
is 300 Hz (Given, C = 0.01µF and R1 = 12 k Ω)
R1 R2

V0

C
R

R C

π
55. Two thin conducting half planes at φ = 0 and φ = , are insulated from each other along the
6
π  60φ
Z-axis. Given that the potential function for 0 ≤ φ ≤ is V =  −  V, the energy stored
6  π 
between the half planes for 0.1 ≤ r ≤ 0.6 m and 0 ≤ Z ≤ 1 m (assuming free space) in nJ is
_______.

56. If a Zener diode (Vz = 5 V and I Z = 10 mA) is connected in series with a resistance and 20 V
is applied across the combination, then the maximum resistance one can use without spoiling
Zener action, (in k Ω) is _________.

57. If the lowering of melting point of ice per atmosphere increase of pressure is 0.0072°C and the
saturated vapour pressure at 0°C = 460 mm while at 1° C = 4.94 mm, then the temperature
(in °C) at triple point is __________.

58. Consider the Fermi-Dirac distribution function f (E ) at room temperature (300 K), where E
refers to energy. If E F is the Fermi energy, then f (E F ) has a value of _________.
π
59. The scattering amplitude in partial wave analysis is given by f (θ) = cos 2θ + i 2 sin θ +  .
 2
Propagation constant is k = π. The value of imaginary part at θ = 0 is __________.

60. One mole of helium and one mole of oxygen contained in two vessels of same volume
have same entropy. If temperature of He be 300 K, then temperature of oxygen (in K) is
_________.

Answers
1. (c) 2. (c) 3. (a) 4. (b) 5. (a) 6. (d) 7. (a) 8. (d) 9. (b) 10. (c)
11. (a) 12. (c) 13. (b) 14. (b) 15. (b) 16. (c) 17. (d) 18. (a) 19. (c) 20. (a)
21. (d) 22. (c) 23. (c) 24. (d) 25. (b) 26. (b) 27. (c) 28. (a) 29. (a) 30. (d)
Answers with Explanations
Section A Multiple Choice Questions rmin 1− e
=
rmax + rmin 2
1. (c) 2rmin
or e = 1−
rmax + rmin
r −r
60° e = max min
rmax + rmin
Now, by conservation of angular momentum rv is
constant throughout the motion
l l mgl rmaxv min = rminv max
Change in PE = mg − mg cos 60° =
2 2 4 rmax − rmin v max − v min
1 2 mgl Thus, = =e
lω = rmax + rmin v max + v min
2 4
1
2∫
⇒ 2
ω =
mgl
=
mgl 5. (a) WE = ε 0 E 2dV
2l ml 2
2× 2
3 ε0 h + 0. 5 2 π 0. 05 10 5 

⇒ ω=
3g =
2 ∫h ∫0 ∫0. 01  r  rd rd φ dz
2l
ε0
= 105[In r ]hh + 0. 5 ⋅ [ φ] 20π ⋅ [ z ] 00.. 05
01 = 0.224 J
2. (c) By definition ∂L 2
px = = 2ax
∂x 6. (d) At triple point temperatureTt can be found using any of
∂L the two relation and must be unique
and py = = 2 by
∂y 3050 3750
The Hamiltonian is therefore, 19.5 − = 23 −
Tt Tc
H = px x + py y − L 3050 3750
− + = 23 − 19.5
p x2  ap 2 bp y2
p y2  Tc Tc
= + −  x2 + − kxy 
2a 2b  4a 4b 2
 700
= 3.5
2 Tc
p x2 p y 700
= + + kxy or Tc = = 200K
4a 4b 3.5
3. (a) The electric field due to an infinite line charge distribution 7. (a) AC current/voltage is a sinusoidal curve. The current or
at point P potential is represented as,V = V0 sin ωt
1 λ
E1 =
2 πε 0 a I (V )
And that is due to an infinite line charge distribution at
t
point Q
1 λ
E2 =
2 πε 0 a
Their ratio where, ω → angular frequency
1 λ 1 V0 → peak voltage
2 πε 0 a a V0
E1 : E2 = = or 1: 1 Root mean square of potential, i. e. Vrms =
1 λ 1
2
2 πε 0 a a
In AC circuit, the potential is given in terms of rms hence,
rmax 1 + e
4. (b) As, =
rmin 1 − e
Adding 1 on both sides, 100 V R
rmax + rmin 2
or =
rmin 1− e
Here, Vrms = 100 V
Practice Set 3 ●
207

8. (d) The energy in nth state of hydrogen atom is given as, 12. (c) B = ∇ × A
13.6
En = 2 eV.  $i $j k$ 
n 
B= ∂ ∂ ∂ = $i  ∂Az − ∂Ay 
1   
⇒ En ∝ 2 ∂x ∂y ∂z 
  ∂y ∂z 
n Ax Ay Az 
1
Thus, energy in hydrogen atoms is proportional to the .  ∂A ∂Az 
n2 ⇒ Bx $i + By $j + Bz k$ = + $j  x − 
 ∂z ∂x 
 cos θ 0 sin θ   ∂Ay ∂Ax 
+ k$  − 
9. (b) We have, A =  0 1 0   ∂x ∂y 
 
− sin θ 0 cos θ  Q B0 is not along z-direction.
cos θ 0 − sin θ  ∂Ay ∂Ax
⇒ Bz = 0 ⇒ − =0
⇒ A = 0
−1
1 0  ∂x ∂y
  ∂Ay ∂Ax
 sin θ 0 cos θ  ⇒ =
∂x ∂y
 cos θ 0 sin θ  cos θ 0 − sin θ 
∴ −1
= 0 0   0 0  B x B y 
AA 1 1 Only A =  0 , 0 , 0 satisfies the above condition.
     2 2 
− sin θ 0 cos θ   sin θ 0 cos θ 
13. (b) V(α ) = 0 for 0 < x < a
 cos2 θ + sin2 θ 0 − cos θ sin θ + cos θ sin θ 
  V = ∞ for x ≤ 0 or x ≥ a
= 0 1 0 
2 2
When it is in ground state
− sin θ cos θ + sin θ cos θ 0 sin θ + cos θ 
  a  3a 
at   < x <  
 4  4
 1 0 0
= 0 1 0 Then its probability 
 3a a  2a a
−  = =
   4 4 4 2
0 0 1
 a 1 
So, its value is  +  .
10. (c) The coriolis force is −2mω × u.  2 π
ω and u are mutually perpendicular. If ω is taken along
14. (b) The acceleration due to earth, on the surface is given
Z -axis and u radially outward, then ω × u = ω u k$ × r$ = ω u θ
as
11. (a) The magnetic field due to a circular wire of radius a P
h
carrying current i, at a point on its axis at a distance x from
the centre is given by Re

µ i a2 P
B= 0 d Re
2 (a + x 2 )3/ 2
2

θ1 P GMem
θ2 mg =
R2
GMe
For n turns, it becomes ⇒ g=
µ i na 2 Re2
B= 0
2 (a + x 2 )3/ 2
2 ⇒ log g = log (GMe ) − 2 log Re
∆g 2 ∆Re
Consider n dx circular wires at a distance x from P, ⇒ = 0−
2
g Re
µ 0i na dx
dB = ∆g 2 ∆Re
2 (a 2 + x 2 )3/ 2 ⇒ =−
g Re
µ i x +d dx
B = 0 na 2 ∫ ∆g  ∆R 
2 x (a 2 + x 2 )3/ 2 ⇒ % = − 2  e%
g  Re 
Let x = a cot θ
∆R
µ nia 2 θ 2 a cosec2θ µ ni Here, = − 1%
2 ∫θ1 a 3cosec3θ
B=− 0 dθ = 0 (cos θ 2 − cosθ) R
2 ∆g
  So, in% = − 2( −1)% = 2%
µ ni x +d x g
B= 0  − 
2  a 2 + (x + d )2 a 2 + x 2  Hence, g increases by 2%.
208 IIT JAM Physics Solved Papers & Practice Sets

15. (b) E = E0 cos (ωt − k. r ) 19. (c) Using Ampere’s law


= E0 cos(ωt − kx cos θ1 − ky cos θ 2 − kz cos θ 3 ) ∫ B ⋅ d I = µ 0i
Y d
⇒ B ⋅ 2 πd = µ 0 j 0 ∫0r × 2πr dr
^
k x cos 60° d3
= µ 0j0 2π
60° 3
µ0
30°
X ⇒ B= j0d 2φ$
^
k cos 30°
3

From figure
d
= E0 cos (ωt − kx cos 30° − ky cos 60° )
 3 1 
= E0 cos ωt − kx − ky 
 2 2 

16. (c) Here, E = E(t )( −k$ ) and B = B (t )( − φ$ )


20. (a) Experimental evidence show that the distribution of
1
Hence, S = ( E × B) nuclear matter is nearly uniform, so that the nuclear
2µ0
matter density ρm is also approximately constant. Since,
E(t )B(t ) $ $ E(t )B(H ) $ nuclear mass is almost linearly proportional to the mass
= (k × φ ) = (r )
2µ0 2µ0 number A, this means that.
17. (d) The efficiency of Carnot heat engine may be defined as ρm ~ A /V = constant
Q1 − Q 2 T1 − T2 i. e. the nuclear volumeV ∝ A. Assuming a spherical
η= = shape of the nucleus with a radius R, we get
Q1 T1
4
where, Q1 is the heat taken from the source by working V = πR 2 ∝ A
3
substance and Q 2 is the heat given by working or R ∝ A1/ 3
substance to sink respectively,T1 and T2 are the
R = r0A1/ 3 ...(i)
temperatures of source and sink, respectively.
where, r0 is a constant known as the nuclear radius
Hence, in question
parameter.
Q1 = 450 kJ/s, Q 2 = ? 4
T1 = 22 + 273 = 295 K, For 16
8 O, the volume V = πR03 ...(ii)
3
T2 = 273 K From Eqs. (i) and (ii), we have
450 − Q 2 4
Hence, = V0 = π(r03 A )
450 3
295 − 273 and charge radius of oxygen is
=
295
R0 = r0 (16)1/ 3 ...(iii)
273 × 450
or Q2 =
295 Similarly, for 128
54 Xe, the charge radius R Xe is
= 416.44 kJ/s ≅ 417 kJ/s RXe = r0 (128)1/ 3 ...(iv)
 r From Eqs. (iv) and (iii), we have
18. (a) F (q ) = ∫ exp iq ⋅  ρ(r )d 3r
 h RXe  128
1/ 3
=  =2
ρ(r ) is a delta function and we know R0  16 
∫ f (r ) ⋅ δ(r − r0 )d r = f (r0 )
3
⇒ RXe = 2R0
At r = r0 and at r0 = 0 Therefore, the volume of 54 Xe128 is
4
∫ f (r ) ⋅ δ (r − r0 ) d
3
r = f ( 0) VXe = π ( 2R0 )3
3
 iq ⋅ r  4 
f (r0 ) = exp   = 8  πR03 = 8V.
 h  3 
r0 = 0
 iq ⋅ 0
21. (d) The magnetic field due to a circular wire carrying a
0
f ( 0) = exp   =e =1 current i is given by
 h 
µ 0i a2
 iq ⋅ r  B(x ) =

3
or exp   δ(r − r0 ) d r = 1. 2 (a + x 2 )3/ 2
2
 h 
Practice Set 3 ●
209

where, a is the radius of the circular wire and x is the 3 NkT


NkdT = − dV
distance from the centre of the circle to the point on the 2 V
axis where the magnetic field is found out. dT 2 dV
+ =0
Z T 3 V
2
C2
In T + In V = constant
3
i
or TV 2/ 3 = constant
2a
or pV 5/ 3 = constant
C1
Y 25. (b) The path of the particle described inside the region of
i the magnetic field is circle of radius r is given by
mv
r =
X qB
Magnetic field at the centre of the ring is obtained by The equation of a circle of radius r, centre at ( 0, r ) is
setting x = 0 x 2 + (y − r )2 = r 2
µ i
B(x = 0) = 0 Y
2a
In the present problem it is required to find the magnetic
field at the centre of the circular wire C1. The magnetic (0,r)
fields at (0, 0, 0) due to the current in C1 and C2 are in
θ
the same direction. X
µ i µ i a2 $ After emerging from the region, the particle will travel in
So, B= 0 + 0 2 3/ 2 
k
2a 2 ( 2
+ 4 ) a straight line tangent to the circle. The slope of the
 a a 
tangent at x = d is obtained after differentiation.
µ i  1 
B = 0 1 + 3/ 2  k$ dy
2a  5  2x + 2(y − r ) =0
dx
22. (c) For bound state dy x
⇒ =
n 2π 2h 2 dx (r − y )
V0 − | E | =
2ma 2 At x = d , y = ± r 2 − d 2 + r
n 2π 2h 2
⇒ V0 = + |E | mv mv m 2v 2
2ma 2 Let r = ⇒ y = ± −d2
qB qB q 2B 2
There are only three bound states.
dy d
Hence, n = 2 to n = 4 So, =
dx 2 2
x =d mv
2π 2h 2 8π 2h 2 −d2
⇒ 2
< V0 < q 2B 2
ma ma 2
mv
If > d , then angle of deviation is
23. (c) 0.25100 = 25 rad = 352.4° qB
At sending end d
tan−1
 j 40 + j 60 tan 352.4° m 2v 2
Z in = 60  = j 29.4 −d2
 60 − 40 tan 352.4° 
q 2B 2
Z in
V ( z = 0) = V0 = Vg If
mv
< d , then the particle will not have the region of the
Z in + Z g qB
 j 29.4  magnetic field. It will turn back into the region.
=  10∠ 0°
 j 29.4 + 50 − j 40
26. (b) The deflection is given by the formula
= 5.75∠102° V ωg  2h 
1/ 2
x =   cos θ
24. (d) E = internal energy 3 g
dE = − dW (for adiabatic process) where, θ is the latitude at the equator, θ = 0
3 3/ 2
Energy, E = NkT ωg  2h  7.2 × 10−5 ( 2 × 100)3/ 2
x =   =
2 3 g 3 10
3
which gives, dE = NkdT where, we have substituted, ω = 7.2 × 10−5 rad/s and
2
NkT g = 10 ms−2
dW = pdV = dV
V x = 2.15 cm.
210 IIT JAM Physics Solved Papers & Practice Sets

27. (c) Here, QDQCQBQ A = 1010 and SI = 0101 Potential at r > r2


q
After 1st pulse, output = 0010 V = − ∫ E. dr =
After 2nd pulse, output = 0110 4π ε 0r
and potential at centre
28. (a) Here, E = E0 exp[k. r - ωt ] 0
V =−∫ E. d I
∇ 2E = k 2E0 exp[i. k. r - ωt ] ∞
r2 q  r1 q 0
∂ 2E =−∫   dr − ∫ dr − ∫ 0. dr
⇒ ∇ 2E = k 2E and = ω 2E ∞  4 πε r 2  r 2 4 πε r 2 r1
∂t 2 0 0

So, for points r > r2


1  ∂ 2E 2 
Now, ∇ 2E =  2 + ω′ E  (Given) V =
q
c2  ∂t  4 πε 0r
1 2
⇒ k 2E = [ω E + ω′ 2 E]
c2
⇒ c 2k 2 = ω 2 + ω′ 2 Section B Multiple Select Questions
ω 2 = c 2k 2 − ω′ 2 31. (a, b) To keep the electron in its orbit of radius a, in a plane
29. (a) Poisson’s equation in spherical coordinates is perpendicular to Z -axis, along which there is a uniform
I ∂  2 ∂I  1 ∂  ∂I  magnetic field B, we have
∇2 I = r +  sin θ 
r 2 ∂r  ∂r  r 2 sin θ ∂θ  ∂θ  mv 02
= B0ev 0
1 ∂ 2I −ρ a
+ = ⇒ Linear momentum, p = B0ev 0
r sin θ ∂φ 2 ε 0
2 2
Q ρ is independent of φ. Angular momentum, L = r × p
∂ 2I = a × B0ea = B0ea 2
⇒ =0
∂φ 2
32. (a, c, d) Here, ω = ck + m and k , m > 0
In this case, I (r , θ ) can be written as ω m
v phase = =c +
 a3 k k
I (r , θ ) = −E0 r − 2  cos θ …(i) dω
 r  v group = =c
dk
The contribution of induced charge is Hence, v phase > v group
3
=
E0 a
cosθ N 6.02 × 1023
33. (a, d) Concentration of boson = =
r2 V 27 × 10−6 m3
Induced charge density using Eq. (i), we get = 2.2 × 1028 m−3
∂I  2q 3  Now, gs = 2S + 1 = 1 (Q S = 0)
σ(θ ) = − ε 0 = cos ε 0E0 1+ 3 
∂r r =a  r  Boson temperature
r =a
2 πh 2  N 
σ(θ ) = 3ε 0E0 × cos θ Tb =   = 3.1 K
mk  2.612 × gV 
3 3
σ(θ ) θ = 30° = 3ε 0E0 × cos 30° = ε 0E0
2 34. (a, b) [α x α y ,α y ] = α x α y α y − α y α x α y
30. (d) Inside the metal sphere = α x α 2y + α x α y α y
E = P = D = 0 and D = εE = α x + α x α 2y
[Q α y α x = −α x α y and α 2x = α 2y = 1]
= α x + α x = 2α x
r2 r1 Now, [α x α y α z β, β] = α x α y α z ββ − β α x α y α z β
= α x α y α z β 2 + α x βα y α z β
Q $ = α x α y α z β 2 − α x α y βα z β
D= r for all points r > a
4 πr 2 = α x α y α z + α x α y α z ββ
 q $ = α xα yα z + α xα yα z
r for r1 < r < r2
 εr 2 = 2a x α y α z
∴ E =  4πq
 r$ for r > r2 1
2 ⇒ α x α y α z = [α x α y α z β, β]
 4πε 0r 2
Practice Set 3 ●
211

35. (a, c) ε = 2K 2 + K + 1 40. (a, d) We can express, E = $i Ex + $j Ey + k$ Ez = ( E ⋅ ∇ ) r


dε E = uniform,
⇒ = 4K + 1
dK ∴ E = ∇( E ⋅ r ) ...(i)
1 dε 4K + 1 In case of static electric field
v = ⋅ =
h dK h E = − grad φ = − ∇φ ...(ii)
2
d ε or ∇( − φ ) = ∇ ( E ⋅ r ) [by comparing Eqs. (i) and (ii)]
Now, =4
dK 2 i. e. φ = − E.r …(iii)
h2 h2 Now, for uniform B
Effective mass = 2 2
=
d ε / dK 4 curl ( B × r ) = B div r − ( B. ∇ )r (using vector triple product)
a b B( 3) − B = 2B [div r = 3]
36. (a, c) Here, U (r ) = − + 9
r r 1
⇒ B = curl ( B × r ) ...(iv)
dU (r ) a 9b 2
We have = 2 − 10
dr r r But according to definition
dU (r ) B = curl A ...(v)
For stable compound =0
dr r = r 0 Comparing Eqs. (iv) and (v), we get
1
a 9b curl A = curl ( B × r ) ...(vi)
⇒ − =0 2
r02 r010
1/ 8
So, from Eqs. (iii) and (vi), we get
9b  φ = − E. r
⇒ r0 =
 a 
1
A = (B × r )
The energy corresponding to r = r0 is 2
−a b
U = + 9
r0 (r0 )
Putting the value of r0, we get Section C Numerical Answer Type Questions
−a  8
U =   41. 1.21 to 1.23
r0  9 
c 3 × 108 3
c′ = = = × 108 m/s = 1.22 × 108 m/s
37. (b, d) In given figure, available energy per cycle is εr µr 3×2 2
 1 
Q1 − Q 2 = 2T1 × S1 + T1 × 2S1 − 2T1S1 = T1S1 42. 2 to 2
 2 
Shift in number of fringes,
and Q1 = 3T1S1
−6
Q − Q2 . − 1) × 2 × 10
(µ − 1) t (15
Efficiency, η= 1 n= = = 2 fringes
Q1 λ 5000 × 10−10
T1S1 1 43. 68 to 72
= =
3T1S1 3 From dQ = dU + dW
dW = dQ − dU = 110 − 40 = 70 J
38. (c, d) (i) According to classical physics a particle in
state of motion has any energy, i. e. its energy in 44. 1.55 to 1.60
different state is continuous. Total resistance of the circuit
(ii) The energy of an electron orbiting in nth orbits 7r 7×2 7.6
of the hydrogen is proportional to = + 0.1 = + 0.1 =
12 12 6
(n −2 ), i. e. En ∝ n −2 E 2×6
Current, I = = = 1.58 A
(iii) According to quantum theory, the energy of a 7.6 / 6 7.6
particle in a box is given as, En ∝ n 2
45. 7.850 to 7.852
(iv) The energy of an oscillator is given as
Given, sin 8° = 0.1392
 1
En ∝ n +  Lattice spacing (a ) = 2.82 Å = 2.82 × 10−11 m
 2
Since, 2d sin θ = nλ
39. (b, c) The angle of incidence θi = tan−1 (n ) ⇒ 2 × 2.82 × 0.1392 × 10−10 = nλ
⇒ θi = tan−1 (1.5) = 56° For n = 1, λ = 5.64 × 0.1392 × 10−10
and the polarisation vector is perpendicular to the λ = 0.7851 × 10−10
plane of incidence. = 7.851× 10−9m = 7.851nm
212 IIT JAM Physics Solved Papers & Practice Sets

46. 989 to 992 ⇒ cos θi = sin θT


R  α  Ro 1
Voltage gain, Av = β o =   ⇒ sin θT =
Ri  1 − α  Ri 5
2
 0.99   10 kΩ 
=  ×  = 990 sin θi
⇒ n= 5 =2
n
 1 − 0.99   1kΩ  As =
sin θT 1 1
47. 0.40 to 0.42 5
2v 3v 3 53. 4.00 to 4.02
Given, = ⇒ l2 = l1
2l1 4l2 4 We have, E2 = 100 cos ( 6 × 108 t + 4x ) ...(i)
3 330  v  Comparing this with E = a cos (ωt + kx ), we get
⇒ l2 = × as, 2l = 300
4 2 × 300  1  ω = 6 × 108 ...(ii)
∴ l2 = 0.41m k =4 ...(iii)
ω 6 × 108
48. 3.9 to 4.1 Velocity of emf is given = =
1/ 3 1/ 3 k 4
 nM   4 × 60.2 
Lattice constant a =   =  v = 1.5 × 108 m/s ...(iv)
 Np   6.02 × 10 23
× 6250 
By Eq. (iii) refractive index of the medium
⇒ a~− 4 Å.
c 3 × 108
n= =
49. 2.4 to 2.6 v 1.5 × 108
Mean free path, λ = v τ = 105 × 2.5 × 10−14
⇒ n = 2. ...(v)
= 2.5 × 10−9 m = 2.5 nm The dielectric constant K = (n 2 )
50. 9.5 to 9.7 K = ( 2)2 = 4.0
2
∆mc
P = 54. The frequency of oscillation f0 is exactly the resonant
t
P × t 10000 × 24 × 60 × 60 frequency of the balanced Wien bridge and is given by
⇒ ∆m = 2 = 1 0.159
c ( 3 × 108 )2 f0 = = …(i)
2πRC RC
= 96 × 10−10 kg = 9.6 × 10−9 kg
Assuming that the resistors are equal in value and the
51. 5.972 to 5.974 capacitors are equal in value in the reactive lag of the Wien
m = 2 , n = 1, β p = 12 , f = 6 GHz bridge.
2
ω f  R1 R2
⇒ βp = 1−  c 
v f  Amplifier
9 2
2π × 6 × 10 f 
or 12 = 1−  c  V0
3 × 108  6
⇒ fc = 5.973 GHz.
52. 1 to 3 R
Angle of incidence
C
n^ C Feedback
θ1
θ3i circuit

90°
θT Wien bridge oscillator
1
From Eq. (i), 300 =
k ⋅ n$ 1 2πR × 0.01× 10−6
cos θi = =
| k ||⋅ n$ | 5 1
⇒ R= = 53 kΩ
2 2π × 300 × 10−8
⇒ sin θi =
5
55. 1.50 to 1.52
As, the reflected light is polarised therefore, reflected and To find the energyWE , stored in a limited region of space,
refracted light beam are perpendicular to each other we must integrate the energy density through the region.
∴ θi + θT = 90 Between the half planes
Practice Set 3 ●
213

1 ∂  60φ  Increase in melting point


E = − ∇φ = − −  aφ
r ∂φ  π  0.0072 × ( 755.4 − 0.34 t )
=
60  V 760
= aφ  
πr  m  0.0072 × ( 755.4 − 0.34 t ) 
or melting point = 0 +
and so,  760 
ε0 But this is the temperature t at a triple point. Thus,
2 ∫
WE = E 2dV
0.0072 × ( 755.4 − 0.34 t )
ε 1 π / 6 0. 6  60
2 t=
= 0∫ ∫ 760
2 0 0 ∫0.1  πr 
  rd rd φ dz
or 760t = 5.4388 − 0.002448t
300 ε 0 or 760.00 − 2448t = 5.4388
= In 6
π 5.4388
t= = 0.007156° C
⇒ WE = 1.51nJ 760.002448
56. 1.49 to 1.51 58. 0 to 1
The principal use of a Zener diode is in a voltage regulator. 1
As, f (E ) =
In a voltage regulator, the voltage across the load remains exp [(E − EF ) / kT + 1]
constant over a range of current.
For E = EF
V − VZ 1 1
So, Rmax = f (EF ) = =
I exp [ 0] + 1 2
20 − 5
= −3 = 1.5 kΩ 59. 0 to 4
10 × 10
Here, f (θ ) = cos 2θ + i 2 sin (θ + π / 2)
57. 0.007154 to 0.007158  π
Imf (θ ) = 2 sin θ + 
Let the triple point correspond to a temperature t °C and  2
pressure p mm of Hg.
Imf ( 0) = 2 sin( 0 + π /2 ) = 2
Saturated vapour pressure at 0°C = 4.60 mm.
Saturated vapour pressure at 1° C = 4.94 mm. 60. 37 to 38
∴Increase in saturated vapour for 1°C rise in temperature Entropy of a gas containing N-particles is given,
= 4.94 − 4.60 = 0.34 mm   2πmkT  3/ 2 5 
δ = Nk In   + 
Hence, increase in vapour pressure for t ° C = 0.34t  h2  2

Therefore, saturated vapour pressure at triple point t °C is where, m is mass of particles.
given by
∴ For the given situation (N = Na = Avogadro’s number)
p = ( 4.60 + 0.34t ) mm
  2πmHekTHe 3/ 2 5 
Now, the melting point of ice at 760 mm (1 atmosphere) = Nak In   + 
pressure is 0°C. But at the triple point the pressure is  h2  2

( 4.60 + 0.34t ).
  2πm0kT0  3/ 2 5 
Hence, decrease in pressure = 760 − ( 4.60 + 0.34t ) = Nak In   + 
 h2  2
= ( 755.4 − 0.34t ) mm 
It is given that a decrease in pressure of 760 mm ∴ m0T0 = mHeTHe
(1 atmosphere) will increase the melting point of ice by
m T 4 T 300
0.0072°C. Therefore, by a decrease of ( 755.4 − 0.34 t ) mm or T0 = He He = × THe = He = = 37.5K.
m0 32 8 8
in pressure.

You might also like